Download as pdf or txt
Download as pdf or txt
You are on page 1of 329

S. K.

Ukarande

Irrigation
Engineering
and Hydraulic
Structures
Irrigation Engineering and Hydraulic Structures
S. K. Ukarande

Irrigation Engineering
and Hydraulic Structures
S. K. Ukarande
K J Somaiya Institute of Technology
Affiliated to University of Mumbai
Mumbai, India

ISBN 978-3-031-33551-8 ISBN 978-3-031-33552-5 (eBook)


https://doi.org/10.1007/978-3-031-33552-5

Jointly published with ANE Books India


The print edition is not for sale in South Asia [India, Pakistan, Sri Lanka, Bangladesh, Nepal and Bhutan]
and Africa [all countries in the African subcontinent]. Customers from South Asia [India, Pakistan, Sri
Lanka, Bangladesh, Nepal and Bhutan] and Africa [all countries in the African subcontinent] please order
the print book from: Ane Books Pvt. Ltd.
ISBN of the Co-Publisher’s country edition: 978-93-8365-689-9.

1st edition: © Author 2018


© The Author(s) 2023

This work is subject to copyright. All rights are solely and exclusively licensed by the Publisher, whether
the whole or part of the material is concerned, specifically the rights of translation, reprinting, reuse
of illustrations, recitation, broadcasting, reproduction on microfilms or in any other physical way, and
transmission or information storage and retrieval, electronic adaptation, computer software, or by similar
or dissimilar methodology now known or hereafter developed.
The use of general descriptive names, registered names, trademarks, service marks, etc. in this publication
does not imply, even in the absence of a specific statement, that such names are exempt from the relevant
protective laws and regulations and therefore free for general use.
The publishers, the authors, and the editors are safe to assume that the advice and information in this book
are believed to be true and accurate at the date of publication. Neither the publishers nor the authors or
the editors give a warranty, expressed or implied, with respect to the material contained herein or for any
errors or omissions that may have been made. The publishers remain neutral with regard to jurisdictional
claims in published maps and institutional affiliations.

This Springer imprint is published by the registered company Springer Nature Switzerland AG
The registered company address is: Gewerbestrasse 11, 6330 Cham, Switzerland
Dedicated
to
My Parents, Wife Asmita
and Daughter Prajakta
Preface

This book, written in a very concise form as a textbook of Irrigation Engineering


and Hydraulic Structures will be useful to undergraduate students. It will also be
useful for elective subjects like: 1. Applied Hydrology and Flood Control and
2. Design of Hydraulic Structures. It is written in a simple language, so that average
student will understand the subject with self-study.
Unlike a research document, a textbook covers all basic useful information and
gives in a palatable form to the students in a self-explanatory manner. In doing so, I
have utilized existing state of knowledge available in the form of textbook, treatises
and research publication by earlier contributors in these fields which I gratefully
acknowledge. A list of such contributors is given in the references at the end.
I do not claim any originality of basic concepts in this book. The manuscript
is checked thoroughly to make this book, near to perfect. I will be grateful to the
users/readers of this book to point out mistakes, if any, so that they can be rectified
in the next edition of this book.
It would be difficult to mention names of all the persons who have given their
valuable help in making this project successful. However, I would like to place on
record my gratitude towards Dr. P. R. Mehta, Retired Principal of SVRCE, Surat
and Ane Books Pvt. Ltd. for publishing this book.

Dr S. K. Ukarande

vii
Contents

Prefaceiv
1. Introduction 1–4
● Practice of Irrigation in India ● Scope ● Status of Irrigation in India ● Impact of Irrigation
on Human Environment ● Irrigation Systems ● Command Area Development ● Development
of an Irrigation Project

2. Water Requirement of Crops 5–25


● Crops and Crop Seasons ● Definitions of Terms Used in Calculation of Irrigation
Demand ● Quality of Irrigation Water ● Soil Water Relationship ● Soil Characteristics From
Irrigation Considerations ● Consumptive Use ● Field Irrigation Requirement (FIR) ● Frequency
of Irrigation ● Irrigation Methods ● Exercises

3. Hydrology 26–68
● Scope of Hydrology ● Rainfall ● Surface Run–Off ● Flood Flow Calculation ● IUH, S and
Synthetic Unit Hydrograph ● To Get Unit Hydrograph of Different Duration from that of Given
Duration ● Rainfall/Flood Frequency Analysis ● Exercises

4. Ground Water 69–94


● Ground Water Occurrence and Resources ● Well Irrigation ● Well Hydraulics ● Recuperation
Test ● Well Shrouding ● Well Development ● Collector or Radial Well ● Well Losses ● Unsteady
Groundwater Flow Toward Wells ● Exercises

5. Reservoir Planning 95–119


● Introduction ● Types of Reservoirs ● Investigation for Reservoir Sites ● Zones of Storage
in a Reservoir ● Sedimentation in Reservoir ● Useful Life of Reservoir ● Factors Affecting
Selection of Reservoir Site ● Determination of Reservoir Capacity from Mass Curves ● Reservoir
Losses ● Reservoir Flood Routing ● Fixing Height of Dam from Reservoir Working
Table ● Exercises

6. Design of Gravity Dams 120–154


● Introduction ● Forces Acting on Gravity Dams ● Theoretical Profile of Gravity Dam ● High
and Low Gravity Dam ● Factor of Safety for Design of Gravity Dam ● Practical Profile of Gravity
Dam ● Drainage Gallery ● Joints in Gravity Dam ● Foundation Treatment ● Types and Function
of Galleries ● Exercises

ix
x Irrigation Engineering and Hydraulic Structures

7. Spillways and Gates 155–177


● General ● Ogee Spillway ● Design of Stilling Basins ● Design of Ski-Jump Bucket and Roller
Buckets ● Siphon Spillways ● Limiting Head of Siphon Spillway ● Spillway Crest Gates ● Design
of Air Vent for Vertical Lift Gates Provided in Supply Sluices of the Dam ● Cavitation and
Vibration Problem in Hydraulic Structures ● Exercises

8. Arch and Buttress Dams 178–189


● Introduction ● Types of Arch Dams ● Design of Arch Dam by Thin Cylinder Theory ● Other
Varieties of Arch Dam ● Buttress Dams ● Exercises

9. Earth Dams 190–217


● Earth Dam and its Component Parts ● Estimation of Seepage Through Homogeneous Earth
Dam ● Design Considerations ● Causes of Failure of Earth Dam ● Filters: Criteria and
Design ● Exercises

10. Canal Head Works 218–240


● Barrage and Weirs ● Different Units of Canal Head Works ● Design of Weir ● Bligh’s and
Khosla’s Theories ● Khosla’s Method of Independent Variables ● Salient Features of Design of
Weir on Pervious Foundation ● Numerical Problems on Weir Design: ● Sediment Control at
Canal-Head Works ● River Training Works for Canal Head Works ● Exercises

11. Distribution System 241–262


● General ● Canal System ● Alignment of Canals ● Losses and Types of Cross-Section for
Canal ● Design of Alluvial Channels ● Waterlogging ● Canal Lining ● Design of Lined
Canals ● Exercises

12. Canal Structures 263–279


●Distribution and Measurement Structures for Canal Flows ● Canal Outlets ● Cross-Drainage
Works (C.D. Works) ● Exercises

13. Sediment Transport in Alluvial Canals 280–295


● Transport of Sediment in Alluvial Canals ● Threshold Movement ● Shield’s Entrainment
Function ● Bed Load Formula by Einstein ● Bed Load Formula by Meyer–Peter ● Du-Bois
Bed Load Formula ● Suspended Load Concentration ● Tractive Force Method of Design of
Stable Channel ● Exercises

14. River Training Works and Flood Control 296–303


● General ● Classification of Rivers ● River Training Works ● Flood Control ● Exercises

15. River Flow Measurement 304–316


● Need for Discharge Measurement in River Section ● Current Meters ● Cross-Sectional Area-
Velocity Method to Measure Discharge ● Slope Area Method ● Stage-Discharge Relationship

References 317–319
A. Standard Books and Treatises 317
B. Technical Papers and Reports 318

Subject Index 321-322


1
Introduction

1.1 PRACTICE OF IRRIGATION IN INDIA


Water is the most essential commodity required on earth without which both human
and animal life cannot survive and that is why nature has covered three-fourth of
earth’s surface with water in the form of oceans. It is evaporation from oceans and
appropriate wind forces that bring rain on land and this becomes chief source of
potable water for human as well as animal race. In addition to this, water in form
of ice on high peaks of mountains, dew on open ground and ground water below
earth’s surface is also available but human race prefers rain, as it is available freely
as a gift from nature. But this gift of nature may not prove to be a boon each year,
for it's uneven distribution and does not occur as and when required. There will
be large regions of scanty and untimely rainfall every year. This makes cultivators
helpless and sometimes they prefer to abandon farming. For an excellent yield from
farms, what is required is occurrence of rain as and when needed by crops. This
is seldom possible. Hence, it is from prehistoric time, man has attempted to store
rain water and supply it to crops as per crop-requirement, resulting in good yield
of crop. This practice of supplying stored water on to farms is known as ­irrigation
and is being practiced since dawn of civilization.

1.2 SCOPE
Modern times have made irrigation practice more systematic and reliable by
constructing huge reservoirs and canal network with the help of hydraulic structures
such as dams and spillways. Thus, irrigation engineering deals with storage and
supply works to be constructed for the purpose of making water available to
farms as and when needed and in the quantity required for optimum growth of the
crops. A well-irrigated farm can produce hundred per cent yield from land whereas
farms left to availability of rains will give yield as per success of monsoon in
that region. Hence, farming as industry can develop only if the farms are lying in
region covered with irrigation water supplies. Proper management of supply of

© The Author(s) 2023 1


S. K. Ukarande, Irrigation Engineering and Hydraulic Structures,
https://doi.org/10.1007/978-3-031-33552-5_1
2 Irrigation Engineering and Hydraulic Structures

irrigation water will certainly give returns that will recover expenditure incurred on
irrigation works as well as farming system. Thus, scope of irrigation engineering is
as high as any other industry in this country. Recent emphasis is therefore more on
managerial aspects rather than merely on storage and distribution of irrigation water. A
well-managed system will utilize minimum water and will give maximum crop
yield from same area of culturable land.

1.3 STATUS OF IRRIGATION IN INDIA


India is the richest agricultural country as regards its ratio of agricultural land to
geographical land is concerned. This ratio is 45% as against 10% of USSR and
25% of USA. Its total culturable land is 148 million hectares which is third largest
in the world, 1st being USSR with 224 million hectares and 2nd being USA with
193 million hectares.
The amount of annual surface water available in our country is 1800 × 103 MCM
(Million cubic meter) of which utilizable amount is 660 × 103 MCM. Out of this
utilizable amount, we have harnessed only 50% so far; remaining 50% still needs
to be harnessed. One can imagine our progress if we can harness all of our water
resources fully. One of the western economists has made an observation in 1950,
that India is a rich country inhabited by poor people. We have yet to prove that this
observation is wrong.
Pre-British period, India used to practice irrigation on a small scale i.e., with
the help of wells, tanks and bandharas. During British period some major projects
like Upper Ganga Canal, Upper Bari Doab Canal and Krishna and Godavari Delta
systems were taken up and at the time of pre-independence era more works like
Sarda Canal, Krishnarajsagar dam etc. were taken up. After Independence, irrigation
engineering got highest preference at government level and first five “Five Year” plans
were irrigation projects oriented. Bhakhra Nangal, Hirakud, Chambal, Nagarjunsagar
dam, Damodar Valley project, Ukai, Koyna and Idiki Dam projects were some of
the major works, which got completed during first five “five year” plans i.e., from
1951 to 1975. With the completion of major irrigation projects the irrigated area
which was 22.6 mha (million hectares) in 1951 has increased to 68 mha upto 1986,
which is yet 45% of cultivable area of the country.
While on irrigation front we have done noteworthy progress, yet on hydropower
project works our rate of progress is very slow. At the beginning of first five year
plan per capita power consumption in India was 15 kWh per head, the lowest in the
world, which has subsequently increased to 175 kWh per capita during 1982-83,
whereas it is 5300 kWh per capita in USA and 3040 kWh per capita in UK. We have
yet to achieve a lot to come to world figures as regards hydropower development
is concerned.
Introduction 3

1.4 IMPACT OF IRRIGATION ON HUMAN ENVIRONMENT


Unscientific and uncontrolled use of irrigation water may result in ill-effects of
irrigation. This creates a tremendous impact on human environment. The excess
use of irrigation water may lead to creation of waterlogged areas and may breed
mosquitoes. This in turn will pollute human environment. Climatic conditions
may also change. Damp climate may prevail over areas where excessive and
uncontrolled irrigation is being practiced. Excess water results in return of water
to either ground water or to surface water i.e., rivers. The return water may be
containing toxic chemicals left on account of use of pesticides on farming. This
return water may therefore create pollution of river and ground water supplies.
In order that irrigation impact on human environment does not create imbalance
in natural environment, irrigation water should be used scientifically and in a
controlled manner with the provision of proper escape for return water. Suitable
land drainage should also be provided so that water logging does not take place.

1.5 IRRIGATION SYSTEMS


There are two types of irrigation systems in use: 1. Flow irrigation system; 2. Lift
irrigation system.
1. Flow irrigation system covers large areas under irrigation and hence is also
known as major irrigation system. This system consists of storage works
such as dams, spillways, weirs and head works and also distribution works
such as canals, cross drainage works and outlets. It caters need of large
command areas and overall development of irrigated land is very fast.
2. Lift irrigation system consists of well irrigation and area covered is small
compared to flow irrigation and hence is also known as minor irrigation system.
Large irrigation works create problem of land submergence and displacement
of people whereas minor irrigation system is free of this nuisance.

1.6 COMMAND AREA DEVELOPMENT


Irrigation potential on account of major and minor schemes implemented after
independence has been increased from 22 Mha in 1950 to 80 Mha in 1995 and
is likely to reach a target of 110 Mha in ­coming decades. However, available
irrigation potential remains under utilized due to lack of co-ordination between
various agencies existing in a command area. The second irrigation commission
and national commission on agriculture recommended an integrated command area
development programs for ­optimizing benefits from available irrigation potential.
4 Irrigation Engineering and Hydraulic Structures

Command Area Development Authorities (CADA) have been set up in 1973 to


fulfill this need. Main objectives of the program are:
1. Optimization of yield by best crop pattern.
2. Avoiding wastage and misuse of water
3. Increasing area of irrigated land by land development and water managed
program.
4. Providing of field channels for equitable distribution of water to farmer’s
land.
Integrated command area development is the major field of work of CADA.
Different agencies, like, State Department of Irrigation, Agriculture, Animal
Husbandry, Agriculture Finance Corporation and Commercial Banks etc. should
work in co-ordination with CADA.

1.7 DEVELOPMENT OF AN IRRIGATION PROJECT


Irrigation Projects aim at supply of controlled amount of waters to cultivable land in
order to raise its yield and betterment of the command area. These projects consist of
hydraulic structures, which collect and store water from catchment areas and supply
it by means of gravity flow to command area to increase agriculturable outputs.
These projects are considered feasible only when the total amount of estimated
benefits of the projects exceed its total estimated cost. The feasibility of irrigation
project is determined on the basis of preliminary estimates of area of land to be
irrigated, its water requirements, available water supplies, productivity of irrigated
land, and required engineering works. A large irrigation engineering project may take
10-20 years for its completion and the results of its estimated benefits will be felt
there after.
2

Water Requirement
of Crops

2.1 CROPS AND CROP SEASONS


There are three crop seasons in India : Kharif, Rabi and Hot Weather. Besides these
three seasons, there are other two overlapping seasons like eight months season
which overlaps hot weather and kharif and perennial season which overlaps all
three seasons i.e., Kharif, Rabi, and Hot weather.

(i) Kharif season starts around middle of June and ends around middle of
October, having a base period of 120 days and has major crops like rice,
maize, jawar, bajra (millets), groundnuts etc.
(ii) Rabi season starts from middle of October and ends around middle of
February having a base period of 120 days. It has major crops like wheat,
gram, potatoes, tobacco, pulses etc. Rabi crops require less water than Kharif
crops.
(iii) Hot weather season starts from middle of February and ends around middle
of June. It has a base period of 120 days and its major crops are vegetables,
fodder crops, fruits etc.
(iv) Eight months season starts from March and lasts upto October with a base
period of around 200 – 220 days, and its chief crop is cotton.
(v) Perennial season spreads over whole year and has a base period of around
300 – 330 days, its chief crop is sugarcane.

© The Author(s) 2023 5


S. K. Ukarande, Irrigation Engineering and Hydraulic Structures,
https://doi.org/10.1007/978-3-031-33552-5_2
6 Irrigation Engineering and Hydraulic Structures

Table 2.1: Salient Features of Crop Seasons.

Crop Season with Crops Seed Requirement Yield Delta (cm)


base period * (kg/hect.) (kg/hect.)
Kharif Rice 40 4000 200
(15 June to 15 Oct.) Maize 40 3000 20
B = 120 days Jowar 30 3000 20
Bajra 10 3000 20
Groundnut 100 2500 25
Rabi Wheat 100 3000 30
(15 Oct. to 15 Feb.) Gram 30 3000 30
B = 120 days Potato 1500 30000 50
Hot weather
(15 Feb. to 15 June) Fodder Crop _ _ 20
B = 120 days
Eight Month
(March – October) Cotton 15 – 20 200 – 500 50
B = 220 days
Perennial
(Nov. – Oct.) Sugarcane 500 25000 100
B = 300–330 days
* Dates given for beginning and end of irrigation season are on average basis of large variation at
country level, however, base period in days, season-wise, is almost same throughout the country.
Similarly Delta, Yield etc. are on average basis.

2.2 DEFINITIONS OF TERMS USED IN CALCULATION OF


IRRIGATION DEMAND
1. Crop Pattern and Crop Ratio: Crop pattern indicates percentage
distribution of different crops area-wise in a crop season. It depends on soil
characteristics, climatic conditions and local requirement of crop. Command
Area Development Authority (CADA) may fix percentage of Kharif and Rabi
crops known as crop ratio.
Area allocated to kharif crop
Crop ratio =
Area allocated to rabi crop
= 1 : 2 (usually), i.e., kharif area is half the rabi
area

2. Catchment and Command Areas: Catchment area is the area over which
whatever rain occurs, flows by gravity towards a river or a reservoir for
storage and then distribution is carried out for irrigation purposes.
Water Requirement of Crops 7

Command area is the area to which stored water is made available by canal
network for irrigation purposes.
Thus, catchment area indicates source whereas command area indicates
utilization.
3. Kor Period: First watering given to crop after it has attained height of few
cm is called kor watering and specific period over which kor watering must
be applied to crop is kor period. It is 2 to 4 weeks for rice and 3 to 6 weeks
for wheat.
4. Base Period (B in days): Base period of a season is counted in number
of days commencing from pre-ploughing watering to last watering before
harvesting.
5. Crop Rotation: When same crop is repeated season to season, soil may
suffer of fertility and numerous insects will get developed. To check this,
crop rotation is adopted i.e., same crop is not raised again and again on
same land but different crops by rotation are raised such that land does
not loose its fertility and pest control is also achieved. Crop rotation, thus,
helps in increasing crop yield and keeps the land fertile. Following are some
suggested crop rotation:
(i) Wheat – Jowar – Gram (ii) Rice – Gram
(iii) Cotton – Wheat – Gram – Sugarcane.
6. Paleo Irrigation: At the time of seeding for Rabi crops, weather may be
dry and hot and as such soil may be dry. Hence, watering is required prior to
ploughing which is known as paleo irrigation or simply paleo.
7. Duty (D): Duty is defined as irrigating capacity of one cumec of water
supplied throughout the base period of the given crop. If one cumec of water
is supplied through out the base period to irrigate 700 hectares of land for
the crop rice, then duty of water for rice is 700 hectares / cumec. Thus, duty
helps in arriving at total requirement of water for maturity of given crop
over the given area and this helps in fixing canal carrying capacity and also
irrigation demand for fixing reservoir capacity.It is generally expressed in
hectare/cumec.
8. Delta (Δ): Total depth of water required from seeding to harvesting of a
given crop is defined as Delta (Δ) and is expressed in centimeter or meter.
9. Relationship of Duty, Delta and Base Period: If a crop, grown on D
hectares of land is supplied with water continuously over its base period of
8 Irrigation Engineering and Hydraulic Structures

B days at the rate of 1 cumec then total volume of water supplied to the crop
is given by
V = 1 × 60 × 60 × 24 × B m3
= 86400 B m3
and area = D hectares
= D × 104 m2
If total depths of water over this area is Δ m then

86400 B
∆=
D × 104

8.64 B
∴∆ = m
D
Where, B = base period in days
and   D = duty in hectare / cumec.
If Δ is expressed as total depths in cm, then
864 B
∴ ∆= cm
D

10. Outlet Factor: Field channels or water courses supply water to the land to
be irrigated. Field channels receive their discharge through canal outlets.
The duty of water at the outlet is known as outlet factor, and is expressed in
ha/cumec.
11. Capacity Factor: A canal is designed to carry certain maximum discharge,
but it may not carry the discharge at all the time. Ratio of supply discharge
of a canal to its maximum discharge capacity is defined as capacity factor. It
is around 0.8.
12. Time Factor: During watering period, canal must supply water for all days of
watering, but it may so happen that due to certain unavoidable circumstances
it may not work for all these days but may work for less number of days.
Thus, the ratio of actual number of days the canal work during watering
period to the total number of days of watering period is defined as time
factor.
For example, if total number of days for watering period is 20, but the
canal has actually run for 12 days then time factor is (12/20).
13. Overlap Allowance: Crop of one season sometimes overlaps into next season
and in that case some extra water in the overlapped period may be required,
which is known as overlap allowance it is expressed as 5 to 10%.
Water Requirement of Crops 9

14. Water Conveyance Efficiency: It is the ratio of water supplied on field to


the quantity of water diverted to canal from the reservoir.
wf
ηc = × 100
wr

Here
ηc = Water conveyance efficiency
Wf = Water supplied on field
Wr = Water diverted to canal from the reservoir

q SOLVED EXAMPLES
Example 2.1: The base period of rice is 120 days, and duty is 900 Ha/cumec.
Find delta.
Solution:
864 B 864 ´120
D= = =115 cm.
D 900
Example 2.2: A head regulator releases water at the rate of 5 cumecs. If duty at
the field is 100 Ha/cumec and transit losses are 30%, find area of land irrigated.
Solution: With transit loss of 30%, One cumec will irrigate 0.7 (100) hectares of
land; hence, 5 cumec will irrigate 5 × 70 = 350 hectares of land.
Example 2.3: A reservoir with a live storage of 300 MCM is able to irrigate a
land of 40,000 Ha, with 2 fillings each year. The crop season is 120 days. What is
duty?
Solution:
40000 40000 × 120 × 86400
D= = = 691.2 Ha / cumec
 (2 × 300 ×10 ) 6 2 × 300 × 106
 
 (120 × 60 × 60 × 24) 

2.3 QUALITY OF IRRIGATION WATER


Generally, potable water is considered suitable for irrigation purposes. Rain water
which is stored in ­reservoir and made available for irrigation purposes is suitable
and safe if it is free of following impurities:
(i) Sediments in suspension
(ii) Soluble salts
(iii) Higher proportions of sodium ions.
(iv) Presence of toxic elements present in water.
10 Irrigation Engineering and Hydraulic Structures

Table 2.2: Standards for Irrigation Water


Class of Water Electrical Total Dissolved Sodium % Baron (ppm)
and Suitability Conductivity in Salts in ppm of Cations
Micro MHO / cm
Class I
Suitable for 0 – 1000 0 – 700 0 – 60 0 – 0.5
Irrigation
Class II
Suitable for 1000 – 3000 700 – 2000 60 – 75 0.5 – 2.0
Permeable soils
Class III
Not suitable for 3000 and more 2000 and higher 75 and higher Over 2.0
irrigation

Well water having high contents of salinity may not be useful for irrigation
purposes. River waters are generally free of salinity problems. The following
classification given in Table 2.2 of irrigation water as published by USBR may be
useful to judge the quality of irrigation water.

2.4 SOIL WATER RELATIONSHIP


Irrigation water supplied to land is held in soil mass in three different forms:

(i) Gravitational Water (ii) Capillary Water

(iii) Hygroscopic Water

(i) Gravitational water drains out freely under the influence of gravity and is not
useful for plant growth.
(ii) Water content retained in the soil after the gravitational water has drained off
is known as capillary water, which is held in the soil by surface tension. Plant
roots absorb this water gradually and thus it is capillary water in root zone
of plant that is responsible for plant growth. Upper limit of capillary water
is known as field capacity. Limit upto which capillary water is available for
plant growth is known as permanent wilting point (PWP). After wilting point
has reached water is held by adhesive forces of soil particles and hence not
available to plant. This water in soil mass is known as hygroscopic water.
Weight of moisture content of soilsample
Field Capacity =
Weight of dry sample
Water Requirement of Crops 11

..........................................
.......................................... GL
..........................................
..........................................
..........................................
..........................................
..........................................
..........................................
..........................................
..........................................
..........................................
..........................................
..........................................
..........................................
..........................................
.......................................... Gravitational
..........................................
..........................................
..........................................
..........................................
..........................................
.......................................... water
..........................................
..........................................
..........................................
..........................................
..........................................
..........................................
..........................................
..........................................
..........................................
..........................................
Field capacity

Water readily
available to Capillary water
plant

Wilting point

Water not
available to Hygroscopic water
plant

Fig. 2.1: Soil water relationship


Let d = depth of root zone
ws = weight density of soil
w = weight density of water
S = specific gravity of soil = ws / w
for unit area of soil, weight of soil = ws × 1 × d
weight of water held by soil = w × 1 × depth of moisture
w × 1× depth of water held bysoil
FC = Field Capacity =
w s ×1× d
Depth of water held by soil of unit area of field capacity,
w
= s d (FC) = S d (FC)
w
Similarly, depth of water held by soil
at permanent wilting point (PWP) = S d (PWP)
Depth of available water for plant growth = S × d × (FC – PWP)

q SOLVED EXAMPLE
Example 2.4: Root zone of a soil has FC of 30 % and wilting point 12 %. What
is the depth of moisture in root zone? How much water is available if root zone
depth is 1.0 m, specific gravity of soil is 1.4?
Solution:
Depth of moisture = S d (FC)
= 1.4 × 1 × (0.30 / 100) = 0.42 m.  
Depth of available water = S d [FC – PWP]
= 1.4 × 1 × [0.30 – 0 .12] = 0.252 m  
12 Irrigation Engineering and Hydraulic Structures

2.5 SOIL CHARACTERISTICS FROM IRRIGATION


CONSIDERATIONS
Soil consists of finely divided organic and mineral matters and they are classified as:
1. Alluvial Soil: This is the largest and most important type of soil of India.
It consists of alluvials derived from deposition caused by Indus, Ganga and
Brahmaputra river systems. Such soils are very fertile and if provided with
irrigation facilities, their production is remarkably very high. For example,
after construction of Bhakra – Nangal Project, Punjab and Haryana and
North Rajasthan have attained highest agricultural yield and these areas
are known for green revolution that they have undergone due to irrigation
facilities, over post independence era.
2. Black Soils: This is a typical soil derived form Deccan Trap and is known as
“black cotton soil”. Maharashtra, M.P., Andhra Pradesh, and south Gujarat
region possess this type of soil. These soils are suitable for cultivation of
rice, sugarcane and cotton. It swells when wet and cracks when dry and has
poor drainage property.
3. Red Soils: They are formed due to weathering of crystalline rocks. Such soils
are found in Tamil Nadu, Karnataka, Goa, M.P., Odisha, and Bihar. It is also
found in West Bengal and UP. Its natural fertility is quite high, as can be seen
from large forest areas that have developed in such soil regions.
4. Laterite Soils: Large regions of southern states like Tamil Nadu, Kerala,
Karnataka possess these soils. It is good for rice and coconut cultivation.
5. Soil Characteristics: Irrigation practices are generally influenced by soil
characteristics, such as physical properties, chemical properties and soil
water relationship.
6. Physical Properties: Permeability of soil is as important as adequate supply
of nutrients and water. Permeability of soils refers to movement of water
through soil and hence it is dependent on porosity. Porosity is dependent
on soil texture and soil structure. Soil texture is decided by its particle size,
for example clayey soils have clay particles, (smaller in size than 0.002
mm), in abundance and sandy soils have sand (0.05 to 1.0 mm) particles
predominating over other particles. Loam soils possess equal amount of
sand, silt and clay particles. Texture of soil affects flow of water, aeration of
soil and rate of chemical transformation.
7. Soil Structure: Soil structure is decided by pore space between soil particles.
Fine textured soils offer favorable soil structure permitting retention of water,
proper movement of air and penetration of roots, which is essential for plant
growth. Porosity of soil is a good measure of its structure. Porosity of soils is
Water Requirement of Crops 13

the ratio of volume of voids present in soil sample divided by total volume of
soil sample i.e.
V void G
Porosity, n = =1 − b ,
V sample Gs

Where, Gb = Bulk density of soil and


Gs = Relative density of soil

q SOLVED EXAMPLE
Example 2.5: A moist soil sample has volume of 484 cm3 in natural state and
weight of 7.94 N. Dry weight is 7.36 N, specific gravity 2.65. Find porosity, soil
moisture content, degree of saturation, S. and volumetric moisture content.

Solution:
7.36
G b = Bulk density of soil sample = =1.55
484 × 10−6 × 9810
Gb 1.55
Porosity, n = 1 − =1− = 41.5%
Gs 2.65

Weight of moist soil sample − Weight of dry soil sample


Soil moisture =
Weight of dry soil sample
7.94 -7.36
Soil moisture = = 0.0788
7.36
Volumertic moisture = Soil moisture × Bulk density of soil sample
Volumetric moisture = 0.788 x1.55
= 12.21%
Volumetric moisture
Degree of saturation = S =
Porosity
12.2
= = = 29.43%
41.5

2.6 CONSUMPTIVE USE


Water evaporated from leaves and stems of a plant is called transpiration and
that evaporated from adjacent soil around the plant is known as evaporation.
Water requirement of crop thus consists of evapotranspiration or consumptive
use. Consumptive use is thus defined as the amount of water needed to meet
the water loss through evapotranspiration and is measured as depth of water
on irrigated area. Knowledge of consumptive use helps to determine irrigation
requirement of crop, which is the difference between consumptive use and effective
14 Irrigation Engineering and Hydraulic Structures

precipitation. Evapotranspiration, ET, can be correlated to pan evaporation PE by


ET = K (PE)
Where K = Crop factor for given period.
PE is available from Pan evaporation data published by meteorological
department. Values of K are 0.5 for sugarcane and 0.2 for rice, wheat, maize etc
Blaney – Criddle Formula gives consumptive use of crop and is used extensively.
Let u = consumptive use of crop in mm then,
u = 25.4 kf
k = empirical consumptive use coefficient.
f = consumptive use factor, given by
p
f= (1.8t + 32)
100
here, p = percentage of day light hours of the year occurring during the crop period
and t = mean temperature in °C for the period.

2.7 FIELD IRRIGATION REQUIREMENT (FIR)


Plant growth can be divided into three stages, (i) Vegetative, (ii) Flowering and
(iii) Fruiting. Consumptive use of plant continuously increases during vegetative stage,
attains peak during flowering stage and ­decreases during fruiting stage. Vegetables
are harvested during vegetative stage and crops like rice, beans, potatoes, bananas
etc. are harvested during fruiting stage.
Not all the rainfall can be stored in root zone of the soil. Part of it goes away as
surface runoff, and a part goes back to atmosphere as evaporation. Thus, effective
precipitation is only that part of ­precipitation, which is available to plants as soil
moistures; and this is obtained by subtracting run off, evaporation and percolation
losses from the total precipitation. If consumptive use exceeds effective precipitation,
and that is the normal case, the difference has to be met by irrigation water. Thus,
FIR, field irrigation requirement, is given by
Det − (D p − Dlosses )
FIR (mm) =
Ea
Where, Det = Depth of evapotranspiration, mm
Dp = Depth of precipitation, mm
Dlosses = Run off, evaporation and percolation into grounds.
Ea = Efficiency of irrigation application.
Average depth added to root zone storage
=
Average depth applied to field
= 60 to 80% depending on type of soil.
Water Requirement of Crops 15

(Dp – Dlosses) = effective rainfall


= 0.8 Dp – 25, if Dp > 75 mm per month
= 0.6 Dp – 10, if Dp < 75 mm per month.

q SOLVED EXAMPLES
Solved Examples for calculation of evapotranspiration loss, and field irrigation
requirement,:
Example 2.6: Determine evapotranspiration loss and field irrigation requirement
for wheat of irrigation efficiency 0.8 and average percentage P of monthly sunshine
hours is 0.7 from following data:

Pe t˚C
p(4.6t + 81.3)
Months (effective Rainfall in cm) (monthly temp) f = (cm)
100
November 2.8 20 12.11

December 3.0 16 10.84

January 3.5 14 10.20

February 2.7 15 10.52

∑ Pe = 12.00 ∑ f = 43.67

Solution: V = evaporation loss = k∑f, (k = 0.8, given)


= 0.8 (43.67) = 35 cm
V − ΣPe 35 − 12
FIR = = = 28.75 cm
η 0.8
Example 2.7: Nine litres of water were required to be added to evaporation
pan, 1.22 m in diameter to make up water level. If recorded rainfall during this
period was 4.4 mm, find e­ vaporation from pan.
9 π
Solution: 9 lit = cum = (1.22) 2 × h
1000 4
∴ h = Depth of water in pan
= 7.7 × 10–3 m = 0.77 cm
4.4
∴ Evaporation from pan = 0.77 +
10
= 1.21 cm
16 Irrigation Engineering and Hydraulic Structures

Example 2.8: Using the data given in the following table, find FIR month-wise
if irrigation efficiency is 60% (i.e., Ea = 0.6)

Given To Find Out

Month K PE (mm) (Dp – DLosses) Det (mm) FIR (mm) =


col. (1) col. (2) (3) mm (4) = K (PE) (5) [(5) – (4)] / Eff.

Nov. 0.2 118 6.0 23.6 29.33

Dec. 0.36 96 16.0 34.5 30.93

Jan 0.75 90 20.0 67.5 79.17

Feb 0.9 105 15.0 94.5 132.50

Mar. 0.8 140 2.0 112.0 183.33

Total FIR = 455.26 mm

Δ = 4.5 m

Solution: Det = K (PE) = 0.2(118) = 23.6


Det − (D p − Dloss )
FIR =
Ea
23.6 − (6) 17.6
= = = 29.33 mm (for Nov.)
0.6 0.6
Similarly for remaining months, calculate FIR and enter in the last column
of the table. Total FIR = 455.26 mm.

2.8 FREQUENCY OF IRRIGATION


Frequency of irrigation is defined as interval of days between two successive
waterings. Irrigation water is to be applied before the soil moisture has depleted
below permissible limit, which is generaly 75% of available moisture. Frequency
of irrigation is then equal to depth of water stored in root zone during each watering
(D) divided by consumptive use of water per day, i.e.
D = depth of water stored in root zone during each watering.
= Sd [0.75 (FC – PWP )]
where,
S = specific gravity of soil
d = effective depth of root zone.
Water Requirement of Crops 17

PWP = permanent wilting point


FC = field capacity
D
Frequency =
Consumptive use per day

q SOLVED EXAMPLES
Example 2.9: After how many days, watering is required in order to ensure
healthy growth of a crop, given the following data:
(i) Field capacity, FC = 29%
(ii) Wilting point, PWP = 11%
(iii) Specific gravity of soil = 1.3
(iv) Effective depth of root zone, d = 700 mm
(v) Daily consumptive use =12 mm
Solution:
D = Depth of water stored in root zone during each waterings
= Sd [ 0.75 (FC – PWP)]
= 1.3 (0.7) [0.75(0.29 – 0.11)]
= 1.3 × 0.7 × 0.75 (0.18)
= 0.122 m
= 122 mm

Daily consumptive use = 12 mm


D in mm.
Watering interval or frequency of irrigation =
Daily consumptive use in mm.
122
= = 10 days (approx)
12

Example 2.10: A crop has consumptive use of water as 2.8 mm/day. Determine
irrigation and depth of water to be applied when the amount of water available in
soil is 25% of maximum depth of available water in root zone, which is 80 mm.
Irrigation efficiency, is 65%.

Solution:
80 (1 − 0.25)
(i ) Frequecy of irrigation = = 21days
2.8
80 (1 − 0.25)
(ii ) Depth of water to be applied = = 92.3mm. Say 93mm.
0.65
18 Irrigation Engineering and Hydraulic Structures

2.9 IRRIGATION METHODS


Technique of applying water on to the farms from source is known as method of
irrigation. Source may be field channel in case of flow irrigation system or a well in
case of lift irrigation system. Methods of applying water may be surface methods or
subsurface methods or sprinkler method. These are given in detail below:
q Surface Methods of Irrigation
In this type, water is allowed to spread on farms either in a controlled manner
or uncontrolled manner. Uncontrolled methods are employed only when
irrigation system is of inundating type that is canal system gets its supply
directly from rivers when they have rise in water level due to floods. If
irrigation water is available from perennial irrigation system i.e., canals get
water form reservoir

Border strip

Field
channel
Border strip

From
distributary

Fig. 2.2: Border strip method of irrigation

or head works, the controlled methods of surface irrigation are adopted. Chief
among this type are:
1. Border Strip Method 3. Check Basin Method
2. Furrow Method 4. Contour Method
5. Sprinkler Method 6. Drip Irrigation Method
1. Border Strip Method: In this method farm is divided into number of strips
of size (3 × 100) to (20 × 400) sqm. Slope of the strip should be between
0.002 to 0.004. This method is generally suitable to all types of crops but
requires preparation of land for making strips of suitable size. This makes
the method costly.
2. Check Basin Method: In this method entire field is divided into number of
plots by constructing surrounding levees. Water is admitted from farmer’s
water course to these plots turn by turn. Loss of water can be minimised and
irrigation of entire farm can be carried out satisfactorily. Irrigation efficiency
is higher in this method but constant supervision and field work is required.
Water Requirement of Crops 19

3. Furrow Method: In surface irrigation methods such as border strip and check
basin methods, flooding of entire land surface is carried out. As an alternative
to this, number of small field channels be provided in the farms known as
“furrows” or “corrugation”. Water flowing through these furrows will infiltrate
into the land to saturate the soils. Furrow depths vary from 8 cm to 20 cm
depending upon whether soil is pervious or less pervious respectively. Their
lengths may also be kept around 100 to 200 m and slope be kept between 3 to
6 per cent. In this method only 50 % of land surface is wetted directly and so
evaporation losses are minimum but labour requirement is higher. Spacing of
furrows is kept between 60 cm to 90 cm. The method is suitable for row crops
such as maize, sugarcane, potato, ground nut, tobacco etc. sectional view of
furrows is given in following figure.

Fig. 2.3: Furrow method of irrigation


4. Contour Method: In hilly areas having steep land slope, this method is
practiced. Here land is divided into series of terraces which are aligned to
follow different contours at a vertical interval of 40 to 60 cm. At the outer
end of each strip an earthen bund is provided so that irrigation or rain water
does not leave the strip. Each strip can then have its irrigation by furrow or
check basin method.
5. Sprinkler Method: This method started with irrigation of nurseries and
orchards, and became popular in fully developed and developing countries.
PVC flexible pipes are provided with overhead sprinkles which rotate on
account of pressure of water leaving the sprinkler. Water gets sprayed over
surrounding areas and gives the benefit of rainy water to orchards. Amount
of water required is small but at moderately higher pressure. It does not
interfere cultivation. Evaporation losses are high but percolation losses are
minimum. Initial cost is high but labour requirement is minimum.
6. Trickle or Drip Irrigation Method: Trickle or drip irrigation consists of
network of small diameter pipes which deliver water at root zone of plants,
through emitters connected to pipes. Lateral PVC pipe lines, 30 mm in
diameter are provided with emitters at a spacing suitable to crop type and
soil condition. Due to its ability of maintaining constant soil moisture in the
root zone, trickle irrigation can have highest efficiency, as high as 100%
and results in greater crop yield. Fruits and vegetables respond very well
to trickle irrigation. This method is not suitable for closely planted crops
20 Irrigation Engineering and Hydraulic Structures

like wheat and cereal grains. Clogging of emitters is a major problem of the
system.

q SOLVED EXAMPLES
Example 2.11: Following data pertain to wheat growing area :
Field capacity, FC = 25%
Permanent Wilting Point, PWP = 9%
Root zone depth = 2m
Soil density = 1400 kg/m3
Effective Precipitation = 36 mm
Daily consumptive use = 16 mm
Find frequency of irrigation and FIR, if irrigation efficiency is = 90%

Solution:
Available Moisture = (FC – PWP) = 25 – 9.0 = 16%
Readity available moisture = 0.75 (16) = 12%
D = Depth of water stored in root zone
Sd = 0.75 ( FC – PWP ) 
= 1.4 ( 2 )( 0.12 ) = 0.336 m = 336 mm
336
Frequency of irrigation = = 21 days
16
Det − (effective rain fall) 336 − 36 300
FIR = = = = 333mm
Irrigation efficiency 0.90 0.9

Example 2.12: Find out capacity of a reservoir for culturable command area of
50,000 hectare for following data:

Crop B (days) D = Duty at field (Hect / cumec) Intensity of Irrigation

Wheat 120 1800 20%

Rice 120 800 40%

Sugarcane 360 1700 20%

Cotton 180 1400 20%

Assume canal loss as 20% and reservoir loss as 10%


Water Requirement of Crops 21

Solution:
Volume of water required crop wise:
(i) Wheat,
Volume of Water
Volume required
of Water = A ×∆
required =A×D
 8.64B 
= 0.2(50, 000)  
 D 
 8.64 × 120 
=10, 000   = 10, 000 × 0.576 = 5760 Ha-m
 1800 
(ii) Rice :
Volume of water required = A × ∆
 8.64 × 120 
= 0.4(50, 000)   = 25920 Ha -m
 800 
(iii) Sugarcane :
Volume of water required = A ×∆
 8.64 × 360 
= 0.2(50, 000)   =18296 Ha-m
 1700 
(iv) Cotton :
Volume of water required = A × ∆
 8.64 × 180 
= 0.2(50, 000)  
 1400 
= 1,1108 Ha-m
Total volume of water required by corps = 5760 + 25920 + 18296 +
11108 = 61084 Ha-m
61084
Required capacity of reservior =
0.8 × 0.9
= 84838 Ha-m

Example 2.13: A canal is required to cater the need of irrigating following crop
pattern:

Crop B (Days) Area (H) Duty (Ha/cumec)


Sugarcane 320 900 600
Wheat 120 800 1600
Bajra 120 600 2000
Vegetables 120 320 600
Cotton 180 700 1400
22 Irrigation Engineering and Hydraulic Structures

Taking time factor of canal as 12/20 and capacity factor of canal as 0.8, find
design discharge of the canal.

Solution:

900   
Q for Sugarcane = = 1.5 cumec (Perennial)
600

800
Q for Wheat = = 0.5 cumec   (Rabi)
1600

600
Q for Bajara = = 0.3 cumec   (Kharif)
2000

320
Q for Vegetables = = 0.533 cumec   (Hot Weather)
600

700
Q for Cotton = = 0.5 cumec   (Eight Month)
1400
Since sugarcane is perennial crop, it will require water during Rabi, Kharif
and Hot weather and cotton is eight month crop and so its water is required in hot-
weather and Kharif

Thus Q for Rabi = 0.5 + 1.5 = 2.0 cumec


Q for Kharif = 0.3 + 1.5 + 0.5 = 2.3 cumec

Q for HW = 0.533 + 1.5 + 0.5 = 2.533 cumec


Thus, Q for canal is 2.533 that of hot weather, highest of any three season.
Time factor of a canal is the ratio of number of days canal has actually run to
number of days it was required to be run. Hence,

Q canal = 2.533 × 20 / 12 = 4.22 cumec


4.22 4.22
Design Q canal = = = 5.27 cumec
Capacity Factor 0.8

Example 2.14: A canal has Gross Command Area (GCA) of 25,000 Ha, of which
80% is culturable. Intensity of irrigation for Rabi is 40%, and 20% for Rice. If Kor
period for Rabi is 4 weeks and 2.5 weeks for rice, find outlet discharge. Outlet
duty for Rabi is 1600 Ha/cumec and for Rice 800 Ha/cumec. Calculate delta for
each case.
Water Requirement of Crops 23

Solution:
GCA = 25,000 Ha
CCA = 0.8 × 25000 = 20,000 Ha
40
Area, A under Rabi = (20,000) = 8000 Ha
100
8000
∴ Outlet discharge = = 5.0 cumec
1600
20
Area under Rice = (20,000) = 4000 Ha
100
4000
∴ Outlet discharge = = 5cumec
800
8.64 B 8.64(2.5 × 7)
∆ for rice = = = 0.189 m.
D 800
8.64 (4 × 7)
∆ for Rabi = = 0.15 m.
1600

Example 2.15: Determine the reservoir capacity and main canal discharging
capacity for following data:
(i) Culturable Command Area, CCA = 50000 Ha.
(ii) Canal losses 10%, Reservoir losses 10%.
(iii) Time Factor – 0.7, capacity factor – 0.7
(iv) Details of crop :

Crop Base period B in Duty D in Ha/ Intensity of


days cumec Irrigation –(I)
Wheat 120 1800 20%
Cotton 180 1500 20%
Rice 120 700 20%
Sugarcane 320 1700 10%
Vegetables 120 700 10%

Solution:

Sl. Area under B Volume of water


Delta(m) ∆ = × 8.64
No. Crop Season Irrigation(Ha) D (ha.m) = A(∆)
(i) Wheat (Rabi) 50,000 × 0.2 = 120 5,760
10,000 = × (8.64) = 0.576 m
1800
(ii) Cotton (E.M.) 10,000 180 10,360
(i.e., HW + ∆= (8.64) = 1.036 m
1500
Kharif)
24 Irrigation Engineering and Hydraulic Structures

Sl. Area under B Volume of water


Delta(m) ∆ = × 8.64
No. Crop Season Irrigation(Ha) D (ha.m) = A(∆)
(iii) Rice (Kharif) 10,000 120 14,800
∆= (8.64) = 1.48 m
700
(iv) Sugarcane 5,000 320 8,131
(Perennial) ∆= (8.64) =  1.62  m
1700
(Rabi + HW +
Kh)
(v) Vegetable (HW) 5,000 120 7,405
∆= (8.64) = 1.48 m
700
Total water Requirement = 46,456 Ha. m
46456
Reservoir capacity = (as canal and reservoir losses each = 10%)
0.8
∴ Reservoir capacity = 58070 Ha-m
Area
Q required in canal = + due to perennial drop
D
10, 000 5000
Rabi : Q = + = 8.49 cumec (Due to wheat and sugarcane)
1800 1700
10000 5000 10000
Kharif : Q = =+ + = 23.88 cumec (Due to cotton, rice
700 1700 1500
and sugarcane)

5000
HW : Q = + 2.94 + 6.66 = 16.74 cumec (Due to cotton, sugarcane and
700
vegetables)
∴ Q is highest in kharif = 14.28 + 2.94 + 6.66 = 23.88 cumec
23.88
∴ Canal Capacity = = 48.73 cumec
0.7 × 0.7

EXERCISES
1. Define: Duty, Delta, Base Period and obtain relationship between them.
2. What is the effect of transit losses on duty? Will duty of rice be same for field
and canal head work? Why?
3. Define: Paleo irrigation, crop ratio, kor period, crop – rotation, Field capacity,
PWP and Consumptive use.
Water Requirement of Crops 25

4. Find frequency of irrigation requirement for a crop having following data: -


Root zone depth 100 cm.
FC = 22%, PWP = 12%, specific gravity of soil = 1.5, Consumptive use =
12.5 mm / day.
(Ans: 4 days)
5. A field has FC = 27% and PWP = 13%. If wheat is required to be grown on
this field, find the storage capacity in 80 cm. depth of soil if dry density of
soil is 1.5 gm/cc. If irrigation water is to be supplied when soil moisture falls
to 18%, find FIR for wheat if irrigation efficiency is 80%.
(Ans: 16.8 cm, FIR =13.5 cm)
3

Hydrology

3.1 SCOPE OF HYDROLOGY


Hydrology is a science which deals with rainfall, run-off and floods occurring in
nature as a result of hydrologic cycle. One-fourth of earth’s surface is land and rest
is ocean. Evaporation from oceans, lakes and ponds generate cloud formations,
which are transported landward by wind and its condensation results in rains. Rains
produce rivers and tributaries which again meet sea and cycle gets repeated. Each
river and tributary has its own catchment area over which rain occurs and its resulting
surface run-off forms river and tributary. Study of catchment characteristics helps
in knowing quantum of surface run-off and its concentrated effects like flood. Thus,
scope of hydrology can be defined as:

(i) Water yield from a catchment which enables engineers to design storage
dams, water supply schemes and hydropower projects.
(ii) Floods, their frequency and intensity, which enable civil engineers to design
spillways, bridges, culverts etc.

3.2 RAINFALL
Rainfall occurring in moderate quantities over catchment areas forms surface-run-
off which feed rivers and nalas, but rainfall occurring with great intensity, which is
defined as storm, results in formation of floods in rivers. Study of surface run-off
helps in getting reliable quantities of water available in a year whereas study of
floods helps in finding out suitable methods for its safe disposal, so that hydraulic
­structures such as dams, spillways, bridges etc. do not suffer any damage and loss
of life and property can be prevented. Thus, most valuable contribution of science
of hydrology is to know the reliable quantum of water supply for irrigation and
power projects and intensity of floods and their frequencies for safe and suitable
design of storage works.

© The Author(s) 2023 26


S. K. Ukarande, Irrigation Engineering and Hydraulic Structures,
https://doi.org/10.1007/978-3-031-33552-5_3
Hydrology 27

3.2.1 Types of Rainfall


Rainfall or Precipitation can be of four different types as described below:

1. Convectional Rainfall: Air in contact with warm earth surfaces in summer


rises higher up into the sky and gets cooled. If it carries vapour with it in
sufficient quantities, then its cooling process results into rainfall. This type
of rainfall is convectional or thermal precipitation.
2. Frontal Precipitation: When two air masses of different temperatures and
densities collide, resulting rainfall is known as frontal precipitation.
3. Orographic Precipitation: If moist air masses are lifted up due to barriers
like mountain ranges then resulting rainfall is known as orographic
precipitation.
4. Cyclonic Precipitation: On account of pressure difference in atmospheric
air created by unequal heating of earth surface, moist air gets lifted up into a
low pressure belt resulting in cyclonic precipitation.
There are two types of cyclonic precipitation (i) tropical cyclone, 300 to
1500 km in diameter and causing heavy precipitation along with high wind
and (ii) extra tropical cyclonic precipitation of diameter upto 3000 km with
wide spread frontal type precipitation.

3.2.2 Measurement of Rainfall


Rainfall measurement is carried out with the help of network of rain gage station,
spread out over given catchment area. In plain area, one rain gage station is provided
to command 500 km² area. For hilly and elevated area one rain gage station is
provided per 130 km² to 260 km². There are two types of rain gages: 1. non-recording
type and 2. recording type.
1. Non-recording Type: Symon’s rain gage (see Fig. 3.1) is a non-recording
type and consists of a funnel of 12.7 cm diameter put into a glass bottle
of 10 cm diameter. This bottle is mounted on a concrete pedestal of
60 cm × 60 cm × 60 cm. The funnel top should be 30 cm higher than the
surrounding ground. If the receiving bottle is full, it measures 1.25 cm of
rainfall. Rain water collected in the bottle is measured by a measuring glass
cylinder. Symon’s rain gage can give measurement of previous 24 hours
rains that has occurred during morning 8.30 am to next morning 8.30 am.
28 Irrigation Engineering and Hydraulic Structures

Fig. 3.1: Symon’s rain gage


2. Recording type Rain gage: Automatic or self-recording type of rain gage
consists of a funnel, catch bucket, pointer and graph drum and is installed
such that funnel top is 75 cm above ground surface (see Fig. 3.2).

Fig. 3.2: Self-recording type rain gage

Average Annual Rainfall (AAR)


Average Annual Rainfall is the mean of rainfall observed over past 35 years. If
average annual rainfall is less than 40 cm, the region has arid climate, if AAR is
greater than 40 cm, but less than 75 cm, it is semi-arid climate and if AAR is greater
than 75 cm, it is humid climate.
Hydrology 29

Estimate of missing Data of Rainfall record:


It is generally worked out by simple proportion method, illustrated by following
example.

q SOLVED EXAMPLES

Example 3.1 Rain gage station O was in operation for past few weeks, during
which a storm occurred. The storm rain recorded at surrounding stations P, Q, R,
was 9, 6, 10 cm respectively. If AAR values for the station are 81, 90, 70 and 90
cm respectively, find storm rainfall at O.
Solution: By Simple Proportion
Let P0 is storm rainfall at station P.

9 6 10 P
= = = o
81 90 70 90
19 6 10 
P0 =  × 90 + × 90 + × 90 
3  81 90 70 
1 29
= [10 + 6 + 13] = = 9.66 cm.
3 3
For large area (greater than 50 Km²), mean rainfall is worked out by any of following
three methods:
1. Arithmetic Mean Method
2. Thiessen Polygon Method
3. Isohyetal Method
They are given in detail with illustrative examples below:
1. Arithmetic Mean Method:
∑ Pi
Pav =
n
where, ΣPi = sum of rainfall at individual station
n = number of rain gage station.
This method is good if rain gage stations are uniformly distributed over the area.
2. Thiessen Polygon Method: If rain gage stations are not distributed
uniformly over the given area then the method used is Thiessen Polygon
Method. It is also known as weighted area method.
∑ Ai Pi
Pav =
∑ Ai
30 Irrigation Engineering and Hydraulic Structures

ΣAi = total area of basin with i no. of rain gage stations.


ΣAiPi = sum of product of area covered by rain gage station and rainfall P
measured at that station. The area is obtained by constructing polygon
as indicated in the illustrative example (1) below:
3. Isohyetal Method: In this method point rainfalls are plotted on a graph paper
and lines are drawn through points representing equal rainfalls. These lines are
called “Isohyetals” such as P0, P1, P2, P3 over a given area of A km2

Fig. 3.3: Isohytes

 P0 + P1   P1 + P2 
Pav =   (Area bet. iso. 0 and1) +  (Area bet. iso. 1, 2 )
 2   2 
 P +P  
+  2 3  (Area bet. iso. 2,3)  / (Total Area A)
 2  
Example 3.2 Figure 3.4 shows a catchment area of 400 sq km with following six
number of rain gage stations with rainfall records:
P1 = 6 cm P4 = 9 cm
P2 = 7 cm P5 = 10 cm
P3 = 8 cm P6 = 11 cm

Note –
1. P1, P2---P6 – Rain gage
stations
2. ID, HE, GA etc
perpendicular bisectors
3. ABCDEA is polygon for
station P6 , A B F G is
polygon
(III) fordstation P2 etc
Fig. 3.4: Theissen polygon method
Hydrology 31

Calculate Average Rainfall by


(i) Arithmetic Mean Method
(ii) Thiessen Polygon Method

Solution: (i) By Arithmetic Mean Method

P1 + P2 + P3 + P4 + P5 + P6 51
Pav = = = 8.5cm.
6 6

(ii) By Thiessen Polygon Method: Join rainfall stations P1, P2, P3 etc. Draw
perpendicular bisector for lines P1P2, P2P3, P3P4, P4P5, P5P1, P1P6, P2P6, P3P6,
P4P6, P5P6. These perpendicular bisectors shall intersect each other forming
of polygon, such as ABCDE and other surrounding polygons. The central
polygon will give area for rain gage station P6. Area of polygon ABFG is for
station P2 etc. The areas are given below for each station. The areas can be
calculated by planimeter or graph paper.

Rainfall Station Polygon Area

11 cm P6 ABCDE 90 sq km A6

7 cm P2 ABFG 120 sq km A2

8 cm P3 BCJF 100 sq km A3

9 cm P4 CDIJ 90 sq km A4

10 cm P5 DEHI 90 sq km A5

11 cm P1 EAGH 130 sq km A1
640 sq km

P1A1 + P2 A 2 + P3 A3 + P4 A 4 + P5 A5 + P6 A 6
Pav =
A1 + A 2 + A3 + A 4 + A5 + A 6
(130 × 11) + (120 × 7) + (100 × 8) + (90 × 9) + (90 × 10) + (90 × 11)
=
640
1430 + 840 + 800 + 810 + 900 + 990 5770
= = = 9.0 cm.
640 640
32 Irrigation Engineering and Hydraulic Structures

Example 3.3 Figure 3.5 shows a basin of 80 sq km area and Isohyetals indicating
rainfall of 17,18, 19 and 21 cm are drawn there of. Areas between Isohyetals are
given below:

17 cm. 18 cm. 19 cm. 21 cm.

* 20.5 cm.

10 sqkm. B
* 18.7 cm.
* 16 cm. 15 sqkm. * 22 cm.
20 sqkm.
20 sqkm.
15 sqkm.

A
* 17 cm.

P2 P2 P3 P4

Fig. 3.5: Isohyetal method


Between cover point A and Isohyetal 17 cm, Area = 20 sq km.
Between P1 and P2, 15 sq km.
Between P2 and P3, 20 sq km.
Between P3 and P4, 15 sq km.
Between P4 and cover point B, 10 sq km.
Find average rainfall by Isohyetal Method.
20 + 15
Area Represented by P1 = = 17.5 sq km.
2
15 + 20
Area Represented by P2 = = 17.5 sq km.
2
20 + 15
Area Represented by P3 = = 17.5 sq km.
2
15 + 10
Area Represented by P4 = = 12.5 sq km.
2
17.5 × (17) + 17.5(18) + 17.5(19) + 12.5(21)
Hence Pav =
80
297.5 + 315 + 332.5 + 262.5 1107.5
= = = 13.8cm.
80 80
Adequacy of rainguage stations:
Let N = number of rain gage station required, then
2
 Cv 
N=  (3.8)
 e 
Hydrology 33

Where e = percentage error in adequacy of number of stations required


σ
Cv =
x
x = mean value of rainfall recorded at given number of stations say n,

(x − x)
2

σ= (3.9)
n −1

Example 3.4 Rainfall recorded at seven stations are given below in cm. Take
permissible error as 10% in estimation of average rainfall, check adequacy of
existing number of stations.
x = 154, 109, 139, 144, 124, 112, 172 cm
Solution:

Station x x–x (x – x)2


1 154 154–136=18 (18)2 = 324
2 109 –27 729
3 139 03 09
4 144 08 64
5 124 –12 144
6 112 –24 576
7 172 36 1296

Σx = 954 ∑ (x – x) 2
= 3038

954
∴x = = 136
7
1
Σ (x − x)2  3038 
∴σ = = = (506) 2 = 22.5
n −1  7 − 1 
σ 22.5
∴ Cv = = = 0.1682
x 136
2
 Cv 
N = no. of station required =  
 e 
2
 0.1682 × 100 
= 
 10 
N = 2.72 = 3.
34 Irrigation Engineering and Hydraulic Structures

Hence, existing station are in excess by 7 – 3 = 4 Nos.


Thus, 4 number of stations are not required.
Double Mass Curve Technique or Test for Consistency of Rainfall Record:
If conditions relevant to the recording of rain gage stations have undergone a
significant change, during this period of record, then inconsistency would result in
rainfall data of that station. This can be checked by double mass curve technique.

Method
A minimum of 5 number of stations having reliable data are chosen as base
stations. The average annual rainfall for these stations is procured for at least
5 to 10 years and its cumulative record is prepared. A similar cumulative record
of rainfall at stations X for which consistency check is required is also prepared.
Then a graph between cumulative rainfall of base station on X axis and cumulative
rainfall of station X on Y axis is drawn such that latest year record comes first and
subsequent years record comes last. If station X has consistent data, this graph will
be a straight line or else it will deviate from the year from which inconsistency in
data has arises. This can be set right by considering the slope of the straight line
if it has deviated and slope of undeviated straight line. This will be clear from
the following Fig. 3.6.

45
Cumulative Annual Rainfall at Station x

50

55
Year of
60
Deviation
a
65 c
70
75
c
80 Correction ratio = a
85 c
90 Corrected value for year 1955 = x55 × a
19

Cumulative Annual Rainfall of 10 station average


3
Σpav of station (in 10 cm units)

Fig. 3.6: Double mass curve

Example 3.5 Test the consistency of rainfall data from following Table for 12
years for station X and 12 surrounding stations using double mass curve technique.
Hydrology 35

S. P at station Pav at Surrounding Σ Pav of surrounding


No. X (cm) Σ Px (cm) station (cm) station (cm)
1 32 32 24.3 24.3
2 40.4 72.4 36.5 60.8
3 45.2 117.6 31.7 92.5
4 33.7 151.3 28.8 121.3
5 28.9 180.2 26.7 148.0
6 33.5 213.7 29.7 177.7
7 50.8 264.5 37.6 215.3
8 29.9 294.4 29.9 245.2
9 26.1 320.5 26.6 271.8
10 23.3 343.8 25.1 296.9
11 29.7 373.5 34.8 331.7
12 18.8 392.3 24.9 356.6

Plot Σ Pav on X-axis


Σ Px on Y-axis

Change in data from Sr. No. 7 will be noted as under :


(i) Slope of original line = 1.25
(ii) Slope of deviated line = 0.90
0.9
∴ Correction =
1.25

Cumulative rainfall at Sr. No. 6 = 213.7


∴ its corrected value
0.9
= 213.7 ×
1.25
= 153.87

2. Weighted average of four station method:

1 1 1 1 
 2 P1 + 2 P2 + 2 P3 + 2 P4 
r r2 r3 r4 
Px =  1
1 1 1 1
+ + +
r12 r22 r32 r42
36 Irrigation Engineering and Hydraulic Structures

Where Px is missing rainfall at station X surrounded by four different station


at radial distances r1, r2, r3, r4 etc.
Example 3.6 Station X failed to report rainfall during a storm. The co-ordinates of
four surrounding station are (10,10), (–9,6), (–12, –10) and (5, –12) km respectively.
Find missing rainfall at X if rainfall at the four surrounding station was 70, 90, 68,
60 mm respectively.
Solution: P1 = 7 cm, P2 = 9 cm, P3 = 6.8 cm, P4 = 6.0 cm
r12 = (10)2 + (10)2 = 200

r22 = (−9)2 + (6)2 = 117

r32 = (−12)2 + (−10)2 = 224

r42 = (5) 2 + (−12)2 = 169

 P1 P2 P3 P4 
 2 + 2 + 2 + 2 
r r2 r3 r4 
∴ Px =  1
1 1 1 1
 2 + 2 + 2 + 2 
 r1 r2 r3 r4 
7 9 6.8 6
+ + +
Px = 200 117 224 169
(5 × 10 −3 ) + (8.5 × 10−3 ) + (4.46 × 10−3 ) + (5.9 × 10−3 )

0.035 + 0.076 + 0.03 + 0.035


=
23.86 × 10−3
0.176
= = 7.37 cm
23.86 × 10−3

3.3 SURFACE RUN–OFF


Rain occurring over a given basin or catchment area will flow over the surface as
per general slope of the catchment area and part there of will penetrate the ground.
Some part of flowing water may get detained because of unevenness of area and
some part may get evaporated and also transpirated. Thus, resulting overflow is
always less than the total rain water that has fallen on the catchment area. This
resulting surface water flows by gravity and forms a stream. Part of the ground water
sometimes comes back to stream. The stream flow is thus result of surface run-off
from the catchment area and hence its measurement gives the magnitude of surface
Hydrology 37

run-off. There are some empirical formulae developed by various researchers to


give measure of surface run-off (R) in cm. They are given below:
1. Inglis formula
(a) R = 0.85 P – 30.5 for ghat areas.
Here R indicates run-off in cm and P is average rainfall in cm for all
formulae mentioned below.
(P −17.8) P
(b) R = for plain area
254

2. Lacey Formula for Indo–Gangetic Plain Area:


P
R=
304.8 F
1+
PS
Here F = Monsoon duration factor = 0.5 to 1.5
S = Catchment factor = 0.25 to 3.5
3. Khosla’s Formula for North India:
T
R =P−
3.74
Here, T = mean annual temperature in °C for the given catchment area.
4. Rihand Formula:
R = P – 1.17 P0.85

3.3.1 Strange Curves


Based on empirical observations, strange developed curves giving values of surface
run-off R in cm for given average annual rainfall P. They are given in the Fig. 3.7.

R = Run-off in cm.
P = Rainfall in cm. Wet catchment

Average catchment
R in cm

Dry catchment

P in cm

Fig. 3.7: Strange runoff curves


38 Irrigation Engineering and Hydraulic Structures

3.3.2 Stanford – Watershed Model


Lansley and Crawford developed a computer model at Stanford University. This
model is known as Stanford Water–shed model and gives hourly stream flow if
hourly precipitation data and evapotranspiration losses are fed as inputs.

3.3.3 Rational Approach


(a) A rational method to obtain yield from given catchment area is based on
finding appropriate catchment coefficient C and then using the rational
relation.
Yield = C × A × P.
Here yield is total water available from catchment for the purpose of storage
reservoir, A is catchment area and P is average annual rainfall of the catchment
area. Value of C depends on catchment characteristics such as type of soil,
general slope of the ground, vegetable and temperature prevailing in the
area. C = 0.3 is general value adopted.
(b) Surface runoff calculations can also be carried out by ascertaining the
losses that may take place before water from catchment appears as river
flow. These losses are due to infiltration into the ground, interception due
to uneven surface of the ground and temporary storages due to depression
in the ground. Losses are also due to evaporation from water masses and
evapotranspiration from ground as well as vegetation in the catchment area.
In order to calculate losses, catchment characteristic should be thoroughly
investigated. These characteristics are :
(i) Shape and size of catchment area
(ii) Type of soil and density of vegetation
(iii) Temperature prevailing in the catchment area.
(iv) Wind velocity prevailing in the catchment area.
(v) Month-wise percentage of sunshine hours available into catchment area.
Thus, if losses can be calculated, hydrological equilibrium can be written as

Rexccss = Run off = P – I – S – E – ET,

where P = mean precipitation as available from rain gage station network in


catchment area.
I = Infiltration loss into the ground
S = Loss due to interception and storage in depressions in catchment area.
E = Loss due to evaporation from water bodies
ET =Loss due to evapotranspiration
Hydrology 39

(c) Losses: Determination of all these losses are discussed in detail as under:
1. Infiltration Loss: Rain water entering into soil is infiltration loss for
calculation of surface run-off where as it is a gain for ground water storage.
Rate of infiltration will depend on type of soil and slope of ground. A
ground with no slope or gentle slope will have more infiltration loss if the
soil is sandy and loose; but if a ground has appreciable slope and also the
soil is either rocky or clayey, then infiltration loss will be less. Initially
when the rain starts ground surface is dry and it itself will absorb water,
but once this is done, water will start entering voids, cracks and cuts and
then initially infiltration rate will be high, but as time passes and as the
ground soil gets saturated, there will be fall in infiltration rate. This has
been studied by Horton who has given following equation:
Horton Equation
f = fc + (fo – fc)e–kt(3.1)
where, f = rate of infiltration at any time t.
fo = initial rate of infiltration
fc = constant rate of infiltration
Equation 3.1 can be rewritten as under:
f – fc = (fo – fc) e–kt
∴ ∫ (f − f c )dt = ∫ (f o − f c )e − kt dt

Now fc = area of shaded portion in the Fig. 3.8


= ∫ (f − f c )dt

= no. of square units if graph is down

Infiltration
rate Shaded Area = fc
Horton’s curve
fO

ft
fc = constant
Rate of infiltration.
t=0 time (hrs)
t

Fig. 3.8: Horton's curve (infiltration capacity curve)


40 Irrigation Engineering and Hydraulic Structures

(f o − f c )
Now, ∫ (f o − f c )e− kt dt =
k
fo - fc
Thus, k =
Fc
As an alternative, k can be decided by taking natural logarithms on both sides of
equation (3.1),
ln (f – fc) = ln (fo – fc) – kt
Equation (3.2) on plotting will give a straight line of the form
y = ax + c
Here y = ln (f – fc)
And ax = –kt and c = ln (fo – fc). Slope of the line will give value of K and
intercept on y-axis will give fc.

q SOLVED EXAMPLE
Example 3.7 A 24 hour storm occurred over a catchment area of 2 sq km and total
rainfall observed was 12 cm. Infiltration curve has fo = 1 cm/hr and fc = 0.3 cm/hr
after 15 hours, with Horton’s k value = 5 per hr.
An evaporation pan installed indicates a fall in level by 0.6 cm during 24 hours.
If pan coefficient is 0.7 find run-off and volume of run-off from the catchment.

24
− kt
Solution: Fp = ∫ [fc + (fo − fc )e ]dt
0
24
= ò [0.3 + (1.0 - 0.3)e-5t ]dt
0
24
é 0.7 ù  0.7   0.7 
= ê 0.3t + ú = 0.3 × 24 − 5×24  − 0 − 0 
êë -5e úû 0
5t  5e   5e 

 0.7 
= 7.2 − + 0.14 = (7.2 – 0.107) + 0.14 = 7.233 cm.
 5(1.3) 

Run-off = P – Fp – E = 12.0 – 7.233 – 0.7(0.6) = 4.347 cm


4.347
Volume of run-off = (2 × 106 ) = 8.694 × 104 = 86940 m3
100
2. Infiltration Indices: If rainfall hyetograph i.e., rain in cm on y axis and
time of duration on x-axis is drawn as shown in Fig. 3.9, then infiltration
curve as obtained by Horton’s equation can be superimposed on it and
infiltration loss and run-off can be obtained as shown in Fig. 3.9.
Hydrology 41

P Run-off
(cm) Horton’s Curve
φ index
fo

fc

1 2 3 4 5 6

Fig. 3.9: Infiltration indices

Here the difficulty will arise if the curve line is above part of hyetograph and
has a variation. To avoid this difficulty, average rate of infiltration is worked out
and is expressed as a straight line on hyetograph (dotted horizontal line).
Thus, ϕ index can be defined as average rainfall intensity above which rainfall
volume is equal to run-off volume.
P −Q −S
Windex = (3.2)
t
Here, P = total precipitation corresponding to time interval t,
Q = run-off
S = interception and storage losses in catchment
Thus
Windex = ϕindex – S

q SOLVED EXAMPLE
Example 3.8 The rate of rainfall for successive interval of 1 hr, of 6 hr storm are
1.5, 3,5, 2.5, 2, 1.0 cm/hr.
The corresponding surface run-off is 6.0 cm. Find ϕ and W indices.
Solution:
Σ(p − φ)t = R e xcess = 6 cm.
∴[(1.5 − φ) + (3.0 − φ) + (5 − φ) + (2.5 − φ) + (2 − φ) + (1 − φ)] × 1 = 6 cm.
∴15 − 6φ = 6
∴φ = 1.5 cm/hr (1st trial)
Ptotal = 1.5 + 3 + 5 + 2.5 + 2 + 1 = 15 cm
P − Q 15 − 6
∴ Windex = = = 1.5 cm/hr
t storm 6
If ϕ = 1.5 as obtained in 1st trial is correct.
42 Irrigation Engineering and Hydraulic Structures

P 5.0
cm/hr 5
φ index
3.0
3 2.5
2
1.5

1 2 3 4 5 6 7

t
hrs

Fig. 3.10: ϕ-Index

Then Rexcess as per shaded area in Fig 3.10 should be equal to 6.0 cm. but it is
(1.5 + 3.5 + 1 + 0.5) = 6.5 cm
∴ As a second trial, take Q = 1.6 cm then (1.4 + 3.4 + 0.9 + 0.4) = 6.1 cm and
hence it can be accepted .
∴ ϕindex = 1.6 cm/hr
Note : thus ϕ index can be obtained by trial and error.
3. Loss on Account of Evaporation and Evapotranspiration: Loss on
account of evaporation can be measured either by using evaporation pans
on the concerned site of the catchment area or by making use of empirical
equations.

(a) Evaporation Pans: It is a cylindrical vessel of about 1.25 m in diameter


and 0.3 m depth and is open at top and is made up of GI sheet or copper
sheet. It is placed about 15 to 20 cm above ground level and water level
inside the pan will be 5 to 10 cm below vessel’s rim. The change in water
level over specified duration of time is noted and loss of volume of water
due to evaporation is worked out, which when multiplied by pan coefficient
will give rate of evaporation at the given side.

Actual evaporation from site


Pan coefficient =
Measured evaporation from Pan
= 0.7, generally
Hydrology 43

There are four varieties of the pans available for measurement of evaporation loss:
1. U.S. class A evaporation Pan: It is a land Pan and is shown in Fig. 3.11 given
below:
121 cm dia.

5 cm GI sheet
30 cm or copper sheet
25 cm depth of
water
15 cm
G.L

Fig. 3.11: US class a evaporation pan


2. IMD Pan: Pan developed by Indian metrological department which is
similar to above.
3. Colorado Sunken Pan: It is 920 mm × 920 mm square in plan and 460 mm
deep, and is buried into the ground such that 50 mm is above ground level and
water level in the pan is kept at ground level.
4. Floating Pan developed by U.S. Geological survey department: It is a
900 mm square and 450 mm deep and kept on drum floats which are kept
floating in reservoir.
(b) Impirical Equations :
1. Dalton’s law of evaporation : E = k(ew – ea) According to Dalton,
evaporation is proportional to difference in vapour pressures of water
mass and surrounding air mass about 2 m above water surface. They are
denoted by ew and ea in the above equation. Higher the temperature and wind
velocity, greater is the evaporation, however if greater is humidity, smaller is
evaporation, this can be inferred from Meyer’s equations given below:
2. Meyer’s Equations:
 V 
E = k m (e w − ea ) 1 + 9  mm / day (3.3)
 16 
Here
Km – is coefficient and is generally 0.36
ew = saturated vapour pressure at water surface in mm of mercury.
ea = saturated vapour pressure of air mass about 2 m above the water surface
= (% of relative humidity of air mass)
V9 = monthly mean wind velocity in km/hr at about 9m above the ground and
can be worked out by :
V9 = V2(9)1/7
where
V2 = Wind velocity in km/hr at 2m above the ground or
Vh = v2(h)1/7 where h is any height in meter above the ground
44 Irrigation Engineering and Hydraulic Structures

q SOLVED EXAMPLE
Example 3.9 A reservoir with a surface area of 300 Ha had following data for
a week :
Water temperature = 22.5°C
Relative humidity = 40%
Ew = 20.44 mm of Mercury
Coefficient km = 0.36 in Meyer’s equation
 V 
E = k m (e w − ea ) 1 + 9  mm / day
 16 
Estimate daily evaporation from the reservoir and volume of evaporation loss
from the reservoir during the week.
V2 = 16 kmph, wind velocity at 2m from ground.
Solution:
ew = 20.44 mm of mercury
ea = (relative humidity %) (ew) = 0.4 (20.44)
V9 = V2 (9)1/2 = 16(9)1/7
V9 = 22 kmph.
km = 0.36
 V 
∴ E = 0.36(e w − ea ) 1 + 9  mm / day
 16 
 22 
= 0.36(20.44)(0.6) 1 +  = 10.5 mm/day
 16 
Evaporation loss for reservoir during the week
10.5
= 7 × (300 × 104 ) × = 0.22 MCM
1000

3. Methods to Reduce Reservoir Evaporation Loss:


(i) Reservoir site should be chosen such that it gives least area of surface
exposed to atmosphere at given height. This is highly impossible.
(ii) By growing tall trees all around reservoir.
(iii) By spraying chemicals on reservoir surface. This is effective.
Cetyl alcohol (C16H33OH) (hexadecanol) be applied on the reservoir surface
and it develops a thin film of 0.015 micros and will not allow water molecules
to pass to atmosphere but wind v­ elocity should be within tolerable limit. 2.2 kg
of the cetyle alcohol is required to spread on 1.0 Ha of reservoir surface area.
Hydrology 45

4. Estimation of Evapotranspiration Loss or Determination of Consumptive


Use of Water:
(a) Blaney Craddle Formula:
U = Σkf , (in cm )
where
p(4.6t + 81.3)
f= cm
100
k = monthly consumptive use coefficient.
f = monthly consumptive use factor
t = mean monthly temperature, ˚C
p = monthly percentage of sunshine hours
U − ΣPe
then, FIR = field Irrigation requirement =
η
η = water application (irrigation) efficiency
Pe = effective rainfall for given month.
(b) Penman’s Equation:
Evapotranspiration (ET) loss can also be worked out by making use
of Penmen’s equation, which gives potential evapotranspiration (PET) as
under:
AH n + E L γ
PET = mm / day.
A+γ
Here,
A = slope of a graph of vapour pressure v/s temperature.
Hn = net radiation in mm of evaporable water in a day.
ew = saturated vapour pressure at given temperature
ea = (% of relative humidity)
γ = Psychromatic constant = 0.49
EL = Evaporation loss
 V 
    = k m (e w − ea ) 1 + 2  mm / day
 16 
V2 = wind velocity in kmph at 2 m from G.L.

q SOLVED EXAMPLE
Example 3.10: Calculate PET for following data:
Sunshine hours – 9
Wind velocity at 2m from G.L. = 16 kmph
ew
A = Slope of curve = 1 mm/ºC
temp
46 Irrigation Engineering and Hydraulic Structures

Hn = 1.99 mm of water per day


Relative humidity = 75%
ew = 16.5 mm of mercury
r = 0.49

Solution: ea = (75%) (ew)


     = 0.75 (16.5) mm of mercury
   EL = as per Meyer’s equation with Km = 0.36
 v 
    = Km (ew – ea) 1 + 2 
 16 
 16 
∴ EL = 0.36 (ew – 0.75 ew) 1 +  = 0.36 (16.5)(0.25) (2) = 2.97 mm/day
 16 
AH n + E L γ (1)(1.99) + 2.97(0.49)
∴ PET = =
A+γ 1 + 0.49
3.45
= = 2.3 mm / day
1.49

3.4 FLOOD FLOW CALCULATION


Flood estimation is carried out by
(i) Using flood Formula developed empirically
(ii) Using unit hydrographic method.

3.4.1 Empirical Formulae for Flood Calculation


1. Dicken’s Formula:
Q = C A3/4 m3/sec
where,
C = catchment constant varying from 11 to 25
A = catchment area in sq km
Q is in cumec and gives flood discharge in cumec
This formula is used in North and Central India.
2. Inglis Formula:
124 A
Q= cumec
A + 10.4
A = catchment area in sq km.
This formula is used in Madhya Pradesh, Maharashtra and Gujarat.
Hydrology 47

3. Ryve’s Formula for South India:


Q = CA 2/3 cumec
where, C = 6.8 for areas within a distance of 80 km from Coast.
= 8.3 for areas between 80 to 24 km from coast.
= 10 for areas near hills.
and A is in sq km.
4. Fuller’s Formula:
This is developed in USA, however it has its importance as it considers
concepts of probability of flood and its return period.
Qav = CA0.8 , here C = 0.18 to 1.3
QT = Qav ( 1 + 0.8 loge T )
QT = flood occurring once in T years.
And a is in sq km and Q is in cumec.

3.4.2 Unit Hydrograph Method


In 1932, L.K. Sherman developed unit hydrograph method and is found extremely
useful for flood estimation. It not only gives flood peak value but also gives entire
flood hydrograph corresponding to anticipated storm. Unit hydrograph method is
much more rational than empirical flood formulae.
Unit Hydrograph, here after referred as UH, can be defined as the direct run-off
hydrograph corresponding to unit effective precipitation, i.e., 1 cm rainfall excess,
in a specific time duration, occurring uniformly over the given catchment.
Three important conditions are implied in above definition of UH:
(i) Unit Effective Precipitation, i.e., one cm of rainfall excess, i.e., net rainfall
of 1 cm available as run-off.
(ii) Time duration is to be specified for each storm. i.e., UH for 2 hr means
hydrograph resulting from 1 cm rainfall excess for a storm of 2 hr duration.
It could be 3 hr duration or 4 hr duration or any duration. The unity is for
rainfall excess only.
(iii) UH does not include base flow, thus while preparing UH, base flow must be
deducted from direct run-off hydrograph.

Assumptions:
(i) Base period of a direct run-off hydrograph corresponding to storms of same
duration but different intensities, for a given catchment, is always constant.
(ii) UH can be looked upon as the catchment response to an input of 1 cm rainfall
for some specified time.
48 Irrigation Engineering and Hydraulic Structures

(iii) Principle of linearity and principle of superposition are applied. This is


explained with the help of following illustration:
In the above Fig. 3.12, two storm hydrographs A and B, of same duration
but of different intensities are shown. If the intensity of A is just double that
of B, then resulting hydrograph is also doubled, this is principle of linearity
and is shown in following Fig. 3.13.

Storm A

Storm B
Q

Base period
T

Fig. 3.12: Storm hydrograph

Figure 3.14 shows two storms of same intensity and same duration but with a
time lag of T hr, generating two identical hydrographs. The composite hydrograph
ordinates will be equal to sum of ordinates of the two hydrographs. This is the
principle of superposition.

Construction of UH:
(i) For a given storm of given intensity and given duration of time, plot the
flood hydrograph.
(ii) Separate out base flow from the flood hydrograph.
T
Hyetograph
(Indicates rain
fall excess) Storm A
Ordinate of hydrograph
of storm A = 2 (a1)

Storm B
Q

a1

Base period
T

Fig. 3.13: Storm hydrograph

(iii) Obtain ordinate of direct run-off (DRO) by deducting base flow ordinate
from flood hydrograph i.e., from Q1 subtract Q2.
Hydrology 49

3 hrs. 3 hrs.

3 hrs.
Shift in time T
3 hrs.
Fig. 3.14: Storm hydrograph with time lag

Step I: Find “direct Run-off Depth” by following formula.


Σ o× t
Direct run-off depth = 0.36 cm
A
Σo = sum of direct run-off ordinates
t = time internal in hours between successive ordinates.
A = catchment area in sq km.

Ordinate of direct run-off


Step II: Then, ordinate of UH =
Direct run-off depth

Base Flow Separation:


Locate a point E (Fig. 3.15) on recession limb of the flood hydrograph such that
run-off after point E is only due to base flow. Similarly locate point A on rising
limb. Join AE. Line AE represents base flow separation. To locate point E, a rough
guides line is to find out N, given by N = 0.84 A0.2
Here N is in days. Just directly below point N is point E on the hydrograph
as shown in Fig. 3.15.
tp N
B
N

C
Flood
hydrograph
Q

Q1
3 3
Q2 E
A E
F
Base flow
2 1 T 2
Fig. 3.15: Base flow separation
50 Irrigation Engineering and Hydraulic Structures

DRH

Q Ordinate of DRH

UH
Ordinate of UH

Fig. 3.16: DRH and UH


Alternatively, draw a tangent at A to meet the vertical from B at F. Join FE,
and then AFE represents base flow separation line. In this case separation line falls
from A to F and rises from F to E, as shown in Fig. 3.15.

Limitation of UH:
(i) Applicable only for catchment areas less than 5000 sq km.
(ii) Catchment characteristics have to be uniform and storm should occur with
uniform intensity over entire catchment area, which rarely happens and so
only small catchment areas are suitable for UH.
(iii) Selection of design storm and its time duration has to be done carefully or
else UH will not give correct results.

q SOLVED EXAMPLES
Example 3.11: For a catchment area of 250 sq km, first two columns of the
following Table indicate time in hour and flood flow in cumec. Assuming base
flow of 10 cumec, derive and plot 6 hr. unit hydrograph. Also calculate rainfall
excess for the storm.

Time (hrs.) Base Flow Direct Run-off UH Ordinate (cumec)


Col. No. Q (cumec) (cumec) Ordinate (o) Col (4)
Col.(5) =
(1) (2) (3) (cumec) (4) 8.7
00 10 10 0 0
06 190 10 180 20.6
12 305 10 295 33.9
18 227 10 217 24.9
24 148 10 138 15.8
30 94 10 84 9.6
36 81 10 71 8.1
42 35 10 25 2.8
48 10 10 0 0
Σ o = 1010
Hydrology 51

0.36 ∑ o × t
Direct Run-off Depth =
A
0.36 ×1010 × 6
= = 8.7 cm. = Rexcess
250

Column (5) indicate UH ordinate which can be plotted against time to give
UH for 6 hr.
Example 3.12 A storm gave rainfall excess of 4 cm., 5 cm and 6 cm at successive
4 hr. interval. Work out storm hydrograph for 4 hr. storm from the UH ordinates
given below. Assume constant base flow of 10 cumec.

Time 04 08 12 16 20 24 28 32 36 40 44 48 52
4 hr. UH ordinate 0 150 350 500 470 390 300 210 130 80 40 20 0

Solution:

4 hr. Ordinate
UH Rainfall Base for storm
Time ordinate excess Surface run off from Rexcess flow Hydrogph
hrs. (Cumec) (cm) (cumec) (cumec) (cumec)
Col. Col. (2) Col. (3) (4) = Col. (5) = Col. (6) = Col. Col. (7) Col. (8)*
(1) (2) × 4 (2) × 5 (2) × 6
– – – 4 cm 5 cm 6 cm – –
04 0 4 0 – – 10 10
08 150 5 150 × 4 0 – 10 610
= 600
12 380 6 1400 150 × 5 0 10 2160
= 750
16 500 2000 1750 150 × 6 = 900 10 4660
20 470 1880 2500 2100 10 6490
24 390 1560 2350 3000 10 6920
28 300 1200 1950 2820 10 5980
32 210 810 1500 2340 10 4660
36 130 520 1050 1800 10 3380
40 80 320 650 1260 10 2240
44 40 160 400 780 10 1350
48 20 80 200 480 10 770
52 0 0 100 240 10 350
56 0 120 10 130
60 0 10 10
*Note: Col. ( 8 ) = Col. ( 4 ) + Col. ( 5 ) + Col. ( 6 ) + Col. ( 7 )
Plot Col. (8) against time to give storm hydrograph.
52 Irrigation Engineering and Hydraulic Structures

Example 3.13: Two successive storms of rainfall excess 2.5 cm and 3 cm


occurring at 6 hr interval give a hydro graph of which the ordinates against time
are given below. Derive and plot 6 hr UH, for the catchment of 116.8 sq. km area

Time (hr) 0 3 6 9 12 15 18 21 24 27 30 33
Hydrograph (cumec) 0 25 70 110 126 119 66 38 23 12 6 0

Solution:
Total run off hydrograph is obtained by multiplying the ordinates of shifted
UH by corresponding rainfall excess i.e., 2.5 cm and 3 cm, and adding. Thus, if
U0, U1, U2, U3 etc. are ordinates of UH and Q0, Q1, Q2, Q3 are ordinates of total run
off hydrograph, then

Q3

Run-off

Q2
Q

U3 U3 UH of 6 hr each
U2 with a shift of 6 hr.
Q1 U2

U1 U1
U0
0 3 6 9 12 15 18 21 24 27 30 33
Time (hrs.)

Fig. 3.17: Run-off hydrograph

U0 = 0 Q0 = 0
2.5 U1 = Q1 = 25 ∴ U1 = 10
2.5 U2 + 3 U0 = Q2 = 70 ∴ U2 = 28
2.5 U3 + 3 U1 = Q3 = 110 ∴ U3 = 32
2.5 U4 + 3 U2 = Q4 = 126 ∴ U4 = 17
2.5 U5 + 3 U3 = Q5 = 119 ∴ U5 = 9.7
2.5 U6 + 3 U4 = Q6 = 66 ∴ U6 = 6.0
2.5 U7 + 3 U5 = Q7 = 38 ∴ U7 = 4.0
2.5 U8 + 3 U6 = Q8 = 23 ∴ U8 = 2.0
2.5 U9 + 3 U7 = Q9 = 12 ∴ U9 = 0
ΣU = 108.7
Hydrology 53

Here Q3 = 2.5U3 + 3U1


Where U1 and U3 are UH ordinates.
This is represented in the following table.

Time DRO UH1 × 2.5 UH2 × 3.0


0 0
Q1 03 25 U1 –
Q2 06 70 U2 0
*
Q3 09 110 U3 U1
Q4 12 126 U4 U2
Q5 15 119 U5 U3
Q6 18 66 U6 U4
Q7 21 38 U7 U5
Q8 24 23 U8 U6
Q9 27 12 0 U7
Q10 30 06 0 U8
Q11 33 0 0 0

* DRO = 110 = 2.5 U3 + 3 U1 = 2.5 (32) + 3 (10) = 80 + 30 = 110


Volume of U1, U2 etc. can be plotted with a shift of 6 hr, to give two UH of 6 hr.
duration each.
To check whether UH hydrograph ordinates U1, U2, U3 etc. are correct,
A = 116.8 sq. Km.
t = 3 hrs. since there are two 6 hr UH and so t = 6/2 = 3
0.36 ΣU × t 0.36 ×108.7 × 3
Rexcess = =
A 116.8
= 1.0 cm., which is correct, since UH represents 1 cm Rexcess.
Example 3.14: Stream flows due to 3 successive storms of 3.5, 4.5 and 2.5 cm of
duration 6 hr. each, and given below for a catchment area = 45.4 sq km. Assume
base flow of 10 cumec and average storm loss of 0.25 cm/hr. Find 6 Hr UH ordinate.

Time 0 3 6 9 12 15 18 21 24 27 30 33 36 39
Hydrograph ordinate 10 14 18 32 46 54 58 49 36 25 17 12 11 10

Solution:
Steps: (1) Here storm loss of 0.25 cm/hr will give
P1 = 3.5 – 0.25(6) = 2 cm P2 = 4.5 – 0.25 (6) = 3 cm P3 = 2.5 – 0.25 (6) = 1 cm
54 Irrigation Engineering and Hydraulic Structures

(2) Subtract BF = 10 cumec from given value of hydrograph-ordinates then


calculate ordinate of unit hydrograph (UH) by formula,
Qn = P1Un + P2Un–2 + P3 Un–4
i.e., Q1 = P1U1
∴ 14 – 10 = 4 = 2U1 ∴ U1 = 2
  Q2 = P1U2 + P2U0
∴ 18 – 10 = 8 = 2U2 + 3(0) ∴ U2 = 4
Q3 = P1U3 + P2U3–2
32 – 10 = P1U3 + P2U1 = 2U3+3(2)
∴ 22 = 2U3 + 6 ∴ U3 = 8
Q4 = P1U4 + P2U4–2 + P3U4–4
46 – 10 = 2U4 + 3U2 + (1) U0
∴ 36 = 2U4 + 3(4)
∴ U4 = 12
54 – 10 = 44 = 2U5 + 3U3 + 1U1
∴ 44 = 2U5 + 3(8) + 1(2)
∴ U5 = 9
58 – 10 = 48 = 2U6 + 3U4 + 1U2
∴ 48 = 2U6 + 3(12) + 1(4)
U6 = 4
49 – 10 = 39 = 2U7 + 3(U5) + 1(U3)
= 2U7 + 3(9) + 1(8)
39 = 2U7 + 27 + 8
∴U7 = 2
36 – 10 = 26 = 2U8 + 3U6 + 1U4
26 = 2U8 + 3(4) + 1(12)
∴ U8 = 1
27 – 10 = 17 = 2U9 + 3U7 + 1U5
17 = 2U9 + 3(2) + 1(9) ∴ U9 = 1
12 – 10 = 2 = 2U10 +3 (U9) + 1(U8)
i.e., U10 will be negative and hence U9 can be taken as zero.
Thus; UH-ordinates are
U1 = 2 cm; U2 = 4 cm; U3 = 8 cm; U4 = 12 cm; U5 = 9 cm; U6 = 4 cm; U7 =
2cm; U8 = 1 cm; U9 = 0.
Σ U = 42 cm
Hydrology 55

Check, direct run-off = 0.36(ΣU)t


A
A = catchment area = 45.4 sq km
0.36(42)3
t = given time interval = 3 hrs = = 0.99 cm = 1 cm = Rexcess for UH
45.4

3.5 IUH, S AND SYNTHETIC UNIT HYDROGRAPH


IUH, Instantaneous Unit Hydrograph, IUH was first developed by Clark in 1945.
The development of IUH has removed the difficulty of using a unit hydrograph of
specified time duration. IUH is a unit hydrograph as the result of an instantaneous
application of 1 cm rainfall excess to a given catchment.

S Hydrograph:
S, or Summation Hydrograph is used when unit hydrograph of any other duration
from an available unit hydrograph of given duration is required to be obtained.
S hydrograph corresponds to rainfall excess falling for an infinitely long duration.
The S hydrographs have a typical S type rising limb and then they become
asymptotic to a constant value Q. If an S hydrograph is displaced through
T1 hours the difference in the ordinates between original S hydrograph and displaced
S hydrograph would give ordinate of unit hydrograph of T1 hour duration as indicated
in Fig. 3.18.

S hydrograph

Q S hydrograph with
a lag of T1 hr

Ordinate of UH of
T1 hr duration

Time
T1 hours

Fig. 3.18: S-Hydrograph

Synthetic Unit Hydrograph:


This is developed by Snyder in 1938 for region where no flood data is available.
By making use of empirical relationship for similar watershed flood data values
are worked out and unit hydrograph is prepared, which is known as Synthetic unit
Hydrograph.
56 Irrigation Engineering and Hydraulic Structures

3.6 TO GET UNIT HYDROGRAPH OF DIFFERENT DURATION


FROM THAT OF GIVEN DURATION
If from a given unit hydrograph of N hour duration, another unit hydrograph of
N’ hour duration is ­required to be obtained (here N’ =N) then S-curve technique
is most suitable.
Let NhUH represents N hour Unit hydrograph and N’hUH represents N’ hour
Unit hydrograph. To get N′hUH, let S1 curve be drawn from NhUH and S2 curve be
drawn with same ordinates as S1 but with a lag of N′ hour, as shown in Fig. 3.19.
1
Let two hyetographs as shown in Fig. 3.19 be drawn, each representing
N
cm/hr rain intensity. The difference between two hyetographs represents a storm
1 N′
of duration of N’hour with intensity of cm/ hr; and therefore a rainfall cm.
N N
Let S1 and S2 be two S graphs drawn with a lag of N′ hour. Both S1 and S2 represent
NhUH but with a lag of N′ hour. The difference between ordinate of S1 and S2 has
N′
to represent direct run-off due to rainfall of cm. Hence, if this difference of
ordinate is multiplied N

1 1/N cm/hr

lag 2 1/N cm/hr


of N′ hr

Q
(cumec)

S2

S1
S(t)
Ordinate for N′ hUH
N′
= = [(S(t)) – S(t – N′)]
N

lag of N′ hr.
Time T. hrs

Fig. 3.19: UH of different duration


 N′ 
by   then, it represents 1 cm run-off and so ordinate of NhUH can be calculated
N
N′
from formula = [ (s(t)) − s(t − N′) ]. This will be clear from following numerical
N
problem.
Hydrology 57

q SOLVED EXAMPLES

Example 3.15: 4 hrUH ordinates against time are given in the following Table.
Find out ordinates of 8 hour UH.

Solution:

4hr UH
ordinates S-curve S-curve S-curves Difference 8 hr UH
Time (cumecs) additions ordinates lagged by of colm (4) ordinates colm
(1) (2) (3) (4) 8 hr = (5) & (5) = (6) (6) × 4/8 =(7)
U(t) s(t – 4) s(t) s(t – 8)
0 0 – 0 – 0 0
4 20 0 20 – 20 10
8 80 + 20 → 100 0 100 50
12 130 + 100 =
→ 230 20 210 105
16 150 + 230 = 380 100 280 140
=

20 130 380 510 230 280 140
24 90 510 600 380 220 110
28 52 600 652 510 142 71
32 27 652 679 600 79 40
36 15 679 694 652 42 21
40 5 694 699 679 20 10
44 0 699 699 694 5 2.5
48 – 699 699 699 0 0

Here N = 4 hr and N’ = 8 hrs i.e., N’ > N.


4
Ans: is given in col (7) i.e., ordinates of 8 hr UH = Column (6) ×
8

Note: Col(3) S curve additions are obtained from col(4) as shown by arrows.
Col(4) = col(3) + Col(2) = ordinates of S curve' col(5) is a s curve lagged by 8 hrs
and so it starts with zero against 8 hr time and all subsequent ordinates of S curve
col (4) are then obtained in col(5) as shown by arrows.
58 Irrigation Engineering and Hydraulic Structures

Example 3.16: Derivation of 2 hr UH from 4 hr UH i.e., N' < N. Ordinates of 4 hr


UH are given in the table below, find out ordinates of 2 hr UH by S curve technique.
Solution:

Time S-cuve
(t) hrs Ordinates S-curve S-curve lagged by Difference col Ordinates
(1) of 4hr UH additions ordinates 2 hrs (4)–Col. (5) of 2hr UH

(2) (3) (4) (5) (6) (7)


U(t) s(t–U) s(t) s(t–2) 4
Col.(6) +
2
0 0 0 – 0
2 36 36 0 36 72
4 91 + 0 91 36 55 110
6 93 36 129 91 38 76
8 68 91 159 129 30 60
10 49 129 178 159 19 38
12 34 159 193 178 15 30
14 23 178 201 193 8 16
16 13 193 206 201 5 10
18 6 201 207 206 1 2
20 1.5 206 207.5 207 0.5 1
22 0 207 207 207 0 0
24 0 207 207 207 0 0
4
Col(7) = col(6) × = 2 hrUH ordinates
2
Column showing S-curve additions can be explained by this argument: Let S
curve be derived from N hr UH, then if we take difference of two S-curves lagged
by N hr, the difference graph is unit hydrograph of N hr UH. Hence, ordinate of
N hr UH at any time t is given by
U(t) = s(t) – s(t – N)
∴ s(t) = U(t) + s(t – N)
i.e., ordinates of S graph = ordinates of given UH + ordinates of S graph with
a lag of N hrs.
For example:
Ordinates of S graph at 6hr col(4) = Valule of UH at 6 hr in col(2) + value of s
ordinate at (6 – 4) hr in col(4) i.e., 129 = 93 + 36.
The second term of the RHS of the equation given above i.e., s(t – N) is called
S curve additions entered in col(3).
Hydrology 59

Example 3.17: The 3hr UH of a catchment area = 20 km2 at one hour interval
are given below. Obtain 2hr UH and find value of Qc for the S graph.
0, 0.40, 1.4, 4, 7.70, 10.0, 9.2, 6.6, 4.5, 3.8, 2.7, 2, 1.30, 0.9, 0.4, 0.
Solution:

Ordinate
S-curve Diff. of
3hr UH S-curve S-curve of 2 hr UH
Time lagged Col.(4) –
m3/s additions ordinate Col  (6) ×
2 hrs Col. (5)
3/2
(1) (2) (3) (4) (5) (6) (7)
0 0 0 0 0
1 0.4 0.4 0.4 0.6
2 1.4 1.4 0 1.4 2.1
3 4
+ 0 4.0 0.4 3.6 5.4
+
4 7.7 0.4 8.1 1.4 6.7 10.0
5 10
+ 1.4 11.4 4.0 7.4 11.1
6 9.2 4.0 13.2 8.1 5.1 7.65
7 6.6 8.1 14.7 11.4 3.3 4.95
8 4.5 11.4 15.9 13.2 2.7 4.05
9 3.8 13.2 17.0 14.7 2.3 3.45
10 2.7 14.7 17.4 15.9 1.5 2.25
11 2 15.9 17.9 17.0 0.9 1.35
12 1.30 17.0 18.30 17.4 0.9 1.35
13 0.90 17.4 18.30 17.9 0.4 0.6
14 0.40 17.9 18.30 18.30 0 0
15 0 18.38 18.30 18.30 0 0

2.78A 2.78(20)
Qc = = = 18.53m3 / sec
N 3

Plot time v/s col(7), it gives 2 hr Unit hydrograph.

3.7 RAINFALL/FLOOD FREQUENCY ANALYSIS


Hydrologic events like rainfall, floods can be analysed by applying statical methods
and maximum probable rainfall or flood can be worked out and its probability can
also be calculated.
60 Irrigation Engineering and Hydraulic Structures

Suppose a given sample size is N (number) i.e., N no. of values are given for
either rainfall or flood year wise, and it is required to calculate how many times a
particular flood value will occur or exceeded in an interval of say 100 years, then
arrange the given number of events in descending order, i.e., start with highest value
first, lower value next and so on such that lowest value is the last.
Let m = descending order number
N = sample size i.e., total number of events, then
m
Probability, P =
N +1

1 N +1
or, Recurrence Interval = T = =
P m
i.e., it will repeat after an interval of T yrs.
The probability of occurrence of event r times in n successive years is given by
n!
Pr,n = p r q (n − r)
(n − r)!r!
Here q = 1 – p
Also, probability of event not occurring at all in n successive years is given by
Po, n = qn = (1 – p)n
Probability of events occurring once in n successive years is
n!
P1, n = (p)1(q)(n −1)
(n − 1)!
P1 = 1 – qn = 1 – (1 – p)n = risk factor
Here P1 = probability of event occurring at least once in n successive years.
Van T Chow (1951) has expressed that probability distribution function applicable
to hydrology is

x T = x + k T σ N −1
Here:
xT = value of variate x of random hydrologic series with a return period of T.
Σx
x = mean variate =
   N
N = total number of events
Σ(x − x)2
σN–1 = standard deviation of variate =
N −1
YT − Y n
KT = frequency factor =
sn
Hydrology 61

  T 
YT = reduced vairate for a given T = − log e  log e 
  T − 1  

Y n = reduced mean = 0.533 for N between 20 to 30


= 0.577 for N > 100
Sn = reduced standard deviation = 1.2825.
For calculation of recurrence interval T following formulae are available
N
(i) California method : T =
m
2N
(ii) Allen-Hazen formula : T =
2m − 1
N +1
(iii) Weibull formula : T =
m
N +1
(iv) Gumbel formula : T =
m + c −1
but with c = 1, (Gumbel’s correction factor), It will be same as Hazen’s formula
N +1
i.e., T =
m
N +1
If not mentioned, T = is generally acceptable value.
m

q SOLVED EXAMPLES
Following solved examples will help in understanding above statistical approaches.
Example 3.18: Estimate flood peak with return period of 75, 125 and 900 years
by Gumbel’s method from following 13 years data of flood.
Values: 3300, 4200, 1300, 3250, 2700, 1900, 1950, 4510, 3175, 2640, 2840, 3504,
1775 (cumec) and N = 13
Solution:

N + 1 13 + 1
T= = = 14
Values in descending order (m) m 1
4510 14
4200 7
3504 4.65
3300 3.5
3250 2.8
62 Irrigation Engineering and Hydraulic Structures

N + 1 13 + 1
T= = = 14
Values in descending order (m) m 1
3175 2.3
2840 2.0
2700 1.75
2640 1.55
1950 1.40
1900 1.25
1775 1.16
1300 1.07
ΣQ = 37044

ΣQ 37044
∴Q = = = 2850
N 13
1
 Σ(Q − Q) 2  2
σ N −1 =  
 N − 1 
= 950
Now
YT − Y n
KT = , T = 75, 125 & 900
Sn

  T 
But, YT = − log e  log e 
  T − 1  
For T = 75
y75 = 4.31

4.31 − 0.533
∴ K 75 = = 2.91
1.2825
∴ Q75 = Q + K 75σ N −1
= 2850 + 2.91(950)
   = 5616 cumec
Similarly Q125 = 2850 + (3.5)(950) = 6175
and Q900 = 2850 + (4.896) (950) = 7501
Hydrology 63

Example 3.19: Recorded annual rainfall year-wise starting from 1951 are given.
Find the probability of rainfall equal to or exceeding 11 cm.

Year 1951 1952 1953 1954 1955 1956 1957 1958 1959 1960 1961 1962
Rainfall(cm) 12 7.6 14.5 16 9.5 8 12.5 11.2 8.9 9 7.5 6

Solution: Arrange given rainfall values in descending order and work out return
period T = N + 1 where m is order number of rainfall
m
N + 1 12 + 1
Return period T = = –
S. No. Rainfall (cm) m m (R – R)2
1 16 13 33.4
2 14.5 6.5 18.3
3 12.5 4.33 5.2
4 12 3.25 3.16
5 11.2 2.60 1.0
6 9.5 2.16 0.49
7 9 1.85 1.48
8 8.9 1.625 1.74
9 8 1.44 4.32
10 7.6 1.33 6.86
11 7.6 1.18 7.39
12 6 1.08 17.8
2
ΣR = 122.7 Σ (R – R) = 101.25

122.7 101.15
∴R = = 10.22 cm & σ N −1 = = 3.03
12 12 − 1
R T = R + k T σ N −1
 T   12 
T = 12   ∴ YT = − ln ln  = − ln ln  = 2.44
 T − 1  11 

YT − 0.533 2.44 − 0.533


KT = = = 1.489
1.2825 1.2825

∴ R12 = 10.22 + 1.489 (3.03) = 14.33 cm


1
For R = 11 cm what is P? i.e., = ?
T
64 Irrigation Engineering and Hydraulic Structures

R T = R + K T σ N −1
∴ 11 = 10.22 + KT(3.03)
11 − 10.22
∴ KT = = 0.257
3.03
YT − 0.533
∴ = 0.257 =
1.2825
∴ YT – 0.533 = 0.257 (1.2825)
= 0.329
∴ YT = 0.862
 T 
∴ 0.862 = −  ln ln 
 T −1
T
∴ = 1.525
T −1
∴ T = 1.525T – 1.525
0.525T = 1.525
1.525
∴T =
0.525
T = 2.9
1 1
∴P = = = 0.344
T 2.9

P = 34.4%

Example 3.20: Flood values for a river are


(i) Q = 600 m3/sec with return period of 50 years.
(ii) Q = 700 m3/sec with return period of 100 years.
Using Gumbel’s method estimate flood value for a return period of 150 years and

return period for a flood value of 900 m3/sec. Given : Yn = 0.54, Sn = 1.1284.
Solution:
(i) For return period of 50 years
 50 
YT = Y50 = − ln  ln  = 3.9
 49 
YT − Y n
∴ K T = K 50 =
Yn
3.9 − 0.54
K50 = = 2.97
1.1284
Hydrology 65

(ii) For return period of 100 years


æ100 ÷ö
YT = Y100 = - ln ln çç = 4.6
çè 99 ÷÷ø

4.6 − 0.54
K100 = = 3.59
1.1284

∴ Q50 = Q + K 50 σ

   600 = Q + 2.97σ …(1)


   700 = Q + 3.59σ  …(2)
Subtracting (2) from (1)
(3.59 – 2.97) σ = 700 – 600 = 100
100
∴σ = = 161.2
0.62
Q = 600-2.97 × 161.2

∴ Q = 600 − 479 = 121 cumec

∴ Q150 = Q + K150 σ = 121 + K150 (161.2)

Y150 − Y 0   150  
But, K150 = and, Y150 = − ln ln    = +5.0
Sn   140  
5.054
∴ K150 = = 3.95
1.1284

∴ Q150 = 121 + 3.95 (161.2) = 757.74 m3/sec

Now T = ? for Q = 900 m3/sec


∴ Q = 121 + KT (161.2)
∴ 900 = 121 + KT (161.2)
900 − 121
∴ KT = = 4.83
161.2
YT − YN
4.83 =
Sn

YT − 0.54
∴ 4.83 =
1.1284
∴ YT = 0.54 = 5.453
é æ T öù
\ YT = 5.99 = - ln ê ln çç ÷÷ú
êë çè T - 1÷øúû
66 Irrigation Engineering and Hydraulic Structures

T
∴ = 1.002
T −1
∴ T = (1.002)T – 1.002
∴ 6.002T = 1.002
1.002
T= = 501 years
0.002

Example 3.21: Analysis of a 50 year flood data for a river gave Q = 1200 m3 /s
and Standard deviation = 700 m3/sec.For what discharge would you design a structure
on this river to provide 90% assurance that it will not fail in next 60 years. Given
Yn = 0.54 and Sn = 1.1284.
Solution:
Risk factor = 1 = (1 – p)n
∴ 100 – 90 = 10% i.e., 0.1 = 1 – (1 – p)60
(1 – p)60 = 0.9
∴ (1 – p) = (0.9)1/60 = 0.998
∴ p = 0.002
1
∴T = = 500 yrs
p
Now find Q500 by Gumbel’s method

∴ Q500 = Q + K 500

 500 
YT = Y500 = − ln ln   = 6.213
 499 
Y500 - Yn
K T = K 500 =
Sn

   K 6.213 − 0.54
500 = = 5.027
1.1284
Now, QT = Q + KTσ
∴ Q500 = 1200 + 5.027(700) = 4719.23 m3/sec

Example 3.22: The 50 year-24 hour record of maximum rainfall at a station


is 16 cm. Determine the probability of 24 hour rainfall of magnitude exceeding
16 cm, occurring at the station.
Hydrology 67

(i) once in 10 successive years


(ii) twice in 10 successive years
(iii) at least once in 10 successive years.

Solution: We have
n!
Pr,n = (p) r (q) n − r
(n − r)!r!
1 1
Here q = 1 – p and p = = = 0.02
T 50
(i) P1,10 for case (i)
10! 1
P1,10 = (p )(1 − p)9
9!(1!)
= 10(0.02)(0.98)9 = 0.2(0.83)
= 0.166 i.e., 16.6% Probability

10!
(ii) p 2,10 = (p 2 )(2 − p)8
(8!)(2!)
= 45 (0.02)2 (0.98)8 = 0.0153
= 1.53%
(iii) P1 = 1 – (1 – p)n
= 1 – (0.98)10
= 1 – 0.817
= 18.3%

EXERCISES
1. (a) Define: Orographic, frontal and convectional precipitation.
(b) Explain how you would calculate missing precipitation data?
(c) State permissible area per rain gage station
2. (a) Define: Hyetograph and Isohyet.
(b) A square area of sides AB, BC, CD, and DE of 10 km length each,
has rain gage stations at A, B, C, D and at O, which is ht center of the
diagonal. The rainfall recorded by these station is 10, 12, 16, 22 and 25
cm respectively. Find mean precipitation by:
(i) Arithmetic mean method and
(ii) Thiessen polygon method
68 Irrigation Engineering and Hydraulic Structures

Draw the Thiessen polygons showing areas incharge of each station on a


graph paper.
(Ans: (i) 15 cm , (ii)14.7 cm)
3. A catchment of 2300 sq km. gave following hydrograph for a 6 hr storm.
Derive and plot 6 hr. UH. What will be the rainfall excess due to the storm?
Time (hrs) 00 06 12 18 24 30 36 42 48
Flow (Cumecs) 15 190 305 227 148 94 61 35 15
Base Flow (Cumecs) 15 10 5 7 8 9 11 13 15

Ans.: (1) UH Ordinates 0 19.22 32 23 15 9 5.35 2.35 0


Ans.: (2) Rexcess = ( ΣU)t 0.36 × 997 × 6
= 0.36 = = 9.36 cm.
A 230

4. (a) What is UH? What are its limitations?


(b) In a catchment area, three successive storms of 2.5 cm, 4 cm. and 3.5 cm
occurred at six hour interval. Draw the storm hydrograph when 6 hr. UH
ordinates are given below. Take a constant base flow of 12 cumecs.

Time 06 12 18 24 30 36 42 48 54 60 66 72
(hrs.)
6 hr. UH
ordinates 0 255 570 750 705 585 450 315 195 105 30 0
(Cumecs)
Ans.: 12 650 2457 5059 6769 6919 5944 4646 3334 2156 1189 899
4
Ground Water

4.1 GROUND WATER OCCURRENCE AND RESOURCES


Rain water that infiltrates into the ground get stored either in confined or unconfined
aquifers. Pervious soil layer above an impervious soil stratum forms unconfined
aquifer if it has water with free surface known as water table. If ground water occurs
in a pervious layer, which is sandwitched between two impervious, layers then it is
confined aquifer and water level in piezometric tubes represent imaginary ground
water table. Thus, ground water is found below earth surface either in confined aquifer
or in ­unconfined aquifer. Estimate of present ground water resources in India is of
order of 650 cubic kilometers as against 1880 cubic kilometers of surface water.
GL.

Capillary zone
Water table

Ground water in un-confined aquifer Sand, Gravel etc.

Impervious
layer

Confined aquifer

Impervious
layer

Fig. 4.1: Aquifers

4.2 WELL IRRIGATION


Open dug wells or tube wells penetrating confined or unconfined aquifers form
source of ground water and power is required to lift this water from the ground to
surface so as to be useful for irrigation purposes and in that case it is lift irrigation
or well irrigation.

© The Author(s) 2023 69


S. K. Ukarande, Irrigation Engineering and Hydraulic Structures,
https://doi.org/10.1007/978-3-031-33552-5_4
70 Irrigation Engineering and Hydraulic Structures

4.3 WELL HYDRAULICS


A well is a hydraulic structure, which gives water from aquifers. When water is
pumped from well, water table in case of unconfined aquifer or piezometric surface
in case of confined aquifer gets lowered down and form a cone of depression
around the well. Horizontal distance from the center of well to the point where
cone of depression ends on the water table is known as radius of influence of well
(g0). Difference between initial water table and its lowered level due to pumping
from the well, is known as draw down. Well yield per unit draw down in the well
is known as specific capacity of well.
Ground water moment was first studied by Darcy in 1856, and according to
him velocity of ground water is given by,
V = Ki, (Darcy’s law)
∴   Q = AV = KAi
Here: A = cross sectional area of aquifer through which ground water movement
occurs, at a velocity of V m/sec, under a hydraulic gradient i,
h − h1
∴∴    i = 2
L
where h2 and h1 refers to depth of water table at two points, L meter apart, If c/s
area of aquifer is unity and hydraulic gradient i is also unity then,
Q=k×1×1=K

i.e., permeability of aquifer or hydraulic conductivity K, can be defined as rate of


flow through the porous media under unit hydraulic gradient through unit cross-
sectional area and has dimensions of velocity i.e., L/T. It is expressed either in m/
day or cm/sec. Darcy’s Law is valid for laminar how through porous media having
value of reynold number, R ≤ 1.
However upto R = 10, Darcy’s law is assumed as applicable without appreciable
deviation in the results.
Here Reynold number R is given by

ρVD
R=
μ
where
D = mean diameter of soil particle
V = Velocity of flow through soil particles.
μ = dynamic viscosity of water and
ρ = mass density of water.
Ground Water 71

4.3.1 Steady State Discharge from Unconfined Aquifer


Let p be a point on the cone of depression at radius r and depression head h then
applying Darcy’s Law,
Q = K A i, here K is permeability of aquifer in, m/s
dh
= K (2πrh)
dr
dr
∴Q = 2πkhdh
r
o = discharge from well
GL

r0

r
p Cone of depression

h0
h
hw

Impervious
layer

2 rw = d = diameter of well

Fig. 4.2: Case of unconfined aquifer

r0 ho
 dr 
∴Q= ∫   = 2π k ∫ hdh
 r 
rw h w

π k ( h o2 − h 2w ) 1.36 k (h o2 − h 2w )
∴Q = =
r  r 
ln  o  log  o 
 rw   rw   (4.1)

If s = draw down at the well = ho – hw, then

h o = s + h w Note :here h w = strainer length


ho + h w = s + 2 h w
h o2 − h 2w = (h o + h w ) ( h o − h w ) = (s + 2 h w ) (s)
π ks (s + 2h w ) 1.36 ks (s + 2h w )
∴Q = =
 ro  r 
ln   log  o 
 rw   rw  (4.2)
72 Irrigation Engineering and Hydraulic Structures

Since s = h o − h w and if s is small, then h o ≅ h w


Now h o2 − h 2w = (h o + h w ) (h o − h w )
= 2h o (h o − h w ) = 2h o s,
Putting this valuein equation (4.1), we get
(4.3)
2πKh o s
Q =
r 
ln  o 
 rw 

For an unconfined, the coefficient of transmissivity


For an unconfined is given
aquifer, the by of transmissivity is given
coefficient
T = kh o (4.4)
 T
2πk   s
 k 2πTs
∴Q = =
r  r 
ln  o  ln  o 
 rw   rw 

Co-efficient of transmissivity or transmissibility T is defined as the rate of flow


through vertical strip of aquifer of unit width and extending for full saturated height
under unit hydraulic gradient. If aquifer thickness is b, then,

dh
T = kAi = k ( b ×1) (i), but, i = = unity
dr
∴T = k b for confind aquifer of thickness b
= k h o for unconfind aquifer of thickness h o

Note:
1. Value of ro falls in range of 150 m to 300 m
2. ro can be calculated from Sicherdt expression, ro = 3000 s k

4.3.2 Steady State Discharge from Confined Aquifer


From Darcy’s law,
dh
Q = kAi = k (2πrb)
dr
dr
Q = 2πkb dh
r
ro ho
 dr 
Q ∫   = 2πkb ∫ dh
r 
r  h
w w
Ground Water 73

2πkb(h o − h w )
Q= ..... (Thiem's Equation) (4.5)
r 
ln  o 
 rw 
If s = draw down = ho – hw, and k b = T, then
2π Ts 2.72 T s
Q= =
 ro  r 
ln   log  o 
 rw  r 
 w  (4.6)
If there are two observation wells at radius r1 and r2 giving piezometric water level
at h1 and h2 then
2πkb (h 2 − h1 )
Q= (4.7)
 r2 
ln  
 r1 

Q = discharge from well


GL

r0

Water table
(Imaginary)
r
Cone of depression

h0
Impervious
h layer
hw
Confined aquifer of thickness b
Impervious
layer
2 rw = d = diameter of well

Fig. 4.3: Confined aquifer

Again s 2 = h o − h 2 and s1 = h o − h1
2πkb (s1 − s 2 ) 2πT (s1 − s 2 )  (4.8)
∴Q = =
r  r 
ln  2  ln  2 
 r1   r1 
Here s1 and s2 are draw down in observation well no. 1 and 2 respectively, and
T transmissibility of the aquifer.
74 Irrigation Engineering and Hydraulic Structures

Q = discharge from well


GL

r2 r0

s2
s1 r1

h2 h1 h0
Impervious
layer
hw
b

Impervious
layer
2 rw = d = diameter of well

Fig. 4.4: Case of confined aquifer

q SOLVED EXAMPLES
Example 4.1: A tube well of 30 cm diameter penetrates fully an unconfined
aquifer, calculate its yield from following data: -

Draw down = 3m
Strainer length = 10m
k = 0.05 cm / sec
Radius of influence = 300 m
Solution: As per equation (4.2),
πks (s + 2h w ) 1.36 ks(s + 2h w )
Q= =
r  r 
ln  o  ln  o 
 rw   rw 
.05
here, h w =10 m , s = 3m, k = m/s,
100
15
ro = 300 m and rw = m
100
1
1.36 × .05 × × 3(3 + 2 ×10)
100 1.36 ×15 × 23×10−4
Q= =
 300 ×100  3.3
log  
 15 
=142 ×10−4 = 0.0142 cumec =14.2 lit/s
Ground Water 75

Example 4.2: A tube well penetrates fully a confined aquifer of thickness 30 m


and k = 4.6 × 10-4 m/s. Find well diameter if its yield has to be .04 m3/s, under a
draw down of 4.1 m. Take radius of influence = 250 m.
Solution: Using equation (4.6),
2.72 kbs 2.72 × 4.6 ×10−4 × 30 × 4.1
Q= =
 ro   250 
log   log  
 rw   rw 
 
 250 
∴log   = 3.8474
 rw 
250
∴ = 7037.849
rw
250
∴rw = = 0.0355 m
7037.849
diameter of well = 0.07 m = 7 cm
Example 4.3: A pumping test on a tube well of 20 cm dia. penetrating unconfined
aquifer gave following data:
Q = 200 m3/hr.
RL of water level in well before pumping = 138 m
RL of water level in well at constant pumping = 134 m
RL of impervious strata at well bottom = 108 m.
RL of water in observation well = 137 m
Radial distance of observation well = 40 m
Find (i) Permeability of aquifer
(ii) Radius of influence of the well
Solution:

Q=
( 2 2
1.36 k h 222 − h112 )
 r2 
log  2 
 r11 
20
r11 = =10 cm = 0.1m = rw
w
2
r22 = 40 m
h0 = 138 – 108 = 30 m
h1 = 134 – 108 = 26 m
h2 = 137 – 108 = 29 m
Q = 200 m3/hr
76 Irrigation Engineering and Hydraulic Structures

1.36 k (292 − 262 )


∴200 =
 40 
log  
 0.1 
200 × 2.6
∴k = = 2.3m/hr = 6.4 ×10−4 m = 55 m/day
1.36 (3× 55)
1.36k(h o2 − h 2w )
Also, Q=
r 
log  o 
 rw 
1.36 (2.3) (302 − 262 )
200 =
 r 
log  o 
 0.1 
 r  1.36 × 2.3× 56 × 4
log  o  = = 3.5
 0.1  200
ro
∴ = 3186 ∴ ro = 318.6 m.
0.1

Example 4.4: Constant pumping rate from a well of 30 cm dia. penetrating


unconfined aquifer is 1800 LPM. A test well 30 m away gives draw down of 2 m
and another test well 60 m away gives draw down of 1 m. Depth of water in well
before pumping = 50 m. Find radius of influence and transmissibility.

1.36 k(h 22 − h12 )


Solution: Q=
r 
log  0 
 γ1 

1.36 k (502 − 482 ) 1.36 k (502 − 492 )


∴Q = =
 r0   r 
log   log  0 
 30   60 
 r 
log  0 
(98 × 2)  30 
=
(99 × 1)  r0 
log 
 60 
log (r0 ) − log (30)
1.97 =
log (r0 ) − log (60)
x − 1.47
∴1.97 = ; here x = log ( r0 )
x − 1.77
1.97 (x − 1.77) = x − 1.47
1.97 x − 3.5 = x − 1.47
0.97 x = 2.03
Ground Water 77

1.97 (x – 1.77) = x – 1047


1.97 (x – 3.5 = x – 1.47)
0.97 x = 2.03
2.03
x= = 2.09 = log (r0 )
0.97
∴ r0 =123m
Now, Q = 1800 lpm = 0.03m3 /s.

1.36 k (502 − 492 )


∴0.03 =
 123 
log  
 60 
0.03× 0.31
∴k = = 6.94 ×10−5 m/s
1.36 × 99 ×1
Transmissivity = T = kb = 6.94 ×10−5 × 50 ,since b = 50

∴ T = 3.47 ×10−3 m 2 /s

Example 4.5: A tubewell penetrates confined aquifer fully the thickness of aquifer
is 25 m and K = 40 m/day. If yield from well is 30 LPS under a draw down of 3.5 m,
find radius of well. Radius of influence can be taken as recommended by Sicherdt.
Solution: As per equation (4.6)
2.72kbs
Q=
r 
log  0 
 rw 

Here S = 3.5 m, r0 = 3000s K = 226m


40
where K = 40 m/day = = 4.62 × 10–4 cm/sec, b = 25
24 × 3600
2.72 × 4.62 × 10−4 × 25 × 3.5
Q=
 226 
log  
 rw 
30
Now Q = 30 LPS = = 0.03 m3/sec.
1800
2.72 × 4.62 × 10−4 × 25 × 3.5
∴ 0.03 =
 226 
log  
 rw 
78 Irrigation Engineering and Hydraulic Structures

 226 
∴ log   = 3.685
 rw 

226
∴ = anti log (3.665) = 4623
rw

226
Radius of tube well = rw = = 0.048 m = 48 mm .
4623

Example 4.6: A 20 cm diameter fully penetrating well in an unconfined aquifer


30 m deep was pumped at 2800 lit/sec, and draw downs in two test wells at 50 m
and 110 m from the pumped well are 4 m and 2m respectively. Find R and T.
Solution: Here draw down
S1 = 4 m at r1 = 50 m
and S2 = 2m at r2 = 110 m
and T = KH = Transmissibility
2.72KH(S1 - S2 )
Now, Q= (as per equation 4.9)
log10 (r2 / r1 )

2800 2.72(T)(4 − 2)
=
1000  110 
log10  
 50 

 110 
2.8log  
∴T =  50  = 0.176 m 2 /sec
2.72 × 2
= KH = K(30)

0.176
K=
30

= 5.86 × 10–3 m/sec


Draw down at R (radius of influence) will be zero.
∴ S2 at r2 = R will be zero in above equation

2.72T(S1 )
∴Q =
log(R / r1 )

2.72 × 0.176 × 4
∴ log(R / r1 ) = = 0.683
2.8
Ground Water 79

∴ R/r1 = 4.828
R = 4.828 × 50
R = 241m
Example 4.7: A 30 cm diameter well peretrates 25 m below ground water table.
If steady state discharge from the well, is 90 LPS giving a draw down of 0.53m
at 90 m and 1.11m at 30 m from the main well, find T, R and draw down in the
main well.

Solution: As well penetrates below ground water table, it is a case of unconfined


aquifer

2πKH(S1 − S2 )
∴Q =
r 
ln  2 
 r1 

Here H = 25m, S1 = 1.11m at r1 = 30 m and S2 = 0.53 m at r2 = 90 m and Q = 90 LPS.


90 2πK(25)(1.11 − 0.53)
∴ =
100 ln(90 / 30)
157K(0.58)
0.09 =
1.098
0.09 × 1.098
∴K = = 1.08 × 10−3 m / sec.
157 × 0.58

∴ Transmissibility, T = KH = 1.08 × 10–3 × 25 = 0.027 m2/sec


To find draw down in the well,
Here h1 = 25 – S1= 25 – 1.11 = 23.89
and r1 = 30 m, rw = 0.15
πk(h12 − h 2w )
Q=
r 
ln  1 
 rw 

π(1.08 × 10−3 )(23.892 − h w 2 )


0.09 =
 30 
ln  
 0.15 

0.09 × 5.298
∴ 23.892 − h 2 w = = 0.14 × 103
π × 1.08 × 10−3

∴hw2 = 570 – 140 = 430


∴ hw = 20.73 m
80 Irrigation Engineering and Hydraulic Structures

∴ draw down in well = 25 – 20.73 = 4.27 m


To find R:
πK(H 2 − h w 2 )
Q=
ln(R / rw )

R  π(1.08 × 10−3 )(625 − 430)


∴ ln  =
 rw  0.09
0.661
= = 7.34
0.09

R
\ = 1547
hw
∴ R = 1547 × 0.15 = 232 m

4.4 RECUPERATION TEST


In recuperation test water is pumped from the well so that sufficient depression
head is developed. Then water level is allowed to rise to the level which was in
the well prior to pumping and time taken is noted. Then yield of the well can be
worked out as under:-
Referring to Fig. 4.5
Let h1 = depression head when pumping was stopped
h2 = depression head at time t after pumping was stopped.
Thus water level in well recuperated = h1 – h2 in time interval t.
dh = small recuperated head, generating
volume dv = Adh, in interval of time = dt
If Q is recharge rate into the well at time t, under depression head h, then
Q
GL
Original water level
h2
h
h1 dh
Water level when
pumping stopped
hw b

Cross sectional area at bottom =

Fig. 4.5: Recuperation test


Ground Water 81

dv = Q dt, here Q = C h , (C is constant )


But, dv = –Adh, (–ve since upward movement of water )

A dh
∴ dt = −
C h

Let h = h 2 after time t, then

h
A 2  dh  A  h1   (4.9)
t=− ∫   = + ln  
Ch  h  C  h2 
1

C 2.3 h 
∴ = log  1 
A t  h2 

C
= Specific yield or specific capacity of open well which is defined as
A
volume of water that perco-
lates into the well per unit time under unit depression head, which is expressed as
cubic meter per hour per sq. m. of area of well under unit depression head.

4.4.1 Safe Yield Under a Constant Depression Head


C
Knowing the value   as above, the rate of yield from a well under a constant
A
depression head ‘h’ may be found out:
C
Q = Ch = (A.h)
A

C π
Now put value of   as per eqn (4.9) and given A = d 2
A
  4
where d = diameter of well and h = given depression head

 2.3  h   π 
Q= log  1    d 2  (h)  (4.10)
 t  h 2    4 

4.4.2 Interference of Wells


If two or more wells are located in such a may that their draw down curves intersect
each other they are known as interfering among themselves. Due to this interference
82 Irrigation Engineering and Hydraulic Structures

discharge of each individual well is ­reduced. Discharge from interfering wells can
be calculated by making use of Muskat formulae given below:
1. For confined aquifers of thickness b:
2πKb(H − h w )
Q1 = Q 2 =
 R2 
ln  
 rw D 
where, R = Radius of influence
b = thickness of confined aquiter
             rw = well radius
          D = distance between two wells.
2. For unconfined aquifers:
πK(H 2 − h w 2 )
Q1 = Q 2 =
 R2 
ln  
 rw D 
Note that R > D, as shown in Fig. 4.6.
Q1 Q2

H H

D

R rω

Fig. 4.6: Interference of wells

q SOLVED EXAMPLES
Example 4.8: Two tube wells of 25 cm diameter each are spaced 80 m apart and
penetrate fully a confined aquifer of depth 12 m. Find discharge if only one well
is discharging under a draw down of 2.5 m. what will be percentage decrease in
discharge if both well discharge under draw down of 2.5 m. Take R = 230 m and
k = 60 m/day.
Ground Water 83

Solution: Here S = 2.5 m, b = 12m


25
rw =
= 12.5cm
2
2.72kbs
Q from one well =
log(R / rw )
2.72 × 60 × 12 × 2.5 4896
   = = = 1500 m3/day
 230  3.264
log  
 0.125 
1500
= = 0.01736m3 /sec = 17.36 LPS.
24 × 3600
When both wells are discharging under a draw down of 2.5 m,
2πKb(H − h w )
Q1 = Q 2 =
 R2 
ln  
 rw D 
But, H - hw = S = 2.5 m
2π × 60 × 12 × 2.5 11304
∴ Q1 = Q 2 = = = 1318 m3/day
 (230) 2 8.573
ln 
 0.125 × 80 
 
= 15.2 LPS
17.36 − 15.2
∴ % Increase in Q = = 0.124 = 12.4%
17.36
Example 4.9: A well penetrates an unconfined aquifer of thickness 100 m. If 12
m draw down gives a discharge of 250 LPM, find discharge at draw down of 18 cm.

Solution: Case I: for draw down of 12 m,


hw = 100 – 12 = 88 m.
πK(H 2 − h w 2 )
Q=
2.3log10 (R / rw )

πK
∴ 250 = (1002 − 882 )
2.3log10 (R / rw )

πK 250
∴ =
2.3log10 (R / rw ) (188 × 12)
84 Irrigation Engineering and Hydraulic Structures

Case II: draw down = 18 m, hw : 100 – 18 = 82m

∴Q =
(
πK 1002 − 822 )
2.3log10 (R / rw )
250
= × 182 × 18
188 × 12
= 363 LPM       

4.5 WELL SHROUDING


Shrouding is a layer of course material such as gravels and rubbles packed around
strainer so that fine material of aquifer does not clog the strainer. (See Fig. 4.7)
Shrouded well has greater specific capacity than the one without shrouding.
Shrouding material is introduced between casing pipe and well pipe and then casing
pipe is removed.

GL.

Soil
Well pipe

Impervious
layer

Aquifer

Impervious
layer
Shrouding

Strainer Concrete plug at the bottom

Fig. 4.7: Well shrouding

4.6 WELL DEVELOPMENT


Well development is the process of removing fine material from the aquifer surrounding
the strainers. It increases specific capacity of well, prevents aquifer sand flowing
into the well, and life of well can be increased.
It is carried out by any of the following methods.
(i) By pumping and surging.
(ii) By back washing
Ground Water 85

(iii) By compressed air


(iv) By making use of chemicals.

4.7 COLLECTOR OR RADIAL WELL


Collector or radial well is used when high rate of discharge from a relatively
thinner aquifer is desired. They are normally provided near river bank or in the
river bed by providing horizontal strainers along circumference of well casing
and these strainers penetrate horizontally into the aquifers, causing high rate
of yield. Baroda city water supply system is dependent on such wells driven in
bed of River Mahi.

q SOLVED EXAMPLES
Example 4.10: A 30 cm diameter well fully penetrates an unconfined aquifer.
For an effective length of strainer of 12 m, draw down of 3.7 m, radius of influence
of 280 m, and permeability coefficient of 42 m/day, find yield of the well.

Solution:
1.36 ks (s + 2h w )
Q=
r 
log  o 
 rw 
here, k = 42 m per day = 4.8 × 10−4 m / sec,
s = 3.7 m, h w = 12 m, r0 = 280 m and rw = 15 m
1.36 × 4.8 ×10−4 × 3.7 × (3.7 + 2 (12))
Q=
 280 
log  
 0.15 
24.15 ×10−4 (27.7)
= = 204.5 × 10−4 m3 /s = .02 m3 /s or 20.4 lit/sec
3.27

Example 4.11: A 10 cm dia tube well is working under a draw down of 4 m,


and has 10m long strainer. The thickness of aquifer is 15 m, K = 30 m/day, find
yield. Also Calculate yield for 100% increase in diameter and 50% increase in
draw down.
Solution: From Sichardt’s expression,

ro = 3000 s k
= 3000 (4) 30 / (24 × 3600)
= 223 m.
k = 30 m / day = 3.47 × 10−4 m/s.
86 Irrigation Engineering and Hydraulic Structures

1.36 ks (s + 2h w ) 1.36 × 3.47 ×10−4 (4) × (4 + 2 × 10)


Q= =
r   223 
log  o  log  
r
 w  .05 

              1.36 × 3.47 × 10−4 × 4 × 24


= = 124 × 10−4 m3 /s
3.65

= 0.0124 m3 / s =12.4 lit/sec

2nd case :

New dia = 2 × 10 = 20cm


s =1.5 × 4 = 6m

ro(new) = 3000 (6) (0.018) = 324

1.36 × 3.47 ×10−4 (6) × (6 + 20) 736 × 10−4


Q new = =
 324  3.51
log  
 0.1 

= 209 × 10 −4 = 0.0209 m3 /s or 20.9 LPS

4.8 WELL LOSSES


Actual draw down is slightly higher than the draw down due to logarithmic equation
for yield from the well. The difference is known as well loss. This is due to the fact
that logarithmic equation represents loss due to movement in aquifer only, whereas
the loss that is occurring on account of flow through strainer and turbulent condition
near the well is not accounted in that equation. Hence, actual draw down is higher
and this difference between actual draw down and theoretical draw down is the
well loss (see Fig. 4.8).

Fig. 4.8: Well losses


Ground Water 87

4.9 UNSTEADY GROUNDWATER FLOW TOWARDS WELLS


If cone of depression does not vary with respect to time, it is steady ground water
flow towards wells but, if cone of depression changes with respect to time it is a
case of unsteady ground water flow towards wells (see Fig. 4.9).
Mr. Thies in 1935 developed partial differential equation governing unsteady
flow toward's wells,
(4)s = draw down at radial distance r at time t after pumping started
   Jacob obtained value of s as  (4.10)
2.3Q  2.25Tt 
s= log  2 
4πT  r S 
Q
GL

Water table
s (Piezometric surface)
r
P
Pais a point at radius r at time
t after pumping started
H h
Impervious
strata
Confined aquifer

Impervious
strata
Fig. 4.9: Unsteady flow towards well
Storage coefficient, S : It represents the volume of water released by a column of
confined aquifer of unit cross - sectional area under a unit decrease in piezometric
head & S and T are known as formation constants of an aquifer and play very
important role in unsteady flow through porous media.
Here S is storage coefficient or storativity, T = transmissivity, r is the radius
at which point P on the depression curve is chosen and h is piezometric head.
Solution of the equation as obtained by Thies is

Q e− u Q × W(u)
H−h =s= ∫
4πT u u
du =
4πT
r 2S  (4.11)
(1) here, u = ,
4Tt
(2) h = H for t = 0 i.e. before pumping started.
∞ −u
e
(3) W(u) = ∫ du
u
u
88 Irrigation Engineering and Hydraulic Structures

(4)s = draw down at radial distance r at time t after pumping started


Jacob obtained value of s as
2.3Q  2.25Tt 
   s = log  2 
 r S  (4.12)
4πT

Q   4Tt 0  
s=  ln  2  − 0.5772  (4.13)
4πT   r S  
If s1 and s2 are draw down at time

Q   t 2 
(s 2 − s1 ) = ln     (4.14)
4πT   t1  
If the draw down S is plotted against t on a semilog paper (S on Y-axis, ordinary
scale and t on x-axis log scale) the plot will be a straight line. The slope of this line
enables to determine storage coefficient – S i.e., when S = 0, t = to in the eqn (4.13)

Q   4Tt o  
i.e., 0=  ln  2  − 0.5772 
4πT   r S  

 4Tt 
∴ ln  2 0  = 0.5772
 r S 

4rt o 2.25Tt o
∴S = 2 0.5772
=  (4.15)
r e r2

here r is distance between observation well and pumped well. In using equation
(4.15) value of T is required, which can be obtained as under:
From the graph as plotted above, take any two values s2 and s1 at time t2 and
t1 respectively and then using equation (4.14), for given Q, T can be calculated.
After this, use of equation (4.15) gives storage coefficient S. This is illustrated in
following problem.

q SOLVED EXAMPLES
Example 4.12: A 20 cm well penetrates a confined aquifer fully and is pumped
at constant rate of 1000 LPM. The draw downs at an observation well at 30 m from
pumping well are observed as under:
Time (t) min 1.0 2.5 5 10 20 50 100 500 1000
Draw down S 0.2 0.5 0.8 1.2 1.8 2.5 3.1 4.4 5.0
(m)

Calculate aquifer parameters, S and T.


Ground Water 89

Solution: Figure 4.10 shows the plot of T v/s S which is a straight line for t ≥ 10
min and when this straight line cuts X-axis time t = t0 at s = 0 is noted.
At s = 0, t = to = 2.5 min from graph (see Fig. 4.9)
s1 = 3.1 m at t1 = 100 min
t (min)

10 100 1000
0 log scale

2 Extend this line


3 to cut X axis so

S
/=0
that t = t0 at S
4

Fig. 4.10: Plot of S vs T

s2 = 5.0 m at t2 = 1000 min


Q = 1000 LPM = 0.016 m3/sec.
From Eq. (4.14)

Q t 
s 2 − s1 = ln  2 
4πT  t1 

0.016  1000  0.016(4.6)


∴ 5 − 3.1 = ln  =
4πT  10  4πT

0.016(4.6)
∴T = = 3.08 × 10−3 m 2 / sec
4π(1.9)

2.25T t 0 2.25(3.08 × 10−3 )(2.5 × 60)


= =
r2 (30) 2
= 1.155 × 10–3
∴ Storage coefficient = 0.001155
here r = 30 m = radial distance of observation well from pumped well.
90 Irrigation Engineering and Hydraulic Structures

Example 4.13: The time draw down data from an observation well at 64 m from
pumping well are given below. Find S and T if Q = 1.9 m3/min.

Time (min) 1.5 3 4.5 6 10 20 40 100


S(m) 0.2 0.65 1.05 1.45 2.45 3.75 5.15 6.95

Solution: We know
s1 = 2.45 m at t = 10 min
s2 = 6.95 m at t = 100 min
Q = 1.9 m3/min = 0.0316 m3/sec

Q t 
s 2 − s1 = ln  2 
4πT  t1 

0.0316  100 
6.95 − 2.45 = ln  
4πT  10 
0.0316 × 2.3
∴T = = 1.28 × 10−3
4π × 4.5
2.25T t o
S=
r2

Here to = 2.5 min = 150 sec at s = 0 (from graph see Fig. 4.11), r = 64 m

2.25 × 1.28 × 10−3 × 150 432


∴S = =
64 × 64 64 × 64
S = 1.05 x 10–4
Y

Ordinate
Value 8

S 4
cm
2

x log scale
2.5 10 100 1000
t0 min time (min)
at
S=0

Fig. 4.11: Plot of T vs S


Ground Water 91

Example 4.14: During a recuperation test water level was depressed by 3 m and
it recuperated to 2 m in 90 min. Find discharge for a depression head of 3.3 m in a
6 m diameter well and size of well to yield 15 LPS for a draw down of 2.8 m.
Solution: Here
s2 – s1 = 3 – 2 = 1 m in 90 m.
2.3 s  2.3 3
C= log  2  = log   = 7.5 × 10–5
T  s1  90 × 60 2

π 2 π 2
A = cross-sectional area of well = d = (6) = 9π.
4 4
Q = CAH = (7.5 × 10–5) (9π) (3.3) = 0.007 m3/sec
Case (2): Q = 15 LPS = 0.015 & H = 2.8m.
∴ 0.015 = CAH = (7.5 × 10–5) (A) (2.8)
π
∴ A = 71.4 = d 2
4
∴ d = 9.5 m.

Example 4.15: Estimate Q from a well in a confined aquifer of 30 m depth.


Distance of observation well from main well is 100 m and draw down values are
1.5 m after 4 hrs and 2.0 m after 16 hrs. Take S = 0.0003
Solution: Plot graph of S v/s t and get to = 3.6 min,
which is the value of t at S = 0
2.25T t 0
∴S =
r2
2.25T(3.6 × 60)
0.0003 =
(100) 2

∴ T = 6.13 × 10−3 m 2 / sec

Q t 
s 2 − s1 = ln  2 
4πT  t1 
Here s1 = 1.5 m at t1 = 4 hrs and
s2 = 2.0 m at t2 = 16 hrs.
Q  16 
∴ 2 − 1.5 = ln  
−3
4π(6.13 × 10 )  4 
∴ Q = 0.0278 m3/sec
92 Irrigation Engineering and Hydraulic Structures

Note: Just as draw down values in observation well against time gives aquifer
constant S and T, similarly draw down values in observation wells located at given
distances from the main well will also give S and T will be clear from following
numerical problem.
Example 4.16: A well in a confined aquifer was pumped for 2 hrs at a constant
rate of 1600 LPM and the draw down in the seven nearly observation wells are given
below. Determine aquifer constants, S and T.

Observation wells A B C D E F G
Distance (m) from main wall 5 10 20 40 80 120 200
Draw down (m) 5.35 4.35 3.35 2.35 1.4 0.8 0.3

Solution: Plot distances (m) on log scale on X axis and S(m) on Y-axis in ordinary
scale.
For S = 0, ro = 210 m from the graph as shown in Fig. 4.12
Also from the graph
∆s = 3.25 m per log cycle of r (distance)
i.e., s value at 10 = 4.35 m
and s value at 100 = 1.10 m
∴ Δs = 3.25 m

7
6
5
4
draw down
3
S (m)
r = 210 m
2
for S = 0
1
log scale
10 100 1000
Distance (m)

Fig. 4.12: Plot of distance vs draw down

Now for data giving different values of s in wells situated at different values of r,
use the formula:
2.3Q
∆s =
2πT
Ground Water 93

 1.6 
2.3  
 60 
∴ 3.25 =
2πT

∴ T = 3 ×10−3 m 2 /sec 

2.25T t
S=
r02

(Here, ro = 210 m as available from graph and t = 2 hrs given).

2.25(3 × 10−3 )(2 × 60 × 60)


=
(210) 2
= 1.1 × 10–3

EXERCISES
1. A 30 cm diameter well penetrates fully an unconfined aquifer of 25 m depth.
After steady state pumping at the rate of 90.0 LPS, from a test well at 90
m from the main well draw down observed was 0.53 m and in another test
well at 30 m distance from main well gave draw down of 1.11 m. Find
transmissibility of the aquifer and draw down in the main well.
[Ans: (i) 1.68 m2/min and (ii) 13.0 m]
2. A bore well of diameter 15 cm. Penetrates fully a confined aquifer of
20 m. depth. If well gives a maximum discharge of 20 LPS, what will be the
strainer length which has 15% area of opening and safe entrance velocity for
the given aquifer is 0.02 m/s?
Hint : Q = 0.02 m3/s
Ao = Area of opening per m length of strainer
= π D (0.15) = 0.070 m² / m length of strainer.

Vo = Safe entrance velocity


= 0.02 m/s.
 Q 0.02 
 Ans: strainer length = = = 14.2 m. 
 Vo A o 0.02 × 0.070 
94 Irrigation Engineering and Hydraulic Structures

3. In an artesian aquifer (confined aquifer) of 8 m. thickness, a 10 cm, diameter


well gives a steady state discharge of 6000 LPH. Two observation well
located at 10 m. and 50 m from the main well gave steady state draw
down of 3 m and 0.05 m respectively. Find transmissivity T and hydraulic
conductivity K (i.e., permeability) of the aquifer.
[Ans: T = 12.5 m² / day K = 1.563 m / day]
4. A well of 50 cm diameter penetrates a confined aquifer of 20 m thickness
having hydraulic conductivity equal to 8.2 × 10-4 m/s. What is the maximum
yield that can be expected from this well if draw down in the well is not to
exceed 3m? Take radius of influence as 250 m.
[Ans: 45 LPS.]
5. (a) Define: Transmissibility, permeability and storativity aquifers.
(b) A recuperation test was conducted on an open well of diameter 6.5 m,
giving following data:
(i) RL of water table = 238 m.
(ii) RL of water level when pumping stopped = 230 m.
(iii) RL of water level after 2.5 hours after pumping was stopped = 234 m.
Find safe yield of the well if working head is 3 m.
[Ans: 7.5 LPS.]
5
Reservoir Planning

5.1 INTRODUCTION
Rivers have very little flow during post-monsoon and high flow during monsoon
months. In order that high flow water is retained to take care of period of scarcity,
usual procedure is to construct a dam and create a reservoir just upstream of it
so that it can supply water during low flow or during drought. Thus, reservoirs
are useful to regulate flow in rivers, prevent flood havoc and are useful during
drought.

5.2 TYPES OF RESERVOIRS


Reservoirs are classified as follows:
1. Multipurpose Reservoirs : Reservoirs catering to needs like irrigation,
water supply, industrial requirement of water, hydropower production
and navigation are called multipurpose reservoirs.
2. Flood Control Reservoirs : Reservoirs constructed for single purpose of
controlling water during flood and releasing when the flood intensity
becomes tolerable, then such reservoirs are known as flood control
reservoirs. These are of two types: (a) Retarding and (b) Detention
reservoirs.
(a) Retarding Reservoirs : These are meant for holding flood water such
that discharge passing over ungated spillway is not causing damage to
downstream side.
(b) Detention Reservoirs : In such reservoirs outlets and spillways are
controlled by means of gates to allow quick disposal of flood water
and can also be closed to detain requisite amount of flood water, which
may be made useful during other period of demand.
3. Distribution Reservoirs : These are small storage reservoirs used for water
supply to city and industrial areas. During peak demands, these need to
be supplied by pumping water from external sources. During low demand
period these can be used for storage.

© The Author(s) 2023 95


S. K. Ukarande, Irrigation Engineering and Hydraulic Structures,
https://doi.org/10.1007/978-3-031-33552-5_5
96 Irrigation Engineering and Hydraulic Structures

5.3 INVESTIGATION FOR RESERVOIR SITES


Three types of surveys are carried out for investigating suitable site for reservoirs.
viz., 1. Engineering Surveys, 2. Hydrological Surveys and 3. Geological Surveys.
1. Engineering Surveys : The area of dam site and reservoir site i.e., valley
region just upstream of the dam is surveyed and contour plan is prepared.
From this contour plan, area-elevation and volume–elevation curve
(Fig. 5.1) of the reservoir sites are prepared by plotting.
(i) Reservoir site areas against various elevations for dam site, known as
area-elevation curve.
(ii) By knowing contour areas of reservoir sites at various elevations,
one can work out volume by making use of either of the following
formulae:
(a) Trapezoidal Formula
h
V=∑
( A1 + A 2 )
       2
where, h is contour inerval between contour areas A1 and A 2 .

(b) Cone Formula


h
V=∑
3
(
A1 + A 2 + A1A 2 )
(c) Prismoidal Formula:
h
V = ∑ ( A1 + 4A 2 + A3 )
6
By plotting cumulative volume against elevation, volume-elevation curve is
obtained.

2
1

Elevation
in m.

1. Volume (ha-m)
2. Area (ha)
Fig. 5.1: Area elevation and volume-elevation curves
Reservoir Planning 97

With the help of area-elevation curve and volume-elevation curve, height


of the dam for requisite volume of water to be stored in reservoirs can be fixed.
These curves help in determining evaporation losses at various elevations
by knowing areas at these elevations and respective evaporation loss from
experimental data. These curves also help in deciding area of submergence
of land for storage at various elevations. Thus, these two curves are starting
point for design of a dam and reservoir.
2. Hydrological Surveys : Reservoir planning depends on hydrological
surveys, and hence they are required to be carried out to know the following:
(i) Study of run-off pattern of river at the proposed dam site to determine
storage capacity and height of dam.
(ii) Study of flood hydrographs at the dam site to determine spillway
capacity and flood control measures.
Area of reservoir at the height of the dam is known as water spread area,
whereas the area up stream of water spread area, which feed rainwater to reservoir
site, is known as Catchment area. Hydrological surveys are therefore required to
include the surveys for Catchment areas also. This will provide reliable run-off data
and flood flow data at dam site. The hydrological data if available for at least past
thirty-five years, then future pattern of floods and run-off can also be predicted by
flood frequency analysis. This is a must for any multipurpose reservoir planning.
3. Geological Surveys : Geological investigations of dam and reservoir sites
are carried out to determine,
(a) Suitability of foundation for dam site.
(b) Water tightness of reservoir basin.
(c) Location of quarry site for materials of construction required for dam
and spillways.
(d) To ascertain whether dam site is in seismic zone or not.
(e) Whether sound rock foundation is available or not is the prime
information required to determine the type of the dam. Hence, detailed
foundation explorations are carried out and faults, cavities, fractures
etc. if present in rock strata are located and ascertained whether they
are permissible or not.

5.4 ZONES OF STORAGE IN A RESERVOIR


The zones of water storage in reservoir are as follows:
1. Dead Storage: Volume of water held below Dead Storage Level (DSL) is
known as dead storage. This volume may get filled up by sedimentation
in reservoir that may take place year by year during life of reservoir. This
storage is not useful and hence is known as dead storage.
98 Irrigation Engineering and Hydraulic Structures

2. Live Storage: Water stored between maximum water level (i.e., FRL,
full reservoir level) and minimum water level (LWL, lowest water level)
is known as live storage or useful storage. If upper part of reservoir i.e.,
between spillway crest level and FRL is supposed to hold flood water for
some time to moderate peak flood value then volume of water between FRL
and spillway crest level is known as surcharge storage or flood absorption
capacity of reservoir, and in that case live storage is only between spillway
crest level and LWL. Live storage also includes evaporation and seepage
losses from reservoir.

FRL Free board

Surcharge storage Spillway


Spillway crest
Level

Live storage
Supply sluice
LWL

Carry over storage


DSL

Dead storage Scouring sluice

Fig. 5.2: Zones of storage in reservoir


In absence of suitable data, dead storage is taken as 10% of live storage and
losses are taken as 5% of live storage.
3. Carry Over Storage: Volume of water stored between DSL and LWL is
known as carry over storage, and is useful for bad year or delayed monsoon
period. This provision is not made always, but it depends on the type of
reservoir.
4. Gross storage: It is the sum of all types of storage i.e., dead, carryover and
live. It gives tentative height of the dam by using volume-elevation curve.

5.5 SEDIMENTATION IN RESERVOIR


Catchment characteristics such as soil type, land slope, vegetal cover, climatic
condition like temperature, intensity of rainfall have great significance in the
production of sediment load in the form of suspended particles and in the form of
movable bed load (Fig. 5.3).
weight of sediment in sample
Sediment load, ppm = ×106
weight of sample
Reservoir Planning 99

If Qs = Suspended sediment load transported by the flowing water,


and Q = Discharge of flowing water, then
Q s = k Qn
∴ log Qs = log k + n log Q

Qs = k, when Q = 1; where, n is between 2 to 3.


When the sediment laden water reaches reservoir, coarse sediment starts settling
down at the mouth or entry to reservoir and fine sediment along with a part of
coarse sediment gets settled down in dead storage zone and if sediment load is
very high, by opening out scouring sluice gates, the heavily laden sediment water
can be removed from reservoir, see Fig. 5.3. Mean annual sediment production
rate vary from 250 to 2000 tons/km² of catchment, and reservoir loses its storage
capacity around one percent annually.

Floating debris FRL


sluice

Density
currents sluice
LWL
Supply sluice
se Fine
C dim
se oar en
t
di se
m
en DSL
t
Scouring sluice

Fig. 5.3: Sedimentation in reservoir

5.6 USEFUL LIFE OF RESERVOIR


The sedimentation in a reservoir is measured in terms of TRAP efficiency, h, which
is defined as percentage of inflowing sediment that is retained in the reservoir, thus

 Reservoir Capacity  C


η = Trap Efficiency = f   =f 
 Inflow to Reservoir  I
n
 1 
= 100 1 − 
 (1 + ax) 
Capacity
here x = ratio, and a and n are constants
Inflow
100 Irrigation Engineering and Hydraulic Structures

The useful capacity of reservoir lost each year by sediment deposition is


Vs = Qs ηav
Vs = volume of useful capacity of reservoir lost each year.
Qs = annual sediment inflow into reservoir
While allocating space for dead storage i.e., space for sediment deposition
during life of reservoir, trap efficiency is taken as 95% to 90%.

q SOLVED EXAMPLES
Example 5.1: Find probable life of a reservoir with an initial capacity of 4000 ha-m
and average annual inflow is 8000 ha-m, and average annual sediment inflow is
2 × 106 KN. Specific weight of sediment is 11.2 KN/m3. Useful life will terminate
when 80% of initial capacity is filled with sediment. Values of trap efficiency and
capacity-inflow ratio are given below:

Capacity
Ratio Trap efficiency , η
Inflow
0.1 87
0.2 93
0.3 95
0.4 95.5
0.5 96
0.6 96.5
0.7 97
0.8 97.3
0.9 97.4
1.0 97.5

Solution: Annual sediment inflow = 2 × 106 KN


2 ´106
Vs = Volume of sediment inflow = = 1.786 ´105 m3 = 17.86 ha- m
11.2
Initial capacity of reservoir = 4000 ha-m
Average annual inflow = 8000 ha-m
C 4000
Initial Capacity / Inflow = = = 0.5
I 8000
Reservoir Planning 101

Vs × Vol. of
Capacity Trap eff Av value ηav vol. capacity Years
Cap. Inflow η – from of trap of sed. interval to fill
Capacity ha-m ratio given eff. ηav trapped (h-m) col.6
% col.(1) (2) table (3) (4) (5) (6) col.5 (7)

96 + 95.5 (17.86 4000– 800


100 4000 0.5 96 × 3200 17
2
0.956)
80 3200 0.4 95.5 = 95.7 = 17.0 = 800 = 47
60 2400 0.3 95 95.2 17.0 800 47
40 1600 0.2 93 94.0 16.79 800 47.64
20 800 0.1 87 90.0 16.07 800 50.00
Life of Reservoir = 191.64 years.

Example 5.2: A reservoir has capacity of 400 ha-m. Catchment area is 130 Km2
and 12 cm runoff. If sediment production per year is 0.03 ha-m/km2 what is
probable life of reservoir before its capacity is reduced to 20% of initial? Relation
between trap efficiency and (capacity/inflow) ratio is given below:
C/I ratio 0.05 0.1 0.2 0.3 0.4 0.5 0.6 0.7 1.0

η trap eff. 77 87 93 95 95.5 96 96.5 97 97.5

Solution:

12 130 × 106
Average annual inflow = × = 1560 ha-m
100 104
C 400
= = 0.256 → η = 93.5,say
I 1560
Qs = .03 × 130 = 3.9 ha-m
Vs = Qs η = 3.9 × 0.935 = 3.64 ha-m
Similarly calculations are done and entered in following table.

No. of years
to fill the loss
Loss of R ΔC
= Vs years
C C/I h h av Vs = Qs × ηav capacity = ΔC

 94 + 93 
0.256 × 800
400 0.256 94   0.935 400-320 17
 2 
80
320 0.2 93 93.5 = 3.64 = 80 = 22
3.64
102 Irrigation Engineering and Hydraulic Structures

No. of years
to fill the loss
Loss of R ΔC
= Vs years
C C/I h h av Vs = Qs × ηav capacity = ΔC

80
240 0.15 90 91.5 3.57 80 = 22.4
3.57
80
160 0.1 87 88.5 3.45 80 = 23.2
3.45
80
80 0.05 77 82.0 3.20 80 = 25.0
3.2
Total 92.6 years
Life of Reservoirs = 93 Years

5.7 FACTORS AFFECTING SELECTION OF RESERVOIR


SITE
1. Reservoir site should be chosen such that run off from catchment area is
maximum, with least sediment load.
2. Reservoir site should be such that it gives minimum seepage loss i.e., site
should have sound rock formation with least percolation.
3. Reservoir site must be such that it has a suitable dam site i.e., dam axis, such
that dam construction cost is minimum for maximum storage of water.
4. Valley opening should be narrow.
5. Site should be accessible with least cost for its approach and maintenance.
6. Land submergence due to flood should be minimum.
7. Evaporation loss should be minimum. This is possible when reservoir site is
deep. i.e., area of waterspread exposed is minimum with respect to elevation.
8. Minerals and salts contained in rock formation of reservoir site should not be
objectionable, spoiling the quality of water stored.

5.8 DETERMINATION OF RESERVOIR CAPACITY FROM


MASS CURVES
Procedure:
1. From inflow hydrograph prepare mass inflow curve at least for 35 years.
2. Prepare demand curve by calculating demands such as irrigation, water
supply etc per year.
3. From mass curve apex such as A1, A2 etc. draw tangents parallel to demand
curve.
Reservoir Planning 103

4. Measure maximum vertical intercepts such as E1D1, E2D2 etc. The vertical
interval indicates volume by which inflow falls short of demand for example,
C1E1 indicate demand, whereas C1D1 indicate inflow, Hence, E1D1 has to
be provided from reservoir storage i.e., it is shortage. Hence, highest of
the ordinates E1D1, E2D2 etc. indicate reservoir storage capacity. Elevation
against this capacity is read from capacity–elevation curve and that is the
height of the dam.

E2

D2
A2
C2
Demand
Inflow Q

Demand Q
curve
E1
Mass inflow curve Demand
curve
D1
A1
C1
Time (Years)

Time (years)

Fig. 5.4: Mass curve

5.9 RESERVOIR LOSSES


The loss of water from reservoir is due to evaporation, absorption and percolation.
Evaporation loss mainly depends on surface area exposed to atmosphere, temperature
humidity and wind velocity. It can be measured by making use of evaporation pan
and appropriate pan co-efficient. Month-wise evaporation loss values in mm are
given in Table 5.1:

Table 5.1 Monthly Evaporation Loss

Month Evaporation loss in mm in North India In south and central India in mm


Jan 70 100
Feb 90 100
March 130 180
April 160 230
May 270 250
June 240 180
104 Irrigation Engineering and Hydraulic Structures

Month Evaporation loss in mm in North India In south and central India in mm


July 180 150
Aug 140 150
Sept 140 150
Oct 130 130
Nov 90 100
Dec 80 100

Loss of water due to absorption depends on type of soil forming reservoir basin.
This is large in beginning but gradually reduces as pores get saturated.
Percolation or seepage loss is very small in most of the cases, however it
may be significant if severe leakage takes place through hills or base of dam due
to presence of fissured rock. Grouting should be carried out to prevent such loss.

5.10 RESERVOIR FLOOD ROUTING


Consequent upon heavy rains in catchment area, a river may start swelling i.e.,
its rate of flow may increase and if continues to do so, it will start overflowing,
due to this people living on banks may suffer because of floods in river. Certain
reservoirs do make a provision of holding flood water for some interval of time
thereby giving chance to down stream people to get shifted to safe places, but all
reservoirs can not do so. However, regulated release of water from reservoir can be
planned and executed. This is known as flood routing. Thus, flood routing means
control of water levels in reservoir and release of water from reservoir i.e., it’s a
matter of controlling “inflow and outflow” of reservoir during flood time. Flood
routing is thus regulation of reservoir storage and its outflow and also its means
regulation of flood flow in rivers or channels. Thus, flood routing is divided into
two separate studies:
1. Reservoir routing
2. Channel routing
Reservoir routing makes use of hydrologic methods developed by LG Puls , Goodrich
and others , where as channel routing method is known as Muskinghm method which
was first tried on river Muskingm in USA.
Reduction in Peak values of flood due to routing technique is known as “attenuation”.
This is shown in Fig. 5.5. It is the difference in peak value of inflow (flood) hydrograph
and outflow hydrograph. Further peak of outflow occurs after peak of inflow, thus
Reservoir Planning 105

giving difference in time between occurrence of two peak values. This is known
as “time lag”.

T ime lag = Difference in time between


peak value of graph (1)
and (2)

Q
Attenuation = difference
m 3/sec
between peak of
graph (1) and (2)
2
1 Outflow hydrograph
Inflow hydrograph

T ime (hrs)

Fig. 5.5: Attenuation and peak lag

The attenuation and time lag of flood hydrograph at a reservoir are two
important aspects of reservoir operation under flood control criteria. The storage
capacity of reservoir and characteristics of spillway and other outlets from reservoir
control the flood for some time so that downstream population can be warned.
Total flood control is not possible but its detention for certain interval of time
is possible.
Reservoir routing is a process of calculating changes in reservoir levels, its
volume and outflows for a given inflow hydrograph representing a flood approaching
from catchment area. This is based on hydrologic method, which can be expressed
mathematically as
I − O = ± ∆s .....(5.1)
Where :
I = Inflow
O = Outflow
ΔS = Changes in reservoir storage.
By choosing a proper time interval (Δt) between different values of outflow
and inflow, equation (5.1) can be written as :

 I1 + I2   O1 + O 2 
 2  ∆t −  2  ∆t = S2 − S1 (5.2)
   
where,
I1 and I2 are inflow at a Δt interval,
O1 and O2 are outflow at Δt interval and
S1, S2 are corresponding storage value in reservoir.
106 Irrigation Engineering and Hydraulic Structures

Equation (5.2) can also be expressed as :

 2S   2S 
(I1 + I2 ) +  1 − O1  =  2 + O 2  (5.3)
 ∆t   ∆t 
Inflow – Storage – Discharge (ISD) method developed by L.G. Puls, makes
use of equation (5.2) and Goodrich method or modified Puls method makes use
of equation (5.3). By both the methods results obtained do not vary significantly
and hence any method can be adopted. In following numerical problem Goodrich
method is used to illustrate how reservoir routing can be carried out .

q SOLVED EXAMPLES
Example 5.3: Following flood hydrograph is to be routed using modified
puls method. Find out attenuation and time lag by plotting inflow and outflow
hydrographs.
Time (hrs) 0 6 12 18 24 30 36 42 48 54 60 66 72
3
Q (m /s) 60 170 280 350 420 360 310 220 165 80 70 25 20

Outflow from reservoir just before arrival of flood = 200 m3/s.


Also given following values of outflows from reservoir against storage:

Q (outflow) (m3/sec) 0 40 70 85 100 110 250 490 800


S (cumec-day) 10 52 105 163 225 290 365 450 530

Solution:
1
∆t = 6hr = day
4
2S
\ = S´ 8 m3 /day
Dt
 2S 
First step is to plot Q v/s  + Q  Hence from given data of Q and S, prepare
 ∆t 
 2S 
following table and plot Q on Y axis and  + Q  on X axis.
 ∆t 

2S  2s  3
Q (m3/sec) S (cumec-day) = s × 8 (m3/sec)  + Q m /sec
∆t ∆t
  0 10    80    80
40 52 416 456
70 105 840 910
Reservoir Planning 107

2S  2s  3
Q (m3/sec) S (cumec-day) = s × 8 (m3/sec)  + Q m /sec
∆t ∆t
85 163 1304 1389
100 225 1800 1900
110 290 2320 2610
250 365 2920 3170
490 450 3600 4090
800 530 4240 5040

The above values are plotted as shown in Fig 5.6.

800

700

600
Q 500

400
300

200
100

1000 2000 3000 4000

2S
Q
t

 2S 
Fig. 5.6: Graph of  + Q & Q
 ∆t 
Now given value of Q = 200 m3/sec at entry of flood.
2S
Hence, read  + Q  from graph of Fig. 5.6, it is 3000 m3/sec against
 ∆t 
Q = 200 m3/sec.

 2S   2S 
∴  − Q  =  + Q  − 2Q
 ∆t   ∆t 
= 3000 – 2 (200) = 2600 m3/sec.
108 Irrigation Engineering and Hydraulic Structures

Now I1 = 60 and I2 = 170 (given in data)


∴ (I1 + I2) = 60 + 170 = 230
 2S 
∴ (I1 + I2 ) +  − Q  = 230 + 2600 = 2830
 ∆t 
 2S 
=  + Q
 ∆t 
 2S 
Read Q for  + Q  = 2830 from graph of Fig 5.6
 ∆t 
∴ Q = 160 m3 / sec
From this value of Q = 160,
 2S 
 + Q  = 2830 − 2 × 160 = 2510
 ∆t 
The steps are continued further and the results are given in Table below:

Time I (m3/ (I1 + I2)  2S   2S 


 – Q (m3/sec)  + Q (m3/sec) Q(m3/sec)
(hr) sec) m3/sec ∆t  ∆t 

(1) (2) (3) (4) (4) (6)


0 60 – – 3000 200
6 170 230 2600 2830 160
12 280 450 2510 2960 200
18 350 630 2560 3190 230
24 420 770 2730 3500 300
30 360 780 2900 3680 350
36 310 670 2980 3650 340
42 220 530 2970 3500 300
48 165 385 2900 3285 250
54 80 245 2785 3030 210
60 70 150 2610 2760 150
66 25 95 2460 2555 120
72 20 45 2315 2360 110

Plot Time v/s I i.e., column(1) v/s column(2) and get in flow graph.
Plot time v/s Q i.e., column(1) v/s column(6) and get outflow graph.
These graphs are given in Fig. 5.7.
From Fig 5.7, attenuation = 70 m3/sec and time lag = 9 hours.
Reservoir Planning 109

time lag = 9 hours

400 3
70 m /sec = Attenuation

300

200

Q Outflow as per
column (6)

100
Inflow as per
column (2)

6 12 18 24 30 36 42 48 54
T (hrs)
as per column (1)

Fig. 5.7: Plot of T vs. Q

Example 5.4: Route the following flood by ISD method and find attenuation
and reservoir lag.
Data : 1. In flow Hydrograph:

Time (hrs) 0 6 12 18 24 30 36 42 48 54 60 66 72

Q (m3/sec.) 60 170 280 350 420 360 310 220 165 80 70 25 20

2. Outflow at time of arrival of fluid = 200 m3/sec


3. Values of outflow against storage.

Q(m3/sec) 0 40 70 85 100 110 250 490 800

S cumec-day 0 52 105 163 225 290 365 450 530

 Q   Q 
Solution: Calculate  S + ∆t  and  S − ∆t  for Q values given and plot the
 2   2 
graph (Fig. 5.8) calculations are given in the table below:
∆t
Δt = 6 hours = 0.25 day ∴ = 0.125 day
2
110 Irrigation Engineering and Hydraulic Structures

Q Q Q
S Q ∆t = 0.125 Q S+ ∆t S– ∆t
2 2 2

Cumec-day m3/sec cumec-day cumec-day cumec-day


0 0 0 0 0
52 40 5.00 57.00 47.00
105 70 8.75 113.75 96.25
163 85 10.62 173.62 152.38
225 100 12.50 237.50 212.50
290 110 13.75 303.75 276.25
365 250 31.25 396.25 333.75
450 490 61.25 511.25 388.75
530 800 100.00 630.00 430.00

For routing the flood hydrograph


(i) I1 = 60 m3/sec, I2 = 170 m3/sec.
I1 + I2
∴ = 115 m3 / sec.
2
 I1 + I2 
Q1 = 200 m3/sec, Δt = 0.25 day,   ∆t = 115 × 0.25 = 28.75
 2 
Q∆ t
S
2
800
700 Q∆ t
S+
2
600
500
400

300
200

100

100 200 300 400 500 600 700

Q∆ t Q∆ t
S S+
2 2

 Q∆t   Q∆t 
Fig. 5.8: Polt of  S −  vs. Q and  S +  vs. Q
 2 2 
Reservoir Planning 111

For Q1 = 200 m3/sec from graph of Fig. 5.8,


 Q ⋅ ∆t 
 S1 −  = 325 cumec − day
 2 
 Q ∆t   I + I   Q ∆t 
∴  S2 − 2  =  1 2  ∆t +  S1 − 1  = 28.75 + 325 = 353.75 cumec-day
 2   2   2 
3
This gives Q2 = 160 m /sec   From Graph (2) of Fig. 5.8
Now calculations for routing the flood can be entered into following tabular form:
Time I(m3/sec) cumec-day cumec-day cumec-day Q m3/sec
0 60 – – – 200
6 170 (28.75) 325 353.75 160
12 280 56.25 300 356.25 161
18 350 78.75 300 378.75 200
24 420 96.25 325 421.25 275
30 360 97.50 350 447.50 330
36 310 83.75 370 453.75 340
42 220 66.25 375 441.25 300
48 165 48.13 360 408.00 250
54 80 30.63 340 370.63 195
60 70 18.75 320 338.75 150
66 25 12.00 290 302 110
72 20 7.00 275 282.00 105

400 70 m /s
3

9 hrs.

300
3
Q m /s

Outflow
200

100
Intflow

6 12 18 24 30 36 42 48 54 60 66 72
Time (hrs.)

Fig. 5.9: Graph of T vs. Q and T vs I


112 Irrigation Engineering and Hydraulic Structures

Plot : Time v/s Q and Time v/s I as shown in Fig 5.9. From the graph read
attenuation = 70 m3/sec (420 – 350)
One time lag = 9 hours.
Which is the same as obtained by modified Pul’s method of previous problem.

5.11 FIXING HEIGHT OF DAM FROM RESERVOIR


WORKING TABLE
As explained in article 5.8, height of dam can be fixed with the help of mass inflow
and mass outflow curves, But in absence of such data it can be tentatively fixed on
the basis of reservoir working table as explained below:
1. First workout irrigation, water supply and industrial demand for a year, 5% of
this demand can be assumed as losses due to evaporation and seepage. Total
of this (demand + losses) forms live storage.
2. To this live storage add dead storage as 10% of live storage as a space for
accumulation of sediment. Total of live storage and dead storage will give
gross storage.
3. Corresponding to this gross storage, read elevation from volume – elevation
curve (Fig. 5.1) this is first approximate height of dam.
4. On the basis of reservoir full against gross storage in the month of October,
prepare reservoir working table as given in Table 5.2.
5. Generally the reservoir is full in the month of October. Deduct the demand for
irrigation, water supply and losses for October month from the inflow due to
rain in catchment area for month of October. Thus difference between month
wise outflow and inflow is to be either added or subtracted as the case may be
from gross storage and resulting storage will give starting elevation for next
month.
6. Like this proceed month wise; around May or June there will be minimum
elevation of water in reservoir. If this level is equal to level on account of
dead storage, height of dam against gross storage is all right, but if this
level is higher than DSL, then volume of water between minimum level in
reservoir and DSL is surplus water. This surplus volume of water is to be
removed from gross storage and entire procedure is to be repeated till there
is no surplus water in reservoir. This is the economic height of dam. The
entire procedure is explained by way of a working example and preparation
of reservoir working table given below.
Data
Catchment area = 1200 sqkm
Average annual rainfall = 55 cm
Loss (infiltration etc) = 5 cm
Reservoir Planning 113

Rexcess = 50 cm = (50/100) m
Runoff = C × Rexcess × Catchment area
(Where C = Coefficient of catchment = 0.3)
Runoff = 0.3 × (50/100) × 1200 × 106
     = 180 × 106 m3
     = 180 MCM

The above yield is called monsoon yield.


Assume Riparian rights as 2 MCM per month
Note: Riparian rights- Water required to be allowed to flow in river through out
the year for the benefit of down stream people. It is around 10% of annul runoff.
Here it is taken as 2 MCM/month.
90. % of total yield (0.9 × 180 = 162 MCM) is assumed as total yield in monsoon
months of (June, July, August and September) which is distributed month-wise in
the following percentages:

Table 5.2: Yield in Monsoon Months

Month Percentage (%)


June 15
July 25
August 30
September 30

The remaining 10% of 180 MCM (i.e., 0.1 × 180 = 18) is taken as yield available
in the post monsoon months i.e., October to May, which is distributed month-wise
in the following percentages:

Table 5.3: Yield in Post-Monsoon Months

Month Percentage (%)


October 20
November 20
December 15
January 15
February 10
March 10
April 5
May 5
114 Irrigation Engineering and Hydraulic Structures

Calculation of Irrigation Demand


Table 5.4: Irrigation Demand

D = Duty B = Base Period 8.64B


Season Crop (Ha/cumec) (Given) (Days) D = Delta = m
D
Rabi Wheat 1800 120 0.567

Kharif Rice 800 120 1.296


Hot Weather Fruit/Vegetable 700 120 1.480
Eight Months Cotton 1400 180 1.110
Perennial Sugar cane 1700 300 1.524

Assuming Cultivable Command Area = 3000 hect.


and 80% is intensity of irrigation;
Area under irrigation = 0.8 × 3000 = 2400 hect.

Table 5.5: Irrigation Demand

Crop percentage Duty Ha/cumec Base period Delta Area Volume


Season (%) (Given) (Given) (days) (m) (1) Ha (2) (1) × (2)Ha-m
Rabi 25 1800 120 0.567 600* 345.6
Kharif 25 800 120 1.296 600 777.6
Hot Weather 10 700 120 1.480 240 355.2
Eight Months 15 1400 180 1.110 360 399.6
Perennial 25 1700 300 1.524 600 914.4
Total =2792.4

25
where, × 2400==600
x2400 *
600*
100
Total irrigation demand = 2792.4 Ha-m = 2792.4 × 104 m3 = 27.92 MCM
= 30 MCM/year (rounded figure)
Calculation of month-wise irrigation demand:
For perennial season:
Total demand for perennial season = 914.4 Ha-m
(from Table 5.5) = 914.4 × 104 m3
= 9.414 MCM (million cubic meter)
Similarly, for other season month-wise irrigation demand can be calculated, which
is given in Table 5.6.
Reservoir Planning 115

Table 5.6: Month-wise Irrigation Demand


Month Perennial Eight month Rabi Kharif Hot weather Total
(MCM) (MCM) (MCM) (MCM) (MCM) (MCM)
Oct 0.762 1.944 2.706
Nov 0.762 0.4995 0.864 2.1255
Dec 0.762 0.4995 0.864 2.1255
Jan 0.762 0.4995 0.864 2.1255
Feb 0.762 0.4995 0.864 2.1255
March 0.762 0.4995 0.888 2.1495
April 0.762 0.4995 0.888 2.1495
May 0.762 0.4995 0.888 2.1495
June 0.762 0.4995 0.888 2.1495
July 0.762 1.944 2.706
Aug 0.762 1.944 2.706
Sept 0.762 1.944 2.706

Calculation of Water Supply Demand


Storage for water supply demand can be considered on per capita requirement.
Assume Population = 2.5 lakh
Water consumption = 200 litre / capita / day
Water supply demand = 200 × 365 × 2,50,000
= 18.25 × 109 litre (yearly)
= 18.25 × 106 m3
= 18.25 MCM
= 20 MCM (rounded figure on higher side)
Water supply demand per month = 20/12 = 1.667 MCM
Live storage = Irrigation demand + Water supply demand
= 30 + 20 = 50 MCM
Dead storage = 10% of Live storage = 5 MCM
Losses = 10% of Live storage = 5 MCM
Carry Over Storage (requirement for weak monsoon) = 50% of
Live storage = 25 MCM
Gross storage = L.S + D.S + Losses + C.O.S
= 50 + 5 + 5 + 25 = 85 MCM
Seepage loss = (5% of Live storage) / 12 = 2.5/12 = 0.2083 MCM/month
Lowest Water Level (L.W.L) = Dead Storage (D.S) + Carry Over (C.O)
= 5 + 25 = 30 MCM
Elevation of LWL against storage of 30 MCM = 243.75 m (From volume-
elevation graph; see Fig. 5.1)
Now to get the required height of the dam, trial reservoir working tables are
worked out till difference of water level at minimum value over 12 month and low
water level fixed for reservoir are coinciding with each other, in case they do not
coincide, then volume of water equivalent to difference in these values is required
to be removed from gross storage and one more trial working table should be
prepared. This is to be repeated till the difference is negligible as can be seen from
Tables 5.6, 5.7 and 5.8.
Table 5.7 Reservoir Working Table – I (first trial)
116

Flood
Reservoir Volume of Irrigation Water
Gross Level Evaporation Evaporation Seepage on Supply Riparian Total Inflow –
Month Storage (FRL) Area Loss Loss Loss Demand Demand Demand Outflow Inflow Outflow Overflow
(MCM) (m) (Ha-m) (cm) (MCM) (MCM) (MCM) (MCM) (MCM) (MCM) (MCM) (MCM) MCM
(1) (2) (3) (4) (5) (6) (7) (8) (9) (10) (11) (12) (13) (14)
OCT 85 286.25 165 × 104 13 0.2145 0.2083 2.7060 1.667 2 6.7958 3.6 –3.1958
NOV 81.8 284.25 161 × 104 9 0.1449 0.2083 2.1255 1.667 2 6.1457 3.6 –2.5457
DEC 79.25 282.5 158 × 104 8 0.1268 0.2083 2.1255 1.667 2 6.1276 2.7 –3.4276
4
JAN 75.82 280.5 155 × 10 7 0.1085 0.2083 2.1255 1.667 2 6.1093 2.7 –3.4093
4
FEB 72.41 278.25 152 × 10 9 0.1368 0.2083 2.1255 1.667 2 6.1376 1.8 –4.3376
MAR 68.07 275.5 148 × 104 13 0.1924 0.2083 2.1495 1.667 2 6.2112 1.8 –4.4172
Irrigation Engineering and Hydraulic Structures

4
APRIL 63.65 272.5 143 × 10 16 0.2296 0.2083 2.1495 1.667 2 6.2544 0.9 –5.3544
MAY 58.29 269.0 138 × 104 27 0.3740 0.2083 2.1495 1.667 2 6.3988 0.9 –5.4988
4
JUNE 52.79 265.5 133 × 10 24 0.3192 0.2083 2.1495 1.667 2 6.344 24.3 17.956
4
JULY 70.75 277.25 150 × 10 18 0.2709 0.2083 2.7060 1.667 2 6.8522 40.5 33.6478 24.6478*
AUG 85 286.25 165 × 104 14 0.231 0.2083 2.7060 1.667 2 6.8123 48.6 41.7877 41.7877
4
SEP 85 286.25 165 × 10 14 0.231 0.2083 2.07060 1.667 2 6.8123 48.6 41.7877 41.7877

Since minimum level = 265.5 m; Corresponding volume = 52.79 MCM and LWL = 243.75 m Corresponding volume = 30.00 MCM
Surplus water in reservoir = 52.79 – 30.00 = 22.79; hence Gross storage (new) = 85 – 22.79 = 62.21 MCM
FRL is to be obtained from graph of volume –elevation for new value of Gross storage. i.e., 62.21 MCM, corresponding level is 271.5
* Overflow will take place only when (I-O) added to reservoir at the beginning of month become more than 286.25 m, the starting FRL in the month of October.
Table 5.8 Reservoir Working Table – II (second trial)

Flood
Reservoir Volume of Irrigation Water
Gross Level Evaporation Evaporation Seepage on Supply Riparian Total Inflow –
Month Storage (FRL) Area Loss Loss Loss Demand Demand Demand Outflow Inflow Outflow Overflow
(MCM) (m) (Ha-m) (cm) (MCM) (MCM) (MCM) (MCM) (MCM) (MCM) (MCM) (MCM) MCM
(1) (2) (3) (4) (5) (6) (7) (8) (9) (10) (11) (12) (13) (14)
4
OCT 62.21 271.5 142 × 10 13 0.1846 0.2083 2.7060 1.667 2 6.7959 3.6 –3.1959
4
NOV 59.04 269.5 139 × 10 9 0.1251 0.2083 2.1255 1.667 2 6.1259 3.6 –2.5259
4
DEC 56.51 267.5 136 × 10 8 0.1088 0.2083 2.1255 1.667 2 6.1096 2.7 –3.4096
4
JAN 53.10 265.5 133 × 10 7 0.0931 0.2083 2.1255 1.667 2 6.0939 2.7 –3.3939
FEB 49.70 263.5 130 × 104 9 0.117 0.2083 2.1255 1.667 2 6.1178 1.8 –4.3178
MAR 45.38 259.75 124 × 104 13 0.1619 0.2083 2.1495 1.667 2 6.1867 1.8 –4.3867
4
APRIL 40.99 255.5 118 × 10 16 0.1888 0.2083 2.1495 1.667 2 6.2136 0.9 –5.3136
4
MAY 35.67 250.75 111 × 10 27 0.2997 0.2083 2.1495 1.667 2 6.3245 0.9 –5.4245
4
JUNE 30.24 244 101 × 10 24 0.2424 0.2083 2.1495 1.667 2 6.2672 24.3 18.0328
4
JULY 48.27 262.25 128 × 10 18 0.2313 0.2083 2.7060 1.667 2 6.8126 40.5 33.6874 24.4374
4
AUG 62.21 271.5 142 × 10 14 0.1988 0.2083 2.7060 1.667 2 6.7801 48.6 41.82 41.82
SEP 62.21 271.5 142 × 104 14 0.1988 0.2083 2.7060 1.667 2 6.7801 48.6 41.82 41.82

Since minimum level = 244 m => volume = 30.24 MCM


LWL = 243.75m => volume = 30.00 MCM
Surplus = 0.24 (Not required) Hence, Gross storage (new) = 62.21 – 0.24 = 61.97 MCM, corresponding level is 271.45.
Reservoir Planning 117
Table 5.9 Reservoir Working Table – III (third trial)
118

Flood
Reservoir Volume of Irrigation Water
Gross Level Evaporation Evaporation Seepage on Supply Riparian Total Inflow –
Month Storage (FRL) Area Loss Loss Loss Demand Demand Demand Outflow Inflow Outflow Overflow
(MCM) (m) (Ha-m) (cm) (MCM) (MCM) (MCM) (MCM) (MCM) (MCM) (MCM) (MCM) MCM

(1) (2) (3) (4) (5) (6) (7) (8) (9) (10) (11) (12) (13) (14)

OCT 61.97 271.45 142 × 104 13 0.1846 0.2083 2.7060 1.667 2 6.7959 3.6 –3.1959
4
NOV 58.8 269.5 139 × 10 9 0.1251 0.2083 2.1255 1.667 2 6.1259 3.6 –2.5259
4
DEC 56.27 267.75 136.5 × 10 8 0.1092 0.2083 2.1255 1.667 2 6.1100 2.7 –3.41

JAN 52.86 265.5 133 × 104 7 0.0931 0.2083 2.1255 1.667 2 6.0939 2.7 –3.3939

FEB 49.46 263.5 130 × 104 9 0.117 0.2083 2.1255 1.667 2 6.1178 1.8 –4.3178
Irrigation Engineering and Hydraulic Structures

4
MAR 45.14 259.75 124.5 × 10 13 0.1619 0.2083 2.1495 1.667 2 6.1867 1.8 –4.3867
4
APRIL 40.75 255.5 118 × 10 16 0.1888 0.2083 2.1495 1.667 2 6.2136 0.9 –5.3136

MAY 35.43 250.75 111 × 104 27 0.2997 0.2083 2.1495 1.667 2 6.3245 0.9 –5.43

JUNE 30.00 243.75 100 × 104 24 0.2412 0.2083 2.1495 1.667 2 6.266 24.3 18.034
4
JULY 48.03 262.0 128 × 10 18 0.2304 0.2083 2.7060 1.667 2 6.8117 40.5 33.6883 24.2686
4
AUG 61.97 271.5 142 × 10 14 0.1988 0.2083 2.7060 1.667 2 6.7801 48.6 41.8199 41.8199
4
SEP 61.97 271.5 142 × 10 14 0.1988 0.2083 2.7060 1.667 2 6.7801 48.6 41.8199 41.8199

Surplus = 0.0; hence height of the dam = F.R.L – River Bed Level = 271.45 – 200 = 71.45 m
Reservoir Planning 119

EXERCISES
1. (a) What are different types of reservoirs and what are their functions?
(b) State sectors those are required to be considered for selection of
(i) Reservoir site and (ii) Dam site.
2. (a) Explain how height of dam is fixed?
(b) What are mass curves?
(c) What is the importance of (i) area – elevation and (ii) volume – elevation
curves?
3. What is trap efficiency? What is its role in deciding life of reservoirs?
4. A reservoir has a capacity of 600 ha-m, the catchment area is 150 sq km
and average annual run-off is 12 cm. If annual sediment production is
0.03 ha-m/sq km how many years will be required to have 10% reduction in
reservoir capacity by sedimentation? Take ηav = 0.95 for C/I ratio between
0.3 and 0.35. (Ans.: 14 years)
5. A reservoir has storage capacity of 740 ha-m. Catchment area is 80 sqkm,
from which annual sediment inflow to reservoir is 0.12 ha-m per sq km. Of
C.A. Taking ηav = 0.80, find annual capacity loss due to sediment in reservoir.

(Ans.: 1.03 % per year)


6. Find probable life of a reservoir having capacity equal to 30 MCM. And
annual inflow of 60 MCM. The annual sediment inflow is 0.2 × 106 tonnes.
Specific gravity of the sediment is 1.2; the useful life of reservoir will be
over when 80% of its capacity is filled with sediment. C/I ratio and η values
are given below :
C/I Ratio 0.1 0.2 0.3 0.4 0.5 0.6 0.7 0.8 0.9 1.0

η% 87 93 95 95.5 96 96.5 97 97 97 97.5

(Ans.: 153 years)


6
Design of Gravity
Dams
6.1 INTRODUCTION
Dam is an obstruction constructed across a river valley with a specific purpose
of creating storage reservoir on its upstream side. Its length is divided into two
zones, non-overflow and overflow. The overflow part of the length of the dam
is defined as spillway and rest is known as non-overflow section of the dam.
When a dam resists the force of water pressure solely by its own weight, it is
known as Gravity Dam. It can be constructed with masonry or concrete and as such
it can be classified as rigid dam. In general, gravity dam is a huge massive masonry
or concrete structure facing water pressure as its main force. In addition to this, there
are other forces that play their role in the design of gravity dams. They are described
in detail below.

6.2 FORCES ACTING ON GRAVITY DAMS


Determination of force, its magnitude and line of action, its effect on shape of
its profile and its importance whether it is basic or secondary are the essential
features the designer should know thoroughly for design of a dam, as it is a very
responsible task that a civil engineer is required to perform. Essentially, entire
irrigation engineering is nothing but design of civil engineering structures, which
deal with storage and distribution of water for irrigation and allied purposes. The
forces can be enlisted as under:
A. Basic Forces:
(i) Weight of body of dam, W
(ii) Water pressure, P
(iii) Uplift force, U
B. Secondary Forces:
(i) Wind pressure
(ii) Wave pressure
(iii) Silt pressure
(iv) Ice pressure

© The Author(s) 2023 120


S. K. Ukarande, Irrigation Engineering and Hydraulic Structures,
https://doi.org/10.1007/978-3-031-33552-5_6
Design of Gravity Dams 121

C. Periodic Type:
(i) Earthquake force
(ii) Cyclonic conditions
Top width
FRL
F.B

U/s and d/s face vertical (zone l)


Pv1
U/s vertical d/s sloping (zone ll)
w2

Pv2 U/s vertical d/s both sloping R


Cuts middle 3rd (zone lll)
PH

Heel w3 High gravity condition


(h/3) w1 Re Rf Toe R cuts inside middle
3rd (zone lV)

h
A/3
A = (ch)

Fig. 6.1: Forces acting on a gravity dam

Detailed Description of Forces:


A. Basic Forces:

1. Self-weight, W: Self-weight W consists of W1, W2 and W3 as shown in


Fig. 6.1

W1 = u/s triangular portion due to u/s slope


W2 = middle rectangular portion due to top width
W3 = d/s triangular portion due to d/s slope
W1, W2, W3 act vertically downward through respective center of
gravity of each portion of the section of dam, and their magnitude is
equal to:
W1 = (s γ) V1
W2 = (s γ) V2
W3 = (s γ) V3
122 Irrigation Engineering and Hydraulic Structures

Where V1, V2, V3 are volumes of respective portion and (S γ) is weight


density of concrete. Here s is specific gravity of concrete, generally 2.4
and γ is weight density of water, generally 1000 kg for 1.0 tonne.
2. Water Pressure, P: Calculation of water pressure is carried out in two
parts: for PH and PV.
PH = horizontal pressure acting on vertical projection of upstream side
face of the dam and its magnitude = ½ γ h². where, h = height of FRL
from river bed level, and is acting at 1/3 h, giving displacing moment
about toe of the dam.
Pv = (PV1+ PV2)V due to weight of water on u/s sloping face of dam
Pv1 = Wt. of water in rectangular portion
Pv2 = Wt. of water in triangular portion
Pv1 and Pv2 act vertically downward through respective centre of
gravity of rectangular and triangular portion.
Hence, P = Total pressure = PH2 + PV2

If u/s face is vertical from top to bottom, then Pv does not exist, and P
= PH
Calculation of PH, PV, W, U etc. are carried out for unit length of dam
and hence volume.
3. Uplift Force, U: Uplift force is due to a possible presence of water below
dam. Ordinarily it is not present as foundation of gravity dam is a sound
rock formation, which is free of cracks, crevices and faults. However,
these may develop at a later stage and hence as a precautionary measure,
it is assumed that certain percentage of hydrostatic force may be acting
upward on the base of the dam. This is known as uplift force, U and is
given by
U = ½ (B) (C γ h), where
C = Coefficient of uplift
  = Fraction of hydrostatic force
  = 0.5, generally, as it can vary from zero to 1.0
U is acting vertically upward through centre of gravity. of the triangular
uplift pressure diagram shown below base of dam in Fig. 6.1, i.e., at B/3
from heel of the dam. Here down stream (tail) water depth is assumed
as zero since even if it is present, it will be at the toe of spillway portion
and not on non-overflow section of the dam.
If drainage gallery provides means of release of uplift pressure then, uplift
force gets reduced to one third of its value at the heel to the point where
pressure release arrangement is provided. This will be described later on
Design of Gravity Dams 123

when drainage gallery is described under the portion of practical profile


of dam. Hence, this effect is shown by means of dotted line in triangular
portion of the uplift diagram below the base. Effect of uplift force is to
reduce the weight of the dam and also to create displacing moment at the
toe of the dam.
In the calculation of forces for elementary or theoretical profile of
gravity dam, only these three forces i.e., W, P and U are considered and
hence they are known as basic forces.
Forces under portion B of the list are known as secondary since they
need to be considered at a later stage as check on the section of the
dam designed on basis of three basic forces. Secondary forces are now
described below.
B. Secondary Forces:
4. & 5. Wind and Wave Pressures: Wind with a velocity of 100 to 150 kmph
may cause exposed face of the dam to have additional horizontal
force either acting u/s to d/s or vice versa depending on direction of
wind velocity and its effect is to be resisted by self-weight of the dam,
which it is capable to do so for it is designed to resist water pressure
which is much more higher than wind pressure. But effect of wind is
also to generate waves on u/s side as it blows over reservoir surface
exposed to wind effect. This may cause an impact force at FRL level
to the body of the dam and also may create conditions of overflow
of water if sufficient free board (FB) is not provided. Hence, FB is
decided on the basis of wave height, hw, which is given by Moliters
formula:
h w = 0.032 VF, for (F > 32 km)
here, h w is in meter,
V is wind velocity in kmph,
F is fetch in km
Crest
FRL FB
hw
Pw
F
(3/8 hw Trough

DSL

Fig. 6.2: Wind and Wave pressure acting on a dam.


124 Irrigation Engineering and Hydraulic Structures

Fetch (F) is defined as maximum length of water surface at FRL level


from dam axis exposed to wind effect i.e., it is the length of the farthest
point on FRL contour from dam axis in km. If value of F is greater
than 32 km, above formula is applied but if it is less than 32 km, then,
h w = 0.032 VF + 0.763 − 0.271 4 F, for (F < 32 km)

h w = wave height,
FB = Free Board = 1.5 h or 3 m , whichever is higher.
FB is also decided as per IS 6512 of 1972 as,
FB = [1.33h + wind set-up] or 3 m,
where, wind set-up in m is given by,
V2F
62000 D
where, D = average depth of reservoir in m.
Now a days, top width of dam provided is always kept either equal to
one or two traffic lane i.e., 3 to 6m, which is sufficient to take care of
any impact load at FRL level
Wave Pr essure can be calculated by,
5
Pwave = ½(23.5h w ) h w
3
=19.62 h 2w KN / m length of dam, h w in m.
and acts at 3 / 8 h w from trough or sill level of wave, see Fig. 6.2

6. Silt Pressure: Over the reservoir portion known as dead storage, silt
may get accumulated in course of life span of the reservoir and hence its
pressure on u/s face of dam be considered as a check force in the design
of dam. This is taken as φ
 1 − sin φ 
Psilt = ½ (γs) (h1 )2  
 1 + sin φ 
where, γs = Submerged weight of silt
h1 = Depth of silt deposited
    f = Angel of internal friction of silt and is acting horizontally at
1 
 h1  from base.
3 
7. Ice Pressure: This is generally not required to be considered in India,
however as per provision of IS 6512 – 1972 its magnitude Pice = 245
KN/m² is taken as acting over contact area of dam face at FRL level.
Design of Gravity Dams 125

C. Periodic Forces: Periodic forces e.g., earthquake force and forces due to
cyclonic conditions may come in effect for a small interval of time in a
life span of a reservoir and hence they are considered as check forces for
the adopted design. However, sometimes earthquake force is required to be
considered if it is suspected that dam site is in seismic zone. As such, full
description of earthquake force is required which is given below. As regards
cyclonic conditions are concerned, wind velocity of 200 kmph if considered
in the design, it will be sufficient.
8. Earthquake Force: It can be defined as that force which structure
situated on earth’s crust experiences when some rupture or collision of
rock plates below earth’s crust has taken place suddenly. The point where
such a rupture or collision has occurred is defined as focus of earthquake
and its depth below earth’s crust is focal depth. Corresponding location
of focus on earth’s surface is epicentre as shown in Fig 6.3.
The energy released by rupture of rock plate travels towards earth’s
surface in the form of:
1. Primary or P waves, which are compression or longitudinal waves
2. Shear or transverse or S waves
3. Raleigh or love waves.

Dam
Epicenter
Earth surface or crust
Focal depth
Focus

Waves of acceleration

Plane of rupture or collision


of rock plates

Fig. 6.3: Earthquake force.

The energy E released at the focal area is quantified as magnitude M on a


logarithmic open ended scale, first introduced by Richter and hence known as
Richter scale representing earthquake intensity M and is defined as
M = log10 (Amplitude of P waves recorded in mm at 100 km from epicenter)
Hence, log10 (E) = 11.4 + 1.5 M
Here E = Energy released in ergs.
M = Richter’s Magnitude of earthquake force.
126 Irrigation Engineering and Hydraulic Structures

However, Engineer’s scale for calculation of earthquake force is not Richter


scale, but is based on adoption of a coefficient, denotes as a, and given by
acceleration due to earthquake
α =
acceleration due to gravity
Hence, horizontal acceleration that structures will have to face on account of
earthquake is given by (ag), where a is a fraction varying from 0 to 0.4. As per IS
1893 of 1984, are given in Table 6.1.

Table 6.1: Effect of Earthquake on Structure


α Values Effect of Earthquake on structure
0.01 to 0.05 Negligible
0.05 to 0.1 Tolerable
0.1 to 0.15 Damage to structure on small scale may take place
0.15 to 0.2 Structure may get damaged on large scale
0.3 and above Full destruction may occur

Hence, a adopted for checking of an adopted design of dam is generally taken


as 0.15
A. Effect of Earthquake Force on Self-weight
W
(a) Horizontal Force due to earthquake =   (α g)
 g 
= aW, where, W = Wt. of dam.
This force (aW) is an additional horizontal force due to earthquake and is
acting horizontally through centre of gravity of the dam section causing on
additional displacing moment about toe of the dam.
(b) Vertical Effect of (ag) on W:
(i) Force = (W) (1 + a)↓
acts vertically downward if (ag) is upward.
(ii) Force = W (1 – a)↓
acts vertically downward if (ag) is downward
For an upward acceleration (ag), the inertia force (aW) would be acting
downwards and hence it would result in an increase in the weight of the dam from
W to W (1 + a). On the other hand, if acceleration (ag) is acting downwards, the
inertia force (aW) will act upward and would result in reduction of the weight of
the dam for W to W (1–a).
Hence, for ag upward, wt.(new) = W (1 + a) and ag downward, wt.(new)
= w(1 – a) both new weights will act vertically downward through their centre of
gravity’s since they are weights. This effect of increase or decrease in weight is
apparent and is for interval of time over which earthquake waves influence lasts
at dam site.
Design of Gravity Dams 127

B. Effect on Water Pressure: Effect of earthquake on water pressure is


illustrated in Fig. 6.4.
FRL

y
Pe

Pe

Parabolic pressure diagram due to effect


of earthquake on water pressure

Fig. 6.4: Effect of earthquake force on water pressure

Additional intensity of pressure at depth y from FRL is given by Zanger in


1952 as,

pe = C α ( γ y)
 θ y  y y y 
here, C = 0.735    2 −  +  2− 
 90   h h h  
 h
If u / s face of dam is vertical

θ = 90°, and if y = h, then


h  h h h 
C = 0.735(1)   2 −  + 2 −  
 h  h h h  
= 0.735 × 2
i.e. , C = 1.47 for y = h and θ = 90°
if we assume α = 0.1,
p h = 1.47 (0.1) ( γ h)

Pe = Total pressure due to earthquake force upto depth y is 0.726 py (y) and
 4
acts at   y from base of pressure diagram.
 3π 
128 Irrigation Engineering and Hydraulic Structures

6.3 THEORETICAL PROFILE OF GRAVITY DAM


Theoretical profile of a gravity dam can be assumed as right angled triangle as it
has to face horizontal pressure of water, which is also a right angle triangle in shape
as shown in Fig. 6.5. The uplift pressure diagram is also triangular one and U and
W are coplanar forces.

FRL

d/s face of dam

h
Vertical
H W from top
P
B/3 R
River bed R Rv
h/3

D E PH
h B

(ch )
B/3

Fig. 6.5: Theoretical profile of a gravity dam

Hence, Rv = W – U, here Rv is resultant vertical force


Rv = W – U, but W = ½ BH (s γ)
and U = ½ B (c g h)

Hence, Rv = ½ B γ h (s–c) acts vertically downward at (B/3) from vertical face


PH = ½ γ h² , acts at (h/3) from base.
Taking moment about 2nd middle 3rd point i.e., E on base, (Fig. 6.5)

h  B
PH   = R v  
3 3
h  B
∴ ½ γ h²   = ½B γ h (s − c)  
3  3  (6.1)
∴ h² = B²(s − c)
h
∴B =
s−c
Design of Gravity Dams 129

h
The equation (6.1), i.e., B = , satisfies requirement of no over turning
s−c
moment about second middle third point. For stability of theoretical section of the
dam, safe design criteria can be listed as under:
(i) No Overturning
(ii) No Sliding
(iii) No Crushing and
(iv) No Tension
From equation (6.1) it can be said that criteria (i) and (iv) both get satisfied, since
resultant cuts the base such that it does not go out of first and second middle third
point. This is evident since if reservoir is empty, resultant W cuts first middle third
point i.e., D and if reservoir is full, resultant cuts second middle third point i.e., E
For satisfaction of condition number (ii), i.e., no sliding, let us consider
equilibrium of horizontal forces:
ΣH = µΣR v , here µ = coefficient of friction at the base
∴ PH = µ(R v )
       ∴ ½ γ h² = µ [½B γ h(s − c) ]  (6.2)
∴ h = µB(s − c)
h
∴B =
µ(s − c)

If µ = 0.7, s = 2.4 and c = 0.4,


h h
we se e that B = = from equation (6.2)
0.7(2) 1.4
h h h
      and also B = = =
1.4
, from equation (6.1),
2.4 − 0.4 2
h
hence B = also
s−c
satisfies condition of no sliding.

For satisfaction of condition (iii) i.e., ‘no crushing’ let us consider stresses that
may develop at foundation plan i.e., base.
By considering both direct and bending effect, we get normal stress as,

Rv  6e 
pn = 1 ± B  (6.3)
B  
where, e = eccentricity of R from midpoint
130 Irrigation Engineering and Hydraulic Structures

2 B  B B
=  B −  for R at E =  −  for R at D
3 2 3 2
B
i.e., e = ±
6
Substituting this in equation (6.3), we get
2R v
p n max = and  (6.4)
B
 p n min = 0
(6.5)

FRL

d/s face of dam


R
RV
h
Vertical
from top PH
P
H W i.e., B

River B/3 R
Ø

bed h/3
C
D E
Pr
Ø

h B
90

(ch ) Ø
B/3
B
Pn max
U

Fig. 6.6: Principal stress in a gravity dam

By substituting value of Rv = ½ B g h (s–c) in equation (6.4), we get

2(½B γ h(s − c))


p n max =
B (6.6)
p n max = γ h(s − c)

Now referring to Fig. 6.6, we consider equilibrium of small triangular prism


ABC at the toe of the base and get,

pi (AC × 1) cos φ = p n max (AB × 1)

where, pi = inclined stress acting normally to plane (AC × 1) = principal stress and
p n max = maximum normal stress at toe acting on plane (AB × 1)
Design of Gravity Dams 131

 AB  1
∴ pi = p n max  
 AC  cos φ
AB 1
= p n max sec ²φ, since = (6.7)
AC cos φ
∴ pi = p n max (1 + tan ²φ )

pH ½ γh²
But tan φ = =
PV ½B γ h(s − c)
h h
= but B = (6.8)
B(s − c), s − c)
1
∴ tan φ =
s−c
Substituting (6.8) in (6.7), we get,
 1 
pi = p n max 1 + , but p n max = γh(s − c)
 s − c 
 s − c + 1
∴ pi = γ h(s − c)   = γ h(s − c + 1)
 (s − c) 
pi
∴h = , but for no crushing σ ≥ pi (6.9)
γ (s − c + 1)
where, σ = permissible compressive stress of conerete
σ
∴h ≤
γ (s − c + 1)
If equation (6.9) is satisfied, there will not be crushing of the dam material.
Hence, all four criteria of stability of theoretical section of gravity dam get
satisfied if
h
B= and
s−c
σ
h≤
γ (s − c + 1)

6.4 HIGH AND LOW GRAVITY DAM


σ
Condition h ≤ is also known as limiting condition for height of low
γ (s − c + 1)
gravity dam i.e., if actual height, H of tan is greater than limiting height h, then it
σ
is known as High Gravity Dam and if it is equal to or less than h ≤ then
γ (s − c + 1)
it is known as Low Gravity Dam.
132 Irrigation Engineering and Hydraulic Structures

If a given dam is falling in the category of low gravity dam, then resultant
can be allowed to cut the base exactly on first and second middle third point and
stress conditions are satisfied in an optimised manner, however if given dam falls
in category of high gravity dam, resultant is required to be kept inside the middle
third such that maximum stress (pi) developed is lower than permissible stress of
material, s.
The portion of dam (H – h1) as shown in Fig 6.7 falls in category of high gravity
dam and should be designed on the basis of satisfaction of maximum stress criteria
i.e., pi ≤ γ H(s – c+1), and even if R cuts

Low gravity
h1 dam height
R exactly on E

H R

D E

Zone of high
R gravity condition

D E

Fig. 6.7: High and low gravity dam

the base little inside of 2nd middle third point, it should be tolerated. However
for low gravity dam design, R must cut 2nd point exactly and no gap should be
tolerated for as it will lead to wastage of concrete on a large scale. Therefore
the portion of the dam (H – h1) is designed by strip method in which stresses
are calculated for Rfull and Rempty conditions and are compared with Σ. In
actual design of dam top width is always provide and as such there may not be
any requirement of providing up stream slope. If for reservoir empty condition
the resultant cuts outside of 1st middle third point i.e., towards u/s face of
dam, then there is a need for providing u/s slope such that R cuts the first
middle third point when reservoir is empty. Thus, fixing u/s and d/s slope by
strip method is a matter of trial and error and is based on the requirement that
R must cut inside the middle third points as well as the maximum stresses
developed are within permissible limit. This will be clear from the following
illustrative example.

q SOLVED EXAMPLE
Example 6.1: Design a gravity dam for following data:
RL of river bed = 100.0 m
FRL = 197.0 m
Fetch = 40 km,
Design of Gravity Dams 133

wind velocity = 100 kmph.


Top width = 6m.
s = 2.4, c = 0.5, µ = 0.7, λ = 300 t/m2
Consider P, W and U only.
State whether it is high or low gravity dam and work out stresses.
Solution: First, let free board be calculated using Moliter's formula,
h w = 0.032 VF , since F > 32 km
here V = wind velocity in kmph =100
F = fetch in km = 40
6m

FB=3m
FRL
Zone l, no
slop on u/s
y1 and d/s

Zone ll, u/s


y2 vertical and d/s
slope assumed
as 0.7: 1 upto y2

Fig. 6.8: Design of gravity dam

∴h w = 0.032 100 x 40 = 2.00 m

∴ FB = 3m.
Total height = 97 + 3 =100 m.
σ
Limiting height for low gravity down = , neglecting c.
γ (s + 1)

300
= = 88m
1.0(2.4 + 1)
Given height 100 m > 88 m
Hence, high gravity dam.
Design upto height = 88 m as low gravity dam
and 100 – 88 = 12 m part be designed as high gravity dam.
134 Irrigation Engineering and Hydraulic Structures

Let dam height be divided into following


zones:
Zone I, u/s and d/s both faces vertical
Zone II, u/s vertical, d/s sloping
 one III, u/s sloping, d/s sloping upto
Z
low gravity height 88m
 one IV (100–88) = 12m part as high
Z
gravity type
Zone I:
Both u/s and d/s face vertical upto y1

W = 6 (3 +y1) 2.4 = 14.4 (3+ y1)

1 Fig. 6.9: Design of gravity dam, zone I.


P = (1) (y1 ) 2 = 0.5 y12 , here
2

γ = 1 tonne/m3

1
U = (0.5) (1) (6) (y1 ) =1.5 y1
2

Take moment about reference line 0-0:


y1
ΣM =14.4 (3 + y1 ) x 3 + 0.5 y12 x −1.5y1 x 2
3
=130 + 43 y1 + 0.16 y13 − 3 y1
ΣV = W − U =14.4 (3 + y1 ) −1.5 y1

= 43 + 14.4y1 −1.5 y1 = 43 + 13y1

ΣM
X f = 4m = ,
ΣV
where, X f indicates position of resultant from reference line 0-0 when reservoir is full

∴ 4(43 + 13y1 ) = 130 + 40y1 + 0.16y13


∴ 42 + 12y1 = 0.16y13
262.5 + 75y1 = y13
Try y1 = 10.0
1012 ≅ 1000
Hence, y = 10 m may be considered as depth upto which no d/s slope is required.
Design of Gravity Dams 135

Zone II:
6m
o
FB=3m
FRL

y1 = 10 m

W2
07
3m 1
y2 = 10 m

o
6m 4.9 m

Fig. 6.10: Design of gravity dam, zone -II

Let u/s face be vertical upto 20 m from top and d/s face be assumed as sloping at
0.7: 1 from y1

W1 = 6(3 + 10 + 7) 2.4 = 288 t M1 = W1 × 3 = 864 t m


W2 = ½(4.9) (7) (2.4) = 41 t M 2 = W2 × (6 + 1.6) = 312 t m
1 8
U = (.5)(1)(10.9)(17) = 46.32 t M v = 46x = 167 t m
2 3
1 17
P = (1)(17)2 = 144.5 t M p = 144.5x = 818 t m
2 3
Reservoir full condition :
ΣV = W1 + W2 − U = 283 ΣM = 1176 + 818 − 167 = 1827
ΣM 1827  2B 
∴ Xf = = = 6.45 ≤ = 2.76 
ΣV 283  3 

Reservoir empty condition:

ΣV = W1 + W2 = 329M ΣM = M1 + M 2 = 1176 t m
ΣM 1176 B 
∴ Xe = = 3.60 =  = 3.60 
ΣV 329 3 

and S V = W1+W2 – U = 283 t


136 Irrigation Engineering and Hydraulic Structures

Zone III:
o
6m

FRL 3m

10 m
PV1

7m
0.7
PV2 W1 1
3m
68 m
75 m
1 W2
0.1
W3

6.8 m 6m 52.5 m

B = 65.3 m

Fig. 6.11: Design of gravity dam, zone III

From the section onward, provide u/s slope of 0.1 : 1 and d/s slope of 0.7 : 1 upto 88 m
W1 = 6 × 88 × 2.4 = 1267 M1 = 1267 × 3 = 3801
1 52.5
W2 = (52.5)(75) × 2.4 = 4725 M 2 = 4725 × (6 + )
2 3
= 4725 × 23.5 = 111037
1 6.8
W3 = (6.8)(68) × 2.4 = 554 M 3 = 554 × = −1257
2 −3
6.7
PV1 = 17 × 6.8 × 1 = 115 M PV1 = 115 × = −393
2
1 6.8 × 2
PV2 = (6.8)(68) × 1 = 231 M PV2 = 231 × = −1048
2 3
1  13 
U = (.5)(1)(65.3)(85) M u = 1387.6 ×  − 6.8 
2 3 
= 1387.6 × (21.76 − 6.8)
= 1387 ×15 = −20758
1 85
P = (1)(85) 2 = 3612 M P = 3612 × = 102354.
2 3
Design of Gravity Dams 137

  SV = 1267 + 4725 + 554 + 115 + 231 + 1387 = 5505


SM = 102354 – 20758 – 393 – 1048 – 1257 + 111037 + 3801 = 193736

ΣM 193736  2B 
∴ Xf = = = 35.2 ≤ − 6.8  ≤ 37 m
ΣV 5505  3 

Reservoir empty condition:


ΣV = W1 + W2 +W3
= 1267 + 4725 + 554 = 6546 t
ΣM = M1 + M2 + M3
= 3810 + 111037 – 1257 = 113581 t m
113581 B 
∴ Xe = = 17.35 m ≥  − 6.8  ≥ 15 m
6546 3 
Zone IV:
o
6m

3m
FRL
13 m
10 m
PV1
Low gravity
zone 88 m 7m
0.7
h=97 m PV2 W1 1
68 m 3m
87 m
1 W2
0.1
W3

High gravity
zone

8m 6m 61 m

B = 75 m

Fig. 6.12: Design of gravity dam, zone IV.

Consider full height of dam 100 m, and continue same u/s and d/s slope. Check for
stresses and also Xf, and Xe, the positions of the resultant for the reservoir full and
empty conditions respectively:
138 Irrigation Engineering and Hydraulic Structures

W1 = 6 × 100 × 2.4 = 1440 t M1 = 1440 × 3 = 4320 tm


1 61
W2 = (61) (87) × 2.4 = 6368 t M 2 = 6368 × (6 + )
2 3
= 6368 × 26.3 = 167690 tm
1 8
W3 = (8) (80) × 2.4 = 768 t M 3 = 768 × = − 2048 tm
2 −3
PV1 = 17 × 8 × 1 = 136 t M PV1 = 136 × 4 = −544 tm
1 8× 2
PV2 = (8)(80) × 1 = 320 t M PV2 = 320 × = −1706 tm
2 3
1 B 
U= (0.5) (1) (75) (97) = 1818 t M u = 1818 ×  − 8
2 3 
= −30918 tm
1 97
P= (1) (97) 2 = 4704 t, M P = 4704 × = 152112 tm
2 3
ΣV = 1440 + 6368 + 768 + 136 + 320 − 1818 = 7214 t
ΣM = 4320 + 167690 − 2048 − 544 − 1706 − 30918 + 152112
= 288906 tm
Reservoir Full Condition :
ΣM 288906  2B 
∴ Xf = = = 40 ≤  − 8  ≤ 42 m
ΣV 7214  3 
Reservoir empty condition:
Σ V = W1 + W2 +W3
= 1440 + 6368 + 768 = 8576 t
Σ M = M 1 + M2 + M3
= 4320 + 167690 – 2048 = 169962 t-m
169962 B 
Xe = = 19.8 m ≥  − 8 = 17 m
8576 3 
Check for Stresses:
Reservoir full condition:
R  6e 
p n = v 1 ± 
B  B
B 
e = Xf −  − 8  R V = ΣV = 7214 m
2 
 75 
= 40 −  − 8  where, B = 75
 2 
= 40 − 29.5 = +10.5
Design of Gravity Dams 139

7214  6 × 10.5 
∴ p n max = 1+
75  75 
7214
= × 1.84 = 176.9 t/m²
75
pi = p n max sec ²φ = 176.9 (1 + tan ²φ)
= 176.9(1 + 0.49), here, tan φ = 0.7
= 263.5 t / m² < 300 t/m² hence safe.
Reservoir empty condition :
8576  6e 
pn = 1+ 
75  B
B 
e =  − 8  − X e = 29.5 − 19.8 = 10.7
 2 
8576  6 × 10.7 
p n max = 1+
75  75 
8576
= × 1.857 = 212 t/m²
75
tan φ = 0.1,
pi = p n max (1 + tan ²φ) = 212 (1 + 0.01)
= 212.2 t / m² < 300 t/m², hence safe.

Fig. 6.13: Gravity dam (section)

Hence, zone falling in high gravity category has got stresses within limit
∴ Design of dam is safe.
140 Irrigation Engineering and Hydraulic Structures

6.5 FACTOR OF SAFETY FOR DESIGN OF GRAVITY DAM


Factor of safety can be defined as ratio of stabilizing moment or force to displacing
moment or force, i.e.,
S.M. S.F.
F.S. = or
D.M. D.F.
There are in general three factors of safety to be considered in the design of gravity
dam.
1. Factor of safety against overturning is defined as the ratio of stabilizing
moment to overturning moment.
stabilizing moment
FS against overturning =
overturning moment
2 
(W − U)  B 
F.S. =  3  , (taking moment about toe)
1 
PH  H 
3 
1  2 
 2 BH s γ − ½ cγ HB  3 B B(s − c) 2B 2B² (s − c)
= = =
1  1  H² H²
 γ H²  H 
2  3 
H
But, B =
s−c
H² (s − c) 2
∴ FS = =2
s − c H²
Hence, theoretical value of F.S. against no overturning is 2
Therefore, for practical profile, it can be prescribed greater than 2.
2. Factor of safety against sliding is defined as the ratio of stabilizing force to
overturning force.
SF
FS against no sliding =
DF
µ(W − U)
= , take µ = 0.7
PH
0.7 [½BHsγ − ½cγHB]
=
½ γH²
0.7(s − c)B 0.7(s − c)H H
= = , sin ce B =
H H s−c s−c
= 0.7 s − c
= 0.7(1.4) , taking s = 2.4, c = 0.4, s − c = 1.4
= 0.98 ≅ 1.0
Design of Gravity Dams 141

Hence, theoretically F.S. against sliding should be equal to 1.0 or greater than 1.0.
For practical profile, shear–friction factor of safety is considered, which is greater
than 4.
µ (w − u) + Bq
Shear-friction F.S. =
PH
Here q = shear stress at base of dam
= p n max tan φ
2R V  PH 
= W − U
B  
2(W − U)PH 2PH
= =
B(W − U) B
∴ Bq = 2PH
µ (W − U) 2PH
∴ Shear Friction FS = +
PH PH
= 1.0 + 2.0 = 3.0

Hence, shear friction F.S. for practical profile is always greater than 3.

6.6 PRACTICAL PROFILE OF GRAVITY DAM


Theoretical profile is triangular one,
but practical profile cannot be so since
top width is always provided which
serves useful purpose of transportation
of machinery, maintenance gang and
also public transport facility. Also free
board has to be provided to prevent
over topping of water. Thus, theoretical
profile gets modified as shown in the
Fig. 6.14.
The shaded portion is addition
to theoretical section, and hence to
balance its effect, small triangular
portion on u/s side in the form of u/s
slope is added.
Also drainage and inspection
galleries and other openings are
provided in the dam section, hence,
following Fig. 6.15 illustrates the Fig. 6.14 Practical profile of gravity dam.
practical profile fully:
142 Irrigation Engineering and Hydraulic Structures

6.7 DRAINAGE GALLERY


A drainage gallery or inspection gallery is always provided in the body of dam
section and is normally below Low Water Level and above Dead Storage Level,
also its u/s vertical face may be along a vertical passing through mid point of top
width. A porous vertical drainage line checks the seeping water in body of dam
and makes it seep into the gallery. The gallery in turn throws it out of dam body
via cross-gallery or pumping arrangement. Air vent in the form of a 20 to 30 cm
diameter pipe connecting gallery to d/s face of dam above FRL is provided. This
provides ventilation in gallery as well as just behind sluice gates. A cut off and
dented bed at foundation level is provided to cut down seepage and to increase bed
friction. Grouting holes from gallery to foundation can also be provided if curtain
grouting is required to be carried out to decrease or cut down seepage.
Top width
4 m to 6m

F.B. F.B. = 1.5 hw or 3m;


F.R.L whicever is higher

Live storage d/s slope (0.7 ro 0.8 H to 1.0V)

Air vent for ventilation in gallery and sluices


L.W.L Porous drain line, meeting gallery
Supply sluice
Carry over store

D.S.L. Drainage or inspection gallery


Cross gallery
Dead storage
Secondary sluice

Cut-off
Dented bed to increase friction

Fig. 6.15: Practical profile of a gravity dam

6.8 JOINTS IN GRAVITY DAM


Joints can be defined as ‘designed crack’ for if joints are not provided, cracks will
develop in the body of the dam in an uncontrolled manner. Also, gravity dam is a
massive concrete work and hence lot of heat of hydration on account of setting of
cement will get released and hence there is a very high temperature gradient between
Design of Gravity Dams 143

the surface and in the interior of the dam section. This calls for proper release of heat
of hydration as well as providing suitable joints. The massive concrete work cannot be
taken up all at a time and hence joints are required to provide constructional facilities
also. Hence, joint is a must and serves following useful purposes.

1. Constructional facilities just as suitable blocks of concrete length-wise


and suitable vertical lift height-wise, here type of joints are key ways and
benching.
2. Temperature control by providing contraction joints between block to block
of concrete work length-wise, elaborate arrangement to stop leakage water
is required, which is illustrated in Fig. 6.16.

Joint gap to take care of expansion or contraction in concrete block due to heat
of hydration. Copper plate 10 mm thick, to prevent leakage. It is further protected
by asphalt poured in space on u/s and d/s of plate.
joint gap to take care of expansion or contraction
in concrete block due to heat of hydration
u/s

Unhealed Copper plate, 10 mm


Dam back Asphalt thick to prevent leakage
width in plan
It is further protected by
Asphalt poured in space
on u/s and d/s of plate
Block A Block B

d/s

2 to 3 mm

Fig. 6.16: Contraction joint

Depending upon the magnitude of concreting work, suitable number and type
of joints are adopted.

6.9 FOUNDATION TREATMENT


Most of gravity dam failures are due to poor foundation treatment. Following cases
of failures indicate that foundation treatment is very important and must be carried
out with utmost care.
(i) Francis dam in California, 62 m high and 210 m long. failed soon after
its construction. This was due to presence of conglomerate in one of the
abutments which was weakened after the reservoir got filled.
144 Irrigation Engineering and Hydraulic Structures

(ii) Malpasset Dam in France, 60 m high arch dam failed due to presence of clay
seam in the abutment rock.
(iii) Austin Dam on Colorado River in Texas failed in 1900 due to large cavities
got dissolved in its limestone foundation.

Hence, foundation treatment in the form of pressure grouting is extensively


carried out prior to dam construction. Pressure grouting may be:
1. Consolidation Grouting, 2. Curtain Grouting, 3. Contact Grouting
1. Consolidation Grouting: Entire Foundation bed of dam is consolidated by
pressure grouting. Shallow holes are drilled into foundation bed upto a depth
of 10 to 15 m and at 5 m to 20 m centre to centre. Cement slurry is forced at a
pressure of 3 to 4 kg/cm2 through these holes. This is known as low pressure
grouting technique.
2. Curtain Grouting: It forms a barrier or curtain against seepage through
foundation. Holes are drilled at 1.2 to 1.5 m c/c upto a depth of 30 to 40% of
u/s water head. Grouting is carried out under high pressure of 0.250 kg/cm2
where D = depth of grouting hole in m.

3. Contact Grouting: To develop a strong bond between abutment areas of


concrete dam with the adjacent rock, contact grouting is carried out. It
is also carried out to prevent seepage along the contact plane of concrete
and adjacent rock produced on account of shrinkage of concrete on
setting.

6.10 TYPES AND FUNCTION OF GALLERIES


Galleries are provided either parallel or normal to dam axis at various elevation,
and are interconnected by vertical shafts filled with lifts. Functions of galleries are
given below :

1. Drainage: Seepage from u/s water face of dam is interconnected by means


of vertical drain lines or hollow vertical shafts as shown in Fig 6.15. They
will join the drainage gallery and seeping water will be removed by pumping.
Drilling into foundation to cut down seepage through foundation is also
carried out from drainage galleries.
Design of Gravity Dams 145

2. Inspection: Study of structural behaviour of dam in post construction period


is takes up through galleries and actual stresses developed in the body of
the dam are measured by providing strain gauages and electronic measuring
devices.
3. Operation of Gates: Galleries provide access to mechanical equipments for
operation of gates.
4. Foundation Gallery: This is very near to foundation rock surface and
its usual size is 2.0 × 2.5 m. It provides space for drilling and grouting of
foundation holes from the floor of the gallery.
Thus, there are three main types of galleries (i) Drainage, (ii) Inspection and
(iii) Foundation.

q SOLVED EXAMPLES
Example 6.2: Find out maximum height of theoretical profile of a dam if
permissible value of compressible stress σ = 350 tonnes/m2. Take specific gravity
of concrete s = 2.4. Find height H and base width B if uplift coefficient c = 0.6.
Solution: Limiting height of dam is given by
σ 350
H= = = 125 m
γ (s − c + 1) (1)(2.4 − 0.6 + 1)

H 125 125
B= = = = 93 m.
and s-c 2.4 - 0.6 1.34

Example 6.3: Draw uplift pressure diagram for a dam holding 60 m depth of
water on u/s and top and base widths are 10m and 40 m respectively.
Consider uplift as effective on 60% of base area and tail water depth = 6m.
What will be the change in uplift pressure diagram if drainage gallery is provided
at 6m from its u/s face?

Solution:
Uplift at heel = 0.6 (60) = 36 m
Uplift at toe = 0.6 (6) = 3.6 m.
1
Uplift at gallery = 3.6 + (36 − 3.6) = 3.6 + 10.8 = 14.4 m
3
146 Irrigation Engineering and Hydraulic Structures

10 m
FRL

60 m

Tail water
Gallery
6m

6m

40 m

3.6 m

14.4 m

36 m

Fig. 6.17: Uplift pressure diagram.

Example 6.4: Determine pressure, line of action and wave height for a dam
120 m high having a fetch of 30 km and wind velocity 100 kmph. Find required
free board.

Solution: As fetch = 30 km < 32 km use formula


1/4
   h w = 0.032 FV + 0.763 - 0.27(F)
Design of Gravity Dams 147

\ h w = 0.032 32 ´100 + 0.763 - 0.27(30)1/4

   = 1.75 + 0.763 – 0.63


= 1.883 m (wave height)

Free Board (FB) = 1.5 hw


= 1.5 (1.883)
= 2.82 m ≃ 3 m.
Wave pressure acting on dam at FRL per meter length of dam
= 19.62 h2w KN/m
= 19.62 (1.883)2 = 69.56 kN/m = 6956 kg/m

and its line of action is at 0.375 hw above FRL

= 0.375 (1.883)
= 0.7 m above FRL.

Example 6.5: Find out base width of a theoretical profile of a gravity dam
holding 60 m, water for given following data:
s = 2.4, c = 0.5, µ = 0.7, γ = 1.0 tonne/m3. Also find principal and shear stresses
at the toe.

Solution:
H 60
B= = = 45.11 m.
µ(s − c) 0.7(2.4 − 0.5)

1 1
W= B H S γ = (45.11)(60)(2.4)(1) = 3248 tonnes.
2 2

Cg hB 1
U = Uplift = = (0.5)(1)(60)(45.11) = 676.65 tonnes.
2 2

∴Rν = W − U

= 3248 − 676.65 = 2571.35 tonnes.

1 1
P= g h 2 = (1)(60) 2 =1800 tonnes.
2 2
148 Irrigation Engineering and Hydraulic Structures

ΣM = Moment at toe
2  h
= Rv  B − P 
3  3
2   60 
= 2571 × 45.11 − 1800   = 77140 − 36000 = 41140 tonnes-meter
 3   3 

ΣM 41140
XF = = = 16.0 m.
Rv 2571

B 45.11
Eccentricity e = - Xf = - 16 = 6.55m < B / 6 hence safe.
2 2

p n = normal stress at toe


RV é 6e ù 2571 é 6(6.55) ù 2571(1.871)
= ê1 + ú = ê1 + ú=
B êë B úû 45.11 êë 45.11 úû 45.11
p n = 106.6 tonnes/m 2

∴ principal stress at toe = Pn sec 2 φ

4511
= p n (1 + tan 2 φ), here, tan φ = = 0.75
60
= p n (1.5625) = 106.6(1.5625)
= 166.56 t m 2 < 300 t m 2 hence safe.

τ = shear stress at toe


= p n tan φ = 106(0.75) = 80t/m 2 .

Example 6.6: A concrete gravity dam stores water to a depth of 70 m and has top
width 8m, fetch 40 km, wind velocity 150 kmph, upstream face vertical and d/s face
starts sloping at 10 m from top at 0.7 Horizontal to 1.0 vertical. If coefficient of uplift
c = 0.5. Specific gravity of concrete (s) = 2.4, friction factor (µ) = 0.7, compressive
stress (σ) = 300 t/m2, shear stress (q) = 140 t/m2.

Find:
(i) Whether dam is high or low gravity dam
(ii) Principal stresses at toe and heel for Rfull and Rempty.
(iii) Factor of safety
(a) against overturning
(b) against shear-friction.
Design of Gravity Dams 149

Assume location of drainage gallery at 8 m from vertical face and at 10 m


above the base of the dam, and tail water nil.
8m

FB = 4 m
10 m

h = 70 m

4m W1
H = 74 m

64 m

P 1 rh2
W2
2 30 m

23.3 m

10 m

45 m Toe
8m

4m
U3
U1
30 m
35 m
U2
1
30 (35) 12 m
2.7 m 3

Fig. 6.18 Forces acting on concrete gravity dam.

Solution: Ordinates of uplift at heel = cγh = 35 t/m2 ordinate at cross-section of


1
drainage gallery = (35) = 12 m (approx)
3
B = 8 + 0.7(64) = 8 + 44.8 = 8 + 45 = 53m.
Fetch, F = 40 km > 32 km
v = 150 kmph.
150 Irrigation Engineering and Hydraulic Structures

∴ h w = 0.032 ∨ F

= 0.032 150(140) = 2.48 m


∴ FB = Free board

= 1.5h w = 1.5(2.48) = 3.71  4m.


σ 300
(i) Limiting height of dam = = = 103m > 74m
r(s − c + 1) 2.9
∴ Given cross-section of dam is low gravity dam.
(ii) Forces acting per meter length of dam,
1. W1 = weight of rectangular portion (8 m × 74 m)
    = 2.4 × 8 × 74 = 1421 tonnes(t)
Acts at 4 m from u/s face and at 49 m form toe.
2. W2 = weight of triangular portion of base 49 and height 64 m.
1
= (45)(64)(2.4)
2
   
2
= 3456 t acting at (45) i.e. = 30 m from toe.
3

1 1
3. P = g h 2 = (1)(70) 2
2 2
1
   = 2450 tonnes acting horizontally at (70) = 23.3 m from base.
3
4. Uplift:
Ordiantes at heel = cγh = 0.5(1) (70) = 35 tonnes/m2
1
Ordinate at cross-section of drainage gallery = (35) = 12 t/m2 (approximately)
3
∴ u1 = 12 × 8 = 96 t at 4 m from vertical face of dam.
1 1
   u 2 = (8)(35 − 12) = (8)(23) = 92 t at 2.7 m from vertical face of dam.
2 2

1 æ 2 ö
u 3 = (12)(45) = 270 t ççat ´ 45 i.e., 30 m from toe÷÷.
2 çè 3 ø÷
(iii) Moment about toe: Moment due to weight is stabilizing and hence treated
as +ve (anticlockwise) and that due to P is –ve (clockwise) about toe.
Design of Gravity Dams 151

Sr. No. Force (t) Lever arm from toe (m) Moment (t – m)
1. W1 = 1421 49.0 1421 × 49 = +69629
2. W2 = 3456 30.0 3456 × 30 = +103680
3. P = 2450 23.3 (Fresh base) 2450 × 23.3 = –57085
4. Uplift
U1 = 96 49.0 96 × 49 = –4704
U2 = 92 50.3 92 × 50.3 = –4628
U3 = 270 30.0 270 × 30 = –8100
Σ M = +98792 t.m
W – U = Σ Rv = 1421 + 3456 – 96 – 92 – 270 = 4419 t
ΣM 98792
∴ Xf = =
Rv 4419
= 22.36 m from toe > B/6 i.e., 17.66 hence safe.
B 53
= = 26.5 From toe.
2 2
B
∴e = − X f = 26.5 – 22.36
2
B
= 4.14 m < (i.e., 8.33) Hence safe.
6
R

B B B
3 3 3 toe
Heel
B B
Second middle
2 2
X third point
First middle
third point

Fig. 6.19: Position of resultant in full condition

(R) Position of resultant when R full is at X f from toe and is within middle
third part of the base, hence safe
R  6σ 
Maximum normal stress = p n(max) = v 1 +
B  B 
4419  6(4.14) 
= 1+
53  53 
=122.46 t/m2 (Compressive at toe hence safe)
152 Irrigation Engineering and Hydraulic Structures

∴ p = Maximum principal stress at toe = p n(max) (1 + tan 2 φ)

= (122.46) (1 + (0.7)2); as tan φ = 0.7


= 183 t/m2 > 30 t/m2 hence safe
SM Moment due to weight
Factor of safety against overturning (RFull) = =
DM Moment due to P and U
69629 + 103680
=
(57085 + 8100 + 5633 + 4704)

173309
=
75522
= 2.92 >2, hence safe.
µ(W − U) + Bq or BT
Factor of safety against shear friction =
P
0.7(4399) + 53(140) 3079 + 7420
= =
2450 2450

= 4.25 > 4, hence safe.


(iv) Calculation of principal stress at heel when reservoir empty
R empty mean P = 0, V = 0 only weight present
ΣM
∴ Xe =
ΣW
173309 173309
= =
1421 + 3456 4877
= 35.55 m from toe > 35.32
∴ not safe but difference is only marginal and hence OK.
(R) Position of resultant when Rempty e = Xe − B / 2
= 35.53 – 26.5 = 9.03 m
Rv  6σ 
p n(heel) = 1 + B 
B  
4877  6(9)  4877 × 2
= 1+ =
53  53  53

 = 184 t/m2 < 300 t/m hence safe.


Design of Gravity Dams 153

pi = p n (1 + tan 2 f), as u/s face vertical tan φ = tan(0) = 0


= pn
= 184 t/m2 hence OK.
Overall result : Dam section is safe.
R
17.66 17.66 17.66

Heel Toe
35.32
X e = 35.53

Fig. 6.20: Position of resultant in empty condition

EXERCISES
1. Distinguish between “low and high” gravity dam. Derive an expression
for base width of elementary profile of a low gravity dam satisfying all the
criteria of the dam.
2. Describe forces acting on a gravity dam.
3. Distinguish between practical profile and theoretical profile of a gravity
dam.
4. Workout the stress at toe and heel of a gravity dam for reservoir full condition
for following data:
• Top width - 6m.
• U/s slope 0.1 H to 1 V and starts at 20 m from FRL.
• D/s slope 0.8 H to 1 V and starts at 10 m from top of the dam.
• Free Board – 3 m.
• Height of the dam – 53 m.
• Density of concrete – 24 KN/m3.
• Uplift Coefficient, C – 0.5
(Ans: At toe pn = 56.6 t/ m2, at heel pn = –1.0 t/ m2, pi = 93 t/m2)
5. Define factor of safety for gravity dam and work out it for dam data of
Question – 4, if m = 0.7.
(Ans: FS against overturning = 2.0 and FS against sliding = 1.38)
6. Explain various purposes of drainage gallery. Give sketch of a gallery and
air vent.
154 Irrigation Engineering and Hydraulic Structures

7. What are joints in concrete gravity dam? Describe them with sketches.
8. In a storage reservoir for irrigation, maximum fetch is 40 km. The wind
velocity over water spread area is 100 km/hr and wind set up of 0.2 m is
expected at FRL. Find minimum free board that should be provided above
FRL.
(Ans: FB – 3.23 m)
9. For an elementary gravity dam profile, the limiting height of the low dam
without considering uplift is ho, and with consideration of full uplift it is h1.
h
Find ratio of 1 . Take specific gravity of concrete as 2.5.
h0
h
(Ans: 1 = 1.4)
h0
10. Find the maximum theoretical height of concrete gravity dam, if permissible
compressive stress of concrete is 30 kg/cm2 and coefficient of uplift 0.6 and
specific gravity of concrete 2.4.
 (Ans: 107.14 m)
7
Spillways and Gates

7.1 GENERAL
Spillway is an overflowing part of the length of the dam. Generally, in accordance
with site condition, central part of the length of dam is chosen as overflowing section
of a dam and is known as spillway. Spillway is a vital part of dam and serves as
safety valve to reservoirs in the time of flood. Spillway should be designed for
adequate capacity such that design flood passes over it safely without causing any
damage on u/s and d/s sides. This is possible by providing sufficient net length of
spillway and head over spillway crest. Design flood is chosen as per importance of
the project from maximum flood value decided by hydrological analysis. Spillway
discharge is given by following formula:
Qdesign = C Le He 3/2
2
where, C = Cd 2g = 2 to 3 and is known as spillway coefficient and its usual
3
value is 2.2.
Le = Effective length of spillway
= Net length – 2 (N Kp + Ka) He
N = Number of piers
Kp = Pier contraction coefficient = 0.01
Ka = Abutment contraction coefficient = 0.1
V2
He = Effective head = H+ a
2g
H = Head over crest
= FRL – Crest Level
q
Va = Velocity of Approach =
(H + h)
q = Discharge per unit net length of spillway
Q
=
Le
h = Height of spillway crest from river bed.

© The Author(s) 2023 155


S. K. Ukarande, Irrigation Engineering and Hydraulic Structures,
https://doi.org/10.1007/978-3-031-33552-5_7
156 Irrigation Engineering and Hydraulic Structures

Types of Spillways:
(i) Ogee or Standard crested spillway
(ii) Side channel spillway
(iii) Shaft or Morning glory spillway.
(iv) Siphon Spillway
(v) Chute or Trough Spillway.
(vi) Tunnel or Conduit Spillway.

7.2 OGEE SPILLWAY


This is the most suitable types of spillway and is provided at all the major project sites
in India. It has two different types of curves at crest level, d/s curve and u/s curve.
F. R. L
a
Crest level Crest point, origin curve

U/s crest with R1 d/s curve upto (dy/dx) = V/H


R2 y
radii R1 and R2
b x
Curve meets d/s slope tangentially
H
h u/s
Face
vertical V

R
y2
River bed R
R=Bucket radius End sill
90o

Heel Toe

Fig. 7.1: Ogee spillway with bucket

Upstream curves has radius R1 from crest point upto a point at horizontal
distance (a) from crest point, and then after it has radius R2 upto a point where it
meets u/s vertical face of the Spillway. Values of a, b, R1 and R2 are given below.
(Note : The crest point is at a horizontal distance of b from u/s face and this
crest point is the origin of the d/s curves as well as u/s curves.)
a = 0.175H, H = Head over crest
b = 0.298H,
R1 = 0.5H,
R2 = 0.2H
D/S Curves follows WES equation, developed by US army corps of engineer,
which is given below:

x1.85 = 2 y (H)0.85 , H = Head over crest.


Spillways and Gates 157

In its general forms,

x n = KyH n −1

 1  n
∴y =  x
 KH n −1 

dy  1  n −1 V
∴ =  nx = = D/S slope of spillway body.
dx  KH n −1  H
d/s curve starts from origin at crest point and meets tangentially d/s face of
the spillway such that
dy V
=
dx H
Then after d/s slope (V:H) continues upto Spillway toe. At the toe suitable
dissipation works such as
1. Roller bucket
2. Ski-jump bucket
3. USBR basins
4. IS (Indian Standard) Basins, etc. may be provided to dissipate maximum
part of Kinetic Energy of flowing sheet of water, Energy dissipation can be
related to Froude Number at toe, F1.
V1 q Q
F1 = , here V1 = , and q =
gy1 y1 Le
Energy dissipation due to hydraulic jump can be worked out by knowing y1,
y2 and F1.
1 
( )
½ 
y 2 = y1  8F12 + 1 − 1
2  
( y2 − y1 )3
and energy head lost = m ; where y1 = pre-jump depth and
4y1y 2 y2 = post-jump depth

Table 7.1: Relationship of F1 and Hydraulic Jump


F1 Type of Jump and % energy dissipation
F1 < 1 Jump will not occur
F1 > 1, <1.7 Undulating jump, head loss 5%
F1 = 1.7 to 2.5 Weak jump, head loss 20%
F1 = 2.5 to 4.5 Oscillating jump, head loss 45%
F1 = 4.5 to 9.0 Stable jump, head loss 45 to 70%
F1 > 9 Strong jump head loss 85%
158 Irrigation Engineering and Hydraulic Structures

7.3 DESIGN OF STILLING BASINS


Energy dissipating works designed on the basis of formation of hydraulic jump are:
1. USBR basins and
2. Indian Standard Basins

y1 Baffle block End sill y2


Chute block
l.5 y1 l.2 y1
Toe of spillway
0.8 y1

L= apron lenght

Fig. 7.2: Hydraulic jump type stilling basin

Figure 7.2 shows a typical hydraulic jump type stilling basin developed by
USBR. Water from d/s face of spillway enters the apron with depth equal to y1
and F1 greater than one. Tail water depth, y2, should be sufficient for formation
of hydraulic jump and it should be equal to ½y1 ( 8F1 ² + 1 − 1)  and then only
energy dissipation as per values shown in Table 7.1 will take place. If tail water
depth is greater than y2, jump gets submerged and if it is less than y2, jump gets
swept off to d/s side. This creates problems of erosion of river bed. Hence, success
of stilling basin is dependent on formation of hydraulic jump within basin. USBR
type basin are classified into four categories i.e., type I, type II, type III and type IV
for different ranges of F1 values and arrangement of length of apron, chute block,
baffle block and end sill is made type wise such that hydraulic jump occurs in the
basin, giving expected energy dissipation. IS basin type I type II are also designed
for various ranges of F1 on similar lines. Types of basin and ranges of F1 values are
given in Table 7.2.

Table 7.2: Stilling Basin Types and Values of F & L/Y2

Types USBR Basin IS Basin Value of L/Y2

Type I 8.5 < F1 > 4.5 F1 ≤ 4.5 4 to 6


Type II 4.5 < F1 > 9.5 F1 > 4.5 2.3 to 2.8
Type III 9.0 < F1 > 17 3.85 to 4.3
Type IV F1 > 17 3.85 to 4.3
Spillways and Gates 159

Relationship of Jump Height Curve (JHC) and Tail Water Rating Curve (TWRC):
JHC can be plotted by calculating various values of Y2 for different values of q,
discharge per unit net length of spillway and TWRC can be plotted by obtaining
field observation for various values of Tail water depth for different values of q,
their possible combination are given in Fig. 7.3.

(1)
(1) JHC
JHC (1) (1)
(2) (2)
TWRC
TWRC (2) JHC = TWRC (2) JHC > TWRC

q q
(I) (ii)

JHC lower thanTWRC


for high ranges of q
(2) (1)
JHC (1) JHC (1) (2)
(1) (1)
JHC above TWRC
TWRC (2) JHC < TWRC TWRC (2) for low ranges of q

q q
(iii) (iv)

Fig. 7.3: (i) to (iv) showing possible combinations of JHC and TWRC against q

Case I where JHC and TWRC coincide for all ranges of q rarely occurs and
if so, it is ideal and a simple apron as energy dissipation work is sufficient, but
usually JHC may be found either above or below TWRC as shown in Fig. 7.3 (ii),
(iii) and (iv) and in that case provision of stilling basin as per different ranges of
Froude number F1 should be made. Last case where JHC and TWRC cross each
other is found generally occurring in most of spillway energy dissipation works.

7.4 DESIGN OF SKI-JUMP BUCKET AND ROLLER BUCKETS


If TWRC is always below JHC and river bed consists of sound rock formation,
then ski-jump bucket type energy dissipation work is economical and more suitable,
which is shown in Fig. 7.4
As shown in Fig 7.4 the bucket is turned upward such that its invert is always
lower then Tail Water Depth, and its lip makes on ∠θ with the horizontal at a height
from invert level equal to R (1 – cos θ).
160 Irrigation Engineering and Hydraulic Structures

F.R.L
Crest level H

Trajectory curve

h R = Radius of bucket
Solid rock bed of river
R y
River bed
level y1
TWD

Invert of bucket

Fig. 7.4: Ski-jump bucket

The lip angel is kept between 30° to 45° and water shoots from the lip in the form
of trajectory curve, with edge of lip as origin, is the equation of trajectory

y = x tan θ −
4KE1 cos 2 θ

where, K = air entertainment coefficient = 0.9 to 0.95

V12
E1 = y1 + = specific energy head at invert of the bucket
2g

V1 = 2g(H + h)
Q
q= = discharge per unit length of spillway
net length of spilling
q V
y1 = , F1 = 1
V1 gy1

y max = maximum height of trajectory curve


V12sin 2 θ
=K
2g
V12sin 2 θ
x = maximum horizontal range = K
g

The energy dissipation is due to air-entertainment and bubble formation and


impact of trajectory with rocky river bed having some tail water.
If ski-jump bucket is not suitable on account of non-availability of sound rock
at river bed, then solid bucket with one roller falling on incoming water and one
roller going out into river bed will be a suitable type of energy dissipating device.
Spillways and Gates 161

Here radius is same as that of ski-jump bucket, but invert of bucket is kept at river
bed level and lip angle is greater than 45°, preferably 60°, as shown in Fig. 7.5:
F.R.L.
H
Crest level

Roller falling on the incoming jet


H
dv/dx1
V
R Outgoing roller

River bed level


y1 R (1-cos ) TWD > JWC

Invert of bucket
R = Roller bucket radius

Fig. 7.5: Roller bucket


R = Roller Bucket Radius
  = 0.6 Hh
or R= 0.305 (10)p, where p = 1.9, Ven Te Chow’s coefficient
Roller bucket is preferred when TWD > JHC. Both these types of buckets remain
submerged in tail water and hence are also known as submerged energy dissipaters.
Radius of bucket R can also be worked out by using Varshney’s formula.
æRö
F1 = 0.09 ççç ÷÷÷ + 1.98,
è y1 ø÷
where F1 = Froude number at toe.
or Varshney and Bajaj formula,
F1 =13(R)1/4 - 19.5
(Note : Formulae for radius of bucket for Ski-jump
bucket and for Roller bucket are same)
Roller buckets were first used for Grand Coulee dam on river Colarodo (USA)
and have become quite popular since then. In India, a number of dams have used
roller buckets. Table 7.3 gives.
Table 7.3: Data used for Roller Bucket in India
Sr. No. Name of Dam q (m3/s/m) Radius of bucket in m Lip Angle
1 Nagarjun Sagar dam 92 21.3 34°
2 Shri Shailam 121.0 21.3 20°
3 Rana Pratap Sagar 61.2 16.76 40°
4 Hirakud 104.1 15.24 40°
162 Irrigation Engineering and Hydraulic Structures

7.5 SIPHON SPILLWAYS


7.5.1 Saddle Siphon
A spillway designed and constructed on the basis of siphonic principle is defined
as siphon spillway. It is automatic in action and has advantage of high rate of
discharge for same head H over crest level compared to ogee type spillway, but
this has a limit that difference between HFL and tail water level should not exceed
7.3 m otherwise its dome may suffer of cavitation and vibration problems. This is
proved in next section (7.6).
High head dam projects can not satisfy this condition and hence this type
of siphon spillways are suitable to low head medium sized projects. It consists
of two legs: Inlet leg kept on u/s side and its entrance should be below FRL
and discharging leg kept on d/s and its outlet should be always under tail water
level and should never be exposed to atmospheric air. On the summit at crest
level there hood or crown attached with deprimer pipe having its leg on u/s side
above FRL so that when water level rises in reservoir, deprimer gets locked,
no air can enter or leave from it. Water rushes through the inlet leg and starts
over flowing the crest and getting the air wiped out from discharging leg when
Deprimer Unit
HFL Hood or Crown
air vent Crest level
H FRL Spill way
X axis

Inlet Exit end always


between tail water
Tail water level
Y axis
0.3 H

Fig. 7.6: Siphon spillway (saddle type)

all the air is removed, siphonic action starts. To speed up the removal of air from
discharging leg several arrangement have been tried, such as provision of baby
siphon below entrance leg or a plate with spring action on d/s of crest so that water
is thrown to strike the upper surface of the discharging pipe and forcing the air to go
out. With all such attempts, this difficulty is not overcome totally, hence other type
of spillway known as volute siphon has evolved. When water level goes down on
u/s sides deprimer leg on u/s gets exposed to atmospheric air first and so air enters
the summit and siphonic act gets stopped.

7.5.2. Volute Siphon


A volute siphon spillway consists of a dome with a funnel of D1 diameter and throat
of D2 diameter such that volutes are provided over (D1 – D2)h space which is shown
in section and plan given in Fig. 7.7.
Spillways and Gates 163

Dome is resting on supports leaving a clear cut entry of water through the
Funnel. Deprimer ends above FRL and dome end below FRL, so that when water in
reservoir rises above FRL, water starts overflowing into the throat via volutes and
thereby a forced vortex flow is generated which will force out air through the exist,

air entry Volute


Deprimer
or exit

Above
D1
FRL D2
FRL
h
Dome
D2
Funnel
(inlet) D1
Below
FRL Support Plan of
to Dome Funnel

Exit
Throat
D2

90° Bend

Fig. 7.7: Siphon spillway (volute type)

the air which gets locked in dome excerts extra pressure on forced vortex motion
and so removal of air from exist is achieved in a very short interval of time, thus
this type is having a clear advantage over the saddle type of quick siphonic action.
However, it suffers off if any clogging in the annular space around funnel.
Discharge Formula:

Q = CA 2g(H − h f )

Here, H = maximum operating head, which is difference between water level


above FRL on u/s and level of centre line through the exist pipe on d/s, and its
limit is 7.3 m.
π
A = crest section area of exit pipe, A = (D 2 ) 2
4
hf = friction loss in the pipe and bend.
With this fall in water level in reservoir below FRL, air enters through deprimer
into the dome and siphonic action stops. Maintenance problems are heavy and so
this type is not preferred for high head and medium head projects.
164 Irrigation Engineering and Hydraulic Structures

7.5.3 Chute Spillway


Normally on earthen dam site, if construction of saddle weir on either side of the
dam is possible then a chute i.e., a channel on sloping side of weir is constructed
to pass the discharging water to join the river on d/s. This type of site is rarely
available and so in practice these types of spillway are very few at global level, and
is as shown Figure 7.8.

Main Dam Saddle Weir

Section

Weir
Dam
Side wall

Chute Spillway
Divide wall
(Channel on sloping
Plan
river → side of weirs)

Fig. 7.8: Chute spillway

7.5.4 Side Channel Spillways


This type differs marginally from chute type in having the side weir not in alignment
of the main dam but at either right angle or at some acute angle with the main dam
axis so that overflow takes place from reservoir rim and is guided by means of side
channel to meet the river on d/s, as shown in Fig. 7.9.
(1) Ogee weir on Guide wall
rim of reservoir

U/S Side Channel Spilway


(2) may be 90° or less
DAM

d/s
river

Fig. 7.9: Side channel spillway

The weir provided along the rim of reservoir at ∠θ may be designed as ogee or
straight drop type and side walls are provided to guide the water flow to join the river
on d/s. This is also not a common type due to non-availability of proper site for it.
Spillways and Gates 165

7.5.5 Shaft Spillway


It is similar to volute siphon type with a difference that it has a circular weir accommodating
on u/s protection of the main dam and water falling over the rim of the weir joins a
tunnel which might have been used as diversion tunnel to divert the river flow at the
time of construction of the main dam. This type is chosen only for this purpose that
diversion tunnel may be used even after completion of the construction of main dam.
Except the ogee spillway and siphon spillway all other types are rarely chosen
as sites for these types are rare and also their working needs continuous attention
and hence maintenance.

q SOLVED EXAMPLES
Example 7.1: Design a ski-jump bucket type energy dissipater for following
data:
(i) Discharge from spillway 9000 m3/s = Q
(ii) Height of spillway above bed h = 60 m
(iii) Design head H = 10 m
(iv) Net length of spillway = 90 m
(v) Rock is found at 2 m below river bed
(vi) Air resistance coefficient = K = 0.95
(vii) Value of p in Ven Te chow’s expression = 1.9
Assume TWD < JHC
Find (i) Radius of bucket
1. Length and height of trajectory
2. Depth of invert of bucket.

Solution:
R = 0.305(10) p = 0.305(10)1.9 = 24.22m
R by 0.6 Hh = 0.6 10 × 60 = 14.7 m
Adopt lip angle as 30°∴θ = 30°
V12 sin 2θ
Length of trajectory = K =114.8 m
g
V1 = 2g(H + h ) = 2 × 9.81 (60 + 10) = 37.0 m/s.
Q 9000
y1 = = = 2.7m
Length of spillway × V1 90 × 37
V1 37 37
F1 = = = = 7.18
gy1 9.81 × 2.7 5.15
166 Irrigation Engineering and Hydraulic Structures

Hence, adopt R = 21m

é F + 19.5 ù 4 æ 7.18 + 19.5 ÷ö4


R by Varshney and Bajaj = ê 1 ú = çç ÷ = 17.74 m
êë 13 úû çè 13 ø÷
or R by Varshney,
æRö
F1 = 0.09 çç ÷÷÷ + 1.98
çè y1 ÷ø
æRö
7.18 = 0.09 çç ÷÷÷ + 1.98
çè 2.7 ø
0.699
R= ´ 2.7 = 20.9 m
0.09

V12 sin 2 q
y max = Height of Trajectory = K
2g
(37) 2 (sin 30) 2
= 0.95 = 16.5 m.
2´ 9.81
V12 sin (2q ) (37) 2 (sin 60)
x max = K = 0.95 = 114.8 m.
g 9.81

Depth of invert of bucket from lip = R(1 − cos θ)


= 21(1 − cos 30) = 2.79m.

Example 7.2: The siphon spillway of a dam has 20 vents of size 2m × 1m. The
maximum water level is 160.0 m from river bed, and tail water Level is 154.0
m. from river bed. Spillway crest is at 158.0 m. Cd = 0.62. Find discharge of the
spillway. If same discharge is to pass over ogee spillway of length = 40 m, find
design head, take C = 2.2. Which type would you recommend? And why?

Solution:
Refer to Fig. 7.10, H = 160 – 154 = 6 m.
Q = Discharge through siphon spillway = Cd A 2gH

= 0.62 (20 × 2 × 1) 2g × 6.0 = 0.62 × 40 × 10.84 = 268.8 m3/s

Q over ogee = C Lnet H3/2


268.8 = 2.2 (40) H3/2
Spillways and Gates 167

Deprimer hood

M.W.L.

Lip of deprimer
F.R.L.

Entrance Lower leg


lip H=160-154=6 m

h Exit lip
below
TWL
T.W.L.

Fig. 7.10: Siphon spillway

1
M.W.L. O M.W.L.

F.R.L.

Entrance Lower leg


lip H

h Exit lip

3
O T.W.L.

Fig. 7.11: Siphon spillway

H = (3.05)2/3 = 2.11 m. = head required over crest is higher than that of


siphon spillway. Hence, siphon spillway is recommended.
168 Irrigation Engineering and Hydraulic Structures

7.6 LIMITING HEAD OF SIPHON SPILLWAY


Let point (1) be at MWL, (2) at throat of spillway and (3) at TWL. [Fig. 7.6 (i)]
Apply Bernoulli’s theorem between points 2 and 3,

p 2 V22 p V2
+ + H = a + 3 + h f 2 −3
γ 2g γ 2g
p 2 = pressure at throat of siphon spillway.
pa = atmospheric pressure at TWL
pa
∴ = 10.35m of water, absolute
γ
V2 = velocity at throat,
pv
Vapour Pressure = = 2.43 m of water, absolute
γ
V3 = velo city at exit lip

If pv = absolute vapour pressure at which vaporization occurs, then

æp p ö æ V 2 - V22 ö÷÷
H = çç a - v ÷÷÷ + ççç 3 ÷ + h f 2-3
çè γ γ ÷ø èç 2g ø÷÷

= (10.35 - 2.43) + h f 2-3 ,sin ce (V3 = V2 )

= 7.92 + h f , h f = (1 - C2v ) H , C v = 0.96,

\ h f = 0.08H, neglected

\ H £ 7.92m, limiting value, for no cavitation at throat

Apply Bernoulli 's Theorem between po int s (1) and (2):


pa p 2 V22
= +
γ γ 2g
æp p ö
V2 = 2g çç a - 2 ÷÷÷ = V3.
çè γ γ ø÷
Limiting value of Velocity at throat = 2g ´ 7.92 = 12.5 m/s.
= V3 = value of velocity at exit
Spillways and Gates 169

q SOLVED EXAMPLES
Example 7.3: Design an ogee spillway to pass a designed flood of 6000 m3/s
with a head over crest equal to 10 m. Height of spillway crest from rocky river
bed = 60 m. D/s slope is 1.0 V to 0.75 H. Crest coefficient = 2.2. What is the
probable area of catchment from which flood is coming? Inglis formula may be
used.
Solution:

Fig. 7.12: Ogee spillway with roller bucket

Step I: Refer Fig. 7.12

Q = CLe H3/2 − − − − > neglecting Velocity of Approach


6000 = 2.2 (Le )(10)3/2 .
6000
∴ Le = = 86.2m.
2.2 × 31.62
L net = Le + 2(NK p + K a )H
Assume 12 gates, hence N = no. of piers =11
Assume k p = 0.01 and K a = 0.1

∴ L net = 86.2 + 2(11 × 0.01 + 0.1)10


= 86.2 + 4.2 = 90.4 m
Assume pier width 3m
Hence, total length of spillway = 90.4 + 3×11 = 123.4 m.
170 Irrigation Engineering and Hydraulic Structures

Step II: For d/s crest curve,


WES equation is x1.85 = 2 y (H)0.85 (1)
Assume x and calculate y., draw curve from crest point as origin and upto x
0.85 0.85
dy
dy = 11 = 1.85(x)
1.85(x)0.85 = 1.85(x)
1.85(x)0.85
dx = = H 0.85 = (10)0.85
dx 0.750.75 H 0.85 (10)0.85
0.85
0.85 11 (10)
(10)0.85 5.10
\ xx 0.85 =
\ = 0.75 ´
´ 1.85 = = 5.10
0.75 1.85
1
1
xx = 0.85
\
\ = (5.1) = 6.72m.
=
(5.1) 0.85 6.72m.
i.e.
i.e., from origin, upto x = 6.72
, from origin, upto x = 6.72 m,
m,
crest
crest curve on d/s be drawn using equation
curve on d/s be drawn using equation (1).
(1).

Step III: Draw u/s curve by taking


R1 = 0 .5H = 0.5 × 10 = 5m
R2 = 0.2H = 0 .2 × 10 = 2m
a = 0.175 H = 1.75m
b = 0.29 H = 2.9m
Step IV: Bucket Radius = 0.6 H × h = 0.6 10 × 60 = 14.69 = 15m.

dy 1
tan ϕ = =
dx 0.75
∴ϕ = 53o
Adopt lip angle θ = 45o ,
and height of lip of bucket = R(1 − cos θ)
= 15(1 − 0.7) = 4.5m.
Step V: Catchment Area = A sq km.
By Inglis formula,

124A
Q=
A + 10.4

124A
∴ 6000 = ≈ 124 A
A + 10.4
2
 6000 
A=  = 2341 sq km.
 124 
Spillways and Gates 171

Example 7.4: Vent size of a siphon spillway is 3m × 1m, and there are 10 vents.
The head over crest is 2 m and crest height from bed is 10 m. Tail water depth
is 2 m above riverbed. Find discharge through siphon spillway such that vapour
pressure at throat is not less than 2.43 m absolute.

Solution:
R 2=Radius of crown
L=3 m

R2 1m

R1 r
R 1=Radius
H=MWL-TWL
Entrance of crest
=10 m
lip

TWD=2 m

Fig. 7.13: Siphon spillway

Refer Fig. 7.13.


Since crest and crown are curved, vortex motion through vent of spillway gets
established and hence:
V1R1 = V2R2 = v r = Constant,
V1 = velocity at crest level, v = velocity at strip level.
1
V2 velocity at crown level ∴ v = V1R1 But V1 = 2gH
r

Here, H permissible head =10.35 – 2.43 =7.92

∴ V1 = 2g × 7.92 = 12.5 m / sec.


dq = L × dr × v , here dq = discharge passing through strip of area (dr × L)
1
dq = L × dr × V1R1
r
R2
dr
  Q = Cv 12.5 (LR1) ∫ r
R1
172 Irrigation Engineering and Hydraulic Structures

R 
= C v 12.5 (LR1 ) log e  2 
 R1 
R 
= C v 12.5 (LR1 ) 2.3log10  2 
 R1 
2
= C v 12.5 (3 × 1) 2.3log10  
1
= 23.36 m3 /s per vent, (C v = 0.9)
There are 10 vents
∴ Q total = 23.36 × 10 = 233.6 m 2 /s
If Q = Cd A 2gH relationship is used.
then
Q = 0.62 (30) (12.5) = 232.5 m3 / sec.
Example 7.5: A reservoir catchment of 4500 sqkm. is in hilly area. Using Inglis
formula find the flood discharge. If an overflow ogee section with crest gates of
12m × 6.5 m is to be provided, find the number of gates. Neglect velocity of
approach and take coefficient C for discharge as 2.1.

Solution:
124 A 124(4500)
Q by Inglis formula = =
A + 10.4 4500 + 10.4
= 8309 Cumec.
Assuming H = 6m, Q = CLH3/2
(since gate size given is 12 m ´ 6.5 m , H can be assumed as 6 m.)
8309 = 2.1 L(6)3/2
8309
\L= = 269m. let , n = no. of gates, n ´12 = 269
2.1´14.7
269
\n= = 22.4 say 23
12
= Number of gates = 23

7.7 SPILLWAY CREST GATES


Gates on the crest of spillways provide additional storage capacity of reservoirs
known as “flood absorption capacity” of reservoirs or surcharge storage. In the
event of flood, the crest gates can be opened out as per requirement of reservoir
flood routing scheme, and flood water is allowed to pass over the spillway crest.
Spillways and Gates 173

Economics of gated and ungated spillway is based on comparison of


1. Cost of gates and cost of additional land that may get submerged.
2. Benefits that can be made available due to additional storage capacity of
reservoirs due to gates.

Type of Gates:
(i) Vertical lift type
(ii) Radial or Taintor gate
(iii) Drum gates
(iv) Stop log or needle gates
Hoisting arrangment

Hoisting wire
Skin plate

L-section to hold
M.W.L.
Arm of taintor gate rubber
Hard rubber seal

(Head on I-Beam embedded


crest) Trunnion
of gate in block A at crest
of spillway
Crest lavel Bracing
Block A
A

Fig. 7.14: Section of taintor gate

Taintor Gates: (Refer Fig. 7.14) Among all four different types, taintor
gate or radial gate is preferred for medium and large scale projects. Taintor
gates have advantage over vertical lift type as resultant water pressure line will
pass through trunion of the gate about which gate rotates, hence water pressure
will not exert any moment on the gate and opening and closing of gate will
be smooth compared to vertical lift type. Also hard rubber seals at bottom of
the skin plate and sides of skin plate will not allow leakage of water. The gate
consists of curved skin plate supported by I beams which are connected to arms
of the gate. The arm rotates about a trunion embedded in pier. The opening and
closing of gate is done mechanically by winch and hoist arrangement provided
at the top of the dam.
174 Irrigation Engineering and Hydraulic Structures

7.8 DESIGN OF AIR VENT FOR VERTICAL LIFT GATES


PROVIDED IN SUPPLY SLUICES OF THE DAM
Vertical lift gates are usually provided in the supply and scouring sluices of the
dam. Just at the back of these gates, air vent connection should be provided to keep
pressure at the back of gate atmospheric or at least not to allow negative pressure
to develop. If gates are partially opened or closed, the cavity formed at the back of
gate will develop negative pressure, which will lead to development of cavitation
and vibrations. Air vent properly designed will prevent this. Function of air vent is
to nullify vacuum effect just d/s of leaf gates of the sluices.
Capacity of air vent should be adequate and is fixed on the basis of volume
of air needed to nullify development of negative pressure. For example, at Mica
project in Canada, airflow of 200 cumec was required in air vent at a maximum
air velocity of 53 m/s to allow a discharge of 1000 cumec of water through sluice
opening in the dam. Empirical relationship between Qair and Qwater are given below:
(i) Qa = 400 C A P
here C = 0.7, A = air vent c/s area
P = differential pressure between atmospheric condition and that just at d/s
of gate leaf in kg / cm²
(ii) (a) Air vent area = 10% of sluice gate area
(b) Air vent velocity = 40 m/s to 90 m/s
(iii) As per USBR recommendations,
Air ventilation

Vertical lift gate


H

Fig. 7.15: Air vent


Air vent discharge = 25 % of Qwater
i.e., Qair = 0.25 Qwater
Spillways and Gates 175

iv)
(iv) Qair = α Q water
here α = 0.03(F1 − 1)1.06 for openings in which jump occurs.
and α = 0.0066(F1 − 1)1.4 for opening in which jump does not occur
F1 = Froude Number of Flow through opening of size (y × b)
V
∴ F1 = , Qw = v × y × b
gy

q SOLVED EXAMPLE

Example 7.6: Design an air vent for gate opening of size 2 × 3.3m. The gate
in front of the opening is vertical leaf type and is under a head of 42 m. The gate
bottom lip angel is 45° and maximum air demand occurs at 80% of vent opening.
Cd for gate lip = 0.81
Solution:

Air ventilation 47 cm diameter

Vertical lift gate

H = 42 m

y 1.6 m
2 m x 3.3 m

Fig. 7.16: Air vent

Refer Fig. 7.16,


Y = 80% (2 m), gate size 2 m × 3.3 m.
  = 1.6 m
176 Irrigation Engineering and Hydraulic Structures

V = Cd 2gH = 0.81 2g ´ 42 = 24m / s


V 24
F1 = = = 6.4
gy 9.81´1.6
a = 0.0066 (F1 - 1)1.4 = 0.07
Qa = aQ w = 0.07[24 ´1.6 ´ 3.3]
= 8.8 Q = AV
p
= d 2 (50), here 50 m / sec is air velocity through vent.
4
∴ Diameter of air vent d = 4.7 m.
Hence, provide air vent of diameter = 47 cm.

7.9 CAVITATIONAND VIBRATION PROBLEM IN HYDRAULIC


STRUCTURES
1. Ungated Spillway Crest: Spillway crests are designed as per WES
equation, xn = KH dn −1 (y) , When effective head on spillway crest, He, is less
than or higher than Hd (here Hd = design head) then the lower nappe of water
will compress against or will get lifted from crest profile respectively and
hence will exert positive pressures or negative pressures on crest profile
.The negative pressure if falls below vapour pressure (2.43 m absolute) then,
cavitation may take place and crest profile may get damaged. This can be
controlled by providing adequate ventilation. Tests have shown that sub
atmospheric pressure is of the order of 0.2 Hd where Hd = 0.75 He. Ungated
spillway crest is shown in Fig. 7.17.

Sub-atmospheric
pressure
on spillway
Hd

Fig. 7.17: Ungated spillway crest


Spillways and Gates 177

2. Gated Spillway Crests: Tests have shown that crests of gated spillway may
also develop negative pressure just downstream of gate and its magnitude
may be
Hd
if H d = 0.75 H e
10

3. Cavitations problems may develop in throat section of saddle siphon


spillways if pressure in throat falls below vapour pressure.
4. Cavitations and vibration may also occur just downstream of vertical leaf
gates provided in supply sluices or scouring sluices in the dam section. This
can be controlled by providing atmospheric air just d/s of gate by means of air
vents. Gates may also suffer of vibrations due to instability of flow passing
through them. Instability of flow may occur due to crest profile may not be
truly congruent to lower nappy of water sheet flowing over it. Velocity and
pressure fluctuation may get amplified due to instability of flow and hence
may lead to induced vibrations in the structure. This can be controlled by
adopting proper shape of crest profile of opening of gate and lining it with
material, which can withstand cavitation effect.

EXERCISES
1. State different types of spillways and describe in detail any one type.
2. State different types of gates and distinguish between vertical lift type and
radial type.
3. A spillway discharges 5400 cumecs through 6 spans of 10 m each and is a
part of concrete gravity dam of height 66 m and FB = 3m. Find crest level
of spillway and Catchment area Pier thickness is 0.75 m, slope of d/s glacis
beyond tangent point is 0.8 H to 1.0 V. Spillway coefficient = 2.5, Kp =
0.01, Ka = 0.1. Find the coordinates of lower nappe and also u/s curvature of
spillway surface. (Ans:. 50 m above river bed)
4. Sketch a stilling basin if Froude number at toe is 4.5
5. Sketch and write down specification for radial gate.
6. Design an air vent for a sluice of size 3.3 m × 2.2 m height, closed by vertical
lift gate operating under a head of 42 m. Maximum air demand occurs when
the gate is lifted to 78% of its height, take lip angle = 45o, Cd = 0.78, Cv =
0.95(Ans:. Diameter of air vent = 0.6 m.)
7. Explain the terms: Cavitations and vibration in hydraulic structure.
8. Siphon spillway of a gravity dam has 25 vents of size 3 m × 1 m height. The
MWL for spillway is 160 m for design head, crest of siphon at RL of158 m,
and RL of tail water = 154 m, Cd = 0.62, find discharging capacity of siphon
spillway.(Ans:. 25 × q = 25 × 20.1)
8
Arch and Buttress
Dams

8.1 INTRODUCTION
Gravity dams resist water pressure by self-weight and hence are known as
gravity dams. Similarly, Arch dams resist water pressure by arch action and
pass the load to valley sides, i.e., abutments. Basic requirement for arch dam
is therefore sound rock formation along valley line of the dam site. In our
country we have constructed only one arch dam so far because non-availability
of suitable site for this type of dam. Resistance to water pressure by arch action
and transferring the load to valley line at different levels reduces its thickness
considerably. Gravity dams need as high as 85% of its height as base width,
whereas arch dams need only 5% to 10% of height as its thickness, and hence
if arch dam is possible to construct, first preference should be given to it
compared to Gravity type.
The only arch dam in India, so far, is Iddiki dam situated near Cochin in Kerala
State. It is constructed in narrow V-shape gorge and is having double curvature. It
has a base width of 19.8 m and top width of 7.6 m over a height of 169 m. Its length
is 366m. It is one of the arch dams of world, which has thinnest section.

8.2 TYPES OF ARCH DAMS


There are four main types of arch dams depending upon mode of its design. They are:
(i) Constant radius arch dam
(ii) Constant angle arch dam
(iii) Variable radius and angel type
(iv) Double curvature type
The constant radius arch dam has upstream face in the form of a vertical cylinder
and hence radius of extrados remains constant from top arch to bottom arch, however
intrados of arches will have variable radius as the thickness of arch from top to
bottom increases (Fig. 8.1).

© The Author(s) 2023 178


S. K. Ukarande, Irrigation Engineering and Hydraulic Structures,
https://doi.org/10.1007/978-3-031-33552-5_8
Arch and Buttress Dams 179

180
160 V shape (elevation)

80 m 140
Ph

120

100

Extrados for all arches


B
Intrados at level 140 m
Intrados at level 120 m
Intrados at level 160 m Plan Intrados of bottom most
Intrados at level 180 m
arch, level 100 m
Fig. 8.1: Constant radius arch dam

8.3 DESIGN OF ARCH DAM BY THIN CYLINDER THEORY


The arches are designed by thin cylinder theory. The constant radius arch dam has
relatively large thickness of arch and it increases uniformly with depth, as can be
seen from the theoretical analysis based on thin cylinder theory: (Fig. 8.2)

U/s water pressure

Abutment
RV
r sin ( /2) r sin ( /2) R sin
RH ( /2) = Rv
/2

r
/2

Fig. 8.2: Constant angle arch dam


180 Irrigation Engineering and Hydraulic Structures

PH = pressure force on projected area of arch at depth h from water line.


θ
= γ h × 2r sin × 1.0, Here, radius of intrados of arch = r
2
ht. of arch = unity,
thickness of arch = t.
density of water = γ
Now PH = 2R v
here R v = Component of reaction R in direction
opposite to that of Water Pressure
θ
= R sin
2
 θ θ
∴ γ h  2r sin  = 2R sin
 2 2
∴ R = ( γ h ) r here, γ h = intensity of pressure at depth h,
= σ × ( t × 1) ( t × 1 = A = cross-section of arch )
( γh ) r
∴σ =
t
here, σ = permissible compressive stress of concrete.
 γ h
∴ t =  r
 σ
∴ t α r and also t α h
Cross -sectional area of arch = A = t × 1
∴A α r
∴ A = k r, K = constant
For unit height of arch,
For unit height of arch,
volume of
volume of concrete
concreteininarch,
arch,
V = rθ A
= Kθ r 2
 θ  span of Arch, L L
Sin   = =
2 2r 2r
L
∴r =
θ
2 sin  
2
2
 
 L 
∴V = Kθ  
 2sin θ 
 2
Arch and Buttress Dams 181

KL2 θ
=
4 θ
sin 2  
2

dV θ  θ θ
∴ = sin 2   − θ  sin cos  = 0
dθ 2  2 2
θ
∴ tan   = θ
2
∴θ = 133°34'
Hence, volume of concrete for a given arch of height unity and thickness t at
a depth h from FRL is least if its subtended angle θ = 133° 34´. In practice this
angle may vary from 120° to 140°.

8.3.1 Limitations of Thin Cylinder Theory


This cylinder theory of Arch Dam has following limitations:
1. It does not take into account “Rib Shortening.”
An arch ring of arch dam is a part of circular ring supported at its ends and
on account of uniform radial loads of water pressure, curved length of arch
tends to be shortened. This is known as “rib shortening”. It ends are rigid,
span of arch does not change and consequently arch ring is deformed, which
produces additional stresses in the arch. These are known as rib shortening
stresses and should be considered in the design.
2. Stress distribution along the thickness of arch cross sections are assumed to
be subject to similar stress distribution, whereas the formulae gives higher
stress at the intrados. This is true only near the abutments, while at the
crown, stresses are reversed. Thus, top arches and bottom arches, conditions
are quite opposite.
3. Thin cylinder theory is based on no yielding of abutments, but actually
abutments are not rigid they spread apart slightly under arch action producing
additional stresses in the arch.
4. Stresses produces due to temperature changes and shrinkages of concrete are
not considered in this cylinder theory.
5. This cylinder theory assumes that the valley is symenetrical, whereas
actually it is not so.
6. Earthquake force and silt pressure may not act radially.
On account of limitations as stated above, thin cylinder theory is adopted only
as preliminary design and final design is carried out by
1. Elastic arch theory or
2. Trial load analysis.
182 Irrigation Engineering and Hydraulic Structures

8.3.2. Elastic Arch Theory


This theory assumes transfer of entire load to abutments through arch actions only
and the entire dam consists of horizontal arch rings placed one over the other, with
their ends fixed into the abutments and
dS

B
H H
MA MB
B
f Arch A
Span o
V x

Fig. 8.3: Elastic arch theory


each ring is independent of the other. Rib shortening effect, temperature changes,
shrinkage and yielding of abutments are considered is this theory. Thin cylinder
theory takes into account effect of uniform radial pressure, but elastic theory considers
variable loads along the length of the arch. Refer Fig 8.3 for cross-section of Elastic
Arch.
At a cross section x, moment M is given by
x
M = µ + M A + (M B − M A ) + Hy
L
here, µ = Bending moment (BM) on a straight horizontal freely supported beam.
M = BM at any point x on arch
MA and MB = fixed end moments at A and B.
H = horizontal thrust at abutments,
V = vertical thrust at abutments, such that
R = resultant = H 2 + V 2
By bending stress formula,
f M f M
= and strain = =
y I E EI
Hence, E = modulus elasticity and
I = moment of inertia of the crest section under consideration
x x x x x
M µy ds (M A − M B ) xy ds y2
∴ ∫   y ds = ∫ ds + µ ∫ y ++ ∫ EI +
+ H ∫ EI ds = 0
0 
EI  0
EI 0
EI 2 0 0
Arch and Buttress Dams 183

For symmetrical arch , MA = MB

M µy yds y 2 ds
∴∫ yds = ∫ ds + M A ∫ + H∫ = 0 (8.1)
EI EI EI EI

M = MA + Hy (8.2)

ds y
MA ∫ + H ∫ ds = 0  (8.3)
EI EI

By solving equations (8.1), (8.2) and (8.3)


Arch can be designed as per elastic theory.

8.3.3 Trial Load Analysis


The assumption of thin cylinder arch theory and elastic theory that any horizontal
arch ring is independent of arch ring above and below it is not valid. Trial load
analysis developed by USBR overcomes this difficulty by assuming that the dam
is made up of two systems of elements:
(i) horizontal arches transmitting thrust to abutments and
(ii) vertical cantilever, fixed at foundation as shown in Fig 8.4. Loading is
assumed to be divided between them such that deflection at the common
element is same. Dam dimensions fixed by following either thin cylinder
theory or elastic arch theory are analysed by this method and modified
to arrive at final design. The trial load analysis method is widely used in
USA for arch dam designs and finite element method is applied to solve
the equations involved on high speed electronic computers. In Fig 8.4, AB
is horizontal arch, CD is vertical cantilever and is common element on AB
and CD. Deflection of this common element should be same by assuming
load distribution between arch and cantilever action. If it is not same, then
new load distribution is tried.
Arch Dam
C

D B
A

Fig. 8.4: Cross-section of arch dam


184 Irrigation Engineering and Hydraulic Structures

8.4 OTHER VARIETIES OF ARCH DAM


8.4.1 Constant Angle Arch Dam
In constant angle type of arch dam, subtended angle of arch remains constant but
radii of arch go on decreasing as depth of water from FRL goes on increasing
towards bottom of the valley. This results in double curvature of up stream face of
the dam. Hence, this type of dam does not have vertical cylindrical face at its up
stream. Constant angle arch dam has got benefit of least consumption of concrete
if the subtended angle adopted is 133°.

8.4.2 Variable Radius (or Angle) Arch Dam


The third variety of arch dam is a mix of the above two types and is known as
variable radius or variable angel type of arch dam. The arch thickness also increases
towards abutment, i.e., splayed arch, which is more suitable rather than arch of
constant thickness.

8.4.3 Doubly Curved Type Arch Dam


If design procedure adopted considers curvature in both horizontal as well as vertical
direction, the dam is known as Double curvature type Arch Dam. Here part load
of water pressure is resisted by arch action and remaining part by cantilever from
foundation. Hence, it can also be said to be Arch cum Gravity type of dam. USBR
has given guidelines for design of such dams. Besides thin cylinder theory, arch
dam can also be designed by Trial load method and method based on Theory of
shells. The design procedure adopts Finite Element Technique to arrive at solution
of differential equation involved. As this is beyond scope of undergraduate studies,
it is not included here.

8.5 BUTTRESS DAMS


Just as counter forted retaining walls retain earth mass on its up side, similarly,
if water is retained on upstream by water tight membrane supported by series of
counterforts or buttresses normal to dam axis, then the type of dam is known as
Buttress Dam (Fig. 8.5). In India, so far no dam of this type has been constructed.
Instead of flat slab as face slab, arches can be provided supported by buttresses,
and in that case it is known as ‘multiple arch dam’. (Fig. 8.6)
The design of buttress is similar to that of gravity dam, but as buttresses are
thinner in sections, they have to be checked against lateral buckling and hence struts
should be provided horizontally as well as vertically in between buttresses, (Fig. 8.4)
Arch and Buttress Dams 185

Road way

F. R. L.

Face slab u/s


Struts

Buttress
spacing

Fig. 8.5: Buttress dam plan Fig. 8.6: Multiple arch dam
in plan

q SOLVED EXAMPLES
Example 8.1: From the profile of a valley, following data is available:

Height from top of dam (m) Width of Valley (m) Remark


0 168 Top of dam
10 142
20 130
30 110
40 96
50 82 Bottom of dam
For a central angle of 133° 34´, find
(i) ri = radius to inner surface of arch
(ii) t = thickness of arch dam.
Solution: Assume net compressive strength = λ − p = σ

λ = 3600 KN / m 2
P = wh
w = 9.8 KN / m3 = weight of water per m3
h = height in m from top.

Draw c/s of the arch dam, Refer to Fig. 8.7.


186 Irrigation Engineering and Hydraulic Structures

168 m
0 Top
t
L
10 142 m

20 130 m

30 110 m
ri ri
0/2
40 96 m
0

50 Bottom
82 m

Fig. 8.7: Cross-section of arch dam


L 168 168
( ri )top = θ
= = = 91.6 m.
2Sin 2 (Sin 66.5) 2 × 0.917
2
142
σ = λ − p = λ − wh , (ri )10 = = 77.4 m.
1.834

σ10 = 3600 − 9.8 × 10 = 3502 KN / m² ,


 whr  9.8 × 10 × 77.4
t10 =   = = 2.16 m.
 σ 10 3502
130
( ri )20 = = 70.8, ( σ20 = 3600 − 9.8 × 20 ) = 3404
1.834
9.8 × 20 × 70.8
t 20 = = 4.07 m.
3404
110
( ri )30 = = 60m.
1.834
9.8 × 30 × 60 9.8 × 30 × 60
t 30 = = = 5.33m
3600 − (9.8 × 30) 3306
96
( ri )40 = = 52.3m.
1.834
9.8 × 40 × 52.3 20501.6
t 40 = = = 6.4m
3600 − (9.8 × 40) 3208
82
( ri )50 = = 44.7m.
1.834
9.8 × 50 × 44.7 21903
t 50 = = = 7.02m
3600 − (9.8 × 50) 3120
Pr ovide top width as 3 m.
Arch and Buttress Dams 187

ri = 77.4m re = 79. 56m


10 2.16m

ri = 70.8m re = 74. 8m
20 4m

ri = 60. 0m re = 65. 33m


30 5.33m

ri = 52.3m re = 58. 7m
40 6.4 m

ri = 44.7m re = 51. 72m


50 7. 02m

Fig. 8.8: Cross-section of arch dam

Example 8.2: Design and draw section of constant radius arch dam for the data
given below:
Depth from top (m) 0 10 20 30 40 50 60
Width of valley (m) 180 155 135 122 108 103 86

Consider, Top thickness of arch = 5.1 m , σ = 3600 kN/m2, angle subtended


at top = 150˚.
Solution: For constant radius arch dam, extrados radius remains constant, but
intrados radius goes on changing due to changes in arch thickness at various levels,
and subtended angle also changes with increases in depth from top of the dam.

At top, to = 5.1 m, h = 0
Span of arch Lo = 180 m
ri = re as t = 0 at top theoretically
θ  150 
L = 2re sin   = 2re sin  
2  2 
= 2 re (0.965)
L0 180
\ re = = = 93.26m
2(0.965) 2(0.965)
3
fc = net compressive strength = s - g h, g = 10kN / m = σ − 10h = 3600 – 10h

g hg
t=
fc
188 Irrigation Engineering and Hydraulic Structures

at top t = 5.1
10(h)(93.26)
5.1 =
3600 − 10(10)
3500 × 5.1
∴h = = 19.1 m
10 × 93.26
i.e., upto h = 19.1 m say 20 m, top width t = 5.1 m can be considered
ri = 93.26 – 5.1 = 88.16 m
from top to h = 20 m.
r (h)(10)
at h = 20, t 20 = i
3400
93.26(20)(10)
=
3400
= 5.5 m
ri at 20 = 93.26 – 5.5 = 87.76 m
and subtended ∠ θ will be given by
θ
L 20 = 135 = 2(87.7)sin  
2
θ 135
∴ sin   = = 0.769
2
  2(87.7)
θ
∴ = 50.3°
2
∴ θ = 100.6°
Similarly make calculations for L = 122m and h = 30 m and so on.
Results are tabulated below:

Given Calculated
Depth from width of r1 = re – t intrades subtended
top (m) valley cms t(m) radius r(m) angle
(1) (2) (3) (4) (5)
0 180 t0 = 5.1 88.1 150
10 155 t1 = 5.1 88.1 120
20 135 t2 = 5.5 87.76 100.6
30 122 t3 = 8.5 84.76 92
40 108 t4 = 11.6 81.66 82.79
50 103 t5 = 15 78.26 82.30
60 86 t6 = 18.6 74.66 70.33
Arch and Buttress Dams 189

Columns (1), (2) are given in data, Columns (3), (4), (5) are answers.
Cross-section is drawn below in Fig 8.9.

5.1 m
t
t1
t2
re
60 m t3
t4
t5
ri 0/2 = 75
o U/S
0/2 re t6

18.6

Fig. 8.9: Cross-section of arch dam

EXERCISES
1. Distinguish between
(i) Masonry dam and arch dam
(ii) Arch dam and buttress dam
(iii) Constant angle and constant radius arch dam.
2. Derive an expression for thickness of an arch dam of constant radius based
on thin cylinder theory. Also find condition for least volume of concrete for
arch dam.
3. Draw the cross-section of a constant radius arch dam for following data:
Height of dam 80 m, σ = Allowable stress in concrete = 300 T/m²,
Top width = 3 m, Radius of dam = 75 m.
4. Explain briefly the trial load analysis of the design of arch dam.
9
Earth Dams

9.1 EARTH DAM AND ITS COMPONENT PARTS


Earth dams are constructed of non-rigid materials i.e., earth, gravel, pebbles, rock
pieces etc. If reservoir site does not have suitable foundation for gravity dam, then
earth dams are preferred on such site. Earth dams are of two types:
1. Rolled Fill Earth Dam and
2. Hydraulic Fill Earth Dams
Only rolled filled type is more common in our country. Earth dams are also
known as per type of their section i.e., homogeneous earth dam, zoned earth dam
and diaphragm type earth dam. For same height of dams, earth dam’s initial cost of
construction is far less than that of gravity dam but maintenance is heavier. Earth
dam projects provide more employment potential and hence serve as socio-economic
solutions of country’s unemployment problems.
Component Parts and their functions (Fig. 9.1).

In general, following are the


components parts of zoned
earth dam: (12)
1. Core Top width
2. Cut-Off F.R.L F B (13)
3. Curtain Grouting (5)
(5) (11)
4. u/s and d/s Shells (10)
(4) (4)
5. Filters D.S.L. (1) (6)
6. Internal Drain (7)
7. Rock Toe (8) (2) (9)
8. U/s Blanket (3)
9. Cross Drain Semi impervious layer
10. Pitching or Rip Rap Impervious (rock) stratum
on u/s Slope
11. Downstream Slope Fig. 9.1: Earth dam section
with Sod
12. Top Width
13. Free Board

© The Author(s) 2023 190


S. K. Ukarande, Irrigation Engineering and Hydraulic Structures,
https://doi.org/10.1007/978-3-031-33552-5_9
Earth Dams 191

Functions of Component Parts


1. Core: This is central impervious component part of earth dam and is
responsible for cutting down seepage from u/s to d/s to a minimum. It
consists of 85% to 90% clay and about 15 % to 10% silt.
2. Cut-off: Core extended upto foundation plane is known as cut-off. It consists
of same material as core and has function of preventing seepage from u/s to
d/s in foundation plane. Cut off rests on impervious stratum but if it is at very
deep level then it is provided upto semi pervious stratum and from that level
to imperious stratum.
3. Curtain Grouting: Curtain grouting is provided such that there is no seepage
from u/s to d/s in the foundation soil.
4. U/s and d/s Shells: It consists of coarse material such as coarse sand,
pebbles, gravels etc. and it provides structural support to the core. The shell
material must have excellent drainage properties so that water drains out of
it immediately.
5. Filters: Filters are provided between core and shells both on u/s and d/s
side and also on u/s side of rock toe and all around internal and cross drains.
Filters have important function of not allowing migration of coarse material
to finer and finer to coarser, thus it is a buffer zone between two different
types of material and maintains their stability. Filters need special design
criteria so that migration of filter material is prevented.
6. Internal Drain: Internal drain is provided at toe of core such that whatever
seepage takes place through core is tapped here and is thrown out via cross
drains. They consist of rock pieces surrounded by filter.
7. Rock Toe: Rock toe is also a drain consisting of rock pieces with a filter
on u/s side. This is provided at toe of dam and hence is known as rock toe.
Its function is to throw out rain water that has seeped through d/s slope
into d/s shell and also it makes dam toe stronger against any failure due to
slunghening of d/s dam material.
8. U/s Blanket: U/s blanket is provided at u/s side horizontally between u/s
shell and foundation soil such that water from u/s shell does not seep into
foundation but in the event of sudden drawdown of water level in reservoir,
water in u/s shell is forced to rush out through open joints of the rip-rap
provided along u/s slope.
9. Rip-Rap: Rip–Rap also protects the u/s slope from getting washed out due
to wave impact on u/s slope at FRL. This is therefore provided upto level
higher than FRL on upper side and lower than DSL on lower side of u/s
slope.
10. Downstream Slope with Sod: Sod is provided along d/s slope to protect it
from getting washed away due to heavy down pour of raining water on d/s
side of dam. Sod is a jungle growth of grass along the d/s slope. D/s slope
has also got berms, so that maintenance party can work on d/s slope side and
maintains it nicely.
192 Irrigation Engineering and Hydraulic Structures

11. Top Width: Top width is provided to allow for at least two traffic lanes on
top of dam for free moment of heavy earth moving machinery. It is therefore
kept around 6 to 8 m.
12. Free Board: FB is provided such that waves due to exposed water surface
at FRL does not over top the dam and washing out of u/s slope at FRL is
prevented by proper maintenance at that level.
FB is either 1.5 (hw) or 10 ft. (3 m), whichever is higher, and hw is given by
hw = 0.032 VF m… (for F > 32 km)
here V = wind velocity in kmph
and F = fetch (in km), it is horizontal distance of farthest point on contour
at FRL level.
Thus, all above 13 component parts have specific function, but in a given
earth dam all of them need not be provided. Site conditions will decide
which one is required and which is to be dropped.
Arc with radius AF and centre at A to intersect
0.3 b CJ at C
F.R.L C
A I
Directrix

H P(X.Y) H
y
o
G 90 J
b F Horizontal
filter of L
x S at toe of
0.7b D
L dam

D = Floor distance between (A) and (F)

Fig 9.2: Seepage through homogeneous earthen dam section


with horizontal filter at the toe

9.2 ESTIMATION OF SEEPAGE THROUGH HOMOGENEOUS


EARTH DAM
Seepage calculations make use of flow net analysis. For this drawing of top most
pheratic line and potential line is essential.
This is explained below with refrence to different boundary conditions at the
base of the dam.

9.2.1 Homogeneous Earth Dam Section with Horizontal Filter at Toe


Let P be any point having coordinate (x, y) on the base parabola APG as shown in
Fig. 9.2 and F is a point on front edge of horizontal. filter and is regarded as origin
for coordinate (x, y) of P.
Earth Dams 193

As per property of base parabola, any point on the parabola is equidistant from
directrix and focus of the parabola.

∴ PG = PH
∴ x 2 + y2 = x + s
∴ x 2 + y 2 = (x + s)2 = x 2 + 2xs + s 2 

∴ y = 2xs + s 2 (9.1)
dy s (9.2)
∴ =
dx 2xs + s 2
Assuming various values for x, y can be worked out and parabola can be plotted.
Starting point of parabola A is required to be shifted to B, a point at FRL and u/s
slope intersection. Base parabola being Ψ line and u/s slope of earth dam when
water upto FRL is present, is Φ line hence both must intersect at 900 at B. A line
from B be drawn such that it meets base parabola APG tangentially at I and hence
B to I be regarded as ingress correction to the parabola. Then BIPG is the phreatic
or top seepage line above which dam section is dry and below it dam section is wet.

Let, q = Seepage per m length of dam at P.


= KAi , (as per Darcy 's Law )
dy
= K (y´1)
dx
s
 = K(y ´1)
2xs + s 2

=K ( 2xs + s 2 ) s

( 2xs + s 2 )
\ q = Ks (9.3)
= permeability of dam material ´ focal length of filter at base.

Hence, seepage q per m length of homogeneous dam section can be worked out.
When x = D and y = H, focal length s can be worked out from equation (9.1) as,

s =  D2 + H 2 − D 
 
Here, D = (length of base of dam) − 0.7b − L , (see Fig.9.2)

∴ q = K  D2 + H 2 − D 
(9.4)  
194 Irrigation Engineering and Hydraulic Structures

9.2.2 Homogeneous Earth Dam Section without Horizontal Filter


at Toe
Base parabola AJP cuts the d/s slope at J as shown in Fig. 9.3. However the phreatic
line must emerge out at K, a point on d/s face, such that it meets the d/s face
tangentially. KF is then known as discharge face making an angle α with horizontal
at the toe. The correction Δa by which the base parabola is to be shifted downwards,
∆a
is found by the value given by Casagrande for various values of a as given
(a + ∆a)
in Table 9.1 below:
0.3 b
A
B

a
H J
a
K

b F G
D

Fig. 9.3: Egress correction base parabola

Table 9.1: Egress Correction


Δa
α (a + Δa)
30° 0.36
60° 0.32
90° 0.26
120° 0.18
150° 0.10
180° 0.00

d d2 H2
Ifα < 30°, then a = − − (9.7)
cos α cos 2 α sin 2 α

If α > 30° but < 60°, then


a = H 2 + d 2 − d 2 − H 2 Cot 2 (α) (9.5)
∆a 180° − α
Alternatively if α < 90°, = (9.5A)
a + ∆a 400
∆a 180° − α (9.6)
but if α > 90° and < 180° then =
a + ∆a 300
Earth Dams 195

9.2.3 Phreatic Line for Zoned Earth Dam


A zoned earth dam consists of core and u/s and d/s shells as shown in Fig. 9.3(a).
Phreatic line will be a base parabola ABFG, which is horizontal from A to B and
then BFG is parabolic. It cuts the d/s core

F.R.L A B

F a

H Core a
Shell R

N P G
L d M

Fig. 9.3(a): Zoned earth dam


at F and FR = Δa, egress correction. As per correction Δa, parabola must meet
d/s of core tangentially at R. Seepage loss q can be worked by q = KS, where S =
H 2 + d 2 − d and K is permeability of core material. Here d is PM as shown in
Fig. 9.3(a).

9.3 DESIGN CONSIDERATIONS


Stability analysis of earth dam section is carried out for following cases:
1. d/s slope stability for steady state of seepage
2. u/s slope stability for sudden withdrawal of water from reservoir, a case known
as “Sudden Drawdown”
3. Stability of d/s and u/s slope against shear
4. Stability of foundation against shear.
Method of stability analysis is Swedish slip circle Method, which is explained
as under:
A
Centre of b b2
slip circle R
o
b cg
Radius of R
slip circle
Wedges h
of equal width b T= W sin (0)
end wedges width 0 N N= W cos (0)
=b1 at bottom W= (bh x 1)y5
=b2 at top b1 T

Fig. 9.4: Slip circle method


196 Irrigation Engineering and Hydraulic Structures

Chose a center as at A and draw an arc of radius R as shown in Fig. 9.4. Divide the
arc in wedges of equal width ‘b’, but width of bottom triangular wedge is b1 and
that of upper triangular wedge is b2
Take central wedge for analysis, let vertical representing weight of wedge W be
passed through c.g. of the wedge and let it be resolved along normal and tangential
components as N and T, then
Displacing or Driving Moment = M d = RΣT
Stabilizing or Resisting Moment = M s = R [ cL + ΣN tan φ]
Here L = length of arc of slip circle
2πRδ
=
360
φ = angle of repose of earth dam material.
= cohesive strength of soil.
δ = angle subtended by arc length at center A.
θ = angle between vertical through c.g. of the wedge and N.
Hence, Factor of Safety (FS) against sliding
MS cL + ΣN(tan φ)
= = ≥ 1.5
Md ΣT

Here, ΣT = ΣW sin θ = Σ (bhγ )sin θ

This can be obtained from T diagram, see Fig. 9.4(a)


h1 sin 01 h2 sin 01 etc.

b

Fig. 9.4(a): T diagram

T1 = ( γb)h1 sin θ1
T2 = ( γb)h 2 sin θ2 etc
γ = weight deinsity of dam material

and ΣN = ΣW cosθ , this can be obtained from N diagram see Fig. 9.4 (b)
= Σ(bhγ ) cos θ
= ( γb)Σ(h1 cos θ1 + h 2 cos θ2 etc)
T1 = ( γb)h1 sin θ1
T2 = ( γb)h 2 sin θ2 etc
γ = weight deinsity of dam material
Earth Dams 197

and ΣN = ΣW cosθ , this can be obtained from N diagram see Fig. 9.4 (b)
= Σ(bhγ ) cos θ
= ( γb)Σ(h1 cos θ1 + h 2 cos θ2 etc)
h1 cos 01 h2 cos 01
etc.

b

Fig. 9.4(b): N diagram


Line AE as shown in Fig. 9.5 can be obtained as under:
At top of dam, draw a line making ∠a2 = 35° with horizontal and along BC,
u/s slope of dam, draw a line at ∠a1 = 25°, for values of a1 and a2 see Table 9.2

Fig. 9.5: Locus of critical slip circle


These lines will intersect at A. Draw line DE at H (m) below base of dam upto
4.5 H from D giving a point E. Then join AE, center of critical slip circle shall
lie on this line AE. Chose 4 to 5 centers and calculate FS for each slip circle for
198 Irrigation Engineering and Hydraulic Structures

these centres; and then find least value of FS, from the plot of FS Vs. location
of center of slip circle. This should be > 1.5, adopt it for design. This is the
critical slip circle.

Table 9.2: Values of slope and angles

Slope of u/s or d/s face Directional Angle


α1 α2
1:1 28 27
1.5:1 26 35
2:1 25 35
3:1 25 35
4:1 25 35
5:1 25 35

9.3.1 Case of d/s Slope for Steady State Seepage


Let the arc of slip circle cut phreatic line at A and D as shown in Fig. 9.6. Take
points B, C along this line and measure the vertical height a1, a2 etc. Then at B,
draw perpendicular FB = γa1 and at C draw perpendicular CE = γa 2 , where
γ = weight density of water. Then join A, F, E, D which gives pore-pressure
diagram, find its area, which gives U = pore pressure of zone where phreatic line
and slip circle cut each other then
cL + tan φ Σ(N − U)
FS = ≥ 1.5
ΣT

F.R.L
A Are of slip
Phreatic line circle
a1
a1 B
a2
F
D
a2 C
E
Filter

Fig. 9.6: Steady state see page


Earth Dams 199

Here, N and T should be worked out by taking value of γdry for dam material and
considering wedge width b as explained earlier,
i.e., N = W cos θ = (bhγ dry ) cos θ
T = W sin θ = (bhγ dry )sin θ
Effect of uplift pressure present in zone below phreatic line is deducted from
N, i.e., tan ϕ Σ(N − U) is considered in above formula, here U = uplift pressure,
which is the area of uplift pressure diagram ABCDEF as explained with the help of
Fig. 9.6. Alternately, for dam material below phreatic line if submerged weight is
taken then no need of calculating U separately.

9.3.2 Stability of Foundation against Horizontal Shear (Fig. 9.7)

Fig. 9.7: Stability of foundation

 h 2 − h 22   φ
Total horizontal shear, P = γ m  1 tan 2  45 − 
 2   2
 

γ d (h1 − h 2 ) + γ f h 2
where, γ m =
h1
γ d = density of dam material
γ d = density of foundation material.
γ f = density of water
P
Average unit shear = Sa =
B× 1
200 Irrigation Engineering and Hydraulic Structures

Smax = maximum unit shear = 1.4 Sa and it occurs at L, which is at 0.4B from J as
shown in the Fig. 9.7.
Shear strength below toe, X
=S
= =C
S11 = C+ + ((g
gff hh 22 )) tan
tan f
f
Shear
Shear strength
strength below
below J, J, S
S22 = C+ ((g
= C+ gm h ) tan f
m h11 ) tan f
S
S1 +S
+S2
S=Average
S=Average shear
shear strength
strength = = 12 2
2
S
S3
\ F.S.=
\ F.S.= S3 1.5
1.5 for
for stability
stability of of foundation
foundation shear
shear
Saa
Sr
Sr3
FS
FS at
at maximum
maximum shear shear pointpoint =
= S 3 1.0
1.0
Smax
max
Here
Here Sr
Sr =
=C C++ ((g
gav hh )) tan
av
f
tan f
(ã h + ã h )
ã av = ( ã dd h + ã ff h 22 )
ã av = (h + hh 22 ))
(h +
hh =
= 0.6
0.6 (h - hh 2 ))
(h1 -1 2

9.3.3 Stability of Earth Dam against Horizontal Shear at the Base


of the Dam
W = overall weight of dam
W tan φ = Shear resistance

Fig. 9.8: Section of earth dam


γh 2
P = Total Horizontal water pressure =
2
W tan φ
∴ FS = ≥ 2.
P
0.4.B
d

Phreatic
line
d / s slope
h=0.6 H

Point of
h1 maximum
shear

Bd

Fig. 9.9: Section of earth dam (d/s slope)


Earth Dams 201

9.3.4 Stability of d/s Slope against Horizontal Shear at Base (refer


Fig. 9.9)
 γ av H 2  2  2 ϕ  γ w h12 
Earth
Earth Pr essure
Pressure =
= H =  =H = tan γ av
 H − 2 +
45 ϕ γ w h12 
Earth Pr essure 2   2 2 + 2 
tan 45 −
d 
 2 d   2
 
γ 1h1 + γ 2 (H − h )
γ h + γ 2 (H − h1 )
1
γ av = = 1 1
γ av H
H
Sd = averageS shear
= strength
average onstrength
d / s side
shearstrength at /base
Sd d= average shear onond/sdside
s side at base
at base
Hd
=
Bd × 1
Smax d = max shear on d/s at base = 1.4Sd
Rd = resisting shear force at d/s
= (Wed ) tan φ + (Bd × 1)
Rd
\ FSd/s = ³2
Hd
g h tan f + C
FS at point of maximum shear =
Smax d

9.3.5 Stability of u/s Slope against Horizontal Shear at Base

Fig. 9.10: Section of earth dam (u/s slope)

 γ H2  φ γ h2 
Total horizontal shear H u =  sat tan 2  45 −  + w 1 
 2  2 2 
Hu
Su =
Bu × 1
Smu = Smax u /s = 1.4Su
R u = Weu tan φ + cb u
Weu = total effective wt of dam below u / s slope
Ru
FSu = ≥2
Hu
h tan φ1 + C
FS at pt of max shear =
Smu
u
Smu = Smax u /s = 1.4Su
R u = Weu tan φ + cb u
202 Irrigation Engineering and Hydraulic Structures
Weu = total effective wt of dam below u / s slope
Ru
FSu = ≥2
Hu
h tan φ1 + C
FS at pt of max shear =
Smu
γ sat h1 tan φ + C
here tan φ1 =
γ sat h1

q SOLVED EXAMPLES
Example 9.1: A homogeneous earth dam has following data:
H = height of dam = 30m
Top width = 6m
u/s slope 4 : 1, d/s slope 3 : 1
Foundation is pervious to a depth of 20m
γ sat = 2160 Kgf / m3
γ sub = 1160 Kgf / m3
C = 1200 Kgf / m 2
φ = 24°
Check the stability of foundation against shear for u/s portion of dam.
0.4 bu

4:1 H = 30m 3:1

18m = 0.6 (30)

bu = 120 m 90 m
120 m

Point of maximum shear

Fig. 9.11: Earth dam section


Solution:  h 2 − h 22   ϕ
P = total horizontal. shear = γ sat  1 tan 2  45 − 
 2   2
 
γ h tan ϕ + C
tan φ1 = sat 1 = 0.4563
γ sat h1
h1 = 30 + 20 = 50 m and h 2 = 20 m
Earth Dams 203

\ j1 = 24.53°
\ P = 9.373´105 kgf
P 9.373´105
Sav = = = 7.8 ´103 kgf / m 2
bu 120

s max = 1.4Sav 10.9 × 103 kgf / m 2

s1 = C + γ f h 2 tan φ = 1200 + 1160 (20) tan 24o


= 11.5 × 103 kgf / m 2
s 2 =1200 + 1160 (50) tan 24o
= 27 × 103 kgf / m 2
s1 + s 2
s = = 19.2 × 103 kgf / m 2
2
s 19.2 × 103
FS = = = 2.4 〉 2 hence OK
sav 7.8 × 103
C + 1160(18 + 20) tan 24o
FSat pt.of max .shear = 1.9 〉1.5 hence OK
s max
Note : h at point of maximum shear = 0.6 (30) = 38 m

Example 9.2: A 24m high earth dam has slopes at u/s 4:1 and d/s 3:1, top width
6.4 m, check the stability of u/s portion against horizontal shear, seepage line is at
4.0 m below top of u/s shoulder.
γ sat = 21KN / m3
γ sub = 11.2 KN / m3
C = 35 KN / m 2
φ = 25°
F. R. L.
4m

4:1 3:1

20m H = 24m

bu = 96m

Fig. 9.12: Section of earth dam (stability of u/s past)


204 Irrigation Engineering and Hydraulic Structures

Solution:
1  ϕ 1
Hu = γ sat H 2 tan 2  45° −  + γ w h12 , here H = 24 m, and h1 = 24 − 4 = 20 m
2  2 2
1  24°  1
= (21) (24)2 tan 2  45° −  + (10) (20)
2
2  2  2
= 2540 + 2000
= 4540 KN
Hu 4540
Su = = = 47.3 KN / m 2
b u × 1 96 × 1
Smu = 1.4Su = 66.2 KN / m 2
1
R u = Weu tan φ + cb u , here, Weu = γ sub (96) (24) × 1
2
1
= (11.2) (96) (24) tan (25) + 35 × 96
2
= 6016 + 3360 = 9376 KN
R u 9376
FSu = = = 2.06 > 2 , hence OK
H u 4540
γ sub h tan φ + C
FS at point of maximum shear =
Smu
(11.2) (0.6 × 24) tan 25° + 35
=
66.2
75.2 + 35
= = 1.66 >1.0 , hence OK
66.2

Example 9.3: A homogeneous earth dam section has top width 6m, u/s slope 3.5
: 1, d/s slope 3 : 1, height = 75 m, FB = 3 m.
Properties of soil of earth dam:
Specific gravity, G = 2.6, void ratio = 0.7
Dam section consists of 55% saturated zone
35% wet zone
and 10% dry zone
For wet zone assume average degree of saturation = 0.5
At base of dam, C = 40 KN / m², φ = 28°.
Check the overall stability of dam section.
Earth Dams 205

Solution:
F B 3m

3.5:1 75m 3:1

bu = 262.5m 6m bd = 225.5m

B = 493.5m

Fig. 9.13: Section of earth dam (overall stability)


Note: (This means finding FS against horizontal shear at the base of dam).
A = Total area of dam section
1
= ½ (262.5) 75 + 6 × 75 + (225) (75)
2
= 9844 + 450 + 8437 = 18731 sq.m.
G = 2.6, η = 0.7
γ (G + η) 1000(2.6 + 0.7)
6, η = 0.7 ∴ γ sat = w = γ w (G += 1.94
η) ×1000(2.6
1000 = 1940 kgf/m3
+ 0.7)
G = 2.6, η(1=+0.7
η) ∴ γ sat(1.7)
= = = 1.94 × 1000 = 1940 kgf/m3
(1 + η) (1.7)
1940 + 940
γ sub = , [ hereit is divided
1940 + by9402 as degree of saturation given is 50 %]
2 γ sub = , [ hereit is divided by 2 as degree of saturation give
3 2
=1440 kgf/m
=1440 kgf/m3
= 0.55(18731) (1940) = 199.8 × 105 kgf = 200 × 105 kgf , say
Wsat = 0.55(18731) (1940) = 199.8 × 105 kgf = 200 × 105 kgf , say
= 0.35(18731) (1440 = 94 × 105
Wwet = 0.35(18731) (1440 = 94 × 105
Wdry = 0.1(18731) (2.6 × 1000) = 2600
= 48.7 × 105 kgf = 48 × 105 kgf , say
Wtotal = (200 + 94 + 48) × 105
= 340 × 105 kgf , (say)

∴ W tan ϕ = 340 × 105 (tan 28°) = 180 × 105 kgf


∴ Shear resistance at base = W tan φ + C(B × 1)
R = 180 × 105 + 4000 × 493.5 = 180 × 105 + 20 × 105 = 200 × 105
Horizontal Force due to water pressure:
1 1
P = γ w h 2 = (1000) (72) 2 = 26 × 105 kgf
2 2
206 Irrigation Engineering and Hydraulic Structures

R 200 × 105
FS against shear at base = = = 7.69 > 1.3 hence
P 26 × 105
F B 4m

3.1 40m 2.5.1

40m

120m 6m
100m

226m

Fig. 9.14: Section of earth dam (FS)

Example 9.4: Determine seepage loss through earth dam having height = 40 m,
FB = 4 m, u/s slope 3:1, d/s slope 2.5:1, horizontal filter length at base = 40 m.,
Top width 6m., K = 25 m / year.
Solution:

S = Focal Length = H 2 + D2 − D ,
here D = Base width – 0.7b – Horizontal filter length, = 226 – 75.6 – 40,
[where, b = 36 × 3 = 108m = horizontal projection of u/s slope at FRL, and
∴ 0.7b = 0.7 × 108 = 75.6m]

∴S = ( 40 − 4)2 + (110.4)2 − 110.4 = 116.4 − 110.4 = 6 m.


 25 
q = KS =  (6) = 48 × 10 −7 m3 / sec/ m length of dam
 365 × 24 × 3600 

Fig. 9.15: Earth dam (seepage loss through dam)


Earth Dams 207

Example 9.5: Check the stability of d/s portion of earth dam against horizontal
shear at the base of the dam. Height of dam 24m., top width 6m, u/s slope 4:1, d/s
slope 3:1, seepage line at 6 m below d/s top shoulder, area of d/s portion below
seepage line 550 m².
Solution:
γ sat = 19.62 KN/m3 or 2000 Kgf/m3
γ sub = 1160 Kgf/m3 , φ = 26°

6m

6m

3.1
24m
18m

bd = 72 m

Fig. 9.16: Section of earth dam (d/s part stability)

C = 4000 Kgf/m²
 γ H2  φ γ h2 
H d =  av tan 2  45 −  + w 1 
 2  2 2 
But
γ h + γ sat (H − h1 )
γ av = sub 1
H
1160(18) + 2000(6)
= = 1370 kgf / m3
24
1  26  1
∴ Hd = (1370) (18) 2 tan 2  45 −  + (1000) (18) 2
2  2 2
= 86556 + 162000 = 0.86 × 105 + 1.6 × 105
= 2.46 × 105 kgf
R d = Resisting force = Wed tan φ + cbd

1
Area of d/s portion of dam = (72) (24) = 864 m 2
2
∴ Area above seepage line = 864 – 550 = 314 m2
208 Irrigation Engineering and Hydraulic Structures

∴ Wed = 550 × 1160 + 314 × 2000


= 6.38 × 105 + 6.28 × 105 = 12.66 × 105 Kg(f )
∴ R d = 12.66 × 105 tan 26 + 4000(72)
= 6.17 × 105 + 2.88 × 105 = 9.05 × 105
R d 9.05 × 105
∴ ( F.S.)d = = = 3.67 > 2,
H d 2.46 × 105
H d 2.46 × 105
d/s average shear stress = Sd = =
bd 72
= 3.4 × 103 kgf /m 2
d/s m ax . shear = Smd = 1.4Sdav
= 1.4 × 3.4 × 103 = 4.76 × 103 kgf/m²
SR = Resisting shear strength = γ sub h tan ϕ + C
where h = 0.6 H = 0.6 × 24 = 14.4 m
∴ SR = 1160(14.4) tan 26° + 4000
= 8147 + 4000 = 12147
= 12.147 × 103 kgf/m²
12.147 × 103
FS(d)max shear = = 2.55 > 1.5
4.76 × 103

Example 9.6: In order to find factor of safety of d/s slope during steady state
seepage, a scale of 1 cm = 10 m was adopted for drawing homogeneous dam
section and following results were obtained on a slip circle:
N diagram = 16.4sq.cm.
T diagram = 7.45 sq.cm.
U diagram = 3.15 sq.cm
ϕ = 30°, C = 4000 kgf / m 2
γ soil = 2100 kgf / m3 , γ w = 1000 kgf / m3
Solution:
L = Length of trial arc = 12 cm.
Find F.S. of the d / s slope
CL + Σ(N − U) tan φ
F.S. =
ΣT
Earth Dams 209

Σ(N − U) = 16.4 × (10) 2 × 2100 − 3.15(10) 2 × 1000


= 31.35 × 105 kgf .

4000(12 × 10) + 31.35 × 105 tan 30°


∴ FS =
7.45 × 102 × 2100
4.8 × 105 + 18.1 × 105
=
15.6 × 105
22.9
= = 1.468
15.6
= 1.5 say (just safe)

Example 9.7: Calculate seepage through earth dam having anisotropic


permeability, for the data given below:

Fig. 9.17: Section of earth dam (anisotropic permeability)

Height of dam = 60m FB = 2.5m.


u/s slope 2.75 : 1 Topwidth = 8m.
d/s slope 2.5 : 1 Filter length = 120m.

K x = coeff . of permeability in x direction = 4 × 107 m / s.


K y = coeff . of permeability in y direction = 1 × 107 m / s.

Solution: As it is a case of anisotropic permeability, first step is to transform the


original section by transformation parameter,
Xt = Xo K y / K x
x0
= X o 1 × 10−7 / 4 × 10−7 =
2
with respect to transformed section,
210 Irrigation Engineering and Hydraulic Structures

D = 46.0 and h = 60 – 2.5 = 57.5


K = K x K y = 4 × 10 −7 × 1 × 10 −7 = 2 × 10 −7 m/s

S = (46) 2 + (57.5) 2 − 46 = 73.6 − 46 = 27.6


q = KS
= 2 × 10 −7 × 27.6
= 55.26 × 10−7 m3 /s/m
Example 9.8: When the foundation consists of pervious stratum and extends
over large depth yf, a horizontal u/s ­impervious blanket is provided to reduce the
seepage through foundation. The thickness (yb) and length (x) of such blanket
depend on permeability of the blanket material, which is usually between 0.6 to
3.0 m per day.

F.R.L.

H
Shell Core Shell

yb U/S
X Xd
Yf
Pervious stratum of thickness

Fig. 9.18: Design of u/s blanket

Solution:
Let Kf = permeability of foundation soil
  Kb = permeability of blanket material.
Then value of constant for u/s blanket is given by

Kb 2
a= and hence length of the blanket x =
K f Yb Yf a
e2ax − 1
X r = equivalent resistance =
a(e2ax + 1)

 Xr 
h o = head dissipated through blanket =  (H)
 X r + x d 
 Xr 
here,  = percentage reduction in seepage due to blanket
 X r + x d 
Earth Dams 211

Example 9.9: Design u/s blanket for earth dam section shown in the figure below.
Blanket thickness = 1.5 m, Xd = 51 m.
6m

H= 12m 2:1 15m 2:1

1.5m
X 51m
Yf = 10m
Pervious stratum of thickness

Fig. 9.19: u/s blanket


Solution:
Kb = 0.08 m/day Kf = 67 m/day.

Kb 0.08 1
a= = =
K f Yb Yf 67 × 1.5 × 10 112

2
Length of blanket = x = = 112 2 = 159 m.
a
e2ax − 1
Xr = = 99
a(e2ax + 1)

h o = head dissipated through blanket


Xr
= (H)
X r + Xd
99
= (12) = 0.66 × 12 = 7.9m and
99 + 51
% reduction in seepage due to blanket = 0.66 × 100 = 66%

Example 9.10: Find out factor of safety for the slip circle arc shown in the
following figure for sudden draw-down condition of u/s slope, given the following
data:
γ sat = 2.10 KN / m3
γ sub = 1.10 KN / m3
C = 2.45 KN / m 2
ϕ = 35°
212 Irrigation Engineering and Hydraulic Structures

∠θ made between N and W, arc length, and area of different slices 1 to 7 are given
in first four columns of the table given below

R= Radius of slip circle

35o
R
1
2
3 h
b
4 Slip surface for
6 5 u/s slope of the
7 O
o N
dam, 2H: 1V
w 90
T

Fig. 9.20: Slip circle

Slice Arc Area T component N component


No. θ° Length of slice W cos θ
(m) W=bhγsat Wsin q W=bhγsat KN/m
(1) (2) (3 ) (4) KN/m (5) KN/m (6) (7) (8)
1 54.5 6.7 12.26 25.74 20.9 13.48 7.8

2 41.0 3.8 19.51 40.97 26.8 21.46 16.2


3 31.0 3.5 21.37 44.87 23.1 23.50 20.1
4 22.0 3.35 20.90 43.89 16.4 23.00 21.3
5 13.0 3.05 19.97 41.93 9.4 22.00 21.4
6 5.00 3.05 16.72 35.11 3.0 18.40 18.3
7 -3.5 3.05 12.08 25.36 –1.5 13.20 13.2
L = total arc length
ΣT = 98.1 ΣN = 118.3
= 26.50

CL + tan φΣN
Factor of Safety =
ΣT
(2.45) (26.5) + tan 35°(118.3)
=
98.1
64.9 + 82.8
= = 1.506 > 1.5, hence safe.
98.1

Example 9.11: The following table gives various slices and the angle of
the tangents drawn at the base of these slices. Mid ordinate and width of each
slice are also given. Find factor of safety given γ = average unit weight of
soil = 18KN / m3, C = 20 KN / m², ϕ =30°, Arc Length = 130 m.
Earth Dams 213

Data
No. of Slice 1 2 3 4 5 6 7 8
Width of each slice (b) m. 15 15 15 15 15 15 15 15
Mid ordinate (h) m. 6 14 20 24 26 25 20 10
θ in degree –10 –3 5 14 25 35 45 58

Solution:
W = bh γ = wt. of each slice 1620 3780 5400 6480 7020 6750 5400 2700
T Component = W sin θ (KN) –281 –198 470 1567 2966 3871 3818 2289

N Component = W cos θ (KN) 1595 3774 5379 6287 6362 5529 3818 1430
Adding T and N components we get,
ΣT = 14500 KN
ΣN = 34194 KN

CL + tan φ ΣN
FS =
ΣT
(20 × 130) + tan 30(34194)
=
14500
2600 + 19741
= = 1.54 > 1.5
14500

Example 9.12: In a zoned earth dam of height 50m, water is stored upto 47m.
Top width 6m, u/s slope 3.5 H : 1 V and d/s slope 3 H to 1 V. Core section has
top width of 3m and height of 48m with side slope as 0.1:1 on both sides. If K of
shell is 3 × 10–3 cm/sec and K of core is 1.0 × 10–4 cm/s. Determine seepage flow
through core section.
Solution:
See Fig. See
− 9.3(a)
Fig. − 9.3(a)
L = Length
L =of horizontal
Length projection
of horizontal of u/s slope
projection of slope
of u/s core of core
L = 47 ×L0.1 = 4.7m.
= 47 × 0.1 = 4.7m.
CoreBase = (48 × 0.1)
CoreBase × 2×+0.1)
= (48 3 =×12.6m.
2 + 3 = 12.6m.
d = 12.6 − 4.7 = 7.9 m
= 12.6 − 3.29 = 9.31
S = d2 + H2 − d
S = (7.9) 2 + (47)2 − 7.9 = 47.6 − 7.9 = 39.76
q = KS
 1 × 10−4 
=  (39.76)
 100 
= 39.76 × 10 −6 m3 / S / m
S = d2 + H2 − d

214
S = (7.9) 2 + (47)2 − 7.9 = 47.6 − 7.9 = 39.76
Irrigation Engineering and Hydraulic Structures
q = KS
 1 × 10−4 
=  (39.76)
 100 
= 39.76 × 10 −6 m3 / S / m

9.4 CAUSES OF FAILURE OF EARTH DAM


Many things can be learned from failures. Failures reveal weaknesses entered during
design and construction. Proper study of failures will lead to better design and better
achievements of construction and maintenance.
Earth dam failures can be grouped as under:
(i) Failures due to hydraulic reasons. Remove underline such as overtopping,
erosion of u/s slope due to wave action and wind set up, erosion of d/s slope
by wind and rain, sloughening of material at toe.
(ii) Seepage or piping through dam and foundation may lead to failures.
(iii) Failures due to structured defects such as sliding of u/s and d/s slopes,
liquefaction slides, earthquake etc.
Remedial Measures:
1. Overtopping of water should never be allowed during lifetime of earth
dam. For this adequate free board and adequate spillway capacity should be
provided. Free board kept should be higher than 1.5 (hw + wind set up S), or
3 m whichever is higher. Wind set up S, is the result of piling up of water on
one end of reservoir on account of horizontal driving force of the blowing
wind. The magnitude of the rise of water surface is called the wind set up or
wind tide. The wind set-up S is calculated from Zuider Zee formula,
V2F
S= m
62000 D
here V = Velocity of wind in kmph
F = Fetch in km
D = Average depth of Reservoir in m along Fetch line
2. Rock toe should be provided at toe of dam to prevent sloughening of dam
material. Rock toe is a drain provided at toe, it must be provided with a filter
on its u/s side.
3. Piping: Seepage through body of dam and its foundation, if uncontrolled,
results in piping in d/s portion of the dam. Piping is the progressive erosion
of embankment material due to leaks, which develop in body of dam and in
foundation. This is the result of poor construction, inferior compaction adjacent
to concrete outlets or pipes which have weak bond with surrounding. Weak
bond between embankment and abutment is also responsible for leaks. When
forces resisting erosion, (mainly cohesion, interlocking stresses, weight of soil
Earth Dams 215

particles) are less than the erosive forces of seepage of water, soil particles get
washed away along leaking water and piping begins. Earth slides or sloughening
are related to piping.

Fig. 9.21: Gravel packed relief well

Sloughening begins when a small amount of material at the d/s toe has
eroded leaving behind steeper slope, which eventually gets saturated due
to seepage of water and slumps again. This may continue to cause complete
failure of the dam.
To check the effect of seepage, seepage line, should be kept well within the
d/s face and seepage water is collected and disposed off without causing dam
material loss. Core, cut-off, rock toe, relief wells, grouting etc are remedial
measures to check ill effect of piping.
4. Relief Wells: Relief wells (see Fig. 9.21) are provided when pervious strata
of embankment foundation are too deep to be penetrated by cut – off or
toe drains. Relief wells can penetrate most pervious strata and relieve uplift
pressures effectively. Spacing between relief wells should be small enough
to lower water pressure to desired value. It has interior perforated pipe of
minimum 15 cm diameter or larger. The annular space is packed with gravel,
however near the surface impervious compacted back fill is provided so that
upward flow of water outside relief well may not occur.
216 Irrigation Engineering and Hydraulic Structures

5. Chimney Drains (see Fig. 9.22):They combine the advantage of horizontal


blanket drain as well as toe drain. These drains keep seepage line well within
the d/s face. These drain material must have high value of permeability so
that it can discharge seepage water without excessive head and hence no
piping may take place.

Fig. 9.22: Chimney drain

9.5 FILTERS: CRITERIA AND DESIGN


Soil particles lying in embankment surrounded by other soil particles cannot move
due to seepage forces but finer soil particles near the boundaries may get washed
away into void space of coarser particles. To prevent such migration of finer to
coarser particles, fillers are provided, for example between core and shell, shell
and rock toe, drains and adjacent zones etc.
Filters have to satisfy following criteria:
(i) Filter layer must be more pervious than the protected soil so that filter acts as
a drain, and
(ii) Particle size of filter layer must be small enough to prevent migration of
particles from adjacent layers into the voids of the filter layer.
(iii) Filter layer, should be sufficiently thick to provide good distribution of all
particle sizes throughout the filter.
Following filter criteria are generally adopted while designing filter layer:
D15 of filter
(i) ≥5
D15 of protected soil

D15 of filter
(ii) ≤5
D85 of protected soil

D50 of filter
(iii) ≤ 25
D50 of protected soil

(iv) Gradation Curve of filter should have approximately same shape as that of
protected soil.
(v) Where protected soil contains large percentage of gravels, filter should be
designed on the basis of gradation curve of the portion of material which is
finer than one inch sieve.
Earth Dams 217

(vi) Filter should not contain 5% of finer materials passing through no. 200 sieve.
Here D15, D50, D85 represent particle sizes, which are respectively coarser than
finest 15, 50, and 85% of the soil by, weight.
These filter criteria are based on studies with non-cohesive soils and take into
consideration only the grain size of the protected soil.
Theoretically thickness of horizontal filter required is 15 cm for sand and
30 cm for gravel. However, a minimum of 1.0 m is desirable. For vertical or inclined
filters minimum width should be 2 to 3 m for convenience of construction.

EXERCISES
1. (a) What are rigid and non-rigid dams?
(b) Distinguish between:
(i) Earth Dam and Gravity Dams.
(ii) Rock filled and Roll-filled Dams.
2. (a) What are causes of failures of earth dam?
(b) What are remedial measures for same/
3. (a) What precautions should be taken during and after construction of earth
dam?
(b) Explain Swedish slip circle method and location of critical slip circle.
4. Draw neat sketch of an earth dam and show there on all component parts.
State the function of these components parts.
5. What design criteria are adopted for safe design of earth dam?
6. (a) Define and draw sketch of ‘phreatic line’ in homogeneous earth dam
section with and without horizontal filter at its base.
(b) What are egress and ingress corrections to be applied to base parabola?
7. Prove that for a homogenous earth dam section with a horizontal filter at its
base of focal length equal to S, seepage loss per meter length of dam is given
by q = KS, where K is permeability of dam material.
What correction factor is required to be applied to this expression if dam material
is non-homogeneous or anisotropic?
8. Explain with sketches:
(a) Rock Toe (b) Chimney Drain
(c) Relief wells (d) Openings through earth dam.
9. What is ‘pore-pressure’ and its role in design and construction of earth dam?


10
Canal Head Works

10.1 BARRAGE AND WEIRS


A weir is an obstruction to river flow provided with a specific purpose of raising
water level in river such that canal can receive water in it by gravity flow. To increase
water level, weir crest is raised above river bed and if need be, shutters are also
provided on the top of the weir. These shutters may get dropped down during flood
so that afflux is minimum. Afflux is defined as difference in water level between
upstream and downstream of weir under free flow condition. Controlling of levels
by means of shutters becomes difficult when the crest level is higher than 2.0 m.
In such cases, a fully gated weir, known, as barrage is preferred. Barrage and weir
are similar structures with a difference that barrage has low level crest and gates all
along its length. Barrages are considered better than weirs as they keep upstream
region free of sediment deposition so that sediment free water enters the canal (See
Fig. 10.1).

Fig. 10.1: Sectional view of barrage

10.2 DIFFERENT UNITS OF CANAL HEAD WORKS


Canal head works are basically diversion works consisting of a weir or a barrage
and a canal head regulator, as shown in Fig. 10.2.

© The Author(s) 2023 218


S. K. Ukarande, Irrigation Engineering and Hydraulic Structures,
https://doi.org/10.1007/978-3-031-33552-5_10
Canal Head Works 219

A deep stilling pocket of water upstream of weir wall is created and through
canal head regulator water enters the canal. This pocket is separated from other part
of river by means of divide wall. A fish ladder is

Fig. 10.2: Canal head works in plan

provided near the divide wall. Sediment excluder near the canal head regulator is
also sometimes provided. This arrangement may be either on left or right bank or
both banks of the river, as shown in Fig. 10.2.

10.3 DESIGN OF WEIR


A weir consists of following component parts:
(i) Weir wall
(ii) Crest gates
(iii) u/s and d/s aprons
(iv) u/s, d/s and intermediate sheet piles
These are considered in detail as under: Weir crest is normally flat and it is then
known as broad crested weir. Crest width should be 2.5 times head over weir or a
minimum of 2 m. U/s slope of weir wall may be kept as 2 H: 1V and d/s slope 3H: 1V
The d/s horizontal pucca apron (horizontal floor) should be such that maximum
dissipation of energy takes place through stable hydraulic jump on it. Total length
of impervious floor includes d/s basin length, glacis, weir crest, and u/s floor. Glacis
means u/s and d/s sloping part of weir wall. The impervious floor in conjunction with
d/s cut off (sheet pile) should result in safe exit gradient. Length of d/s horizontal
floor is kept 5 to 6 times height of jump i.e., difference of depth before jump and
after jump.
220 Irrigation Engineering and Hydraulic Structures

The u/s and d/s sheet piles are always provided to guard against scouring and
piping effects. The depth of sheet piles should be such that its bottom is lower
than the level of possible flood scour at that section. The scour depth is given by
Lacey’s equation,
1/3
 q2 
R = 1.35  
 f 
here q = flood discharge per unit width of river
f = silt -factor
R = socur depth measured below HFL

The length of horizontal floor should be such that exit gradient GE, is less than safe
exit gradient for the soil under consideration.
The recommended values of safe exit gradient are:
1 1
(i ) to for gravel
4 5
1 1
(ii ) to for coarse sand
5 6
1 1
(iii ) to for fine sand
6 7
The thickness of impervious floor is decided by uplift pressure acting on the
floor which can be decided either by Bligh’s creep theory or Khosla’s method,
given hereafter.

10.4 BLIGH’S AND KHOSLA’S THEORIES


10.4.1 Bligh’s Creep Theory
According to Bligh (1910), water percolating through pervious foundation below
weir wall, creeps along the base profile of the structure as shown by dotted line in
the Fig. 10.3 giving a total creep length L as,
L = b + 2(d1 + d 2 + d3 )

Weir wall

H Hydraulic gradient = H/L

d1 d2
d3 (Depth of
Creep length b sheet pile)
(dotted line) (Full length of apron)

Fig. 10.3: Weir section showing creep length


Canal Head Works 221

Here d1, d2, d3 are u/s, intermediate and d/s sheet pile depths, and b = total
length of the floor. Hydraulic gradient will then be equal to
H H
=
L (b + 2d1 + 2d 2 + 2d3 )

As hydraulic gradient is constant, and if L1 = creep length upto point 1 on the


floor, then residual uplift pressure at that point will be H1 as given below.
H   L 
H1 = H −  L1  = H 1 − 1 
L   L

L
The reciprocal of hydraulic gradient , is known as Bligh’s Creep Coefficient,
H
L
C, Hence, C =
H
Bligh has recommended safe values for C for different soils in order to get
safe creep length L
L = CH

Table 10.1: Values of C and GH for Different Soils.

Sr. No. Soil Value of C GH = safe hydraulic Gradient = 1/C


1. Mud 18 1/18
2. Fine sand 15 1/5
3. Coarse Sand 12 1/12
4. Gravel 5 to 9 1/9 to1/5

The floor thickness, t, is given by


4 H1
t=
3 (G − 1)
Here, G = specific gravity of floor material
H   L 
H1 = H −  L1  = H 1 − 1 
 L   L
L1 = creep length upto a point on floor where residual pressure head is H1
Limitation of Bligh’s Creep Theory:
1. Bligh made no distinction between horizontal and vertical creep length.
2. Bligh did not consider safe exit gradient.
3. Bligh did not make any distinction between outer and inner faces of sheet
piles.
222 Irrigation Engineering and Hydraulic Structures

4. According to Bligh, d/s sheet pile is not essential.


5. Exit gradient may not be safe even if hydraulic gradient of Bligh (1/C) is
safe.

10.4.2 Lane’s Weighted Theory


On account of number of limitation to Bligh’s creep theory, Lane put forward his
weighted theory in 1932. According to Lane, horizontal creep length, b, is less
effective compared to vertical creep length due to piles. Hence,
1 
L=  b  +2(d1 +d 2 +d3 ), i.e., weightage of horizontal creep length is reduced to
3 
one third compared to its original value whereas value of vertical creep length is
retained same.

10.4.3 Khosla’s Theory


Dr. A. N. Khosla and his associate published their findings in 1954 for structures,
which were designed as per Bligh’s theory but have not given satisfactory performance
According to Khosla,
(i) Outer faces of end sheet piles were more effective than inner ones.
(ii) Intermediate sheet piles if smaller in length than outer ones, are ineffective.
(iii) Undermining of floors starts from tail end. If hydraulic gradient at exit is more
than permissible (critical) gradient for that soil, the soil particles will start
getting washed away with seeping water causing progressive degradation of
subsoil. This is known as piping and it leads to failure of structure.
(iv) d/s sheet pile is more effective to control piping.
Khosla also gave formula for safe exit gradient,

H 1
GE =
d π λ
here, G E = safe exit gradient for soil
H = head difference between water level on u/s and d/s
side of weir wall,
d = depth of sheet pile
b = total length of pucca floor

λ = ½ 1 + 1 + α 2 
 
b
α=
d
Canal Head Works 223

If P = uplift pressure head at a point on pucca floor, and H= total seepage


head, then
P 1 æ x ö÷
f= = cos-1 çç ÷÷
H p èç b 2 ø÷

Pressure variation along apron length given by above formula for f is shown
in Fig. 10.4. From this figure, it is clear that d/s half of the floor has more
uplift pressure than that given by Bligh.

Fig. 10.4: Variation of pressure head along apron

H H

C1 E
d b b d
D1 D

Fig. 10.5 (i): Sheet pile at u/s end Fig. 10.5 (ii): Sheet pile at d/s end

H
E C
b b2
d
D D’
D’ b
b

Fig. 10.5 (iii): Case of intermediate Fig. 10.5 (iv): Case of depressed
sheet pile      floor
224 Irrigation Engineering and Hydraulic Structures

10.5 KHOSLA’S METHOD OF INDEPENDENT VARIABLES


Seeping water does not creep along the bottom contour of pucca apron as envisaged
by Bligh, but it moves along a set of elliptic streamlines. The case of steady seepage
in a vertical plane for homogeneous soil can be mathematically expressed by Laplace
equation,
∂ 2φ ∂ 2φ
+ =0
∂x 2 ∂y 2
here φ = Kh,
Where K is permeability of soil and h is residual pressure head at point whose
coordinates are (x, y). Dr. Khosla used method of independent variable and obtained
solution for Laplace Equation for simplified cases as illustrated in Fig. 10.5 (i) to (iv).
Case (i) and (ii): For sheet pile either at u/s or d/s end, Khosla’s solution gives
1 λ−2
φE = cos −1   , Fig 10.5 (ii)
π  λ 
1  λ −1
φD = cos −1   , for Fig 10.5 (ii)
π  λ 
φCφ1 C=1 100
= 100
− φ−EφEforfor
FigFig 10.5
10.5 (i)(i)
φDφ1 D=1 100
= 100
− φ−DφDforfor
FigFig 10.5
10.5 (i)(i)
1 1 (1 + 1 +2α 2 )
λ =λ = (1 + 1+ α )
22
bb
αα==
dd
Case (iii): For sheet pile at intermediate point on the pucca floor (see
Fig. 10.5 (iii)),
1  λ −1
φE = cos −1  1 
π  λ2 
1 λ 
φD = cos −1  1 
π  λ2 
1  λ +1
φC = cos −1  1 
π  λ2 
Here,
1
λ1 =  1 + α12 − 1 + α 22 
2  
1
λ 2 =  1 + α12 + 1 + α 22 
2  
b b
α1 = 1 and α12 = 2
d d
1
λ1 = 1 + α12 − 1 + α 22 
2 
1
λ 2 =  1 + α12 + 1 + α 22  Canal Head Works 225
2  
b b
α1 = 1 and α12 = 2
d d
Case (iv) : For case of depressed floor, i.e., pucca floor having depth d from
bed level,
2 3
φ D ' = ϕD − ( φ E − φ D ) + 2
3 α
φ = 100 − φ
D1' D1
Here, fE and fD are same as given in case (ii)
Note: C, D, E and C1, D1, E1 are key points in all of above cases for which f values
are obtained as per above formulate. In these solutions for f, floor thickness t is not
considered. If actual values with consideration of floor thickness are expected, then
following corrections should be applied to values as obtained above.
A. Correction due to Floor Thickness:
(i) Corrections for pile at u/s end [see Fig. 10.6 (i)]
 φ − φC 
Correction for pressure at C1 =  D  t1 , + ve
 d1 

 φ − φC 
Corrected value of pressure at C1 = φc1 = φc +  D  t1
 d1 
t1 = floor
E C
thickness
C1
d1 +ve
d1 = Depth of pile at u/s end

Fig. 10.6 (i): Pile of u/s end

(ii) Correction for pile at intermediate point

E C t2
E1 C1
d2 -ve +ve

Fig. 10.6 (ii): Pile at intermediate point


226 Irrigation Engineering and Hydraulic Structures

∴ Corrected value at pt E1,

 φ − φD 
φE1 = φE −  E  t2
 d2 

and corrected value at C1 is given by


 φι − φC 
= φc1 = φc +  D  t2
 d2 

 φ − φD 
(iii) Correction for pile at d/s end =  E  t 3 , ( − ve), ( Fig. 10.6 ,iii)
 d3 

E C t1

d3 E1
-ve

Fig. 10.6 (iii): Pile at d/s end

 φ − φD 
Corrected value at po int E1 = φE1 = φE −  E  t3
 d3 

B. Correction for Mutual Interference of Piles: [see Fig. 10.6(iv)]

Fig. 10.6: (iv) Mutual interference of pile.


Canal Head Works 227

Formula for correction due to mutual interference of pile is

D d+D
C = 19  
b'  b 
b' = distance between piles under consideration
b = pucca Floor length

D = Depth of pile whose influence is to be considered on the neighboring piles.


The correction C is additive to points in rear and negative for points on forward
side of the pile whose influence is to be considered. For example, if influence of
pile no. (2) is to be considered on pile no. (1), then correction C is to be added to
φC1, value and if influence of pile 2 is to be considered on pile 3, then correction C
is to be subtracted from φ­E3 value but in this case b’ will be distance between pile
2 and 3 and D will be same as d3.

C. Corrections in φ Values for Sloping Apron:


The correction for the sloping apron is applied to f values of key points of that pile
which is fixed at either the beginning or at the end of the slope. [See Fig.10.6 (iv)].
The correction is positive for slope decreasing in direction of flow and negative for
slope increasing in direction of flow. For example, correction due to slope is applicable
to pile no. 2 in Fig. 10.6 (iv) and is additive for f value at E2. If bs is horizontal
length due to sloping apron, and b1 is horizontal distance between two piles between
b 
which the sloping apron is located, then slope correction C = CS  1s  . The value
b 
of Cs depends on slope of the apron and its values are given in Table 10.2 below:

Table 10.2: Cs Values for Different Slopes.

Slope V : H 1:1 1:2 1:3 1:4 1:5 1:6

Cs, % of φ value 11.2 6.5 4.5 3.3 2.8 2.5

q SOLVED EXAMPLES
Examples 10.1: A weir on pervious foundation is shown in Fig. 10.7. Find
hydraulic gradient according to Bligh and Lane. Also find uplift pressure at points
A, B, C. Take G = 2.65. Find floor thickness also.
228 Irrigation Engineering and Hydraulic Structures

H = 6m

A B C

6m d2 = 4m
d1 = 6m

6m d3 = 6m
12m 6m

24m

Fig. 10.7: Weir on pervious foundation

Solution:
According to Bligh,
Creep Length, L = 2 ( d1 + d 2 + d3 ) + b
= 2 (6 + 4 + 12) + 24 = 68m
H 6
∴ Hydraulic gradient = =
L 68
1 1
= < hence safe if soilis coarse grain
11.3 6
According to Lane, Creep Length
b
= 2(d1 + d 2 + d3 ) +
3
24
= 2(6 + 4 + 12) + = 52m
3
H
∴ Hydraulic gradient =
L
6 1 1
= = <
52 8.66 6
 L   18 
h A = Uplift pressure at A = H 1 − A  = 6 1 −  = 4.41m
 L   68 
L A = Creep length upto A = 2 × 6 + 6 = 18
Floor thickness at A = tA
4  h  4  4.41 
=  A =   = 3.65m
3  G − 1 3  2.65 − 1
Canal Head Works 229

hB = Uplift pressure at B:
L B = 2 × 6 + 2 × 4 + 12 = 32m
 32 
∴ h B = 6 1 −  = 3.17m
 68 
4  3.17 
tB =   = 2.56m
3  1.65 
C = uplift pressure at C : −
hC = Uplift pressure athC:
LC = 2 × 6 + 2 × 4 + 18 = 38 m
 38 
h C = 6 1 −  = 2.64m
 68 
4  2.64 
tC =   = 2.13m
3  1.65 
Example 10.2: Using Khosla’s method of independent variables, obtain residual
pressures at key points for the weir given in Fig. 10.8 and plot Hydraulic Gradient
line (HGL).
Pond level RL 170m

(a) RL 268.32m
(b) RL 266.92m
(b1) RL 266.51m
V:H H
RL 266m (c) 266.50m
RL 265m
1:2 1m V

RL 265m

E1 C 1
d u/s pile 1m
2m 5m 2m bs=7m E C E C
D1 5m
RL 260m Intermediate d/s pile
pile D
b1=16m 258m RL 258m RL
b=21m
Fig. 10.8: Cross-section of weir
Solution:
(i) Upstream pile, depth d = 266 – 260 = 6 m, b = 21 m
b 21 1
α= = = 3.5, λ = 1 + 1 + α 2  = 2.32
d 6 2  
1 λ−2
φE = cos −1  
π  λ 
1 1 π 82
= (82°) = (82) = = 0.45 = 45%
π π 180 180
230 Irrigation Engineering and Hydraulic Structures

φC1 = 100 − ϕE = 100 − 45 = 55%

1  λ − 1  55
φD = cos −1  = = 0.3 = 30%
π  λ  180
φD1 = 100 − ϕD = 70
Thickness correction for
φD − φC1 70 − 55
φC1 = 1 t= (1) = 2.5% , + ve
d 6
Correction due to interference of intermediate pile on f value at C1
D d+D 266 − 258  6 + 8 
= +19   = +19  
b1  b  16  21 
 14 
= +19 ( 0.7 )   = 9 % + ve
 21 
∴φC1 corrected = 55 + 2.5 + 9 = 66.5%

H = 270 – 265 = 5
66.5
∴φC1 = (5) = 3.32
100
∴ RL of residual pressure at C1 = 265 + 3.32
= 268.32 shown at (a) in Fig 10.8
(ii) Intermediate pile, depth d = 265 – 258 = 7m
b1 = 16m, b 2 = 5m
b1 16 b 5
α1 = = = 2.3, α 2 = 2 = = 0.7
d 7 d 7
1 2 2
λ1 = 1 + α1 − 1 + α 2
2  
1
= [ 2.5 − 1.2] = 0.65
2
1 1
λ 2 =  1 + α12 + 1 + α 22  = [ 2.5 + 1.2] = 1.85
2  
 2
1  λ −1 1  0.65 − 1 
φE = cos −1  1  = cos −1   = 40.4 %
π  λ2  π  1.85 
1 λ  1  0.65 
φD = cos −1  1  = cos −1   = 38.5%
π λ
 2 π  1.85 
1  λ +1 1  1.65 
φC = cos −1  1  = cos −1   = 15%
π λ
 2  π  1.85 
Canal Head Works 231

 φ − φD 
Thickness correction for φE =  E  (1)
 6 
40 − 38.5
= (1) = 0.25 % , − ve
6
Slope correction for slope of 1V : 2H, Cs = 6.5 from Table 10.2
b  7
∴ Slope correction = CS  s  = 6.5  
 b1   16 
= 2.84, + ve
Correction for interference of u/s pile on φE
264 − 260  4 + 6 
= 19   = 4.5 − ve
16  21 
φE corrected = 70 − 4 + 2.84 − 4.5
= 64.34%
64.34
Corresponding RL for φE = 265 + (5)
100
= 265 + 3.2
= 268.2 m shown at (b) in Fig.10.8
φ D − φC
Similarly thickness correction for φc = t
d
38.5 − 15
= (1)
7
= 3.35, + ve
Correction for interference of d/s pie on C
6 ( 6 + 6)
= 19 = 11.86, + ve
5 21
∴φc corrected = 15 + 3.35 + 11.8 = 30.21 %
Corresponding RL of φc = 265 + 0.3021 (5)
= 266.51, shown as b1 in Fig. 10.8

(iii) D/s pile: d = 6m


b = 21m
b 1
α= = 3.5, λ = 1 + 1 + α 2  = 2.32
d 2  
1  λ − 2  82
φE = cos −1  = = 45%
π  λ  180
232 Irrigation Engineering and Hydraulic Structures

1  λ −1 1 55
φD = cos −1  −1
 = cos (0.56) = = 30%
π  λ  π 180
45 − 30
thickness correction for φE = × 1 = 2.5% , − ve
6
6 6+6
Interference of intermediate pile on φE = 19   = 11.86 % − ve
5  21 
∴φE corrected = 45 − 2.5 − 11.86 = 30.64 %
∴ RL for φE at d / s pile = 265 + .3(5)
= 265 + 1.5 = 266.5, shown at (c) in Fig. 10.8
HGL is shown as (a b b1 c) in Fig. 10.8

Example 10.3: A weir is 4 m high and has 16 m long, 1 m deep apron on u/s
side and 36m long, 2 m deep apron on d/s side. The depths of u/s and d/s piles
are 8m and 8.5 m respectively. Find uplift pressure u/s and d/s of piles at top
and bottom by Khosla’s Theory. Present in tabular form data required to plot
HGL.
Refer to Fig. 10.9,

H = 4m
16m 36m

El C1 t1 = 1m E C
t2 = 2m
d1 = 8m C1
E
d2 = 8.5m

D1
b = 52m
D

Fig. 10.9: Section of weir (pressure at u/s & d/s piles)

Solution:
b = 16 + 36 = 52
b 52
α1 = = = 6.5
d1 8
1
λ1 = 1 + 1 + α12  = 3.7
2 
 
Canal Head Works 233

1 λ −2 1 −1  3.7 − 2 
φE = cos −1  1  = cos   = 34%
π  λ1  π  3.7 
1  λ −1 1  3.7 − 1 
φD = cos −1  1  = cos −1   = 24%
π  λ1  π  3.7 
φC1 = 100 − ϕE = 100 − 34 = 66%
φD1 = 100 − ϕD = 100 − 24 = 76%

 φD − φC1 
Correction for thickness in value of φC1 =  1  t1
 d1 
76 − 66
= (1) = 1.25, (+ ve)
8
Correction due to interference of pile at d/s end

D d+D 8.5  8 + 8.5 


= 19   = 19   , here b1 = b = 52
b1  b  52  52 
= 19 × 0.40 × .32 = 2.41
∴φ = corrected φC1 = 66 + 1.25 + 2.4 = 69.66%
C11

b 52
d/s pile : − d 2 = 8.5 ∴α 2 = = = 6.11
d 2 8.5
1
λ2 = 1 + 1 + α 22  = 3.6
2 
 
1  λ −1 1 −1  3.6 − 1  43.76
φD = cos −1  2  = cos  = = 0.243 = 24.3%
π  λ2  π  3.6  180
1 λ −2
φE = cos −1  2 
π  λ2 
1  3.6 − 2  63.6
= cos −1  = = 0.353 = 35.3%
π  3.6  180
Correction in φE due to thicknes t 2 = 2m
( φE − φD )
= t2 (− ve)
d2
35.3 − 24.3
= ×2
8.5
= 2.58 % , (− ve)
234 Irrigation Engineering and Hydraulic Structures

Correction due to interference of u/s pile

D d+D
= 19  
b1  b 
8  8 + 8.5 
= 19  
52  52 
= 19 × .39 × .32
= 2.38 (− ve)
φE corected = 35.3 − 2.58 − 2.38
= 30.34%

Table for Plot of HGL:

U/S pile D/s pile


H φC
φE1 φD1 φC1 φE φD
4m 100 % 76 % 69.66 % 30.34 % 24.3 % 00
Residual Pressure 4m 3.04m 2.78m 1.212m 0.97m 00

Example 10.4: An impervious floor of a weir on permeable soil is 16 m long


and has sheet piles at both the ends. The u/s pile is 4 m deep and d/s pile is 5 m
deep. The weir creates a net head of 2.5 m; neglecting thickness of apron, calculate
the uplift pressures at the junction of the interfaces of the pile with the weir floor,
using Khosla’s theory. Refer to Fig. 10.10,

H = 2.5m

E C1
E C
16m
4m 5m
D1 D

Fig. 10.10: Weir on impervious f loor.


Solution:

u/s pile d = 4m, b = 16m


b 16
α= = =4
d 4
1
(
λ = 1 + 1 + α 2 = 2.56
2 )
φC1 = 100 − ϕE
1 λ−2
φE = cos −1  
π  λ 
u/s pile d = 4m, b = 16m
b 16 Canal Head Works 235
α= = =4
d 4
1
( )
λ = 1 + 1 + α 2 = 2.56
2
φC1 = 100 − ϕE
1 λ−2
φE = cos −1  
π  λ 
1  2.56 − 2  77
= cos −1  = = 0.429 = 43%
π  2.56  180
φC1 = 100 − 43 = 57%
Correction due to d/s pile interference:
 D  d+D
= +19  1   
b   b 
5  4+5
= +19
  = 19 × .559 × .56 = 5.98 = 6% + ve
16  16 
∴φC1 Corrected = 57 + 6 = 63%
∴ uplift pressure at c1 = 0.63(2.5) = 1.57m

(2) d/s pile:


16
d = 5 m, b = 16 m, α = b d = = 3.2
5
1
λ= 1 + 1 + α 2  = 2.17
2  
1 λ−2 1 −1  0.17  85.5
φE = cos −1   = cos  = = 0.475 = 47.5%
π  λ  π  2.17  180
Correction due to u/s pile interference
4 5+4
= −19   = −5.3%
16  16 
φE corrected = 47.5 − 5.3 = 42.2 %
uplift pressure at E = 0.422(2.5) = 1.05 m

10.6 SALIENT FEATURES OF DESIGN OF WEIR ON


PERVIOUS FOUNDATION
(Refer to Fig. 10.11)
1/3
 q2 
R = 1.35  
 f 
 
L = Creep length = CH = b + 2d1 + 2d 2
b = horizontal creep length = L 2 + B + L1
4 h  L 
t = apron thickness = here h = H 1 − 2 
3 (G − 1)  L 
L 2 = u/s pucca apron length = L − L1 − (B + 2d1 + 2d 2 )
1/3
 q2 
R = 1.35  
 f 
236  and
Irrigation Engineering  Hydraulic Structures
L = Creep length = CH = b + 2d1 + 2d 2
b = horizontal creep length = L 2 + B + L1
4 h  L 
t = apron thickness = here h = H 1 − 2 
3 (G − 1)  L 
L 2 = u/s pucca apron length = L − L1 − (B + 2d1 + 2d 2 )
H
L1 = d/s pucca apron length = 2.21 C
10

2
V /2g
TEL
HFL
(HFL-Afflux)
K Crest width

H=12m
1.5R Weir wall 2R
h
1.5dl
L3
Pucca apron Loose apron
Base width

L2 B L1
d1
d2
(1) (2)

Fig. 10.11: Weir on pervious foundation

L3 = d / s apron length (pucca + loose)


H q
= 18C × with crest shutter
13 75
H q
= 18C × without crest shutter
10 75
V2
TEL = HFL +
2g
K = shutter depth from TEL
2/3
 q 
= 
 1.7 

Crest Level = TEL – K


V2
Level of d/s TEL = HFL before construction +
2g
Canal Head Works 237

Level of u/s TEL = d/s TEL + Afflux


V2
Level of u/s HFL = u/s TEL –
2g
Level of bottom of (u/s) pile = (u/s) HFL – 1.5 R = HFL (u/s),
Level of bottom of (d/s) pile = (d/s) HFL – 2R

10.7 NUMERICAL PROBLEMS ON WEIR DESIGN:


A weir on sandy bed has top width 2 m, height 7 m, and base width 9 m. Depth
of water on u/s side is 12 m above river bed. Discharge per unit length of weir
is 25.7 cumec u/s pile length is 5 m and d/s pile length is 8 m. Design u/s and
d/s solid and loose apron. Take C = 12 (Bligh’s creep coefficient).
HFL

2m

H=12m
Weir wall
7m

L3
9m

L2 B L1
d1=5m d2=8m

Fig. 10.12: Weir on sandy bed

As per Bligh:

L = CH = 12 × 12 = 144m = b + 2d1 + 2d 2
= b + 2(5) + 2(8)
= b + 26
∴ b = 144 − 26 = 118m = Length of horizontal pucca appron
La = u / s pucca apron length = L − L1 − (b + 2d1 + 2d 2 )
H
L1 = d / s pucca apron length = 2.21 C
10
12
= 2.21 × 12 ×
10
= 29m
238 Irrigation Engineering and Hydraulic Structures

∴ L 2 = 144 − 29 − (9 + 10 + 16) = 80 m
L3 = Length of pucca + loose apron on d/s
H q
= 18C ×
13 75
12 25.7
= 18 × 12 × × = 18 ×12 × (0.56) =121 m.
13 75
∴ Length of loose apron on d/s = 121 – 29 = 92 m …
Length of loose apron on u/s = 1.5 d1 = 1.5 × 5 = 7.5m …
t = Thickness of apron below weir wall
  L2  
H 1−
4 h  4   L   4 (.45 × 12)
=  = = = 4.36 m
3  G −1 3  2.65 − 1  3 1.65
 
 

10.8 SEDIMENT CONTROL AT CANAL-HEAD WORKS


(Note: Fig. 10.13, which shows sectional view of head regulator, for plan please
refer Fig. 10.2)
In canal head works, head regulator for canal (Fig. 10.13) is provided just
upstream of the weir wall and canal draws water from the pond created between
weir, divide wall and the bank. The purpose of creating the pond is to exclude entry
of excessive silt into the canal. Undersluices provided in the weir wall remove the
silt deposited in the pond. However, even with this arrangement, if silt entering
into the canal is excessive, then silt excluder is provided just in front of canal head
regulator, and if after entry of water into the canal, silt load is found to be excessive
than silt ejector is provided just little d/s of canal head regulator in the canal reach
so that water flowing into the canal is carrying least silt load.
This is known as sediment control at canal head works. It is achieved by silt
excluder and silt ejector provided on u/s and d/s side of head regulator respectively.

H.F.L.
Approach road

Pier of head regulator Gate Pond level Divide


wall

Canal
Silt excluder
tunnels
Silt ejector

Fig. 10.13: Canal head regulator


Canal Head Works 239

10.9 RIVER TRAINING WORKS FOR CANAL HEAD WORKS


River training structures for canal head works are provided to satisfy following
purposes:
1. To prevent overflow from structures
2. To prevent flooding of land u/s of barrage or weir
3. To prevent entry of silt into canal and to pass silt load to d/s of weir.
For this, following structures are required to be constructed at the canal head
works. (Fig. 10.2)
(i) Guide banks
(ii) Divide wall
(iii) Weir wall with under slucies
(iv) Pond
(v) Head regulator with silt excluder tunnels
(vi) Groynes or spurs to protect the river banks
(vii) Fish ladder to pass fish d/s of weir

EXERCISES

1. (a)  istinguish between Khosla’s theory and Bligh’s creep theory for
D
seepage below weirs on permeable foundations.
(b) How does Lane’s theory differ from Bligh’s theory?
2. Briefly out line Khosla’s theory on design of weirs on permeable foundations.
Enumerate the various corrections that are needed in the application of this
theory.
3. (a) Explain briefly Khosla’s exit gradient concept
(b) What is piping? What are the precautionary methods to avoid the ill
effects of piping?
(c) Distinguish between a weir and a barrage.
4. Design a weir on coarse sand strata having following data making use of
Bligh’s creep theory:
Flood discharge = 300 cumecs.
Value of Bligh’s C = 12 for coarse sand
Length of weir = 30 m.
240 Irrigation Engineering and Hydraulic Structures

Top of weir is at 2m alone LWL


And height of falling shutter = 0.6 m.
Top width of weir 2 m., and bottom width 3.5 m.
5. (a) Draw a sectional view of a weir and show there of various parts.
What is exit gradient? How does it affect the design of a weir?
(b) Find maximum flood discharge that weir having following data can
pass safely:
Number of vertical gates = 51
Span of each gate = 10 m.
FRL = 110 m
Crest Level = 106 m
Kp = 0.02,
Ka = 0.1
C = 1.7 in Bligh’s Formula.
Neglect velocity of approach.
 [Ans:. 6818 cumecs]
11
Distribution System

11.1 GENERAL
Irrigation engineering projects consist of storage works and distribution works.
Storage works are in the form of reservoirs, dams, spillways and distribution works
are head works, canals, regulation and cross drainage works. Success or failure of
project depends on efficient management of distribution system.

11.2 CANAL SYSTEM


A canal is defined as a channel constructed on ground to carry water from river or
reservoir by gravity flow. It has an open surface and is normally unlined and hence
suffers of losses due to evaporation, transpiration and seepage of water.
Depending upon functional role, canals can be classified as:

(i) Irrigation, power or feeder canals


(ii) Main, branch, distributary and field channel or watercourse.

An irrigation canal carries water from its source to agricultural fields situated in its
command area. Command area is demarcation of agricultural area in which canal
water can be provided by gravity flow. Power canals carry water from source to
electrical power generating hydro plants. After utilization for power generation, water
is again fed back to canal system for irrigation purposes. Feeder canal feeds two or
more canals. Irrigation canal system is subdivided into main, branch, distributary
and field channels as per its function. Main canal feeds branch canals through cross
regulators and they in turn supply water to distributaries. Field channels receive
its share of water through regulation works such as outlets provided in distributary
canals. Canals may have to cross natural drainages such as rivers, tributaries etc.
and works provided at such crossing are known as cross drainage works.

© The Author(s) 2023 241


S. K. Ukarande, Irrigation Engineering and Hydraulic Structures,
https://doi.org/10.1007/978-3-031-33552-5_11
242 Irrigation Engineering and Hydraulic Structures

11.3 ALIGNMENT OF CANALS


Based on alignment, (see Fig. 11.1) canals are known as:
(a) Contour Canal
(b) Ridge Canal
(c) Side slope or watershed canal
If a canal has its alignment parallel to a contour of the topography of the land then
it is known as contour canal and if the alignment is along a ridge line it is known
as ridge canal. Canal running along a sloping ground is known as side slope or
watershed canal. A canal has to be terminated into a canal escape, feeding extra
water back to tributary or to river. Ridge canal has advantage over contour canal
that it can irrigate land on both sides.

Contour
Tributary canal Head Regulator
Contours
Weir
RL 100
R
Cross regulator C.D.
I
Work
V
RL 99 E
side slope R
canal
RL 98

RL 97
Ridge
canal

RL 96

RL 95

Fig. 11.1: Alignment of canal

Alignment of canal is generally carried out such that it has to cross minimum
number of tributaries or natural drainages and also such that topography of land
has gentle ground slope. If ground slope is more or changing suddenly along
the canal alignment, a fall is required to be provided such that the canal bed
slope upstream of fall and downstream of fall remains same. Structure known
as canal fall takes care of change in ground level by providing a weir wall and
cistern. Falls are discussed separately. Alignment of canal should be such that
filling and cutting of ground is balanced or is kept minimum. A canal may be
fully above ground level and is known as canal in high embankment. It may
Distribution System 243

be partly above ground and partly in cutting and sometimes fully in cutting.
[See Fig. 11.3(i) to (iii)].

11.3.1 Curves on Canal Alignment


As far as possible curves should be avoided in the alignment of canals because
they become source of siltation or scouring in the canal bed and change its cross
sectional area and create disturbance in the working of the canal system. However,
if they are unavoidable, they should be gentle and according to IS 5968 of 1970 the
radii of the curves should be 3 to 7 times the width of the canal at FSL (full supply
level) subject to the minimum values given ahead:
(i) Unlined Canals

Table 11.1: Radius of Curves(Lined Canals)

Capacity in Cumecs Minimum Radius of curve in meter

80 and above 1500


30 to 80 1000
15 to 30 600
3 to 15 300
0.3 to 3 150

(ii) Lined Canals

Table 11.2: Radius of curve (Unlined canal)

Capacity in cumecs Minimum Radius of curve in meter


280 and above 900
280 to 200 750
200 to 140 600
140 to 70 450
70 to 40 300
40 to 10 200
10 to 3 150
3 to 0.3 100

It may be noted that idle length of canal should be kept minimum while considering
the alignment of canal and rock strata should be avoided.
244 Irrigation Engineering and Hydraulic Structures

11.4 LOSSES AND TYPES OF CROSS-SECTION FOR CANAL


1. Losses in Canals: Right from head works where water enters the canal to
its destination upto fields where it is utilized for irrigation purposes, canal
water undergoes loss, which is known as transit, or transmission loss and
is around 25 to 45 per cent of water that enters the canal at the head works.
These losses are due to
(i) Evaporation
(ii) Transpiration through vegetation on banks
(iii) Seepage through (a) percolation and (b) absorption
Loss on account of evaporation and transpiration is minor compared to seepage
loss, it is of the order of 1% minimum and 10% maximum of the total transit losses.
Depending upon permeability of soil and position of water table, seepage loss
may occur in two ways:
(a) Percolation: When Ground water table is close to ground level, top flow
line meets the water table forming a zone of fully saturated soil below canal
water way section. This is percolation loss and it is dependent on difference
between FSL and GWT. (See Fig. 11.2).
F.S.L
HG
HG
Top flow
GL line D

Fully saturated soil


Ground water table

Fig. 11.2: Percolation loss in canal

(b) Absorption: When the water table is at a considerable depth, top flow line
does not join it, but a small zone of saturation is formed around the water
way section of the canal, and seepage loss will depend only on FSD (full
supply depth) of the canal. This is loss on account of absorption and is small
compared to percolation loss.
Percolation loss may be around three times higher than absorption loss. Canal losses
can be measured by a method known as inflow and outflow method. In this method
a long reach of canal is chosen and except main flow all other outlets are closed.
Discharge measurements are taken at inlet and outlet of the reach and difference
between these values of discharge is the loss occurring in the reach. In the alluvial
soil zone of North India, values of total transmission losses are 17% in main canal
and branch canal, 8% in distributaries and 20% in field channels making a total of
45% of water supplied at the canal head works.
Distribution System 245

2. Types of Canal Sections: Canal sections are usually of following three


varieties depending upon site conditions of the track:
(a) Canal in full cutting
(b) Partly in cutting and partly in filling
(c) Fully in filling i.e., either in low embankment or in high embankment
These are illustrated in the Fig. 11.3(i) to (iii).
A canal is said to be in cutting when ground level is above FSL of the canal. A canal
is said to be in filling when canal bed level (CBL) is higher then ground level (GL).
A canal is partly in cutting and partly in filling when GL is in between FSL and CBL.
Top width of bank as recommended in IS 7112 of 1973 are as shown in Table 11.3.

Fig. 11.3 (i): Canal section in full cutting

Road width
0.5m Dowel F.S.L
m Free board 1.5:1
1:1 1:1
GL 1.5:1 0.5:1 D
2m 2D 2D
C.B.L.
B
Fig. 11.3 (ii): Canal section partly in cutting and partly in filling

Road width 0.5m Dowel


F.S.L.
1.5:1 Free board 1.5:1
1.5:1
1.5:1 1.5:1 D
3D 3D
GL C.B.L.
Hydraulic gradient line B C.B.L. is higher than GL

Fig. 11.3 (iii): Canal in high embankment

Table 11.1: Top Width of Bank

Q cumec Inspection Bank (Road) Non-inspection bank


Upto 7.5 5.0 m 1.5 m
7.5 to 15 6.0 m 2.5 m
15 to 30 7.0 m 3.5 m
246 Irrigation Engineering and Hydraulic Structures

3. Balancing Depth: (see Fig. 11.4) A canal section partly in filling and partly
in cutting is preferred so that material excavated may be used for filling the
banks. If for a canal section, depth of cutting is such that quantity of excavation
is equal to earth filling required in the banks for a given reach then this depth of
cutting is known as balancing depth. This is quite economical and saves time of
construction.

t t

Filling
n:1 n:1
F. S. L. n:1
n:1
h
D Cutting
m:1 y
m:1

Fig. 11.4: Balanced depth of cutting

Let y be the depth of cutting and (h–y) be depth of filling for a given reach of
unit length.[see Fig. 11.4].
Area of cutting = ( B + my ) y
Area of filling = 2 [ t + n(h − y)] (h − y)

Equating these two areas we get

( B + my ) y = 2 [ t + n(h − y)] (h − y)
∴ y 2 (2n − m) − (B + 4nh + 2t)y + h(2t + 2nh) = 0

A canal has usually side slope of 1:1 in cutting and hence m = 1, whereas a side
slope of 1.5:1 in filling, i.e., n = 1.5
Putting these values, we get
2y 2 − (B + 6h + 2t)y + h(2t + 3h) = 0
By putting given values of B, h, t etc,
y = balancing depth can be obtained from above equation.

11.5 DESIGN OF ALLUVIAL CHANNELS


A lined canal is a rigid boundary canal and can be designed as per laws of hydraulics
but an unlined canal has movable bed and sides and as such has to be designed as
per principles of loose boundary hydraulics. A change in discharge will cause only
a change in depth of flow for rigid boundary canals whereas in case of non-rigid
Distribution System 247

boundary canals it will cause changes in cross-section, bed slope, bed forms and
roughness coefficient. The application of the theory of rigid boundary canals to lose
boundary canals therefore is not possible. Empirical approach based on experimental
observations is therefore adopted. Number of talented PWD engineers have worked
extensively on this problem during British regime, Chief among them are:
(i) R. G. Kennedy (1895)
(ii) Lindley (1919) and
(iii) Gerald Lacey (1945)
A channel or river flowing through non-cohesive sediment material generally consisting
of silt, sand and gravel is called an alluvial channel or river. Upper Bari Doab canal
system situated in west Punjab, is a­ lluvial type and RG Kennedy made number of
observations on this canal system and arrived at f­ ollowing conclusions:
(i) Sediment in alluvial channel is kept in suspension solely by vertical
component of the eddies which are formed along channel bed.
(ii) Eddies formed due to sides of the channels do not have sediment supporting
power.
(iii) Sediment supporting power is thus proportional only to depth of channel and
not to its wetted perimeter.

11.5.1 Kennedy’s Method


According to Kennedy,
Vo = 0.55 D0.64
Where Vo = critical velocity in m/s = non-silting and non scouring velocity
and D = depth of flow in m.
According to him bed with of canal does not influence Vo, but silt grade has some
effect. To take care of effect of silt grade, he introduced in his above equation, a
factor m called critical velocity ratio, CVR;
∴ Vo = 0.55 m D0.64

Value of m = 0.8 to 1.2 as per silt grade


For calculation of mean velocity V, Kennedy suggested use of Kutter’s equation
or Manning’s equation.
As per Kutter’s equation,
1  0.00155 
+  23 + 
N  S 
V= RS
 0.00155  N
1 +  23 + 
 S  R
248 Irrigation Engineering and Hydraulic Structures

A
Here R = = hydraulic radius
P
P = Wetted perimeter
S = bed slope
N = rugosity coefficient = 0.025 to 0.03
1 2 3 12
As per Manning 's , V = R S
n
Here n = Manning's coefficient which is usually 0.014 to 0.02

Limitation of Kennedy’s theory:


B
(i) Kennedy did not consider   ratio in the design of alluvial channels.
D
(ii) No equation for bed slope is given which may lead to varied designs.
(iii) Silt charge or silt grade was not given due weightage.
(iv) Value of m decided arbitrarily.
(v) Design of channel by his method is based on trial and error i.e., one has
to assume D and find V by V = 0.55 mD0.64 and then using mean velocity
equation of Kutter or Mainning he has to prove that assumed value of D is
alright.

11.5.2 Lindley’s Equation


In view of limitation of Kennedy’s theory, Lindley in 1919 gave following equation
for stable design of alluvial channels:
V = 0.567 D0.57
B = 7.86 D1.61
He gave equation for bed width B, but did not give equation for bed slope.
However, he was first to put forward the concept of regime for alluvial channels.
As per concept of regime, the dimensions: bed width B, depth of flow D, and bed
slope S of alluvial channel carrying given discharge are all function of time. British
engineers working in Punjab around 1910 to 1920 introduced the term “regime”.
They studied behaviour of some such structures of existing canals where the bed
was in a state of stable equilibrium. These stable reaches had not required any
sediment clearance for several years of canal operation. They called such canals as
“regime channels”. This means that the term regime was used to express no change
in initial condition of bed of channel even after working for several years. In other
words when a channel is constructed in alluvial soil, carrying silt laden water, its
bed and sides will silt and scour until the width, depth and bed slope attain a state of
Distribution System 249

balance and the channel is then said to be in “regime”. Lacey followed this concept
in development of his theory.

11.5.3 Lacey’s Regime Theory


Based on his observation, Lacey (1945) found that eddies generated from bed as
well as from sides of the channel, in the direction normal to surface of generation,
keep the silt particles in suspension. So Lacey considered hydraulic mean depth
(R =A/P) as more pertinent variable rather than depth of flow as considered by
Kennedy. He also introduced a silt factor, f. Hence, according to Lacey,
2
V= f R (11.1)
5
V = mean velocity of flow in m / s
f = silt factor
A
R = = hydraulic mean depth
P
A = c / s area of flow
P = wetted perimeter
He also gave second equation,
Af 2 = 140V5 (11.2)
Multiplying both sides by V,
∴ AVf 2 = 140V 6 = Qf 2
1
 Qf 2  6
∴V =   (11.3)
 140 
P = 4.75 Q (11.4)
1
 q2  3
R = 1.35   (11.5)
 f 
In equation (V), if q = flood discharge per unit width of canal or river then R
stands for scour depth.
5
f 3
S=  (11.6)
 1 
3340  Q 6 
 
S = bed slope
With a set of above six equations, design of stable alluvial channel can be
carried out accurately.
250 Irrigation Engineering and Hydraulic Structures

q SOLVED EXAMPLES
Example 11.1: Design an irrigation canal to carry 36 cumecs with B/D ratio of
2.5. Take CVR = 1, and Kutter’s regosity coefficient =0.025.
Solution:
Assuming bed slope as l in 4000, and using Kennedy’s equation,

V0 = 0.55 m D0.64 Here, m = CVR = 1.

= 0.55 (1) D0.64

1/2:1 1/2:1 D
n:1 n:1

Fig. 11.5: Unlined alluvial canal (trapezoidal section)

For trapezoidal section, as shown in Fig. 11.5,


A = (B + nD)D
 D 1
=  B +  D, for sideslope of H :1V
 2 2
Now Q = AVO
 D
36 = D  B +  Vo
 2
B
= 2.5 and Vο = 0.55 D0.64
D
∴ B = 2.5D
 D
(
∴ 36 = D  2.5D +  0.55 D.64
 2
)
= D 2.64 (3) (0.55)
1
 36  2.64
∴D =  
 3 × .55 

= (21.8)0.378 = 3.2m
∴ B = 8.00m
 3.20  2
∴ A =  8.00 +  3.20 = 30.72m
 2 
Distribution System 251

 5 
P =  B + 2 D  = 15.15m
 2 
A 30.72
R= = = 2.02 = 2
P 15.15
1  0.00155 
+  23 + 
.023  S  RS ,
V=
 0.00155  N
1 +  23 + 
 S  R
1
with S =
4000
43.5 + 23  1 
V= × 1.41 ×  
0.023  63.24 
1 + (23)
1.41
66.5 1.41 68.4
= × = = 1.08 m / s
1.37 63.24 63.24
V0 = .55(D)0.64 = .55(3.2)0.64 
= 0.55 × 2.1 = 1.15 > 1.08
Hence, try S as 1 in 3600, in equation (1) above.
68.4
V = = 1.14, which is nearer to value of V0 =1.15
60
Hence, S as 1 in 3600 is suggested.
Example 11.2: Design a regime channel to carry a discharge of 40 cumecs and
silt factor 1.0.
Solution:
Use Lacey’s theory,
1 1
 Q f 2  6  40 × 1  6
V= =  = 0.81 m / s
 140   140 
 
Q 40
A= = = 49.38 m 2
V 0.81
5  V 2  5  0.812 
R=  =   = 1.64 m
2  f  2  1 
1
P = 4.75 Q = 4.75 ( 40 ) 2 = 30m
1
For trapezoidal channel having side slope as :1,
2
P = B + 5D and A = (B + .5D)D
252 Irrigation Engineering and Hydraulic Structures

∴ 30 = B + 2.23D ∴ B = 30 − 2.23D
 D
49.38 =  B +  D
 2
 D
Subtituting for B, 49.38 =  30 − 2.23D +  D
 2
49.38 = 30 D − 1.7D 2
∴ D 2 − 17.64 D + 29 = 0
(
∴ D = 17.64 ± 311 − 116 / 2 )
17.64 − 13.96
= , positive sign will be unfeasible.
2
∴ D = 1.84 m
B = 30 − 2.23(1.84) = 25.9 m.
5
f 3 1 1 1
S= 1
= 1
= =
6 6 3340 × 1.849 6176
3340 Q 3340 (40)

Example 11.3: Design an irrigation channel by Kennedy’s theory to carry


1
20 cumecs. Take n = 0.02550 in Manning’s equation, m = 1, B/D = 5, side slope :1 .
2
Solution:
 D
Vo = 0.55 D.64 A =  B +  D, here B = 5D,
 2
= 0.55(2)0.64 ∴ A = (5D + 0.5D) D
= 0.85 m / s. ∴A = 5.5 D 2
Q = AV ∴ 20 = (5.5D 2 ) (0.55 D0.64 )
= (3)D 2.64
1
 20  2.64
hence, D=  = 2m
 3 
B = 5D = 10m
A = 5.5D 2 = 22, and P = B + 5D = 14.47
A 22
R= = = 1.52
P 14.47
1 2 1
V = R 3S 2
n
Distribution System 253

2
1 2 1  0.021 
0.85 = (1.52) 3 S 2 ∴ S =  
0.025  1.32 
= l in 3951

B =10 m, D = 2 m

Example 11.4: Design a channel based on Lacey’s theory for a design discharge
of 12 cumecs. Take silt factor = 1.0

Solution: ∴ D = 1.28 m.

∴ B = 16.45 − 2.23(1.28) = 13.6 m.


5
f 3 1
S= 1
= 1
= 1 in 5053
3340 Q 6 3340(12) 16

11.6 WATERLOGGING
Waterlogging is due to rise of ground water table beyond permissible limits. With
increase in irrigation potential, this problem gets developed. Land subjected to
water-logging does not give expected yield. About 5 million hectares (3.3 %) of
culturable area in India suffer of this problem.
A land is said to be waterlogged when soil strata within the root zone of plants
gets saturated, this causes insufficient air circulation and plant’s growth get affected
adversely. Impermeable soil stratum has higher capillary rise and so gets waterlogged
more often than permeable soil stratum. If ground water table is within 1.5 m from
ground surface, waterlogging may result. Water table depth of 0.9 m to 1.2 m is
considered very dangerous for plant growth and needs immediate remedial measures
for control of waterlogging. Presence of alkali salts further worsens the situation.

11.6.1 Causes of Waterlogging


Natural balance between inflow and outflow get greatly disturbed due to excessive
irrigation facilities introduced in that area. Percolation of surface water on fields
for irrigation and also seepage from canal system add to ground water reservoir
resulting in rise of water table. Effective methods of cultivation and irrigation
practices and blockage of natural drainage of surrounding ground add to intensify
the problem of waterlogging.

11.6.2 Effects of Waterlogging


(i) Waterlogging generally creates conditions of poor aeration in root zone
of soil and is responsible for accumulation of salts near ground surface.
Uncontrolled weed growth and difficulties in cultivation process are
254 Irrigation Engineering and Hydraulic Structures

additional ill effects of water logging. Surrounding area may suffer of


mosquito growth resulting in poor health condition of persons staying
in such areas. Water logging prevents free circulation of air in the root
zone and thereby affects chemical process and bacterial activities, which
are essential for proper growth of plant, and hence crop yield is reduced
considerably. Also preparation of land i.e., tillage etc in wet condition is
difficult and hence expensive. Weed growth at the cost of plant growth
may further decrease crop yield.
(ii) Salt efflorescence: High water table may cause upward rise of capillary
water and this water will get evaporated early as it is nearer to the surface.
As a result, chemicals, particularly alkaline salts brought by water from
deeper soil depths get accumulated in root zone of soil masses, making the
soil infertile. Thus waterlogging first creates marshy land and then after if
unattended results in baren land. Hence, waterlogging should be avoided at
any cost.

11.6.3 Remedial Measures to Control Waterlogging


Waterlogging can be prevented if:
1. Methods adopted for irrigation should supply only that much water, which is
required for healthy plant growth. This is possible by adopting drip irrigation
system rather than irrigation by furrow methods.
2. Intercepting drains must be provided to cause effective drainage of irrigated
lands.
3. FSD of canals be kept as low as possible.
4. Obstruction in natural drainage of areas is removed.
5. Running canals by rotation and adopting varabandhi will minimize wastage
of water and hence reduce unwanted percolation in irrigable areas.
6. Rotation of crops will also generate fertility of soils.
7. Drains should be provided along canal lengths with high embankments so
that they collect seeping water and prevent its entry into the ground so that
undue rise in ground water table may not take place.
8. For every crop there is optimum water requirement for maximum yield, and
farmers should be made aware of this fact and advised to practice irrigation
accordingly.
9. Above all good management of irrigation water is surest means of controlling
waterlogging, for, prevention is always better than cure.
10. If canal lining is carried out for entire canal system of the project, seepage
losses will be minimised and water logging can be avoided.
Distribution System 255

11. Closed drain system provided below ground water table such as to see that
water table does not rise above permissible limits is effective remedial
measures for waterlogging but it is very costly.

11.6.4 Design of Closed Drain System for Effective Drainage of


Waterlogged Areas
Q
If Q D = total discharge per unit length of drain then D is the discharge that enters
the drain from either side. (see Fig. 11.6). 2

L 
−x
QD  2
∴ Qy =  
2  L 
 2 
QD
= (L − 2x)
2L
dy
Q y = Ky according to Darcy 's Law
dx

Fig. 11.6: Drainage system to reduce waterlogging

Above equation for QD shows that:


1. Spacing (L) of closed drains is independent of drain size.
2. Increase in drain diameter will make only a little increase in drain discharge.
3. Spacing of drains (L) is usually 1.5 m to 4.5 m c/c and drain diameter is
between 20 cm to 30 cm.

11.7 CANAL LINING


Canals for irrigation purposes have been kept unlined since the inception of irrigation
projects in India. This is mainly due to exorbitant cost towards lining, for in any
irrigative project total length of canal system is quite large and proportion of project
cost consumed by canal system is also as large as 60% to 70% of the total cost of
project.
256 Irrigation Engineering and Hydraulic Structures

Considering advantage of unlined canals, low initial cost is the only factor in
its favour, but disadvantages are many:
1. In unlined canals, non-scouring and non-silting velocity is required to be
maintained very low, around 0.6 m/s to 0.8 m/s, and hence cross-section area
of such canals are quite large.
2. B/D ratio is kept high so that bed width B is around ten times depth of
flow, D. This helps in keeping FSD low, but because of large bed width land
acquisition required is large and is responsible for heavy seepage losses.
3. Weed growth on unlined canal is very heavy and retards its flow and calls for
repeated maintenance works.
4. Bank may get breached due to erosion or due to burrowing animals or due to
both.
As against these disadvantages of unlined canals, lined canals have got many
advantages:
1. It reduces seepage losses and hence reduces chances of waterlogging.
2. Since losses are minimized, more water is available for irrigation purposes.
3. It is quite safe against breaching and weed growth is prevented to a minimum.
4. Hence, maintenance cost is minimum.
5. For well irrigation, lined canals are provided to reduce cost of pumping of
water.
6. Flow velocity permissible is around 1.5 m/s to 2.5 m/s and hence cross-sectional
area required is low.

11.7.1 Economics of Canal Lining


1. Annual Benefits: Let C1 be water charge for irrigation purposes per cumec
of water used. If m cumces of water is saved due to lining of canals, then
benefit due to lining is mC1 Rs. Lining will also reduce maintenance cost.
From previous records, if C2 = annual cost of maintenance of unlined
channels, then saving achieved due to lining is nC2, where n = percentage
reduction in maintenance cost on account of lining. Hence, total annual
benefits = (mC1 + nC2) Rs.

2. Annual Costs: If capital expenditure required on lining is C Rs. and lining


( )
life is Y years, annual depreciation will be C Y Rs. If i is the rate of interest
i
on capital investment, then annual interest charges = C .
100
This interest will have impact of decreasing value of capital due to
depreciation and hence considering average rate of interest,
Distribution System 257

C i 
Annual interest charges =  
2  100 
Hence total annual investment for lining
C C i 
= +  
y 2  100 
Annual Benefits
∴ Benifit cost Ratio =
Annual Costs
mC1 + nC2
=
1 i 
C + 
 y 200 
≥ 1.
For project feasibility, benefit cost ratio should be greater than one, or at
least one. In addition to above direct benefits of savings of water and reduction
in maintenance cost, there will be intangible benefits such as prevention of water
logging, reduced costs of drainage of lands, reduced costs of breaching work
and prevention of public health hazards such as malaria. Hence, canal lining is
economical.

11.7.2 Types of Lining


Lining work can be of following different types:
1. Concrete Lining: This is the best type of canal lining. It can last around
30 years, and hence though initial cost is high, it is cheaper in long run.
Canal banks slopes are kept self supporting i.e., 1.5 H: 1 V so that lining
work is not required to bear earth pressure. Thickness may vary from
7.5 cm to 12 cm, but in India minimum thickness of 10 cm is generally
adopted. Concrete linings are laid without formwork and so workability
of concrete should be quite good. Reinforcement provided is upto 0.4% of
area in longitudinal direction and upto 0.2% of area in transverse direction.
Reinforcement in lining prevents cracking in lining and provides a tie
between adjacent section of lining. Lining may suffer of shrinkage cracks
but this can be overcome by proper construction joint. Cracks can be sealed
with asphaltic compounds. Present practice is not to provide reinforcement
in lining but well constructed concrete only will do the job. However, for
large canals lining needs reinforcement.
2. Shotcrete Lining: Shotcrete lining is constructed by applying cement mortar
pneumatically to the canal surfaces. Thickness is around 5 cm and this type of
lining is good for small canals. No subgrade is required and reinforcement is also
not provided. Hence, it is the cheapest type. It is preferred on curved alignment of
canals.
258 Irrigation Engineering and Hydraulic Structures

3. Brick Lining: Bricks are laid in layers of two with about 1.25 cm of 1:4
cement mortar sandwitched in between. This type has been used extensively
in North India and is quite cheap compared to concrete lining. The Sarda
Power canal has been lined with bricks.

4. Asphaltic Lining: Most commonly used asphaltic linings are:


(i) Asphaltic concrete and
(ii) Buried asphaltic membrane
(i) Asphaltic concrete is a mix of asphalt cement, sand and gravel mixed
at a temperature of 110° C and is placed with laying equipments. The
lining is compacted with heavy iron plates while hot. Asphaltic concrete
lining is smooth, flexible and erosion resistant. It is not suitable for hot
climatic conditions.
(ii) Buried asphaltic membrane can be of two types:
(a) Hot sprayed asphaltic membrane,
(b) Prefabricated asphaltic membrane
(a) A hot sprayed asphaltic membrane is constructed by spraying hot
asphalt on the subgrade to make a layer of 6 mm thick on which
when it is cool a 30 cm layer of earth is provided and due to this,
canal sections are over excavated. The lining is flexible and gets
adapted to the subgrade surface.
(b) Prefabricated asphaltic membrane is prepared by coating rolls of
heavy paper with 5 mm layer of asphalts or 3 mm of glass fibre-
reinforced asphalt. These rolls are laid on subgrade and then
covered with earth material. Maintenance cost of such type of
lining is high.
5. Earth Linings: Different types of earth linings are being used for irrigation
canals. Their initial cost is low but requires high maintenance expenditure.
These earth linings can be of following types.
(a) Stabilized earth lining
(b) Compacted earth lining
(c) Buried bentonite membrane
(d) Soil cements linings.
Use of earth linings is restricted to small irrigation canals of capacity
lower than 10 cumecs and flow velocity permissible is only 1 m/s.
Distribution System 259

11.8 DESIGN OF LINED CANALS


There are two types of sections, which are adopted for lined canals:
(i) A triangular section with circular bottom
(ii) A trapezoidal section with rounded corners.
For canals having discharge less then 85 cumecs and velocity of flow limited
to 1.8 m/s, triangular section with rounded bottom is adopted, whereas for canals
having discharge greater then 85 cumecs trapezoidal section is adopted. IS 10430 of
1982 recommends only trapezoidal section with rounded corners for all discharges.
For concrete lined canals limiting velocity of flow is recommended as 2.7 m/s and
for bricks lined canals it is allowed upto 1.8 m/s. Value of Manning’s constant n, for
lined canals may vary from 0.015 to 0.03.
(i) Trapezoidal section with rounded corners: (see Fig. 11.7)
Q = AV D = FSD
1 2 1
V by Manning = R 3 S 2 , n = 0.015 to 0.03.
n
3
 Vn  2
R= 1 
S 2
A Q
R= and A =
P V
A = central rectangular potion + triangular portion on each end
1  1
A = BD +  θD.D 2 + ( D cot θ D ) 2
2  2
∴ A = BD + D 2θ + D 2 cot θ = BD + D 2 (θ + cot θ)
and P = B + 2Dθ + 2D cot θ = B + 2D(θ + cot θ)

F. S. L

90O θ θ 90O
D cot θ D D cot θ

Dθ Dθ
B

Fig. 11.7: Lined canal (trapezoidal section)

Since A and P are known, B and D can be determined.


260 Irrigation Engineering and Hydraulic Structures

(ii) Triangular section with rounded bottom: (see Fig. 11.8)


F. S. L.
0
D D
90O
90O D D cot 0

D0

Fig. 11.8: Triangular section with rounded bottom

For ∆lar section,

A = D 2 (θ + cot θ)
P = 2D(θ + cot θ)

D = FSD
A D
R= =
P 2

q SOLVED EXAMPLES
Example 11.5: Design trapezoidal shaped concrete lined canal to carry 100
cumecs of flow at a bed slope of 3 × 10-4. The side slopes are 1.5 : 1 and N = 0.015
with limiting value of flow velocity = 2 m/s.
Solution:
3
3   2
 Vn  2
 2 × 0.015 
R = 1  =  1 
= 2.28m
 2 
S 
( )
 3 × 10−4 2 

Q 100 A 50
A= = = 50m 2 , P= = = 22m.
A 2 R 2.28

For trapezoidal shaped section, θ = cot −1 (1.5) = 0.56 radians

A = BD + D 2 θ + D 2 cot θ
50 = BD + D 2 (0.56) + D 2 (1.5) = BD + D 2 (0.56 + 1.5)
∴ 50 = BD + 2D 2
P = B + 2D (θ + cot θ)
22 = B + 2D (0.56 + 1.5) = B + 4D
∴ B = 22 − 4D
Substituting value of B in eqn (i),
50 = (22 − 4D)D + 2D 2 = 22D − 4D 2 + 2D 2 = 22D − 2D 2
A = BD + D θ + D cot θ
50 = BD + D 2 (0.56) + D 2 (1.5) = BD + D 2 (0.56 + 1.5)
∴ 50 = BD + 2D 2 Distribution System 261
P = B + 2D (θ + cot θ)
22 = B + 2D (0.56 + 1.5) = B + 4D …(1)
∴ B = 22 − 4D
Substituting value of B in eqn (i),
50 = (22 − 4D)D + 2D 2 = 22D − 4D 2 + 2D 2 = 22D − 2D 2 …(2)

D 2 − 11D + 25 = 0
11 ± 121 − 100
∴D =
2
11 − 4.58
= , neglect positive sign = 3.2m
2
∴ B = 22 − 4D
= 22 − 4(3.2) = 9.2m

Example 11.6: Design concrete lined canal having triangular shape section with
rounded bottom to carry a discharge of 20 cumecs. Take side slope as 1.25 : 1 and
Manning’s N = .016 and S = I in 8100
Solution:
A D
R= = ,
P 2
cot θ = 1.25 , (since side slope given is 1.25:1)
A = D 2 (θ + cot θ) = D 2 (.675 + 1.25 )
…(1)
A = 1.925D 2
1 2 3 12
V= R S
n
2 1
1 D 3  1  2
=    
0.016  2   8100 
2 …(2)
D 3
∴V =
2.29
…(3)
Q 20
Also V = =
A 1.925D 2
2
D 3 20 …(4)
∴ =
2.29 1.925 D 2
8
∴D 3 = 23.8
3
∴D = ( 23.8 ) 8 = 3.2m.
262 Irrigation Engineering and Hydraulic Structures

EXERCISES
1. What are different types of canal sections? Give sketches and explain their
importance.
2. State factors that influence determination of canal alignment. What is
importance of fall in canal alignment?
3. State types and purposes of canal linings.
4. Compare and contrast Kennedy’s and Lacey’s silt theories. Which is more
practicable? Why?
5. Design an irrigation canal to carry 18 cumecs. Use Kennedy’s theory.
Take Manning’s n = 0.0225, CVR = 1,
B / D = 5, side slopes ½ H to IV.
[Ans: B = 9.8 m, D = 1.96 m, So in 2570.]
6. What is balancing depth of cutting?
7. How curves are provided in canal alignment ?
8. Design a channel using Lacey’s theory to carry a discharge of 60 cumecs.
Take silt factor = 1.0 and side slope ½ : 1.
[Ans: B = 32 m, y = 2.16 m, S is l in 6608]
12
Canal Structures

12.1 DISTRIBUTION AND MEASUREMENT STRUCTURES


FOR CANAL FLOWS
The flow of a main canal bifurcating into a branch canal with the rest flowing
downstream is controlled with the help of a cross regulator across the parent
canal and a head regulator across the branch canal. At times, the flow of a canal
divides into two or three smaller branch canals without any regulating structure by
designing the entrance of the canals in such a way that the flow enters each branch
canal proportionate to its size. Again, from a canal, outlet structures may take out
water for delivery to the field channel or water courses belonging to cultivators.
These outlet works, of course, are generally not provided on the main canal and
branches, but are installed in the smaller distributaries. Apart from these, there
could be a need to measure the flow in a canal section and different structures have
been tried, mostly based on the formation of a hydraulic jump and calibrating the
discharge with the depths of flow. Typical structures of these kinds are graphically
represented in Fig. 12.1 and this lesson deals with each type in detail.

12.2 CANAL OUTLETS


Cultivators get water for irrigation purposes from distributary channel to their
watercourse. An outlet is a structure through which water is admitted from distributary
into a water course or a field channel. It also acts as a water measuring device.
Cultivators prefer an outlet which supplies water at constant discharge, whereas, the
canal management would prefer outlets that can supply water at variable discharge
so that tail end of distributary is neither flooded nor dried. Usual types of canal
outlets are:
1. Non-modular Outlets or simple pipe outlet
2. Semi-modular Outlets and
3. Modular Outlets

© The Author(s) 2023 263


S. K. Ukarande, Irrigation Engineering and Hydraulic Structures,
https://doi.org/10.1007/978-3-031-33552-5_12
264 Irrigation Engineering and Hydraulic Structures

1. Non-modular Outlets: These types of outlets supply water on the basis


of difference of water levels in distributary and the field channel. Thus,
discharge through non-modular outlets varies over a wide range with
variation in the water levels of distributary and watercourse. A shutter at its
upstream end controls such an outlet. It cannot ensure equitable distribution
of water to cultivators. Loss of head in non-modular outlets is less than that
in modular outlets.
2. Semi-modular Outlets: Discharge through these outlets depend only on
water level of distributary and is independent of water level in field channel
subject to a minimum working head required for its working. A semi-module
is thus more suitable for equitable distribution of water to cultivators. It has
however a greater loss of head than in other types.
3. Modular Outlets: These outlets work independently of water levels in
distributary and that in water course within a reasonable working limit. They
may have or may not have moving parts. They are known as rigid modules if
they do not have moving parts. These outlets are preferred on a branch canal
for supplying water to distributary.

12.2.1 Parameters Controlling Behaviour of Outlets


1. Flexibility, F: It is a ratio of rate of change of discharge at outlet to rate of
change of discharge at distributary.
dQ 0 / Q π Q0 = discharge through outlet
∴F =
dQ / Q Q = discharge through distributary

Q = C1D n and Q0 = C2 H m
Here D = depth of flow in distributary
And H = head on the outlet. C1 , C2 are constants and m and n are indices
dQ dD dQ0 dH
∴ =n and =m
Q D Q0 H
m D dH
∴F = × ×
n H dD
For semi-modular outlet, dH = dD
 m  D
∴F =    
 n   H
When a certain change in the distributary discharge causes a proportionate
change in outlet discharge, the outlet is said to be proportional and is semi-
modular type.
∴ For Proportinal semi-module, F = 1
H m
∴ =
D n
Canal Structures 265

H
The ratio is a measure of location of outlet and is termed setting.
D
If F = 1, outlets are classified as proportional
F > 1, hyper proportional and
F < 1, sub-proportional
2. Sensivity, S.: The Sensivity, S, is defined as ratio of rate of change of
discharge at outlet to rate of charge of depth of flow in the distributary.
dQ0 / Q0
∴S =
dD / D
dQ0 / Q0 dQ0 / Q0
F= =
dQ / Q n (dD / D)
1
= S
n
∴ S = nF
Flexibility of a modular outlet is zero and hence its sensivity is also zero.

12.2.2 Non-Modular Pipe Outlets

1.5:1 1.5:1
F. S. L.
(H) F. S. L. of
Water course
Distributing Q D
Pipe of dia. d Q0

Fig. 12.1: Non-modular pipe outlet

Let, H = difference in water level of distributary and watercourse then

V 2 f LV 2 V 2
H = 0.5 + +
2g 2gd 2g

V2  fL 
H= 0.5 + d + 1
2g  
1
 d  2
∴ V = 2gH  
1.5d + fL 
Q0 = discharge of pipe outlet
1
π   d  2
= AV =  d 2  2gH  
4   1.5d + fL 

∴Q0 = CA 2gH so that


H= 0.5 + + 1
2g  d 
1
 d  2
∴ Vand
266 Irrigation Engineering = Hydraulic
2gH  Structures
1.5d + fL 
Q0 = discharge of pipe outlet
1
π   d  2
= AV =  d 2  2gH  
4   1.5d + fL 

∴Q0 = CA 2gH so that


1
 d  2
C= 
1.5d + fL 

q SOLVED EXAMPLE
Example 12.1: A non-modular pipe outlet is to carry a discharge of 25 lit / sec,
with a minimum head of 0.3 m from the distributary. Pipe length is 12 m, and
f = 0.01. Find diameter of the pipe.
Solution:
1
 d  2 π 2
Qo = CA 2gH =    d  2g (0.3)
 1.5d + fL  4 
25
= cumec.
1000
1
0.025 × 4  d  2
∴ =  (d 2 )
π 2g(0.3)  1.5d + 0.01(12) 
1
7.2 × 10−3 × (1.5d + 0.12 ) 2 = d 2.5
By trial and error method,
If d = 0.12, 3.9 × 10−3 ≠ 4.9 × 10−3
Try, d = 0.10, 3.7 × 10−3 ≈ 3.2 × 10−3
Try d = 0.11, 3.8 × 10−3 ≈ 4 × 10−3 , which can be accepted
Hence d = 0.11m
=11cm

12.2.3 Kennedy’s Gauge Outlet


This is a semi-modular type outlet, developed by Kennedy in 1906. It consists
of a bell mouth orifice discharging into a long expanding delivery pipe 3 m long
with a vertical air column above the throat. This arrangement makes the discharge
through the outlet independent of water level in field channel. The cultivators can
easily temper this outlet by blocking the air vent pipe to increase discharge through
the outlet. It is costly and hence not much in use.
Canal Structures 267

Air vent pipe

h1

H0 Supports
Distributary

Orifice

Throat Cast iron


pipe

Field channel
Fig. 12.2: Kennedy’s gauge outlet (semi-module)

12.2.4 Modular Outlet


There are following two types of rigid module outlets:
1. Gibb’s Rigid Module: Inlet pipe receives water from distributary and
through a rising spiral pipe it is supplied to eddy chamber. This arrangement
provides free vortex motion, so that
V × R = Constant
Hence in eddy chamber, outer curve of spiral pipe will have less velocity
and inner one has larger velocity, keeping, V × R = Constant. Baffles are
provided in eddy chambers to cut down excessive energy of flow and thus
ensure constant discharge into field channel. Thus, discharge through outlet
is kept independent of levels of distributory and field channel. Hence, it is
a modular type of outlet and since there are no moving parts it is known as
Gibb’s rigid module. But it is very costly and not much in use. (See Fig. 12.3).

Baffle Eddy chamber

Distributary
o
Rising spiral pipe (180 )

Field channel
Inlet pipe

Baffles
Rising spiral
Distributary o
pipe (180 )

Field channel

Fig. 12.3: Gibb’s module


268 Irrigation Engineering and Hydraulic Structures

2. Khanna’s Rigid Module: It is similar to orifice semi-module with an


additional provision of sloping shoots fixed in arched roof covering. These
shoots cause back flow and thereby keep the outlet discharge constant.
If shoots are blocked, outlet will continue to function as a semi module.
(See Fig. 12.4).

1.5:1 1.5:1 (H)


F. S. L. F. S. L. of
Distributing QD water course
pipe of dia. d Q0

Fig. 12.4: Khanna’s rigid module

q SOLVED EXAMPLE
Example 12.2: A semi-modular pipe outlet of 20 cm diameter, receives water
from distributary having bed level 200.3 and FSL 201.5 m. Maximum water level
in field channel is 201.15. Set outlet for maximum discharge and find its value.
Take C = 0.62 Indicate type of setting.
Solution:

d
H = ( 201.5 − 201.15 ) −  
2
H is measured from centre line of the pipe outlet
0.20
H = 0.35 −
2
= 0.25m
Q = CA 2gH
π 
= 0.62  d 2  2g ( 0.25) )
4 
π 
= 0.62  (0.20) 2  2g (0.25)
4 
= 0.043m3 / s
m D
F = Flexibility =
n H
1 5
m = and n = ; D = 201.5 − 200.3 = 1.2
2 3
m
∴ = 0.3 and H = 0.25
n
1.2
∴ F = 0.3 × = 1.44
0.250
Since F is greater than one, it is hyper proportional module
m D
F = Flexibility =
n H
1 5 Canal Structures 269
m = and n = ; D = 201.5 − 200.3 = 1.2
2 3
m
∴ = 0.3 and H = 0.25
n
1.2
∴ F = 0.3 × = 1.44
0.250
Since F is greater than one, it is hyper proportional module
H 0.25
setting = = = 0.208m
D 1.2

Falls:
Whenever natural ground slope is steeper than canal bed slope, the difference between
these two slopes is adjusted by a structure known as fall (Fig. 12.5).
The necessity of a fall is due to the fact that ground slope usually exceeds
designed bed slope of a canal. In its head reach, irrigation canal is usually in cutting
and soon after it meets with a condition when it has to be in filling. Irrigation canal
in filling is very costly both in initial cost as well as its maintenances. Hence,
structures known as falls are provided to keep the balance between cutting and
filling.

Fig. 12.5: Fall

Types of Falls
(i) Cylindrical fall or well type fall
(ii) Sarda type fall or vertical drop type fall and
(iii) Glacis fall

1. Cylindrical or Well Type Fall: It consists of an inlet well with a pipe


at its bottom, carrying water to downstream well to cistern. The d/s well is
required if fall is greater then 1.8 m or discharge is higher than 0.29 cumecs
(See Fig. 12.6).
270 Irrigation Engineering and Hydraulic Structures

Fig. 12.6: Cylindrical or well type fall

2. Sarda Type Falls: A raised crest fall with a vertical impact was first
introduced on Sarda Canal system in UP. In that area a thin layer of
sandy clay was found above a stratum of pure sand. If canal bed is to
be cut deep and touches sandy stratum seepage losses would be high.
Hence to keep depth of cutting low, large number of falls were required.
Vertical drop or Sarda type being economical is adopted. Here water
falls into a cistern and then enters d/s canal (See Fig. 12.7).
3. Glacis Fall: This differs from above as in the case, that after raised crest,
weir wall has a gentle slope of 2:1, known as glacis, such that hydraulic
jump occurs and energy dissipation due to hydraulic jump takes place.

Fig. 12.7: Vertical drop fall or sarda type fall

12.3 CROSS-DRAINAGE WORKS (C.D. WORKS)


Canals in head reach are normally canals in cutting and are aligned as contour canals.
Here they are required to cross a number of natural drainages or tributaries, and
structures are required to be provided at such crossing are known as cross-drainage
works. After initial reach of cutting, canal soon takes up as ridge canal and is in
filling. Here it is not required to cross many natural drainages but distributaries
from this canal may again be parallel to contour and hence distributaries may have
Canal Structures 271

to cross tributaries. Thus, number of cross-drainage works are provided on branch


and distributary canals. These cross drainage works are usually of three types:
(i) Super passage
(ii) Aqueducts
(iii) Level crossing
If a cross-drainage work is such that natural drainage is taken over the canal
then it is known as superpassage and if the natural drainage has to pass below the
canal then it is known as aqueduct. If natural drainage and canal are crossing each
other at same level, it is known as level crossing. Superpassages are normally
found in initial reach of canal head works whereas aqueducts are found after the
main canal bifurcates into branch and distributary canal system. Level crossing
are rarely provided. Among all the three types, the most commonly found cross-
drainage work is aqueduct.

12.3.1 Aqueducts
Aqueducts are of two different types:
1. Free Aqueducts: Free aqueducts have no uplift pressure on canal bed from
below due to HFL of natural drainage as HFL is considerably below CBL,
but if HFL is slightly above CBL, water of natural drainage will exert an
uplift pressure on canal bed and in that case aqueduct is known as siphon
aqueducts. Natural drainage floor is depressed below canal crossing so as to
minimize uplift pressure on canal bed and water way does not get obstructed
due to siltation of natural floor. (See Fig. 12.8)

Fig. 12.8: Siphon aqueduct

Aqueducts and siphon aqueducts are further classified into following three types:
Type I: Here no fluming in canal bed width is carried out and canal banks are
carried forward as they are. In this type width of CD work is large but expenditure
on canal wings and bank connections is saved. It is suitable for small streams of
low width and low flood discharge.
272 Irrigation Engineering and Hydraulic Structures

Type II: This type of aqueduct is similar to type I with a provision of retaining
walls to terminate outer slopes of canal banks there by reducing the length of the
CD work. This type is preferred for streams of medium magnitude.
Type III: In this type, canal banks are discontinued over its crossing length and
canal water is carried in rectangular RCC trough. The canal banks are connected
to respective trough walls on u/s and d/s side of the natural drainage by means of
wing walls. The canal bed width is also reduced and canal section from trapezoidal
type is transformed to rectangular trough. Transition lengths with expansion and
contraction are provided before and after the crossing as shown in Fig. 12.9.

12.3.2 Super Passage


CD work: where canal water passes below a natural drainage or stream the CD
work is known as super passage. Here FSL of canal is much lower than stream bed
level and if canal water is carried through a pipe it is known as irrigation siphon.
Super passage with canal FSL free of the bottom of stream are just aqueducts with
the only difference that here stream is above and canal is below. Irrigation siphons
are just pipe culverts carrying canal water below natural streams. Thus, irrigation
siphon is just opposite of siphon aqueduct.

Fig. 12.9: Aqueduct type III (Note Le=1.5 Lc)


Canal Structures 273

q SOLVED EXAMPLES
Example 12.3: Design a suitable C-D works for following data:

Canal Natural Drainage (ND)


Discharge = 18 cumecs Q = 200 cumecs
Full supply level = 213.5 HFL = 210
CBL = 212.0 and CB width = 10 m ND bed level = 207.5
Side slope = 1.5 H : 1.0 V G.L. = 212.5

Solution:
Since CBL is above HFL by 2m and ND is large, type III aqueduct is suitable:
(See Fig. 12.10)
Design:
Step I: Design of ND waterway
P = 4.75 Q = 4.75 200 = 67.2 m

Using 8 bays of 7m width, Lnet = 56 m.

Taking number of piers as 7, provide pier width = 1.5 m


ND bed width = 56 + (7 × 1.5) = 66.5 m
Provide end bays with 1.75 m width, so that
ND bed width = 66.5 + (2 × 1.75)
= 70 m , which is greater than P = 67.2 m , hence OK.
Step II: Design of Canal water way :

Canal bed width = 10 m = Bo


Let it be flumed to 5 m = Bf
Taking splay of 2 : 1, contraction length Lc will be,

 10 − 5 
Lc = 2   = 5m and expansion length Le will be,
 2 
Le = 1.5Lc = 5 × 1.5 = 7.5m

Lc and Le represent length of transition curves.


274 Irrigation Engineering and Hydraulic Structures

Step III: Design of Transition curves: (Fig. 12.10)


(ii) Contraction length, Lc = 5m
Using Mitra’s Hyperbolic Transition formula,
Bo Bf Lc
Bx =
Lc Bo − x ( Bo − Bf )
(10) (5) (5) 250
= =
5(10) − x(10 − 5) 50 − 5x
Assume x values between 0 to 5 and worked out Bx values are given in the
table below:
x 0 1 2 3 4 5
Bx 5 5.5 6.25 7.14 8.33 10

Fig. 12.10: Channel transitions

(iii) Expansion length, Le = 1.5 Lc = 7.5m.


Bo Bf Le
Bx =
Le Bo − x (Bo − Bf )
(10) (5) (7.5) 375
= =
7.5(10) − x (10 − 5) 75 − x(5)
Assume × values between 0 to 7.5 and worked out Bx values are given in the
table below:
x 0 1 2 3 4 5 6 7 7.5
Bx 5 5.36 5.77 6.25 6.81 7.5 8.33 9.375 10
Step IV: Design of trough:
Assuming constant depth of flow u/s to d/s,
d = depth in trough = FSD of canal
  = 213.5 – 212 = 1.5 m
Q 18 18
V= = = = 2.4 m / s. < 2.5 m/s
A 1.5 × 5 7.5
Canal Structures 275

Example 12.4: Design a suitable type of CD work for following data:


Canal ND
Q = 20 cumecs HFQ = 250 cumecs
B = 20 m HFL = 207 m
FSD = 1.5 m Bed level = 204 m
CBL = 206 GL = 206.5 m
Side slope 1½ : 1

Solution:
Since HFL is higher than CBL, siphon aqueduct is to be provided (see Fig.
12.8, 12.9 and 12.11)
Design Step I: Waterway
P = 4.75 Q = 4.75 250 = 75m.
Provide nine clear span of 7m each and pier width = 1.5 m
ND bed width = 8 × 1.5 + 9 × 7 = 75 m, which is equal to P, hence
Let V = 2 m/s of ND
250
then waterway area = = 125m 2
2
125
depth of flow = = 1.98m ≅ 2m
9×7
HFL = Bed Level + depth of flow
= 204 + 2 = 206 which is less than given HFL of 207 and hence
Step II: Canal Waterway
Bed width, B of canal = 20m
Let it be flumed to 10m, following a fluming ratio of 1:2
Provide a splay of 2:1 for contraction length
20 − 10
Lc = × 2 = 10 unit m.
2
and a splay of 3:1 for expansion,
20 − 10
Le × 3 = 15m. = 1.5Lc , OK
2
Bo Bf Lc
Bx c =
Lc Bo − x ( Bo − Bf )

20 (10) (10)
=
10(20) − x(20 − 10)

2000
=
200 − 10x
200
=
20 − x
20 (10) (15)
Bx e =
15(20) − x(20 − 10)
20 (10) (10)
=
10(20) − x(20 − 10)
2000
276 =
Irrigation Engineering and Hydraulic Structures
200 − 10x
200
=
20 − x
20 (10) (15)
Bx e =
15(20) − x(20 − 10)
3000
=
300 − 10x
300
=
20 − x
Assume × and work out Bxc, Bxe and enter in the following table:

x Bx c Bx e
0 10 10
2 11.1 10.7
4 12.5 11.5
6 14.2 12.5
8 16.6 13.6
10 20 15
12 – 16.6
14 – 18.75
15 – 20

Step III: Design of CBL at Various sections (4 – 4, 3 – 3, 2 – 2 and 1 – 1 as


shown in Fig. 12.11)

Fig. 12.11: Channel transitions


Canal Structures 277

At section 4 – 4,
When canal returns to normal section, i.e., beyond 4 – 4,
A = (B + 1.5D)D
  = (20 + 1.5 × 1.5)1.5
  = 33.375 m²
Q 20
V4 = = = 0.6 m / s
A 33.375
V42 (0.6)2
= = 0.0183 m.
2g 2g
RL of canal bed at 4 – 4, given as 206
RL of water surface at 4 – 4 = 206 + 1.5
= 207.5
RL of TEL at 4 – 4 = 207.5 + 0.018
= 207.518
At section 3 – 3
Assume constant depth of 1.5 m throughout
Area of trough = 1.5 × 10 = 15 m²
20
V3 = = 1.33m / s
15
V32
= 0.09 m.
2g
 V 2 − V42 
Loss of head in expansion = 0.3  3
 2g 
 
 1.76 − 0.36 
= 0.3   = 0.07 × 0.3 = 0.021
 2g 
RL of TEL at 3 – 3 = RL of TEL at 4 – 4 + Loss in expansion
= 207.518 + 0.021 = 207.521
RL of water surface at 3 – 3 = 207.521 – 0.09 = 207.431
RL of bed at 3 – 3 = 207.431 – 1.5 = 205.931
At section 2 – 2,
Loss between 2 – 2 and 3 – 3 as per Manning’s

n 2 V32 L
HL =
R4 3

(0.016) 2 (1.33) 2 (75)


=
(1.15) 4 3

2.56 × 10−4 × 1.76 × 75


=
1.2
= 0.0281
278 n 2 VEngineering
2
3L
HIrrigation
L =
and Hydraulic Structures
R4 3
(0.016) 2 (1.33) 2 (75)
      =    {Where, Manning’s n = 0.016
(1.15) 4 3
A 15
−4
R= =
2.56 × 10 × 1.76 × 75 P 10 + 2(1.5)
= }
1.2 15
= = 1.15
= 0.0281 13
RL of TEL at 2 – 2, = RL of TEL at 3 – 3 + Loss between 2 – 2 and 3 – 3.
= 207.521 + 0.028 = 207.549
RL of water surface at 2 – 2 = 207.549 – 0.09
= 207.459
RL of bed at 2 – 2 = 207.459 – 1.500
= 205.959
At section 1 – 1 : Loss due to contraction
 V 2 − V12 
= 0.2  2
 2g 
 
 1.33 − 0.62 
2
= 0.2   :( Where, V1 = V4 = 0.6
 2g
 
 1.4 
= 0.2   and V2 = V3 = 1.33)
 2g 
= 0.0143
RL of TEL at 1 – 1 = 207.549 + 0.0143 = 207.563
RL of water surface at 1 – 1 = 207.563 – 0.018 = 207.545
RL of bed at 1 – 1 = 207.545 – 1.5 = 206.045
Step IV: Uplift Pressure on slab of trough :
Assume trough slab thickness = 0.3 m
RL of bottom of trough slab = CBL – slab thickness
= 2.6 – 0.3 = 205.7
Assume afflux = 0.33 m, and loss of head on account of entry of flood water
below trough
V2 (2) 2
= 0.5 = 0.5 = 0.1
2g 2g
∴ Uplift on trough slab = HFL + afflux – entry loss – RL of bottom of slab trough
= 207 + 0.33 – 0.1 – 205.7
= 1.53 m of water
= 1.53 t/m²
Canal Structures 279

Downward load of concrete slab = 0.3 × 2.4 = 0.72 t / m3


(sp. gr. of concrete of slab = 2.4)
The balance of uplift pressure = 1.53 – 0.72 = 0.81 t / m², has to be resisted
by reinforcement to be provided at top of the slab thickness. If there is no flood
water in ND, then the slab of trough has to be designed for water load in canal +
trough dead weight, and this design is to be checked for HFL condition.

EXERCISES
1. Distinguish between
(a) Aqueduct and Superpassage
(b) Irrigation siphon and syphoned aqueduct.
(c) Modular outlets and non-modular outlet
(d) Fall and Weir
(e) Setting and Flexibility.
2. Design a suitable C-D work for following data :
(Ans: Syphon Aqueduct)

Canal Drainage
Q = 350 cumecs HFQ = 4500 cumecs
B = 28 m Bed Level 195 m.
FSD 6 m HFL 198.5 m
FSL 204 m Natural GL 198 m
CBL 198
13
Sediment Transport in
Alluvial Canals
13.1 TRANSPORT OF SEDIMENT IN ALLUVIAL CANALS
Sediment transport is a natural process coupled with flow of water either in rivers
or in canals. Transporting power of flowing water is always beyond prediction:
it can transport anything, right from small sediment particles to boulders, trees,
dead animal bodies and debris. The ill effect of this transportation is scouring of
bed and banks of canals and rivers and erosion of surfaces of hydraulic structures
such as spillways and bridge piers. A systematic study of transport of sediment
in canal is therefore of utmost importance. Many researchers have contributed in
this field such as Kennedy, Lacey, Ven Te Chow, Einstein, Shield, Peter – Meyer,
Du Bois etc. Early works are due to Kennedy and Lacey as has been pointed
out in Chapter 11 on Distribution System. In this section, summary of further
work in this field is detailed
What makes sediment move? The basic mechanism responsible for sediment motion
is drag force exerted by fluid flow on individual grains. The cumulative effect of all
such drag forces is retarding shear stress exerted by contact surfaces on the flow.
Determination of this bed shear is of prime importance in the problem of sediment
transport in alluvial channels.

13.2 THRESHOLD MOVEMENT


Threshold condition of movement of sediment particles along the bed cannot be
defined with absolute precision. The approach is based on dimensional analysis
and experimented studies. This has been carried out by (i) Shield, (ii) Einstein,
(iii) Meyer – Peter and (iv) Du – Bois which are given below:
Basic parameters involved in sediment transport are:
(i) τ0 = Shear Stress at Bed
(ii) ρs = Mass Density of Sediment Grain
(iii) ρf = Mass Density of Fluid Particle
(iv) d = Grain Diameter

© The Author(s) 2023 280


S. K. Ukarande, Irrigation Engineering and Hydraulic Structures,
https://doi.org/10.1007/978-3-031-33552-5_13
Sediment Transport in Alluvial Canals 281

(v)   g = Acceleration due to Gravity


(vi) µ = Fluid Viscosity
Six variables as listed above contain three fundamental dimensions in them
i.e., all L, M and T are present among these six variables. Hence, according to
Buckingham’s π theorem, (6-3) = 3 pie terms can be formed out of these six
variables. They are:

τ0 ρs d τ0 ρf
, ,
ρf gd ρf µ

τ0
Let v* = = shear velocity,
ρf
= Velocity which would give rise to shear stress, τ0
From Darcy – Weishbach equation for pipe flow, it can be shown that
v* f
= where v is flow velocity and f is friction factor.
v 8
τ0 τ
The first π1 term is but 0 = (v* ) 2
ρf gd ρf
(v* ) 2
∴π1 = is a kind of Froude Number, F* for sediment flow
gd
ρs
π2 = = specific gravity of sediment = Ss
ρf
For channel flow , τ0 = γ RS0 , where
A
R= = hydraulic mean depth
P
S0 is the bed slope and γ is weight density of fluid.
d τ0 ρf
The third pie term, π3 =
µ
 1 
Multiply and divide the 3rd pie term by   so that
 ρf 
τ0
d
ρf dv*
π3 = = = R*
 µ  ν
 
 ρf 
here ν (nu) = Kinematic vis cos ity
µ
=
ρf
282 Irrigation Engineering and Hydraulic Structures

∴ 3rd pie term, π3 represents a kind of Reynold number for sediment flow in channels.
Thus the phenomenon of sediment transport in alluvial channel is governed
by F*, R* and Ss.

13.3 SHIELD’S ENTRAINMENT FUNCTION


Shield studied experimentally the relationship between these π terms and developed
a function known as Shield’s entrainment function, given by
τ0
Fs =
γ d (Ss − 1)
Shield has obtained the relation of his function Fs with R*, and is given in following
graph of Fs v/s R* on log-log scale given in Fig. 13.1.

1.0
Threshold
Laminar condition movement

Turbulent
FS (Iog)

condition
0.1

FS = 0.056 at R+= 400

0.01
1.0 10 100 400 1000
R+ (Iog scale)

Fig. 13.1: Plot of Fs v/s R*

Shield obtained value of Fs = 0.056 at R* = 400 from his above curve given
in Fig. 13.1 and accordingly
τ0
Fs = = 0.056
γ d(Ss − 1)
Ss = 2.56 which is usual value of specific gravity for sediment of size not greater
than 6 mm.
Now τ0 is also equal to γ RS0
γ RS0
∴ Fs = = 0.056
γ d(2.65 − 1) (13.1)
∴ d = 11RSo

Above equation gives minimum size of bed material d, which will remain at rest in
channel of hydraulic radius R and bed slope S0. Here d is D-75 size of sediment grain.
Sediment Transport in Alluvial Canals 283

q SOLVED EXAMPLE
Example 13.1: A wide rectangular canal is to carry 2.7 cumecs through a track
of coarse alluvial of size 6 mm, and canal bed slope is 0.001. Assume that banks
are protected by grass, find minimum width of the canal (Take n = 0.015).
Solution:
d = 11RS0
d 0.006
∴R = = = 0.54 m = y = depth of flow
(11)So 11(0.001)
since given canal is wide rectangular section and so R = Y, depth of flow.
By Manning’s Formula,
1 2 3 12
v= R So
n
1 2 1
= (0.54) 3 (0.001) 2
0.015
= 1.36 m / s
q = discharge per unit width of Canal
= vy
= 1.36 (0.54) = 0.73 m3 /s
Q 2.7
∴ canal bed width B = = = 3.7 m
q 0.73

13.4 BED LOAD FORMULA BY EINSTEIN


Einstein assumed that every particle after it is River
dislocated on the bed, travels a certain minimum
distance which is proportional to its grain size width
before coming to rest. Refer Fig. 13.2 below. Unity
[Where L = River length as shown in the above L
figure (between two given section)]
L ×1 Fig. 13.2 Grains dislocation
Number of grains dislocated =
K1d 2
and volume of each grain K2d3
here, K1, K2 = coefficients giving c/s area and volume of grain size d,
p = probability of dislocation of grain
qs = volume of sediment transported per unit width per sec.
K2
= L p d3
K1
284 Irrigation Engineering and Hydraulic Structures

Einstein assumed value of p as proportional to shield’s Function, Fs divided by K2,

 F  K  τ0 
p = K s  =  
 K 2  K 2  γ d(Ss − 1) 
Here K = proportionality constant.
K2
Now, qs = L p d,
K1
K   K  K
∴ qs =  2 L   Fs d  = LFs d
 K1   K 2  K1
K τ0
= L d, here τ0 = γ RSo
K1 γ d(Ss − 1)
K γ RSo
∴ qs = L d
K1 γ d(Ss − 1)
K RSo
= L
K1 ( Ss − 1)
But velocity of flow in channel as per Chezy is given by
v = C RSo , where C = Chezy 's constant
v2
∴ RSo = (13.2)
C2
K v2
∴ qs = L
K1 C2 ( Ss − 1)

From equation (13.2) it can be deduced that


(i) Sediment carrying capacity of river or canal increases with increase in flow
velocity.
(ii) Floods carry more sediment since velocity of flow is high during floods.
(iii) Branching of canal from main canal reduces discharge of main canal more
rapidly than designed value.
Einstein also gave alternative equation for bed load calculation:

é to ù3
qs = 40W0 d ê ú
ê g d(S - 1) ú
ë s û
(13.2a)
2 36u 2 36 u 2
Here Wo = G gd(Ss - 1) and G = + 3 -
3 gd (Ss - 1) gd3 (Ss - 1)
Sediment Transport in Alluvial Canals 285

here υ is Kinematic viscosity of water = 1 × 10−6 m 2 / s


3
' '  n'  2
and τo = γ R So Here R = R  
n
 

13.5 BED LOAD FORMULA BY MEYER–PETER


According to experimental findings of Meyer and Peter, τo (bed shear) is reduced
due to ripple formation at bed.
 3 
 n'  2
∴τeffective = τo   − τc 

 n 
   
1
d 6
here, n ' =
24
n = 0.02 if Q > 11cumecs
= 0.0225 if Q < 11 cumecs
τc = 0.07 d
d = grain size in mm
Let τ0 = RS0 ,
    qs = volume of sediment transported in kg per unit weight per hour, (13.3)
then G s = qs γ Ss
3
 3  2
 n'  2
= 4700 τo   − τc 
 kg / unit weight / hr
 n 
 
 

13.6 DU-BOIS BED LOAD FORMULA


According to Du-Bois, volume of bed load transported is proportional to the excess
of tractive force over critical shear and is given by
q s = K [ τo ( τo − τc ) ] (13.4)
= Volume of sediment transported per unit width of canal per sec.
0.173
K = Proportionality constant = 3
(d) 4
d = Grain size in mm
τo = γ RSo = tractive force
τc = Critical bed shear
= 0.07d, here d = d50 or d60 size in mm.
286 Irrigation Engineering and Hydraulic Structures

q SOLVED EXAMPLES
Example 13.2: Design a wide rectangle channel carrying 30 cumecs with a bed
load concentration of 50 ppm by weight. The bed material consists of grains of
average size 2 mm. Use Lacey’s regime equations and Meyer–Peter formula. Take
n = 0.02.
Solution:
 50 
Bed load transported =  6 × 1000  × 30 = 1.5 kg/sec.
 10 
P by Lacey 's equation = 4.75 Q
= 4.75 30 = 4.75 × 5.47 = 26 m.
Given Channel is wide rectangular section
∴ R = y = depth of flow
Let depth of flow be assumed as 1m
A
∴R = = 1 ∴ A = P = 26 m 2 = By
P
∴ B = 26 m, since y = 1

∴ G s = wt.of bed load transport per unit width per sec


2.5
= = 0.096 Kg/m/se c = 0.096 × 3600
26
= 346 kg/m/hr
By Meyer–Peter formula,
3
 3 2
  n'  2 
G s = 4700  τo   − τc  kg/m/hr
 
 n 
 
1
 2 6
1

(d) 6  0.35
1000 
n' = = = = 0.0147
24 24 24
n ' 0.0147
∴ = = 0.739
n 0.02
τc = 0.07d = 0.07 × 2 = 0.14
3
 3 2 1
∴ G s = 4700  γRSo ( 0.739 ) 2 − 0.14  × kg / m / sec
  3600
Sediment Transport in Alluvial Canals 287

3 3
= 1.3 (1000RS
= 1.3 (1000RS (0.63) − 0.14 ) 2
o (0.63) −o 0.14 ) 2
3 3
= 1.3 ( 630RS
= 1.3 ( 630RS − 0.14 ) 2
o − 0.14 )o2
3 3

∴ 0.096 ∴ 0.096 =×1.3(630


= 1.3(630 1 × So2− 0.14) 2
1 × So −×0.14)

∴ 630So = ( 0.0738 )2 3 + 0.14 = 0.314

0.314 1
∴ So = = = 1 in 2006
630  630 
 
 0.314 
Now, A = 26 y, and according to Manning’s formula,
2 1 2 1
Q 1 1  1 2
= V = R 3 So2 = (y) 3  
A n 0.02  2006 
2
30 y3
= ,
26y 0.895
here, by putting y =1, we get LHS = 1.15 and RHS = 1.12
∴ y = 1 m, assumed as above is correct
∴ B = 26m, y = 1 m and S0 is 1in 2001

Example 13.3: Find rate of bed load transport in wide rectangular channel for
the following data:
Depth of flow, y = 3 m
Velocity of flow, v = 1 m/sec.
Bed slope So = 1 in 5000
Average size of grain, d = 1.14 mm.
Kinematic Viscosity of water ν = 1 × 10−4 Stokes
Make use of Einstein formula.

Solution: Using Manning’s equation,


2 1
1
V= (R) 3 (S) 2
n
Now, for wide rectangular channel, R = y = 3m and v = 1m / sec.
288 Irrigation Engineering and Hydraulic Structures

1
2
1  1 2
∴1 = ( 3) 3  
n  5000 
2.08
∴n = = 0.029
70.7
1 1
1 1  1.14  6 0.32
n′ = (d) 6 =   = = 0.13
24 24  1000  24
3
 n′  2
∴ R′ = R  
 n
3
 0.013  2
=3  = 3 × 0.316 = 0.95m.
 0.029 

Wo = fall velocity = G gd(Ss − 1)

2 36υ2 36υ2
G= + 3 −
3 gd (Ss − 1) gd3 (Ss − 1)

( )
2
36υ2 36 × 1 × 10−6
= , (here υ in m 2 /s =1 × 10−6 )
gd3 (Ss − 1)  1.14 
3
9.81  ( 2.65 − 1)
 1000 
= 1.5 × 10−3
2
∴G = + 1.5 × 10−3 − 1.5 × 10−3
3
= 0.817 − 0.038 = 0.78

(
∴ Wo = 0.78 9.81 × 1.14 × 10−3 (1.65 ) )
= 0.78 × 0.135 = 0.105 m/s
According to Einstein,
3
 γ R 'So 
qs = 40 × w o d  
 γ d(Ss − 1) 
3
 1 
 0.95 × 
−3
= 40 × 0.105(1.14 × 10 )  5000


−3
(
 1.14 × 10 (1.65) ) 

)
∴ qs = 4.23 × 0.00114 × 1 × 10 −3 = 4.82 × 10 −6 m3 / s / m
Sediment Transport in Alluvial Canals 289

Bed load in terms of N/s/m


= qs γ Ss
= 4.82 × 10−6 × 1000 × 2.65 kg / s / m
= 12.77 × 10−3 kg / s / m
= 12.77 × 10−2 N / s / m

13.7 SUSPENDED LOAD CONCENTRATION


Suspended load concentration, C, can be calculated from Hunter Rouse’s equation,
given below. Limitation for using of this equation is that suspended load concentration
at a point y (measured from channel bed) can be calculated provided suspended
load concentration, Ca, at a point “a” above channel bed is known.
 W0 
 
C  a(D − y)   KV* 
= (13.5)
Ca  y(D − a) 
here D = Full supply depth of flow in channel
W0 = Fall velocity of sediment grain in still water, which is usually 0.02 m/s
K = Karman Constant = 0.4
V* = Shear velocity = τo / ρ

q SOLVED EXAMPLES
Example 13.4: In a wide rectangular channel, suspended load sample taken at
0.5 m height from bed had a value of 1000 ppm of sediment by weight. The full
supply depth of channel is 5 m and its bed slope is 1 in 4000. Fall velocity of
sediment grain is 2 cm/sec. Find suspended load concentration at:
(i) Mid depth and
(ii) At surface.
Solution: Using Rouse equation,
 W0 
 
C  a(D − y)   KV* 
=
Ca  y(D − a) 
here, a = 0.5m., D = 5 m., y = 2.5 m, Ca =1000 ppm, Wo = 0.02 m/s, K = 0.4
 0.02 
C  0.5(5 − 2.5)   0.4V8 
∴ =
1000  2.5(5 − 0.5) 
τo γ RSo ρ g RSo
V* = = =
ρ ρ ρ
= g RSo R = D, Since wide rect. channel = 5m,
1
= 9.81 × 5 × = 0.11 m/s
4000
 0.02 
C  0.5(5 − 2.5)   0.4V8 
∴ =
1000  2.5(5 − 0.5) 
290 Irrigation Engineering
* τ o Hydraulic
and γ RSoStructures
ρ g RSo
V = = =
ρ ρ ρ
= g RSo R = D, Since wide rect. channel = 5m,
1
= 9.81 × 5 × = 0.11 m/s
4000

Wo 0.02
∴ *
= = 0.45
KV 0.4(0.11)
0.45
 0.5 × 2.5 
C = (1000 ppm)  
 2.5 × 4.5 
= (1000 ppm) [ 0.37 ]
= 370 ppm at mid depth
C at surface = 0 since D − y = 0
as y = D at surface.

Example 13.5: Determine the bed load transport in a wide alluvial stream for
following condition:
Depth of flow = 2.5 m, velocity of flow = 1.5 m/s
Average size of sediment = 5.0 mm
Specific gravity of sediment = 2.65
Solution:
Using Meyer - Peter equation,
3
 3 2
  n'  2  1
qs = 4700  τ o   − τ c  × kg / m / sec
  n  3600
 
1
1  5 6
 
d6 1000  0.4135
n' = = = = 0.0172
24 24 24
n = Manning 's Constant = 0.0225
n ' 0.0172
= = 0.764
n 0.0225

τo = γ RSo wide rectangular channel, R = D = 2.5m.

= 1000(2.5) (8 × 10−4 ), Where So = free surface slope


= bed slope for uniform flow
= 8 ×10−4
Sediment Transport in Alluvial Canals 291

= 2 Kgf / m 2
τc = 0.07d = 0.07 × 5 = 0.35
3
3
4700  2
∴ qs =  2(0.764) 2 − 0.35
3600  
 
3
= 1.3[ 2 × 0.667 − 0.35] 2
=1.3(0.985)1.5
=1.3 × 0.9775
=1.27 Kgf / m / sec

13.8 TRACTIVE FORCE METHOD OF DESIGN OF STABLE


CHANNEL
Critical tractive force method of design does A
not allow the shear stress anywhere in the M N
channel to a value less than the critical shear P
stress of bed material. T

Consider a particle P on side of a channel C/S of channel


B
making an angle θ with the horizontal, see
Fig. 13.3 (A) and (B).
Fig. 13.3 (A):
Top of side bank of channel
M

P Side Slope of
FD drag force on particle channel
90°
WS sin T
WS cos T Flow
WS T Direction
L

T, Sloping side Flow Channel Bed


makes
R

Fig. 13.3 (B): Isometric view of side MNLR


Let d = size of particle so that its c/s area, A = K1 d2
Where K1 = coefficient.
Ws = submerged weight of the particle
= K2 ( γ s − γ ) d3 , where K2 = coefficient.
292 Irrigation Engineering and Hydraulic Structures

Ws can be resolved into two components,


(Ws) Sin θ along side slope and
(Ws) Cos θ normal to side slope.
Due to flow a shear stress τw acts on the particle P. The drag force FD due to
flow, acting on the particle P is given by
FD = τw (A) , here A = K1d2
This drag force is parallel to side slope and is in direction of flow. Thus particle
P is influenced by drag force FD, and displacing force (Ws) Sin θ.
The stabilizing force is,
( Ws ) Cosθ tan ϕ, where ϕ is angle of response of material, thus

( Ws ) Cosθ tan ϕ = FD2 + Ws2sin 2θ (13.6)


Drag Force FD = τ w A (13.7)
and Ws tan ϕ = τ b A (13.8)
Substituting values of FD and Ws tan ϕ in equation (13.6), we get

τb A Cosθ = τ2w A 2 + Ws2 Sin 2 θ


2
τ A
( τw A Cosθ )2 = τ2w A 2 +  b  sin 2 θ
 tan ϕ 
2
τ  sin 2 θ 
∴  w  = Cos 2 θ −
 τb  tan 2 ϕ
1
τ  tan 2 θ  2
∴ w = Cos θ 1 − 2  (13.9)
τb  tan ϕ 
Equation 13.9 can be re-arranged as under

2
 τw  2 sin 2 θ
  = cos θ −
 τb  tan 2 ϕ
sin 2 θ
= cos 2 θ + sin 2 θ − sin 2 θ − 2
by adding and subtracting sin 2 θ
tan ϕ
 1 
=1 − sin 2 θ 1 + 2 
 , since sin 2 θ + cos 2 θ = 1
 tan ϕ 
 1 + tan 2 ϕ 
= 1 − sin 2 θ 
 tan 2 ϕ 
 
sec2 ϕ
= 1 − sin 2 θ
tan 2 ϕ
2
τ  sin 2 θ
∴ w  = 1 − 2
 τb  sin ϕ
 1 
=1 − sin 2 θ 1 + 2 2
 , since sin θ + cos θ = 1
2 
 tan ϕ 
 1 + tan 2 ϕ 
= 1 − sin 2 θ 
 tan 2 ϕ 
Sediment Transport in Alluvial Canals 293
 
sec2 ϕ
= 1 − sin 2 θ
tan 2 ϕ
2
τ  sin 2 θ
∴ w  = 1 − 2 (13.10)
 τb  sin ϕ
τw sin 2 θ
∴ = 1− 2
τb sin ϕ
= K3
Thus, shear stress due to flow of water on particle P on side slope is always less
then shear stress at bed τb if θ is less than ϕ .
Now bed shear τb = γ RSo and hence τw
Can be worked out

sin 2 θ
τw = K 3 τb here K 3 = 1 −
sin 2 ϕ

τ sin 2 θ
or w = 1 − 2 (for θ < ϕ) (13.11)
τb sin ϕ

Example 13.6: Design a stable non-erodible wide rectangular channel to carry


a discharge of 12 m3/s. Channel bed consists of gravel of size 10 mm and its bed
slope is 1 in 1250, and side slope 2 H to 1 V, Take angle of repose of bed material
as 32°.
Solution:
1
Side slope 2 H to 1 V means tan θ = = 0.5
2
∴θ = 26.56°
,
∴ sin θ = 0.447 and cos θ = 0.894

ϕ = 32o
∴ sin ϕ = 0.529
τ c = 0.9(d mm ) = 0.9 × 10 = 9 N / m2
τ b = 0.9(τ c ) = 8.1 N / m 2
τ b = γ RSo ,
here R = yo = depth of flow as channel is wide rectangular
= γ yo so
294 Irrigation Engineering and Hydraulic Structures

τB 8.1
      ∴ yo = = = 1.04 m. (13.12)
γ so 10000(0.0008)

Sin 2θ (0.447)2
K3 = 1 − = 1−
Sin 2 ϕ (0.529) 2
= 1 − 0.714 = 0.533
      ∴ τ w = K 3 τ b 
= 0.533 (8.1) = 4.36 N / m 2
= 0.75 γ yo so
4.36
∴ yo = = 0.743 (13.13)
0.75 × 10000 × 0.0008

From (13.12) and (13.13), adopt lower value of yo = 0.743 m.


A=By0 = 0.743B, (B is bed width and A is c/s area)
2 1
Q 12 1
V= = = yo3 So2
A 0.743B n
2 1
1  1 2
= (0.743) 3  
0.022  1250 
= 1.05
12
∴B = = 15.3 m.
0.743 × 1.05
Bed width B = 15.3 m., depth of flow = 0.743m.

EXERCISES

1. In a wide rectangular stream, suspended load sample taken at 0.3m height


from bed had a concentration of 900 ppm. The full supply depth is 5 m and
So is 1 in 4000. If fall velocity of bed material is 2 cm/s, find suspended load
concentration at mid depth.
(Ans: 270 ppm.)

2. A canal is carrying a discharge of 600 cumecs. Bed slope is 1 in 1000. Bed


material size is 5 cm, and its angle of repose 37o. Design canal section if it is
unlined and trapezoidal with side slope 3 :1. Manning’s n = 0.025.
(Ans: Y = 2.8m, B = 7m.)
Sediment Transport in Alluvial Canals 295

3. A wide irrigation channel has hydraulic mean depth of 3m. and bed slope of
1 in 6250. The bed material is 0.3 mm size, and has specific gravity of 2.65.
Take Manning’s n = 0.02. Find rate of bed load transported by the channel in
N/s/m width of channel.
(Ans: 0.7 N/s/m)
4. Prove that for stable non-erodible channel,

τw Sin 2 θ
= 1− , if θ < ϕ.
τb Sin 2 ϕ

where τw is shear stress due to influence of flow on sloping side and τb is


shear stress at bed, θ is the angle the sloping side makes with the horizontal
and ϕ is angle of repose.
14
River Training Works
and Flood Control
14.1 GENERAL
Every monsoon we find certain rivers swell due to heavy flood flow and cause
damage on either side of banks and sometimes we find that river changes its course
also. Such situations demand for river training works and arrangement for flood
control. River training works in the form of guide banks, embankments, levees etc.
keep the river confined to its usual track and flood control measures provide warning
time so that loss of life and material can be prevented or minimized.

14.2 CLASSIFICATION OF RIVERS


Right from its mouth to tail, a river passes through four different types of stages:
(1) Rocky, (2) Boulder, (3) Trough and (4) Deltaic.
Along its flow path it may undergo meandering. Rivers flowing through alluvial
plains can therefore be classified as:
1. Meandering type rivers
2. Aggrading type rivers
3. Degrading type rivers
Apex
g
in
ss

Meandering
o
Cr

width

Mw

Meandering length, ML

Fig. 14.1: Meandering type river

© The Author(s) 2023 296


S. K. Ukarande, Irrigation Engineering and Hydraulic Structures,
https://doi.org/10.1007/978-3-031-33552-5_14
River Training Works and Flood Control 297

1. Meandering Type River: A part of meandering type river is shown in


Fig. 14.1. Depth of cross – section over apex part is greater than that over
crossing part. Meandering width MW, meandering Length ML and river
width vary as per square root of river discharge Q according to Inglis.
2. Aggrading Type Rivers: Aggrading type rivers have sediment load in
excess of their carrying capacity and hence this extra sediment load settles
along its bed and consequently changes the bed slope of river.
3. Degrading Type Rivers: Degrading type rivers have less sediment load
than its carrying capacity and hence they suffer from erosion of their bed due
to high velocity of flow. Thus, meandering, aggrading and degrading type of
rivers undergo changes in their bed slope and this may force them to change
their course. Rivers Brahmaputra, Koshi and Sone in north-east region of
our country exhibit this type of behavior and are well known for their flood
havocs.

14.3 RIVER TRAINING WORKS


Main objective of river training works is to keep the river within its channel
section and make it flow along its established alignment, however few special
types of objectives are listed below:
1. Safe disposal of flood flow without over flooding the banks.
2. Out flanking of bridge or aqueduct, which may result as a heavy flood in
river, is to be prevented by river training works.
3. Protection of banks from erosion and efficient transport of sediment load are
also considered important objectives of river training works.
To meet with above requirement of river training, any of the following works can
be adopted:
(i) Marginal bunds or levees
(ii) Guide banks or guide bunds
(iii) Groynes or spurs
(iv) Cut-offs
(v) Pitched islands
(vi) Bandalling

14.3.1 Marginal Bunds or Levees


In order to keep flood waters within the river section, embankments parallel to
river bank and of nominal height should be provided. These are known as marginal
bunds or levees. They are usually designed as per design of embankments for canals
in filling. If the sloping side of the embankment is likely to come in contact with
floodwater, it may be provided with pitching. The effect of these bunds is to raise
the water level during flood and to dispose off flood water quickly.
298 Irrigation Engineering and Hydraulic Structures

14.3.2 Guide Banks or Guide Bunds


Guide banks are constructed such that they provide a straight approach towards
a work such as bridges, weirs, culvert or C–D works across the river. They also
prevent river from changing its course. Bell was first to construct these types of
bunds in India and hence they are also known as Bell’s bunds. Fig. 14.2 (A) shows
half sectional plan and bund along one of the banks of the river and Fig. 14.2 (B)
shows half section of the bund.

River flow
L/2
0.125 L 1.25 L 0.25 L
A

45o A
d/s nose of
120o u/s nose of
guide bank
guide bank

Fig. 14.2 (a): Half plan of guide bank

Fig. 14.2 (b): Half section of guide bank along AA

Length of waterway, L, is worked out from Lacey’s formula

Q = 4.75 Q
∴ L = Centre to center distance between bunds along the river banks

= 4.75 Q.
here Q = flood discharge in the river.
Length of bunds can be kept as 1.25L and length of u/s nose can be kept as 0.25 L
whereas that of d/s nose is kept as 0.125L. Side slope of the bund can be kept as
2:1 and u/s side be pitched.
River Training Works and Flood Control 299

14.3.3 Groynes or Spurs


The main job of river training is done by groynes or spurs. They are also a type
of bund but are provided normal or inclined to the path of river flow and extend
from river bank into the river flow path. They protect the river bank from erosion
by keeping the flow away from it. According to the angle they make with the river
bank, they are classified as
1. Attracting Groyne or Spur,q > 90°
2. Repelling Groyne or Spur,q < 90°
3. Deflecting Groyne or Spur,q = 90°
They are shown in Fig. 14.3 (a), (b) and (c).

>90° <90°
>90°

Fig. 14.3 (a): Attracting Fig. 14.3 (b): Repelling Fig. 14.3 (c): Deflecting
Groyne Groyne Groyne

1. Attracting Groyne: A groyne making θ > 90° i.e., pointing d/s tends to
attract the river flow towards the bank at its back. They are not useful for
bank protecting but can safe guard a part length of the bank on its d/s side.
2. Repelling Groyne: A groyne that makes θ < 90° , i.e., pointing u/s tends to
repel the river flow away from the bank on which it is provided and so are
known as repelling groyne. Head of repelling groyne needs strong protection
since it is subject to direct attack of whirling motion.
3. Deflecting Groyne: A groyne making θ = 90° with the bank tends only
to deflect the flow without repelling it and gives local protection to the
adjoining bank length on either side.

14.3.4 Cut-Off
A river flowing along a meandering path may sometimes abandons particular bend
and follows a straight path between successive bends, creating a path known as
cut-off. Following this natural process artificial cut-offs can also be provided for
river training works in which diversion of flow from curved path is required. This
is shown in Fig. 14.4.
300 Irrigation Engineering and Hydraulic Structures

Cut-off

Fig. 14.4: Cut-off

14.3.5 Pitched Islands


A pitched island is created artificially in river bed to cause redistribution of velocity
and hence tractive force. This arrangement will attract the river currents towards
itself to reduce erosion of river banks. A deep scour occurs around the pitched island
causing heavy concentration of flow around it. Stone pitching along the sloping side
of island and launching apron along river bed from its toe are provided to protect
against maximum scour condition. (Fig. 14.5)

Fig. 14.5: Pitched island


River Training Works and Flood Control 301

14.3.6 Bundalling
It is also one of the methods of river training used for navigation purposes. A low
level flow in a river can be confined in a single track for maintaining required
depth for navigation purpose. After recession of floods and when water level starts
falling, bandalling work is carried out. This work consists of framework of bamboos
driven into river bed at 0.6 m center to center. On these bamboos horizontal ties
and strutting consisting of bamboos are provided. This arrangement is carried out at
an angle of 30° to 40° with downstream for creating a narrow channel of required
depth, see Fig. 14.6.
Bandals check the flow and cause deposition of sand behind themselves. Thus,
a channel confined between bandals is formed with sand banks on either side and
entire river flow is diverted through this channel. In case a pitched island is formed
in river bed, it will be an additional help to bandalling work as shown in Fig. 14.6.
Vertical bamboos
provided @
6m c/c and of
3m to 6m length 3m to
6m

Horizontal Flow
supports

Strutting

Bundalling in plan

Pitched Flow
island

Bundalling in plan

Fig. 14.6: Plan and sectional view of bundalling

14.4 FLOOD CONTROL


If heavy storms occur in catchment area either consecutively or at very close interval
the magnitude of run-off will be large and its concentration in river flow will result
in a flood. Floods are result of natural processes and cannot neither be avoided nor
controlled, but their effect can be suitably modified by following flood routing
techniques. A flood wave entering a reservoir can be routed by ISD Pul’s method
as explained in Chapter 5. Flood peak can be reduced by working out attenuation
and time lag of occurrence of peak values by using reservoir routing techniques. By
302 Irrigation Engineering and Hydraulic Structures

adjusting out, flow from reservoir, flood wave monitoring is possible. Similarly river
flow capacity can also be utilized to allow safe disposal of flood water by following
channel routing techniques. Thus, flood routing process for reservoirs and channels
can help to modify the effect of floods i.e. to reduce peak values and to provide time
lag for occurrence of peak values. This will enable the authorities to give a warning
time to d/s occupants and loss of life and property can be prevented or minimized.

14.4.1 Flood Absorption Capacity of Reservoirs


Reservoirs have large capacity to withhold flood water over spillway
crest level and spillway gate height and this capacity can be utilized
to hold flood water in reservoir by following reservoir routing
technique. This enables the authority to have long interval of time to give warning
and to carry out temporary evacuation works on d/s side of reservoir. Similarly
channel improvements also help in increasing flood carrying capacity of rivers.
Construction of levees along river banks also helps in increasing flood carrying
capacity of rivers.

14.4.2 Flood Forecasting Methods


Flood forecasting can be carried out by making use of any of following methods :
(i) Application of unit hydrograph technique for quick estimation of maximum
flood that may result due to storms of high intensities in the catchment areas.
(ii) Flood frequency analysis by statistical methods. This method will help to
forecast highest value of flood for a given recurrence interval. This method
however requires at least 35 years past record of rain storms and their
resulting floods.
(iii) Empirical flood formulae such as Dicken’s, River’s, Inglis etc. can be used
to predict maximum flood for a given catchment area.

14.4.3 Channel Improvement


A river can carry higher discharge at lower levels or stages by improving its hydraulic
conditions:
(i) Increase in water way area by increasing its cross-sectional area.
(ii) Increase in velocity of flow by deepening, straightening and shortening the
channel length by cut-offs, removing barriers in channel section and lining the
channel sides to improve its coefficient of rugosity.
It may be noted that channel improvement program if not carried out all along its
length will merely shift the point of location of flooding of waters in channel section
from one reach to another reach of the channel. Hence, implementation of channel
improvement should be carried out for the entire channel reach.
River Training Works and Flood Control 303

EXERCISES
1. (a) Define
(i) Meandering type of rivers.
(ii) Aggrading type of rivers.
(iii) Degrading type of rivers.
(b) Explain how guide banks are useful in controlling the river track.
2. Explain the following:
(i) Groynes of various types
(ii) Pitched Island
(iii) Bundalling
3. What is flood control? What are the methods of flood forecasting?
4. Explain how flood routing is useful for flood control purposes.
15
River Flow
Measurement

15.1 NEED FOR DISCHARGE MEASUREMENT IN RIVER


SECTION
Flood hydrographs are essential for flood flow calculations. These graphs can be
prepared by collecting values of flood flow in river at a chosen section against
specified time intervals. A river may have deep or shallow sections and they can
be wide or narrow. Discharge measurement is done by dividing the river section
into small segments and measuring the velocity into small segments by current
meters or float or salt dilution technique. Use of current meter is more reliable and
can be easily handled.

15.2 CURRENT METERS


It works on the principle of rotation of its runner being proportional to the rate of
flow. The runner may have vertical axis or horizontal axis and is that case known as:
(i) Vertical Axis Meter
(ii) Horizontal Axis Meter.

15.2.1 Vertical Axis Meter


A series of conical shape cups are mounted around a vertical axis, see Fig 15.1.
The cups rotate in horizontal plane and a cam shaft attached to the vertical
axis – spindle records generated signals proportional to the revolutions of the cup
assembly. The measurement of velocity is possible in the range of 0.15 to 4.0 m/
sec with 1% accuracy. If river flow has appreciable vertical components of velocity,
then this type of meter is not suitable.

© The Author(s) 2023 304


S. K. Ukarande, Irrigation Engineering and Hydraulic Structures,
https://doi.org/10.1007/978-3-031-33552-5_15
River Flow Measurement 305

Hoist & Electrical Connectors

Vertical Axis Stabilising Unit


Conical Cup

Sounding Weight

Fig. 15.1: Vertical axis meter

15.2.2 Horizontal Axis Meter


A propeller mounted on horizontal axis gets rotated due to flow and its rotational
number per sec are converted into velocity of flow by electrical instruments.
This type is stable against vertical shocks of flow and measures velocity over
range of 0.15 to 4.0 m/sec with 1% accuracy. This type is widely used.

Hoisting

Propeller

Sounding Weight

Stabilizing Unit

Fig. 15.2: Horizontal axis meter

A current meter records rotation of the cup unit or propeller and these rotations
are converted into corresponding velocity of flow by the equation
V = aN + b
Where N = revolution per second and a and b are meter constants usually
a = 0.65 and
b = 0.03, V is velocity of flow in m/sec.
306 Irrigation Engineering and Hydraulic Structures

15.3 CROSS-SECTIONAL AREA-VELOCITY METHOD TO


MEASURE DISCHARGE
At a chosen section on a river length, its cross-section is drawn by taking readings
of depths at regular interval of width along the section. The depths are measured
by sounding rods or sounding weights. Its river section is having more depth, then
eco-depths recording is used. The verticals representing river depths are drawn on
the section and making use of current meter, velocity at either 0.6d, 0.8d, or 0.2d is
measured. (Refer Fig. 15.3)
If a plot of values of velocities along a vertical representing depths d of the
section is drawn it will be observed that velocity is maximum not at free surface of
flow but at 0.2d from free surface and average velocity for shallow depths occurs
at 0.6d, if depths are lesser than average velocity

V0.2d + V0.8d
VAV =
2

(i.e., velocity at 0.2b + velocity of 0.8d)/2


Gives average velocity for d more than shallow depth, shallow depth is usually
less than 2.0 m.
Vertical

Free surface of river

0.2 d
0.6 d
0.8 d
Depth (d)

River Bed.

Fig. 15.3: Velocity at different depths­­

From a station A on left bank, river depths at regular width of ‘b’ are measured
upto station B on right bank and cross-section is drawn as in Fig 15.4. If d ≤ 2m,
reading taken at 0.6d for velocity measurement are if d > 2 m, readings are taken
at 0.2d and at 0.8d, and average value of V is obtained.
1 1 
At a vertical y, area upto b on left and area upto  b  on right is takes as
2 2 
representative area for the depth at y and ∆Q is worked out as
V + V0.8d 
∆Q = bd  0.2d 
 2 
River Flow Measurement 307

l Y
tica
Ver V at 0.2 d
b b b
Left Bank A B Right Bank

V at 0.6 d

V at 0.8 d

Fig. 15.4: Area velocity method

All these ∆Q added will give Q at this section, and this method is known as mid-
section method. In mean sections method, consecutive depths of sections are chosen
and velocity measurement at these section is carried out, then
 d + d 2   V1 + V2 
∆Q =  1  b1 etc
 2   2 
Where d1 = depth as section 1 and
  v1 = velocity at section 1
d2 and v2 are the depths and velocity at section 2
b = width between section 1 and 2.
                              

This is illustrated in the following numerical problem:

q SOLVED EXAMPLES
Example 15.1: A current meter was used to measure velocity at 0.6d of flow.
Compute discharge if given calibration equation is
V = 0.32 N + 0.32
Where N = revolutions per second and
V = velocity in m/sec
Distance 0 1.5 3 4.5 6 9 12 18 15 20 22 23 24
from bank
Depths 0 0.375 0.75 1.24 2.0 2.3 1.9 1.7 1.8 1.55 1.3 0.8 0
N rev/sec 0 0.33 0.54 0.79 1.1 1.16 1.01 0.91 0.95 0.76 0.71 0.58 0

Solution:
V1 = 0.32 N + 0.32
  = 0.32 m/sec at 0 from bank
V2 = 0.32 (0.33) + 0.32
  = 0.425 m/sec at 1.5 m from bank
V3 = 0.32 (0.54) + 0.32
  = 0.492 m/sec at 3 m from bank
308 Irrigation Engineering and Hydraulic Structures

Sample Calculations: ΔQ = by mean section method


 d + d3   V2 + V3 
= 2  (b)   (where b = 3 − 1.5 = 1.5m
 2   2 
 0.375 + 0.75   0.425 + 0.492 
=  (1.5)  
 2   2 
= (0.562)(1.5)(0.4585)
∆Q = 0.386 m3/sec.
Similarly all values of ΔQ are worked out and they are entered in tabular form
given below (Table 15.1):
Table 15.1 Discharge Calculation
Distance from V = 0.32 N to
bank (m) depth d (m) N riv/sec 32 m/sec bm ∆Q m3/sec
0 0 0 0.32
1.5 0.375 0.33 0.425 1.5 0.09
3.0 0.75 0.54 0.492 1.5 0.386
4.5 1.24 0.79 0.57 1.5 0.795
6.0 2.0 1.1 0.67 1.5 1.5
9.0 2.3 1.16 0.69 3.0 4.38
12.0 1.9 1.01 0.64 3.0 4.06
15.0 1.8 0.95 0.62 3.0 3.49
18.0 1.7 0.91 0.61 3.0 3.22
20.0 1.55 0.76 0.56 2.0 2.89
22.0 1.30 0.71 0.54 2.0 1.54
23.0 0.80 0.58 0.50 1.0 1.00
24.0 0.00 0.00 0.32 1.0 0.04
Q = ΣΔQ
= 23.34 m3/sec

Example 15.2: Velocity in m/sec at one meter interval starting from river bed are
given below. Calculate Q per unit width of the stream, depth of flow at measuring
section was 5.0 m.
V (m/sec) 0 0.6 0.8 0.9 0.9
d (m) from bottom (bed level) 1 2 3 4 5

Solution:
V0.2d + V0.8d
Vmean =
2
0.9 + 0.6
= = 0.75 m/ sec
2
River Flow Measurement 309

Q per meter width = (bd) Vmean


= (1 × 5) (0.75) = 3.75 m3/sec/m
Note: In this table depths are from bottom to top whereas in formula for Vmean,
d is from top
∴ V0.8d = 0.9 and
V0.2d = 0.6
Example 15.3: Calculate stream flow using mid-section method from the
following data :
Current meter rating equation is V = 0.04 + 0.75 N, where N is revolution/sec
Distance from bank 0.8 1.4 2.2 3.2 4.0 4.7 5.2 5.8
Depth 1.3 4.3 5.8 6.8 4.8 2.8 1.3 0
0.55 0.87 0.94 0.61 0.54
N 0.3 0.43 –
0.91 1.11 1.3 1.02 0.8

Note: Single reading of N is at 0.6 d double readings are at 0.8d, and 0.2d
respectively.
Solution:
Table 15.2 Discharge Calculation

Distance ΔQ = b×d ×Vmean or


from d 1 
bank b depth N V0.6d V0.8 d V0.2d Vmean  bd Vmean 
2
0 – – – – – – – –
0.8 0.8 1.3 0.3 0.265 – – 0.265 0.1378
1.4 0.6 4.3 0.55 – 0.452 0.722 0.587 1.515
.9
2.2 0.8 5.8 0.81 – 0.692 0.872 0.782 3.629
1.11
3.2 1.0 6.8 0.94 – 0.745 1.015 0.880 5.984
1.3
4.0 0.8 4.8 0.61 – 0.4975 0.805 0.650 2.500
1.02
4.7 0.7 2.8 0.54 – 0.445 0.64 0.542 1.06
0.8
5.2 0.5 1.3 0.43 0.362 – – 0.362 0.117
5.8 0.6 0 – – – – –
ΣΔQ = 14.942 m3/sec
310 Irrigation Engineering and Hydraulic Structures

Sample Calculation:
For 1st section, depth = 1.3, b = 0.8, it is Δlar
1
∴∆Q = bdV
2
1
∴∆Q = (0.8)(1.3)(0.265) = 0.1378.
2
where, V0.6d = 0.4 + 0.75(0.3) = 0.265 m/sec
For 2nd section, d = 4.3, b = 0.6

0.8d = 0.04 + 0.75(.55) = 0.452 m/sec


V0.2d = 0.04 + 0.75(0.91) = 0.722 m/sec
∴ Vmean = (0.452 + 0.722)/2 = 0.587 m/sec
Similarly, calculate ∆Q and enter values in Table 15.2.

15.4 SLOPE AREA METHOD


This is a hydraulic method in which manning’s formula for velocity is applied at
ends of given reach of a river. The discharge is calculated by trial and error method
as under:
(i) Assume V1 = V2, apply Bernoull’s theorem between sections (1) and (2):

V12 V2
Z1 + + d1 = Z2 + 2 + d 2 + h f
2g 2g

hf = (Z1 + d1) – (Z2 + d2), as V1 = V2 (assume)


= fall in water surface levels between section 1, 2.
hf
∴ = slope of energy line = Sf
L
Now K = conveyence factor
Q 1
= = AR 2/3 by definition
sf n

h f Q2
∴ =
L k2

∴ Q can be calculated using above equation


Q Q
Now calculate V1 = and V2 =
A1 A2
River Flow Measurement 311

hf = Sf
2
V /2g
1

V22/2g

d1 1

2 d2
So

Z1 L Z2

Fig. 15.5: Slope area method

with these values of V1 and V2 find hf Repeat the procedure till value of hf is constant.
This method is illustrated in following problem:

Example 15.4: A 10 m wide rectangular channel has depth of flow as 4.0 m and
3.8 m at two section 100 m apart. If drop in water surface elevation is 0.15m, and
n = 0.025, find discharge passing through the channel by slope - area method.
Solution:
Section 1 Section 2
d1 = 4.0 m d2 = 3.8 m
A1 = 10 × 4 = 40 m2      A2 = 3.8 × 10 = 38 m2
P1 = 10 + 2(4)                  P2 = 10 + 2(3.8)
= 18 m      = 17.6 m
A A 38
R1 = 1 R2 = 2 =
P1 P2 126
40
= = 2.15m
18
1
= 2.22 ∴ K2 = (38)(2.15) 2/3
0.25
1
K1 = A1R12/3 = 2538
n
1
= (40)(2.22) 2/3
0.025
= 2730

Average K = K1K 2 = 2632, ∴ Q = 2632 sf−


hf = given value of fall in water surface between (1) and (2)
= 0.15 m
312 Irrigation Engineering and Hydraulic Structures

h f 0.15
sf− = = = 0.0015
L 100
∴ sf 0.038
∴Q = 2632 × 0.038 = 101.9
2
 Q   2632 sf 
   
V12  A1   A1  2
 = (2.548) = 0.33
= =
2g 2g 2g 2g
2
 Q 
2  
V2  A 2  (2.682) 2
= = = 0.366
2g 2g 2 × 9.81

 V2 V2 
∴ h f = fall +  1 − 2 
 2g 2g 
 
= 0.15 + (0.33 − 0.366) = 0.15 − 0.036
hf = 0.11 (1)
2nd step:
h f 0.114
sf− = = = 0.00114,
2 100
∴ sf = 0.033 ∴ Q = 88.86
2
 2632 
 × 0.033 
V12 40
= 
2g 19.62
4.935
= = 0.251
19.62
2
 2622 
× 0.033 
v 2 2  38  = 0.266
=
2g 2g

∴ hf = fall + (0.251 – 0.266)


= 0.15 – 0.015
= 0.135 (2)
River Flow Measurement 313

3rd step:
h f 0.135
sf− = = = 0.00135
L 100

sf− = 0.036

∴ Q = 2632 × 0.036
= 96.7 m3/sec
2
 96.7 
v12  40 
=
2g 19.62
= 0.297

log scale
O (h – a)

Fig. 15.6: Stage-discharge curve


2
 96.7 
v2 2  
38 
=
2g 19.62
= 0.33
∴ hf = 0.15 + (0.297 – 0.33)= 0.117

∴ Q = 2632 sf−
1/2
 0.117 
= 2632  
 100 
= 2632 × 0.0342
= 90.0 m3/sec
∴ Q = 90.0 m3/sec can be adopted as average fall value remains constant around 0.12
314 Irrigation Engineering and Hydraulic Structures

15.5 STAGE-DISCHARGE RELATIONSHIP


The relationship between stream discharging ‘Q’ are gauge height ‘h’ can be
expressed as
Q = k(h – a)n
Here, h = gauge reading when discharge = Q
A = gauge reading when discharge is zero and k, n are constants to be
determined for given stream.
Taking log on both sides of the equation
log Q = log K + n log (h – a)
This is similar to y = mx + c, equation of straight line.
Hence assume values of ‘a’ and plot log Q v/s log (h – a) if it is a straight line
assumed value is OK or else change value of ‘a’ till you get a straight line. The
slope of the line gives value for ‘n’ and value of Q for (h – a) = 1, gives K.

q SOLVED EXAMPLES
Example 15.5: For various stages, discharges in a river section are given below,
obtain an equation for stage discharge relationship and find value of Q for a stage
of 5.0 m and 10 m.
Stage (m) 1.8 2.0 2.3 2.9 3.7 4.5 5.5 6 7 8
Q 1.0 1.5 2.5 5.5 11.7 20.2 33 44 70 90

Solution:
Q = k (h – a)n
Note: ‘a’ is stage value for Q = 0; is stage value for gives Value of Q, K and n and
'a' are required to be determined. Assume ‘a’ and plot (h – a) on x-axis (log scale)
and Q on Y axis (log scale). If this graph is a straight line, assumed value of ‘a’ is
correct, if not change it till you get a straight line.
To get ‘a’ value following method may be followed
h1h 3 − h 22
a=
(h1 + h 3 ) − 2h 2
Choose h1, h2, h3 such that corresponding values of Q1, Q2, Q3 follow:

Q 2 = Q1 Q3

Hence, from given data


Let h1 = 2 giving Q1 = 1.5
h2 = 2.9 giving Q2 = 5.5
h3 = 4.5 giving Q3 = 20.2
River Flow Measurement 315

\ Q 2 = Q1Q3 = (1.5)(20.2) = 5. 5

∴a =
( )
h1h 3 − h 22
h1 + h 3 − 2h 2

2(4.5) − (2.9) 2
=
2 + 4.5 − 2(2.9)
9 − 8.41 0.59
= = = 0.84
6.5 − 5.8 0.70
Assume ‘a’ = 0.9, and plot Q v/s(h – a) It gives a straight line, see fig. 15.7.
In absence of a log-log paper, log values be taken and plotted: see Fig 15.7 For
Fig. 15.7, n = 2.2 and k = 1.2, Q = 1.2(h – a)2.2 = 1.2(h – 0.9)2.2 is the equation for
stage – discharge relationship.
For h = 5m, Q = 26.7 m3/sec and for h = 10m, Q = 154. m3/sec.
Example 15.6: Develop an equation of the form Q = k(y – a)b from following
stage discharge data :
Stage y(m) 216.46 217.54 219.74
3
Q (m /sec) 55 260 1260

Solution:
Here Q1 = 55, Q2 = 260, Q3 = 1260
Q1 55
∴ = = 0.21 and
Q 2 260
Q 2 260
= = 0.81
Q3 1260

i.e., Q 2 = Q1Q3

y1y3 − (y 2 )2
∴a =
(y1 + y3 ) − 2y 2

(216.46)(219.74) − (217.54) 2
=
(216.46 + 219.74) − 2(217.54)

47565 − 47323 242


= = = 201.6
436.2 − 435 1.2
b
Q 2 k(y 2 − a)b  y 2 − a 
Now, = = 
Q3 k(y3 − a)b  y3 − a 
316 Irrigation Engineering and Hydraulic Structures

b
260  217.54 − 201.6 
∴ = 
1260  219.74 − 201.6 
b
 15.94  b
∴ 0.21 =   = (0.878)
 18.14 
∴ log (0.210) = b log (0.878)

0.677
∴b = − = 11.98
0.565
∴ b  12
Q3 1260 1260
Also, K= b
= 12
=
(y3 − a) (214.74 − 201.6) 1.26 × 105

= 9.92 × 10–13
∴ Q = 9.92 × 10–3 (y – 201.6)12
References
A. STANDARD BOOKS AND TREATISES
S. No. Title Author and Publication
1. Water Resources Engineering R. K. Linsley and J B Franzini
Mc Graw Hill Book Co., New York,
1974
2. Ground Water Hydrology D. K. Todd
John Wiley and Sons, New York
3. Irrigation Enginnering S. Leliavsky
Vol I to Vol V Chapman and Hall, London
4. Open Channel Hydraulics Ven Te Chow
Mc Graw Hill Book Co., International
Students Edition.
5. Water Wealth of India Dr. K. L. Rao
Longman Publication
6. Engineering for Dams Coreager, Justin and Hinds
Vol I, II and III John Wiley and Sons, New York,1945
7. Earth and Rock Dams Sherard, James and Others, John Wiley
and Sons,USA,1963
8. Dams and Control Works USBR Publication 1954
9. Irrigation Vol. I to VII Khushlani K. B. and
Manohar Khushlani
Oxford and IBH Publishing Co., New
York
10. Hydrology H. M. Raghunath
Wiley Eastern Ltd., New Delhi
11. Engineering Hydrology K Subramanaya
Tata – Mc Graw Hill Co., New Delhi.
12. Irrigation Engineering G. L. Asawa
Wiley Eastern Ltd., New Delhi, 1993
13. Irrigation Engineering and S. K. Garg
Hydraulic Structures Khanna Publication, New Delhi
14. Irrigation, water Resources and P. N. Modi
Water Power Engineering Standard Book House, New Delhi
15. Design of Small Dams USBR Publication 1960

© The Author(s) 2023 317


S. K. Ukarande, Irrigation Engineering and Hydraulic Structures,
https://doi.org/10.1007/978-3-031-33552-5
318 Irrigation Engineering and Hydraulic Structures

S. No. Title Author and Publication


16. Text Book of Hydrology Jayarami Reddy
Laxmi Publication Ltd., New Delhi
17. Water Power Engineering Dandeker and Sharma
Vikas Publication, Delhi
18. Embankment Dams Sharma H. D.,
Oxford and IBH Publishing Co., New
Delhi, 1991
19. Flow in Open Channels Subramanya K.
Tata – Mc Graw Hill, New Delhi
20. Hand book of Applied Ven Te chow,
Hydrology Mc Graw Hill, New York, 1964
21. Theory and Design of Varshney, Gupta and Gupta
Irrigation Structures Nem Chand and Bros., Roorkee (U.P.,
India)
22. Fundamentals of Irrigation Bharat Singh, Nem Chand and Bros.,
Engineering Roorkie,1983.
23. INDIA 2020 Abdul Kalam and Rajan,
A Vision for the New Penguin Books Pvt. Ltd., New Delhi,
Millennium 1998.
24. River Engineering Peterson M.S., Prentice-Hall, 1986

B. TECHNICAL PAPERS AND REPORTS


S. No. Title Author and Publication
1. Symposium on Optimum Publication No. 93
Requirements and Utilization Central Board of Irrigation and Power,
of water for irrigated crops. Delhi, 1963 (CBIP)
2. Regime Flow in Incoherent Publication No. 20 by Gerald Lacey,
Alluviums Central Board of Irrigation and Power,
Delhi 1939.
3. Canal Lining in India Kanwar Sain,
Proceedings of International Commission
on Irrigation and Drainage, 3rd congress,
San Francisco 1957
4. Symposium on “Water Publication No. 118,
Logging – Causes and CBIP, Delhi, 1972
Remedial Measures”
5. Design of Weirs on Permeable A. N. Khosla, et al, Publication No. 12,
Foundations CBIP, Delhi, 1965
6. Tubewell and Ground Water Publication No. 54,
Resources CBIP, Delhi, 1965
References 319

S. No. Title Author and Publication


7. IS 1893 of 1962 Bureau of Indian Standards, New Delhi.
Recommendations for earth
quake resistant design of
structures
8. Ranga Raju and Asawa, Journal of Hyd. Div. Pros. ASCE, Oct.
“Viscosity and Surface 1977
Tension Effects on Weir Flow”
10. Garde and Ranga Raju, Journal of Hyd. Div. Proc. ASCE, Nov.
“Regime Criteria for Alluvial 1963
Streams”
11. Meyer – Peter and Muller, Proceedings, 2nd IAHR Congress,
“ Formulae for Bed Load Stockholms, 1984
Transport”
12. Einstein, “Bed Load Functions USDA Tech. Bulletin No. 1026, Sept.
for Sediment Transportation in 1950
open Channels”
13. Keenedy R. G., Paper No. 2826, Proc. ICE, London, Vol.
“Prevention of silting in 119, 1895
Irrigation Canals”
14. Casagrande, “Seepage Journal of New England Water works
Through Dams”. Association, June 1937
15. Vittal and Porey, Journal of Hyd. Div. Proc ASCE, 1987
“Design of Cascade stilling
Basins”
16. Report of Irrigation Publication of Ministry of Irrigation and
Commission Agriculture, 1972, New Delhi
17. Proceedings of Workshop on Publication of CBIP, New Delhi.
Sprinklers and Drip Irrigation
18. Varshney and Bajaj, Journal of Irrigation and Power, Oct.
“Ski-jump Buckets on Indian 1970.
Dams”
19. Jogalekar D.V., CBIP Publication No. 60,1971.
“Mannual of River Behavior,
Control and Training”
Subject Index
A Consumptive use of water 16, 17, 45
Alluvial channels 246, 248, 280 Critical slip circle 197, 198, 217
Aqueduct 271-273, 275, 279 Curtain grouting 142, 144, 190
Arch Dam 144, 178, 179
D
Constant angle 178, 179, 184
Divide wall 219, 238, 239
Constant radius 178, 179, 187
Draw down 70, 71, 73
Thin cylinder theory 179, 181, 182 Duty of water 7, 8
Trial load analysis 181, 183, 189
Asphaltic concrete lining 258 E
Earth dams 190
B Causes of failure 214, 217
Balancing depth 246, 262 Design criteria 129, 191, 217
Bundalling 301, 303 Seepage line 193, 203, 207
Barrage 218, 239 Stability analysis 195
Base period 5-9, 23, 24 Earthquake force 121, 125-127, 181
Bed load 98, 283-287, 289 Escape 3, 242
Berms 191 Evaporation losses 19, 97
Bligh’s creep theory 220-222, 239 Exit gradient 219-222, 239, 240
Border strip method 18
F
Branch canals 241, 263
Falls 25, 50, 72
Ski-jump 157, 159, 160 Various types 303
C Fetch 123, 124, 132
Canals 3, 18, 241 Field irrigation requirement (FIR) 14
Alignment 164, 242, 243 Filters 190, 191, 216
Contour 18, 19, 96 Flexibility 264, 265, 268
Distributary 241, 263, 264 Flood control 97, 105, 296
Lining 177, 254, 255 Flood frequency 59, 97, 302
Losses 9, 14, 19 Foundation treatment 143, 144
Free board 123, 124, 133
Ridge 242, 270, 280
Froude number 157, 159, 177
Watershed 38, 55, 242
Furrow method 18, 19, 254
Casagrande 194
Catchment Area 4, 6, 7 G
Cavitation 162, 168, 174 Gates 99, 142, 145
Chute spillway 164 Vertical lift type 173, 177
Command area 3, 4, 6 Gibb’s module 267
Command area development authority Gravity dams 120, 178, 217
(CADA) 6 Groynes 239, 297, 299

© The Author(s) 2023 321


S. K. Ukarande, Irrigation Engineering and Hydraulic Structures,
https://doi.org/10.1007/978-3-031-33552-5
322 Irrigation Engineering and Hydraulic Structures

H P
Head regulator 9, 218, 219 Paleo 7, 24
Head works 3, 18, 218 Permeability 12, 70, 71
Hot weather 5, 6, 22 Perennial season 5, 114
Hydraulic jump 157, 158, 219 Permanent wilting point 10, 11, 17
Hydrograph 46-56, 58, 59 Pipe outlets 265
Flood 18, 26, 46 Piping 214-216, 220, 222
Unit 11, 46, 47 Pitched islands 297, 300
Synthetic 55 Pore pressure 198, 217
Hyetograph 40, 41, 48
Hygroscopic water 10 Q
Quality of irrigation water 9, 10
I
Ice pressure 120, 124 R
Impervious blanket 210 Rabi season 5
Irrigation methods 18, 19 Regime 247-249, 251,286
Irrigation siphon 272, 279 Regulator 9, 218, 219
Cross 3, 70, 87
K
Head 2, 3, 9
Kennedy’s theory 248, 252, 262
Relief wells 215, 217
Key ways 143
Reservoirs 1, 96, 97
Kharif crops 5
Khosla’s theory 222, 232, 234 S
L Ski-jump bucket 157, 159-161, 165
Lacey’s theory 253, 262 Silt factor 220, 249, 251
Lane’s weighted theory 222 Siphon aqueduct 271, 272, 275
Launching apron 300 Super passage 271, 272
Levees 18, 296, 297 T
M Transpiration 13, 241, 244
Main canal 23, 241, 244
Marginal bunds 297 U
Mass curve 34, 102, 119 Uplift force 120, 122, 123
Modular outlets 263, 264, 279 W
O Waterlogging 253-256
Ogee spillway 156, 165, 166 Weirs 3, 218, 239

You might also like